You are on page 1of 212

CRITICAL CARE SELF-ASSESSMENT PROGRAM

2021 ▪ BOOK 1

Pulmonary and
Endocrinology

Series Editors
Bradley A. Boucher, Pharm.D., FCCP, FNAP, MCCM, BCPS
Curtis E. Haas, Pharm.D., FCCP
®
CRITICAL CARE SELF-ASSESSMENT PROGRAM

New!
CCSAP Series
CCSAP 2019–2021 now available

CCSAP Title Release Date BCCCP Test Deadline

2019 Book 1 Infection Critical Care March 15, 2019 September 16, 2019

2019 Book 2 Surgical Patients in the ICU September 16, 2019 March 16, 2020

2020 Book 1 Cardiovascular Critical Care March 16, 2020 September 15, 2020

2020 Book 2 Issues in Critical Care Practice September 15, 2020 March 15, 2021

2021 Book 1 Pulmonary and Endocrinology March 15, 2021 September 15, 2021

Acute Organ Dysfunction and September 15, 2021 March 15, 2022
2021 Book 2
Special Populations

For purchase information, visit www.accp.com/store


®
IMPORTANT INFORMATION ON THE RELEASE OF
CCSAP 2021 Book 1
(Pulmonary and Endocrinology)

TESTING

BCCCP test deadline: 11:59 p.m. (Central) on September 15, 2021.


ACPE test deadline: 11:59 p.m. (Central) on March 15, 2024.

Before submitting a posttest: Check the online errata for any changes or updates to this Critical Care Self-Assessment Program
release.

Tests may not be submitted more than one time. You may complete one or all modules for credit. For information on passing
levels, assignment of credits, and credit reporting, see Continuing Pharmacy Education and Recertification Instructions page
preceding each module.

Important Notice on BCCCP Recertification: Submitting a required posttest for BCCCP recertification attests that you have
completed the test as an individual effort and not in collaboration with any other individual or group. Failure to complete this test
as an individual effort may jeopardize your ability to use CCSAP for BCCCP recertification.

BOOK FORMAT AND CONTENT

E-Media Format: All purchasers of this CCSAP book also have access to the e-media version. Follow these instructions to load
the text and self-assessment questions in this book onto your e-reader, tablet, or Android phone.

Electronic annotation: The online format of this book can be saved to the desktop or printed. The latest version of Adobe Reader
(available free) offers functionality such as highlighting and adding “sticky notes” to the text.

Hyperlinks: To facilitate further learning and research, this publication incorporates hyperlinks to websites administered by other
organizations. Internal and external hypertext links are visible as underlined text in the print book and are active in the Online and
e-Media versions of the book. NOTE: these links are to websites provided by third parties not affiliated in any way with ACCP.
ACCP assumes no liability for material downloaded from or accessed on these websites. It is the responsibility of the reader to
examine the copyright and licensing restrictions of linked pages and to secure all necessary permissions.

Abbreviations, Laboratory Values: Hyperlinks at the start of each chapter and feature lead to tables of selected abbreviations
and reference ranges for common laboratory tests. These tables can be used as a resource in completing the self-assessment
questions.

NOTE: The editors and publisher of CCSAP recognize that the development of this volume of material offers many opportunities
for error. Despite our best efforts, some errors may persist into publication. Drug dosage schedules are, we believe, accurate and
in accordance with current standards. Readers are advised, however, to check package inserts for the recommended dosages
and contraindications. This is especially important for new, infrequently used, and highly toxic drugs.
Senior Director, Professional Development and Member Services: Nancy M. Perrin, M.A., CAE
Director of International Programs and Associate Director of Professional Development: Wafa Y. Dahdal, Pharm.D.
Senior Managing Editor: Edward Alderman, B.S., B.A.
Medical Editor: Kimma Sheldon-Old, Ph.D., M.A.
Information Technology Project Manager: Michael Blackwell, MBA, CISSP, ITIL

For ordering information or questions, write or call:


Critical Care Self-Assessment Program
American College of Clinical Pharmacy
13000 W. 87th St. Parkway
Lenexa, KS 66215-4530
Telephone: (913) 492-3311
Fax: (913) 492-4922
E-mail: accp@accp.com

ISBN-13: 978-1-952291-17-3 (CCSAP 2021 Book 1, Pulmonary and Endocrinology)

Copyright © 2021 by the American College of Clinical Pharmacy. All rights reserved. This book is protected by copyright. No part
of this publication may be reproduced, stored in a retrieval system, or transmitted, in any form or by any means, electronic or
mechanical, including photocopy, without prior written permission of the American College of Clinical Pharmacy.

To cite CCSAP properly:

Authors. Chapter name. In: Boucher BA, Haas CE, eds. Critical Care Self-Assessment Program, 2021 Book 1.
Pulmonary and Endocrinology. Lenexa, KS: American College of Clinical Pharmacy, 2021:page range.

CCSAP™ is a registered trademark of the American College of Clinical Pharmacy.


TABLE OF CONTENTS
Pulmonary and Endocrinology I ������������������������������� 1 Pneumonia in the ICU
Pulmonary and Endocrinology I Panel �������������������������������������������� 3 By G. Christopher Wood, Pharm.D., FCCP, FCCM, BCCCP

Introduction���������������������������������������������������������������������������������������� 95
Mechanical Ventilation and Pulmonary
Epidemiology ������������������������������������������������������������������������������������ 95
Procedures
Pathophysiology�������������������������������������������������������������������������������� 96
By Andrea Sikora Newsome, Pharm.D., FCCM, BCPS, BCCCP
Risk Factors and Prevention ���������������������������������������������������������� 96
Introduction������������������������������������������������������������������������������������������ 7 Diagnosis ������������������������������������������������������������������������������������������ 97
Fundamentals of IPPV������������������������������������������������������������������������ 9 Likely Organisms and Empiric Therapy ���������������������������������������� 98
Fundamentals of NIPPV ������������������������������������������������������������������ 17 Definitive Therapy �������������������������������������������������������������������������� 101
MV Liberation ������������������������������������������������������������������������������������ 18 Options for Nonresponding/Recurrent Infections �������������������� 103
IPPV-Related Organ Dysfunction and Complications ������������������ 19 Conclusion �������������������������������������������������������������������������������������� 105
MV Adjunctive Therapies ���������������������������������������������������������������� 22 References �������������������������������������������������������������������������������������� 105
Other ICU Pulmonary Procedures �������������������������������������������������� 24 Self-Assessment Questions���������������������������������������������������������� 107
References ���������������������������������������������������������������������������������������� 26
Self-Assessment Questions������������������������������������������������������������ 33 Pulmonary and Endocrinology III������������������������� 111
Pulmonary and Endocrinology III Panel�������������������������������������� 113
COPD, Asthma, and PAH
By Paige Garber Bradshaw, Pharm.D., BCCCP; Critical Care Endocrinology
and Maria Guido, Pharm.D., BCPS By Brittany D. Bissell, Pharm.D., Ph.D., BCCCP
COPD and Asthma ���������������������������������������������������������������������������� 37 Endocrine Physiology and Critical Illness Pathophysiology���� 117
COPD Exacerbations ������������������������������������������������������������������������ 40 Adrenal Disorders���������������������������������������������������������������������������� 120
Status Asthmaticus ������������������������������������������������������������������������ 48 Disorders of Anti-Diuretic Hormone �������������������������������������������� 125
Pulmonary Arterial Hypertension �������������������������������������������������� 51 Thyroid Disorders �������������������������������������������������������������������������� 126
PAH-Targeted Therapeutic Options and Drug Administration���� 53 Dysglycemia in the ICU ������������������������������������������������������������������ 128
Conclusion ���������������������������������������������������������������������������������������� 59 Chronic Critical Illness ������������������������������������������������������������������ 129
References ���������������������������������������������������������������������������������������� 59 Conclusion �������������������������������������������������������������������������������������� 130
Self-Assessment Questions������������������������������������������������������������ 63 References �������������������������������������������������������������������������������������� 130
Self-Assessment Questions���������������������������������������������������������� 134
Pulmonary and Endocrinology II���������������������������67
Pulmonary and Endocrinology II Panel������������������������������������������ 69 Immune-Mediated Reactions
By Abby M. Bailey, Pharm.D., BCCCP;
and Jaclyn M. Stoffel, Pharm.D., BCPS
Drug-Induced Pulmonary Disorders
and Pulmonary Drug Delivery Introduction�������������������������������������������������������������������������������������� 137

By Heather P. May, Pharm.D., FCCM, BCCCP, BCPS; Anaphylaxis ������������������������������������������������������������������������������������ 138


and Andrea M. Nei, Pharm.D., BCPS, BCCCP Severe Dermatologic Emergencies ���������������������������������������������� 141
Introduction �������������������������������������������������������������������������������������� 73 Erythema Multiforme �������������������������������������������������������������������� 143
Drug-Induced Interstitial Lung Disease ���������������������������������������� 73 Stevens-Johnson Syndrome and Toxic Epidermal Necrolysis 144
Other Causes of Drug-Induced Lung Injury ���������������������������������� 75 Angioedema ������������������������������������������������������������������������������������ 148
Inhaled Medications ������������������������������������������������������������������������ 79 Pharmacologic Therapy for Acute Angioedema ������������������������ 150
References ���������������������������������������������������������������������������������������� 87 Conclusion �������������������������������������������������������������������������������������� 156
Self-Assessment Questions������������������������������������������������������������ 90 References �������������������������������������������������������������������������������������� 157
Self-Assessment Questions���������������������������������������������������������� 162

CCSAP 2021 Book 1 • Pulmonary and Endocrinology iii Table of Contents


Pulmonary and Endocrinology IV �������������������� 167 Interactive Case: Neuromuscular Blocking
Pulmonary and Endocrinology IV Panel��������������������������������������169 Agents
By Sara J. Hyland, Pharm.D., BCCCP
Interactive Case: Acute Respiratory Interactive Case: Neuromuscular Blocking Agents��������������������192
Distress Syndrome
Hyperlink to activity������������������������������������������������������������������������192
By Stephanie Bass, Pharm.D., BCPS, BCCCP Self-Assessment Questions����������������������������������������������������������201
Interactive Case: ARDS������������������������������������������������������������������ 174
Hyperlink to activity������������������������������������������������������������������������ 174
Self-Assessment Questions����������������������������������������������������������177

Interactive Case: Extracorporeal Membrane


Oxygenation
By Jennifer Falvey, Pharm.D., BCPS

Interactive Case: Extracorporeal Membrane Oxygenation ������182


Hyperlink to activity������������������������������������������������������������������������182
Self-Assessment Questions����������������������������������������������������������186

CCSAP 2021 Book 1 • Pulmonary and Endocrinology iv Table of Contents


A Message from the Editors
As we move into a new edition of the Critical Care Self-Assess- Inside this CCSAP book, you will find user-friendly formatting
ment Program (CCSAP), we wish to thank the many talented as well as graphic elements such as patient care scenar-
volunteers who have given of their time and talents to create ios demonstrating the application of concepts, treatment
this new recertification component for the Board Certified algorithms, descriptions of pivotal studies that may change
Critical Care Pharmacist. Feedback from our readership indi- practice, and summative practice points. New for this series
cates that they appreciate this tool for continuing pharmacy are two electronic formats (i.e., interactive cases, recorded
education and value CCSAP as a source for the latest in evi- webcasts) that enhance our ability to include topic updates
dence-based information for the critical care practitioner or and important new information. As with the traditional
clinician. review-style chapters, these new features include hyper-
links to reference sources, assessment tools, guidelines and
The goal of CCSAP, as for all the Self-Assessment Programs, is resources, and data compilers such as PubMed. We continue
to provide updates that would improve clinical pharmacy prac- our efforts to provide the best information in the most eas-
tice and patient outcomes. The design process for this new ily accessible platforms. Our hope is that CCSAP will have an
series began with a careful review of the content outline devel- immediate and positive impact on the care of patients in the
oped by the Board of Pharmacy Specialties for the Critical ICU and other critical care settings.
Care Pharmacy Specialty Certification Examination. Specific
content was organized on the basis of the systems and patient We gratefully join ACCP’s long tradition of offering the best
care problems that might be expected of the board certified products for continuing pharmacy education and pharmaco-
critical care pharmacy specialist (BCCCP). Invitations then therapy specialist certification, and pledge that, with the help
went out to potential faculty panel chairs, authors, and review- of our many talented contributors, CCSAP will continue that
ers committed to this specialty and to the board certification tradition.
process.
Bradley A. Boucher and Curtis E. Haas, series editors

CCSAP 2021 Book 1 • Pulmonary and Endocrinology v A Message from the Editors
Pulmonary and Endocrinology I
Pulmonary and Endocrinology I Panel

Series Editors: Department of Pharmacy Services


UCHealth—University of Cincinnati Medical Center
Bradley A. Boucher, Pharm.D., FCCP, FNAP, MCCM, BCPS
Adjunct Clinical Instructor of Pharmacy Practice
Professor of Clinical Pharmacy and Translational Science
Department of Pharmacy Practice
Associate Dean for Strategic Initiatives and Operations
and Administrative Sciences
College of Pharmacy
University of Cincinnati, James L
University of Tennessee Health Science Center
Winkle College of Pharmacy
Memphis, Tennessee
Cincinnati, Ohio
Curtis E. Haas, Pharm.D., FCCP
Maria Guido, Pharm.D., BCPS
Chief Pharmacy Officer
Clinical Pharmacy Specialist, Internal Medicine
University of Rochester Medical Center
Department of Pharmacy
Rochester, New York
UC Health—University of Cincinnati Medical Center
Faculty Panel Chair: Cincinnati, Ohio

Alexander H. Flannery, Pharm.D., FCCM, BCCCP, BCPS Reviewers


Assistant Professor
Chris Droege, Pharm.D., FCCM, FASHP, BCCCP
University of Kentucky College of Pharmacy
Lexington, Kentucky Clinical Pharmacy Specialist, Critical Care
Director, PGY2 Critical Care Pharmacy Residency Program
Department of Pharmacy
Mechanical Ventilation and Pulmonary UC Health—University of Cincinnati Medical Center
Procedures Cincinnati, Ohio

Author Zachary Smith, Pharm.D., BCPS, BCCCP

Andrea Sikora Newsome, Pharm.D., FCCM, BCPS, BCCCP Clinical Pharmacy Specialist, Critical Care
PGY2 Critical Care Program Director
Clinical Assistant Professor
Department of Pharmacy
Department of Clinical and Administrative Pharmacy
Henry Ford Hospital
University of Georgia College of Pharmacy
Detroit, Michigan
Augusta, Georgia

Reviewers
The American College of Clinical Pharmacy and the authors
W. Anthony Hawkins, Pharm.D., BCCCP thank the following individuals for their careful review of the
Clinical Associate Professor Pulmonary and Endocrinology I chapters:
Department of Clinical and Administrative Pharmacy
Marisel Segarra-Newnham, Pharm.D., MPH, FCCP,
University of Georgia College of Pharmacy
BCPS, BCIDP
Clinical Assistant Professor
Department of Pharmacology and Toxicology Clinical Pharmacy Specialist, Infectious Diseases/HIV
Medical College of Georgia at Augusta University Antimicrobial Stewardship Program Pharmacy Director
Albany, Georgia Veterans Affairs Medical Center
West Palm Beach, Florida
Brian Murray, Pharm.D., BCCCP Clinical Assistant Professor of Pharmacy Practice
Clinical Pharmacy Specialist University of Florida College of Pharmacy
Department of Pharmacy Gainesville, Florida
University of North Carolina Medical Center
Jeffrey T. Sherer, Pharm.D., MPH, BCPS, BCGP 
Chapel Hill, North Carolina
Clinical Professor
Department of Pharmacy Practice
COPD, Asthma, and PAH and Translational Research
Authors University of Houston College of Pharmacy
Houston, Texas
Paige Garber Bradshaw, Pharm.D., BCCCP
Clinical Pharmacy Specialist—Critical Care/
Emergency Medicine
DISCLOSURE OF POTENTIAL CONFLICTS OF INTEREST
Consultancies: Stephanie Bass (Entasis); Mitchell S. Buckley (Wolters Kluwer); Chris Droege (Deloitte Consulting LLP); Eric Johnson
(Slayback Pharmaceuticals); Patrick M. Wieruszewski (La Jolla Pharmaceutical Company); G. Christopher Wood (ASHP)

Stock Ownership:

Royalties:

Grants: Brittany Bissell (ASHP, University of Kentucky); Chris Droege (ACCP); Alexander H. Flannery (ACCP, American Society of
Nephrology, La Jolla Pharmaceutical Company); Anthony Hawkins (University of Georgia College of Pharmacy); Heather P. May
(ACCP, Mayo Clinic); Andrea Sikora Newsome (NIH); Joseph M. Swanson (Cubist/Merck)

Honoraria: Abby Bailey (ACCP EM PRN); Joseph M. Swanson (United Arab Emirates – Ministry of Health)

Other:

Nothing to disclose: Mahmoud A. Ammar, Paige Garber Bradshaw, Jennifer Falvey, Maria Guido, Leslie A. Hamilton, Sara Jordan
Hyland, Heather A. Johnson, Debbie Liang, Brian P. Murray, Andrea M. Nei, Jaimini S. Patel, Michael A. Peters, Michael A. Rudoni,
Poorvi Shah, Zachary R. Smith, Jaclyn Stoffel, Kyle A. Weant, Maggie Zhao

ROLE OF BPS: The Board of Pharmacy Specialties (BPS) is an autonomous division of the American Pharmacists Association
(APhA). To maintain its strict, independent standards for certification, BPS does NOT endorse or provide review information,
preparatory courses, or study guides for Board Certification Examinations. The Board, through its specialty councils, is respon-
sible for specialty examination content, administration, scoring, and all other aspects of its certification programs. BPS is totally
separate and distinct from ACCP. CCSAP has been approved by BPS for use in BCCCP recertification. Information about the BPS
recertification process is available online.

Questions regarding BCCCP recertification should be directed to:

Board of Pharmacy Specialties


2215 Constitution Avenue NW
Washington, DC 20037
(202) 429-7591
CONTINUING PHARMACY EDUCATION
AND RECERTIFICATION INSTRUCTIONS
Continuing Pharmacy Education Credit: The American College of Clinical Pharmacy is accredited by the Accreditation
Council for Pharmacy Education (ACPE) as a provider of continuing pharmacy education (CPE).

CCSAP Target Audience: The target audience for Pulmonary and Endocrinology is not only ICU and ED pharmacists across the
spectrum of care but also any pharmacist caring for acutely ill patients whose management may be complicated by these
challenging scenarios.

Available CPE credits: Purchasers who successfully complete all posttests for CCSAP 2021 Book 1 (Pulmonary and Endocrinology)
can earn 22.5 contact hours of continuing pharmacy education credit. The universal activity numbers are as follows: Pulmonary
and Endocrinology I, 0217-0000-21-030-H01-P, 5.5 contact hours; Pulmonary and Endocrinology II, 0217-0000-21-031-H01-P,
5.0 contact hours; Pulmonary and Endocrinology III, 0217-0000-21-032-H01-P, 5.5 contact hours; and Pulmonary and Endocrinology IV,
0217-0000-21-033-H01-P, 6.5 contact hours. You may complete one or all available modules for credit. Tests may not be submitted
more than one time.

TO EARN CPE CREDITS FROM THIS CCSAP BOOK

Posttest access: Go to www.accp.com and sign in with your e-mail address and password. Technical support is available from 8 a.m.
to 5 p.m. (Central) weekdays by calling (913) 492-3311. CCSAP products are listed under My Products on your My Account page.

BCCCP test deadline: 11:59 p.m. (Central) on September 15, 2021.


ACPE test deadline: 11:59 p.m. (Central) on March 15, 2024.

BCCCP Recertification Credit: To receive BCCCP recertification CPE credit, a CCSAP posttest must be submitted
within the 6-month period after the book’s release (see above). Only completed tests are eligible for credit; no partial
or incomplete tests will be processed. You may complete one or all available modules for credit. Tests may not be
submitted more than one time.

The passing point to earn BCCCP recertification credit is based on an expert analysis of the assessment items in each posttest
module. Any posttest submitted before the BCCCP test deadline that meets this passing point will earn BCCCP recertification
credits. These credits will be assigned as of the date of test submission and reported within 48 hours to BPS. For statements of
recertification credit, visit www.bpsweb.org.

Questions regarding the number of hours required for BCCCP recertification should be directed to BPS at (202) 429-7591 or
www.bpsweb.org. The ACCP Recertification Dashboard is a free online tool that can track recertification credits as they are
earned through ACCP and schedule new opportunities for credits from upcoming ACCP professional development programs.

ACPE CPE Credit: To receive ACPE CPE credit for a CCSAP module, a posttest must be submitted within 3 years after the book’s
release (see above). Only completed tests are eligible for credit; no partial or incomplete tests will be processed. You may complete
one or all available modules for credit. Tests may not be submitted more than one time.

Any posttest submitted before the ACPE deadline that scores 50% or greater will be awarded the appropriate CPE. These
credits will be assigned as of the date of test submission and reported within 48 hours. For statements of CPE credit, visit
www.mycpemonitor.net.

Posttest answers: The explained answers—with rationale and supporting references—will be posted 2 weeks after the BCCCP
test deadline and will be available to anyone who has either (1) submitted a posttest or (2) waived the right to receive credit from
a posttest (see below). Go to www.accp.com and sign in with your e-mail address and password. Click the CCSAP book on your
My Account page and you will see a link to the explained answers.

Test Waivers: To access the explained answers without submitting a posttest, sign in to your My Account page, select the
CCSAP book, and click on the waiver link for that module. By completing the waiver form for a module, you waive the opportunity
to receive CPE credit for that module. After you submit a waiver, you will see a link to the PDF file that contains the answers for
the module you waived. Answers will be available starting 2 weeks after the BCCCP test deadline.
Mechanical Ventilation and Pulmonary
Procedures
By Andrea Sikora Newsome, Pharm.D., FCCM, BCPS, BCCCP

Reviewed by W. Anthony Hawkins, Pharm.D., BCCCP; and Brian Murray, Pharm.D., BCCCP

LEARNING OBJECTIVES

1. Justify the role of the clinical pharmacist in the care of mechanically ventilated patients.
2. Evaluate the key differences among the different modes of mechanical ventilation.
3. Develop a pharmacist-oriented evaluation and associated interventions for a mechanically ventilated patient.
4. Evaluate the independent effect of inspiratory positive airway pressure on the different organ systems in critically ill
patients and potential for associated pharmacotherapy.
5. Justify the role of adjunctive therapy specific to the mechanically ventilated patient.
6. Account for the therapeutic use of pulmonary procedures in critically ill patients.

INTRODUCTION
ABBREVIATIONS IN THIS CHAPTER
Clinical Challenges of MV
APRV Airway pressure release ventilation
ARDS Acute respiratory distress syndrome Mechanical ventilation is a common modality in the supportive care
BiPAP Biphasic positive airway pressure for critically ill patients. It cannot be viewed in isolation, but is deeply
CO2 Carbon dioxide intertwined with a patient’s overall clinical status and pharmaco-
FiO2 Fraction of inspired oxygen therapy regimen (Cawley 2011). Incorporation of MV assessment is
I:E ratio
Inhalation/exhalation ratio vital for pharmacists to provide optimal pharmacotherapeutic care
IPPV Invasive positive pressure (Cawley 2007, 2011, 2019, Newsome 2018).
ventilation
Epidemiology
MV Mechanical ventilation
In the United States, more than 750,000 patients annually are sup-
NIPPV Noninvasive positive pressure
ventilation ported with IPPV at a cost exceeding $27 billion, or 12% of hospital
NMBA Neuromuscular blocking agent costs overall. Patients supported with IPPV are critically ill and have
O2 Oxygen in-hospital mortality as high as 35% (Marshall 2008, Chant 2015).
PaO2 Partial pressure of oxygen The pharmacotherapy regimens associated with MV are complex,
PC Pressure control with more than 30% of these patients in the ICU setting prescribed
more than 20 medications. These studies showed that 70% of these
PEEP Positive end-expiratory pressure
patients had more than 13 medications prescribed at any given point
PRVC Pressure-regulated volume control
(Uijtendaal 2014, Newsome 2020).
PS Pressure support
RR Respiratory rate Indications
RSBI Rapid shallow breathing index Indications for IPPV are in three broad categories: hypoxemic respira-
SBT Spontaneous breathing trial tory failure, hypercapnic respiratory failure, and apnea (Tobin 2013).
VAP Ventilator-acquired pneumonia In addition, airway protection in patients at high risk of aspiration or
VC Volume control loss of airway and increased work of breathing may be indications
VILI Ventilator-induced lung injury for MV. Whereas breathing accounts for about 1%–3% of total O2
V T Tidal volume consumption in healthy adults, studies of patients with acute hypox-
emic respiratory failure and shock states demonstrate that work
Table of other common abbreviations.
of breathing can account for 20% of total O2 consumption, which

CCSAP 2021 Book 1 • Pulmonary and Endocrinology 7 Mechanical Ventilation and Pulmonary Procedures
is an unsustainable physiologic state (Manthous 1995). Role of the Clinical Pharmacist in the
Understanding the specific underlying pathology for why a Setting of MV
patient was intubated is vital for guiding pharmacotherapeu- Pharmacotherapy and MV management are interdependent
tic care to address the underlying condition. Often, patients in critically ill patients. A working knowledge of IPPV may
are intubated for one indication but must remain intubated for enhance pharmacists’ ability to provide high-quality inter-
an entirely different reason. This cause may be iatrogenic and ventions that improve patient outcomes, such as mortality,
either preventable or reversible with pharmacotherapeutic length of stay, and duration of MV (Marshall 2008, Chant
intervention (Newsome 2018). Box 1 lists common indica- 2015, Newsome 2018).
tions for MV.
Introduction to Associated Medication Therapy

Goals of MV Essential knowledge of the fundamentals of IPPV allows the


Three general principles of critical care may be applied to MV pharmacist to incorporate IPPV as a medication monitoring
goals: (1) reverse the initial indication for MV; (2) provide sup- variable; for example, increasing FiO2 requirements because
portive care during this reversal process; and (3) minimize of pulmonary edema in the face of diuretic therapy may war-
any complications of the first two processes. Specific goals rant escalation of deresuscitative efforts. Further, the goals of
include achieving appropriate oxygenation and ventilation specific medications may be oriented toward IPPV. A patient
(i.e., CO2 removal), minimizing VILI, avoidance of patient– who is too sedated from continuous infusions of sedative
ventilator asynchrony and patient discomfort, and resolv- or analgesic medications to pass an SBT, for example, will
ing the condition with the shortest duration of IPPV feasible remain intubated secondary to mismanagement of pain, agi-
(Tobin 2013), as shown in Figure 1. tation, and delirium. Further, the ICU mnemonic recommends
twice daily evaluation of the 11 F2 AST HUGS BID variables: (1)
feeding; (2) fluids; (3) analgesia; (4) sedation; (5) thromboem-
bolic prophylaxis; (6) head of bed elevation; (7) ulcer (stress)
BASELINE KNOWLEDGE STATEMENTS
prophylaxis; (8) glycemic control; (9) spontaneous breathing
Readers of this chapter are presumed to be familiar trial; (10) bowel regimen; (11) indwelling catheter removal;
with the following: and (12) de-escalation of antibiotics (Vincent 2005, 2009;
• General knowledge of the care of critically ill Hawkins 2019). Notably, of the 11 variables, 9 are medication
patients related and 10 show evidence of improved adherence and out-
• General knowledge of the pathophysiology that comes with pharmacist involvement (Lat 2020).
leads to requirement of mechanical ventilation
Protocol Development and Adherence for
• General knowledge of acid-base disorders and how Patients on MV
to interpret an arterial blood gas
Pharmacists are champions for quality improvement processes
• Knowledge of common ICU drugs such as those for
stress ulcer prophylaxis, continuous infusion that often include but extend beyond core pharmacothera-
analgesics, and sedatives peutic care. In these roles, pharmacists have shown notable
• Consequences of inappropriate pharmacotherapy
regimens in critically ill patients and the role of
pharmacists in the ICU Box 1. Indications for Mechanical
Ventilation
Table of common laboratory reference values.
Hypoxemic Respiratory Failure
• Hypoventilation—obesity-related
ADDITIONAL READINGS • Ventilation-perfusion mismatch—obstructive lung disease,
pneumonia
The following free resources have additional back- • Right-to-left shunt—anatomic shunts
ground information on this topic: • Diffusion impairment—pulmonary fibrosis
• Cawley MJ. Mechanical ventilation: introduction for • Reduced atmospheric pressure
the pharmacy practitioner. J Pharm Pract 2011; Hypercapnic Respiratory Failure
24:7-16. • Chronic obstructive pulmonary disorder
• Cawley MJ. Advanced modes of mechanical • Severe asthma
ventilation: introduction for the critical care • Neuromuscular skeletal diseases—myasthenia gravis
pharmacist. J Pharm Pract 2019;32:186-98. • Decreased respiratory-motor drive—CNS infections,
malignancy, traumatic brain injury, medications
• Fan E, Zakhary B, Amaral A, et al. Liberation from
mechanical ventilation in critically ill adults. an Apnea
official ATS/ACCP Clinical Practice Guideline. • Traumatic brain injury
Ann Am Thorac Soc 2017;14:441-3. • Stroke
• Drug overdose

CCSAP 2021 Book 1 • Pulmonary and Endocrinology 8 Mechanical Ventilation and Pulmonary Procedures
Principles of Treat underlying
Supportive care Minimize harm
critical care cause

Allow time for Medication


Direct ventilator
Application to MV reversal of therapy (e.g.,
induced lung injury
indication antibiotics)

Provide
Associated
adequate Interventions (e.g.,
complications (e.g.,
oxygenation and chest tubes)
infections, SRMD)
ventilation

Duration of MV,
ICU length of stay

Figure 1. Goals of mechanical ventilation.


MV = mechanical ventilation; SRMD = stress-related mucosal disease.
Information from: Tobin MJ. Principles and practice of mechanical ventilation, 3rd ed. New York: McGraw-Hill Medical, 2013.

reductions in mortality and other patient outcomes (e.g., FUNDAMENTALS OF IPPV


MV duration) and significant cost/benefit ratios (Hammond Overall, MV can be classified as invasive or noninvasive
2019, Lee 2019). This vital activity is especially present in ventilation: IPPV and NIPPV. Invasive positive pressure ven-
the setting of MV, for which protocols that center around tilation denotes the use of positive pressure through an
SBT coordination, diuretic management, pain, and seda- invasive airway device, such as an endotracheal tube or
tion titration have proven the beneficial role of pharmacists tracheostomy; whereas noninvasive positive pressure venti-
(Newsome 2018, Bissell 2020). Thus, the importance of phar- lation uses a less invasive measure, such as a face mask.
macist involvement for the outcomes of patients receiving Box 2 summarizes key terminology for MV (Tobin 2013,
MV support cannot be overstated (Marshall 2008, Hahn 2013, Newsome 2018).
Stollings 2015, Louzon 2017, Leguelinel-Blache 2018).

Box 2. Mechanical Ventilation Terminology


Global Terminology • Phase variable: any variable measured and used to start,
• Mechanical ventilation (MV): a technique using an external maintain, and terminate each phase of the respiratory cycle
device to conduct necessary gas exchange for a patient, • Trigger variable: the measured variable to initiate
including oxygenation and ventilation inspiration
• Invasive positive pressure ventilation (IPPV): form of MV • Target variable: the variable that the ventilator device tries to
wherein positive pressure is applied through the use of an achieve and/or maintain before the end of inspiration
endotracheal or tracheotomy tube • Cycle variable: the variable that, when reached, is used to end
• Noninvasive positive pressure ventilation (NIPPV): form of MV the inspiratory phase
wherein positive pressure is applied through a less invasive
Ventilator Settings
measure such as face mask
• Bilevel positive airway pressure (BiLevel): Form of NIPPV that • Tidal volume (V ): the total volume of gas inhaled and exhaled
T
during one respiratory cycle (mL); VT may be expressed in
includes two levels of positive pressure delivered through a
mL/kg of predicted body weight
mask, corresponding to exhalation and inhalation
• Continuous positive airway pressure (CPAP): Form of NIPPV • Respiratory rate (RR): the number of breaths taken (by
machine, patient, or both) per minute (breaths/minute)
that provides a continuous stream of positive pressure
through a mask • Fraction of inspired oxygen (FiO2): the percentage of O2 in the
delivered gas that is administered to the patient (%)
Ventilator Variables • Positive end-expiratory pressure (PEEP): the pressure setting
• Control variable: a pre-determined variable “controlled” by the ma- that maintains positive airway pressures above atmospheric
chine within the equation of motion for the respiratory system pressure during the exhalation phase (cm H2O)

(continued)

CCSAP 2021 Book 1 • Pulmonary and Endocrinology 9 Mechanical Ventilation and Pulmonary Procedures
Box 2. (continued)
• Inhalation/exhalation ratio (I:E ratio): Ratio of time spent ○ Use of APRV is more often associated with lung-protec-
on the inhalation versus exhalation phase in the breathing tive modes and more extreme inhalation/exhalation (I:E)
cycle ratios
• Inspiratory time: the time over which inspiration is delivered; • Biphasic positive airway pressure (BiPAP): from a practical
depending on the mode, this time may be the duration of VT perspective, this mode is almost indistinguishable from APRV
delivery or the time for which a set pressure is maintained; when the same I:E settings are used
inspiratory time is adjusted to alter the I:E ratio. ○ The terms BiPAP and APRV often used interchangeably
in clinical practice because only minor differences exist,
Ventilator Modes largely the result of proprietary distinctions
• Continuous mandatory ventilation (CMV): a mode that pro- ○ In this chapter, BiPAP refers to an invasive form of IPPV
vides only mandatory breaths, allowing for no spontaneous
breaths
• Proportional assist ventilation (PAV): an advanced mode of
ventilation that is synchronized to generate PS in proportion
• Intermittent mandatory ventilation (IMV): a mode that accom- to patient effort; no target flow, VT, or airway pressure targets
modates the patient taking spontaneous breaths between are set
mandatory breaths
• High-frequency ventilation (HFV): this lung protective mode
• Synchronized intermittent mandatory ventilation (SIMV): a uses extremely high RRs and very small VT less than the dead
form of IMV that synchronizes mandatory breaths by using space in the lung to maximize air flow patterns without lung
measured patient effort to trigger the breath damage
• Continuous spontaneous ventilation (CSV): all breaths are • High-frequency percussive ventilation (HFPV): this form of
spontaneously initiated and subsequently dictated by the HFV combines high-frequency ventilation and conventional
patient pressure-cycled ventilation; also known as volume diffusive
• Volume control (VC): a volume-targeted, time-cycled mode respirator (VDR)
wherein the machine delivers the same VT during each
inspiration, which may be initiated by the machine or the
• High-frequency oscillation ventilation (HFOV): this form of
HFV accomplishes gas transport with quasi-sinusoidal flow
patient oscillations; these oscillations act as a mixing method to
• Pressure control (PC): a pressure-targeted, time-cycled mode blend high O2/low CO2 content air with the air from the patient
of ventilation with maximal airway and alveolar pressures
limited by a maximal preset pressure; as such, VT, flow, minute
• High-frequency jet ventilation (HFJV): this form of HFV makes
use of high velocity (“jet”) flow to achieve high RRs with
ventilation, and alveolar ventilation are all dependent on relatively low VT
intrinsic resistance of the respiratory system
• Pressure-regulated volume control (PRVC): a pressure-limited, Ventilator-related Terminology
time-cycled mode of ventilation that targets average VT
○ Similar to PC, a constant pressure is applied throughout an
• Mandatory breath: denotes when a breath is triggered and
cycled (i.e., started and terminated) by the ventilator
inspiration regardless of whether it is a machine-controlled
or -assisted breath
• Spontaneous breath: denotes when a breath is triggered and
cycled (i.e., started and terminated) by the patient
○ The system adjusts the pressure after each breath based
on measured changes in the patient’s airway resistance to
• Minute ventilation (MiV): describes the amount of air that an
individual breaths per minute and is the product of respiratory
deliver the pre-set “goal” VT rate and tidal volume (MiV = RR x VT)
○ To stay within the preset goal ranges, PRVC evaluates each
actual VT with the pre-set VT so that if the delivered volume
• Rapid shallow breathing index (RSBI), Tobin-Yang index: a
screening index for assessment of a patient’s readiness for
is below goal, it can then increase the inspiratory pressure extubation
on the next breath or decrease the volume if the pressure ○ Calculated by division of respiratory rate by tidal volume
is too high (RSBI = RR/VT)
• Pressure support (PS): a mode of pressure-targeted, partial ○ RSBI values less than 105 breaths/minute/L are associated
ventilator support with extubation success (Tobin 2013)
○ Each breath is flow-cycled, patient-triggered, and machine
supported
• Plateau pressure (PPLAT): Pressure at the end of inspiration
applied to the small airways and measured using an
○ Most commonly used to facilitate ventilator weaning and inspiratory hold maneuver
as part of SBTs
• Driving pressure (ΔP): PPLAT–PEEP; has been associated with
• Airway pressure release ventilation (APRV): a pressure- mortality in ARDS
limited, time-cycled (settings: high and low [Thigh and Tlow]),
lung protective ventilation mode that allows for patient initiat-
• Recruitment maneuver: an intervention wherein a sustained
increase in airway pressure is applied with the intent to
ed spontaneous breathing independent of ventilator cycling— reopen (or recruit) collapsed alveoli
the patient can breathe at any point during the breathing
cycle in addition to the machine settings
○ Ventilation occurs between the time-cycled switch between
two pre-set pressure levels (settings: high and low [Phigh and
Plow])

Information from Tobin MJ. Principles and practice of mechanical ventilation, 3rd ed. New York: McGraw-Hill Medical, 2013.

CCSAP 2021 Book 1 • Pulmonary and Endocrinology 10 Mechanical Ventilation and Pulmonary Procedures
Review of Key Terminology and Concepts of
IPPV Phase I: Change from
Invasive positive pressure ventilation is presented herein as a exhalation to inspiration
discussion of the specific variables that characterize a type Trigger: starts
inspiration phase
of breath sequence and are organized by a chosen mode: con-
trol, phase, trigger, target, and cycle.

Ventilator Variables and Settings Phase IV: Phase II:


Ventilator settings may be conceptualized as a series of Exhalation Inspiration
variables—control variables and phase variables—that ulti-
mately help to describe the associated breath sequences and
Target: goal of
ventilator modes. As an equation of motion for the respiratory the inspiration
system, ventilation is a function of three variables: (1) force, phase (ideally
achieved prior to
described as pressure; (2) displacement, described as volume; cycle variable)
and (3) the rate of changes of displacement, described as flow Cycle: stops
(Otis 1950, Tobin 2013). Pressure and volume are the two pri- inspiration phase
mary control variables, with one acting as the independent Phase III: Change from
variable and the other as the dependent variable. Compliance inspiration to exhalation

is a physical property denoting the change in volume of a


gas or elastic stretch of a material as it is subjected to an Figure 2. Conceptualization of phase variables
applied force. Pulmonary (or lung) compliance measures the controlling the breathing cycle.
expansion of the lung and is calculated by dividing volume Information from: Tobin MJ. Principles and practice of
by pressure. Although compliance is a dynamic variable over mechanical ventilation, 3rd ed. New York: McGraw-Hill
time, at any instantaneous point in time it is a constant. As Medical, 2013.
such, pressure can be understood as volume, and vice versa.
The relationship between these variables prevents them from
being controlled separately during the same breath. pressure or flow generated. For an assisted breath, the patient
The respiratory cycle consists of four phases: (1) inspira- triggers a breath, but the ventilator provides some support
tion; (2) the shift from inspiration to expiration; (3) expiration; by an increase in airway pressure above baseline during the
and (4) the shift from expiration to inspiration (Marini 1998). inspiration and/or below baseline during expiration. For a
Phase variables are used to start, sustain, and end each phase mandatory breath, the ventilator triggers and cycles a breath.
and may include pressure, volume, flow, and time. To mimic With these three types of breaths, three breath sequences
this finely tuned physiology, the machine has numerous set- are created: (1) continuous mandatory ventilation (CMV);
tings aimed to control these phases. The most important are (2) intermittent mandatory ventilation (IMV); and (3) contin-
the trigger, target, and cycle settings (Figure 2). One breath uous spontaneous ventilation. Many permutations within
is defined as one cycle of inspiration (e.g., positive flow) and this construct are possible and form the basis of a myriad
expiration (e.g., negative flow) and is characterized by the of ventilation modes, which is beyond the scope of this dis-
start (the trigger) and stop (the cycle) of inspiration. As such, cussion. For example, synchronized intermittent mandatory
the trigger may be either patient- or machine-initiated based ventilation is a common subtype of intermittent mandatory
on preset criteria, including elapsed time since the last breath ventilation, wherein a mandatory breath is triggered by the
or patient-generated negative inspiratory pressure/volume. patient through the setting of the trigger variables—and may
After this trigger is initiated, the breath may be cycled by include a series of patient- and machine-dictated breaths—
either the patient or machine using time, volume, or pressure with the goal to ultimately minimize dyssynchrony. Figure 3
criteria. and Figure 4 illustrate breath sequences.

Breath Sequences Ventilator Modes


Three types of breaths are possible: spontaneous breaths, Holistically, ventilation mode may be thought of as the pre-
which describes patient-controlled breathing; assisted set pattern of patient-ventilator interactions designed to
breaths, which describes patient-initiated but machine-as- achieve patient-specific goals and objectives. Common
sisted breathing; and mandatory breaths, which describes modes of ventilation include pressure support (PS), volume
machine-controlled breathing. For a spontaneous breath, control (VC), pressure control (PC), and pressure-regulated
the patient determines both the timing—when inspiration is volume control (PRVC). In VC, the V T is held constant while
started and ended—and the V T, whereas the machine reacts pressure steadily increases throughout the inspiration as
to the patient-initiated breath by recognizing the negative a result of the steadily increasing volume in the lungs. If a

CCSAP 2021 Book 1 • Pulmonary and Endocrinology 11 Mechanical Ventilation and Pulmonary Procedures
Volume Control

Preset
flow rate
Flow

Preset backup time

Pressure PEEP

Time triggered
Patient triggered
Preset breath
breath
volume
Volume

Figure 3. Patient- versus time-triggered breaths in volume control. In the patient-triggered breath, a negative
inspiratory pressure prompts the machine to initiate gas flow to achieve the preset volume. In the time- (or machine-)
triggered breath, the variable “time” acts as the trigger variable causing the machine to initiate a breath with the preset
volume.
PEEP = positive end-expiratory pressure.
Information from: Tobin MJ. Principles and practice of mechanical ventilation, 3rd ed. New York: McGraw-Hill Medical, 2013.

SIMV-PRVC Flow Pattern


Preset flow rate

Preset backup time


Flow
Pressure-supported
breath

Time triggered breath


Pressure PEEP

Preset volume
Volume

Patient triggered breath

Figure 4. Synchronized intermittent mandatory ventilation (SIMV) ventilation flow pattern. In SIMV, patient-triggered
breaths receive machine support through positive pressure, but the volumes are patient dictated. If the patient does
not initiate a breath, a pre-set trigger (time) will initiate a breath with a pre-set volume or pressure. In patient-triggered
breaths, small negative inspiratory pressures are generated and act as the trigger variable.
PEEP = positive end-expiratory pressure; PRVC = pressure-regulated volume control.
Information from: Tobin MJ. Principles and practice of mechanical ventilation, 3rd ed. New York: McGraw-Hill Medical, 2013.

ventilator monitor is visualized, the pressure curve will be volume curve will vary, and the flow pattern will be deceler-
seen as increasing (as volume increases), the inspiratory flow ating to achieve this flattened curve (similar to pressing the
will be held constant, and a specific consistent V T is targeted gas pedal to accelerate to achieve a certain speed and then
with the volume of each breath (similar to turning on a faucet gradually letting off to maintain that new speed). As a breath-
and allowing it to fill a cup to the brim). In PC, the V T varies to-breath mode of ventilation, PRVC provides volumes within
to obtain a consistent pressure from breath to breath and a target range while simultaneously maintaining pressures
throughout inspiration, creating a “flat” pressure curve. The within a target range. This ability to manipulate both volume

CCSAP 2021 Book 1 • Pulmonary and Endocrinology 12 Mechanical Ventilation and Pulmonary Procedures
Volume Control

• Constant (‘flat’) flow curve pattern,


essentially a constant rate of gas as a goal
Flow
tidal volume is achieved

Pressure • Increasing or variable pressure curve (in


direct relation to increasing volume)

Volume • Each breath has the same tidal volume

Pressure Regulated Volume Control


• Flow achieves the target pressure and then
decelerates to maintain this constant
Flow pressure
• The pressure curve is flat, reflective of the
Pressure decelerating flow pattern, and changes
breath to breath within a preset range
Volume • Each breath achieves a tidal volume within
a present goal range

Pressure Control
• Flow achieves the target pressure and then
Flow decelerates to maintain this constant
pressure
Pressure • Each breath has the same pressure and
constant pressure for each breath (‘flat’
pressure curve)
Volume
• Each breath will have a variable tidal
volume to achieve the goal pressure

Figure 5. Flow, pressure, and volume curve patterns for volume control, pressure control, and pressure-regulated
volume control.
Information from: Tobin MJ. Principles and practice of mechanical ventilation, 3rd ed. New York: McGraw-Hill Medical, 2013.

and pressure within a targeted range is achieved by deter- IPPV: Concepts, Applications, and Pearls
mining the pressure needed to deliver a target V T after every Pressure Curves and Concepts
breath and using a decelerating flow pattern (Tobin 2013). As Mean airway pressure is the average pressure applied during
such, PRVC has a flat pressure curve (e.g., constant pressure a breathing cycle and is associated with overall alveolar
throughout the breath) similar to PC, but ideally PRVC main- ventilation, alveolar recruitment, and patient oxygenation
tains a V T (and minute ventilation) in a pre-specified range (Figure 6). Thus, increasing mean airway pressure is directly
similar to VC. Actual V T will vary in real-life scenarios because associated with increased PaO2. Taking advantage of this
patient effort and asynchrony disrupt the ability of the venti- concept, APRV increases the time spent at a higher pres-
lator to optimize mechanics, which can lead to significantly sure level, as previously described. Because most of the
different minute ventilation from what was expected, and time in traditional ventilator modes is spent in the exhalation
potentially VILI if not closely monitored (Figure 5). phase, the easiest way to increase mean airway pressure is
to increase the level of PEEP, the pressure applied during this
Access
exhalation phase. The pressure-time curve for an inspiratory
Mechanical ventilation can be applied through various hold is shown in Figure 7, and the air-trapping (auto-PEEP)
access modes, including but not limited to an endotracheal that occurs is shown in Figure 8.
device (orotracheal or nasotracheal), tracheostomy, and cri-
cothyrotomy. Orotracheal is perhaps the most common form Pressure Support
of invasive airway used, but nasotracheal is highly common In brief, PS is a pressure-targeted, patient-triggered mode
in the setting of head/neck surgeries (Prasanna 2014). In the of ventilation that provides partial respiratory support syn-
setting of an emergency airway or unanticipated difficult chronized with the patient’s (preserved) respiratory drive.
airway, the final treatment option is a cricothyrotomy, often As the least level of IPPV support, PS is most often used for
performed in an emergency bedside setting (Frerk 2015). a patient who was or remains intubated for nonpulmonary

CCSAP 2021 Book 1 • Pulmonary and Endocrinology 13 Mechanical Ventilation and Pulmonary Procedures
PIP: peak inspiratory Mean airway pressure
pressure
Mean airway pressure PEEP: positive end
Incomplete expiration
expiratory pressure

Pressure
PIP PIP PIP
Pressure

PEEP PEEP PEEP PEEP


Time
Time
Figure 8. Air-trapping, auto-PEEP, or breath
Figure 6. Pressure—time curve. The pressure stacking. In this scenario, before the patient can
during the expiratory pressure is characterized by completely exhale, the machine initiates another
the PEEP, whereas the highest pressure achieved breath or “stacks” a new breath on the old breath
during inspiration is the PIP, which is measured in prior to the previous breath being entirely exhaled,
the setting of positive inspiratory flow. In contrast, which creates a situation of ever-increasing pressure
plateau pressure is the pressure that occurs when in the system, denoted as air-trapping or “auto-PEEP.”
the flow is zero (during an inspiratory hold) and is This emergency scenario will be marked by obvious
thus a true measure of alveolar pressure. The mean patient discomfort, alarm signals, and potential
airway pressure reflects the mean of time spent at all alterations in hemodynamic variables and must be
pressures during the respiratory cycle. rectified by allowing the trapped air to be released
from the circuit and likely a change in the I:E ratio.
PEEP = positive end-expirotory pressure; PIP = peak This scenario is more likely in settings in which
inspiratory pressure. exhalation is hindered, such as chronic obstructive
Information from: Tobin MJ. Principles and practice of pulmonary disease and status asthmaticus, resulting
mechanical ventilation, 3rd ed. New York: McGraw-Hill in longer exhalation times.
Medical, 2013.
I:E = inhalation to exhalation ratio; PEEP = positive end-
expiratory pressure.
Information from: Tobin MJ. Principles and practice of
PIP mechanical ventilation, 3rd ed. New York: McGraw-Hill
Medical, 2013.
Plateau Pressure

reasons, such as airway protection, or for a patient who will


Pressure

soon be extubated. Trials of PS, which may also be called


Inspiratory spontaneous breathing trials in clinical practice, are the cor-
Hold
nerstone of IPPV weaning and liberation and are discussed
later in text. A change from PS to PC, VC, PRVC, or an
PEEP PEEP
advanced mode of ventilation indicates a significant “clinical
status change” for the patient. Alternatively, transition to PS
Time may often be viewed as a sign of overall clinical improvement
(Tobin 2013).
Figure 7. Pressure-time curve depicting an
inspiratory hold. In this conceptualization of the
pressure-time curve, PEEP can be seen as the “baseline Effort-Adapted Modes of Ventilation
value” during exhalation, and PIP is the highest Beyond the “traditional” modes of MV, the next stage of IPPV
pressure achieved during inspiration. If an inspiratory technology includes adaptive support ventilation, propor-
hold procedure is performed, the plateau pressure tional assist ventilation, mandatory minute ventilation, and
(essentially, a mean pressure during inhalation), can
neurally adjusted ventilator assistance (Singh 2014). These
be obtained and monitored for associated lung injury.
modes generally aim to optimize airflow patterns in the
PEEP = positive end-expiratory pressure; PIP = peak lungs, minimize VILI, and maximize patient comfort through
inspiratory pressure.
enhanced synchrony methodology. However, inappropri-
Information from: Tobin MJ. Principles and practice of
ate reductions of machine support in response to increase
mechanical ventilation, 3rd ed. New York: McGraw-Hill
patient effort, such as in a setting of anxiety or sepsis, can
Medical, 2013.
potentially lead to clinical decompensation. The novel nature

CCSAP 2021 Book 1 • Pulmonary and Endocrinology 14 Mechanical Ventilation and Pulmonary Procedures
of these modes means limited information is available on
Inverse Ratio Ventilation
their place in therapy.
Inspiratory time
Thigh PressureHigh Mean airway pressure
Inverse Ratio Ventilation
Historically, when more limited MV options were available,

Pressure
clinicians faced with hypoxemic respiratory failure resorted
to a form of “inverse ratio ventilation.” For this type of ven-
Spontaneous
tilation, the I:E ratio was reversed such that the inhalation respirations
time was significantly longer than the exhalation period (in
both normally breathing patients and in most modes of ven- Expiratory time Machine triggered breath PressureLow
Tlow
tilation, the exhalation period is longer than the inhalation Time
period). These long inhalation times and short expiration
times were achieved through manipulation of PC modes, Figure 9. Airway pressure release ventilation
and the approach was also called bi-phasic ventilation to pressure-time curve. Note the “inverse” graph with
denote the alternation between two distinct pressure levels. the prolonged inspiratory time at the high pressure
Conceptually, the advantage of this mode is that it makes the setting and the relatively short expiratory time. This
inspiratory phase the primary driver of mean airway pres- pattern is dictated by I:E. In APRV, the primary settings
sure. By increasing mean airway pressure, oxygenation can to evaluate daily include FiO2, I:E ratio (in particular
Thigh), Phigh and Plow, and respiration pattern, such as the
be improved without increasing peak pressures known to
respiratory rate and patient- versus machine-initiated
cause VILI (Cawley 2019). This higher mean airway pressure breaths.
ultimately leads to improved alveolar lung recruitment and
APRV = airway pressure release ventilation; FiO2 = fraction
oxygenation. This improvement in oxygenation is achieved
of inspired oxygen; I:E = inspiratory to expiratory ratio; Phigh
at the risk of two issues: (1) patient discomfort with invert- = pressure high; Plow = pressure low; Thigh = time high.
ing traditional I:E ratios; and (2) hypercapnia, because the Information from: Tobin MJ. Principles and practice of
exhalation phase is significantly shortened and the respira- mechanical ventilation, 3rd ed. New York: McGraw-Hill
tory cycle is significantly longer than is commonly used in Medical, 2013.
patients with hypoxemic respiratory failure. The net effect is
fewer full V T breaths per minute, leading to lower minute ven-
tilation. Permissive hypercapnia is a ventilation strategy that
“permits” unphysiologically high partial pressures of CO2, (Figure 9) (Varpula 2003, Putensen 2004, Habashi 2005). This
thus allowing for the use of lung protective ventilation with mode of ventilation has been associated with improved out-
a low V T. comes with severe respiratory failure (Lin 2019).
These nuances of APRV have important pharmacologic
Airway Pressure Release Ventilation implications for the practicing pharmacist. First, although
Building on inverse ratio ventilation, a key technological a pervading perspective is that APRV requires deep seda-
advancement allowed a patient to breathe spontaneously at tion and possibly neuromuscular blockade, this assumption
any phase in the breath cycle. This mode of ventilation is gen- should be questioned and evaluated on an individual patient
erally known as airway pressure release ventilation, but may basis because many patients tolerate this mode of ventila-
be referred to colloquially as other proprietary names such tion while using light sedation. Notably, when APRV is used
as BiVent or BiLevel (Tobin 2013). As an inverse-ratio, pres- in the absence of spontaneous breathing activity, the mode
sure-controlled mode that alternates between two pressures is essentially a time-cycled inverse I:E ratio ventilation strat-
(high and low [Phigh and Plow]) over two time-cycled periods egy, increasing risk of hypercapnia (O’Croinin 2005, Daoud
(high and low [Thigh and Tlow]), APRV allows for spontaneous 2007). Second, hypercapnia is an expected occurrence in this
respirations at any point in the breathing cycle (Habashi mode of ventilation and does not necessarily require pharma-
2005). These spontaneous respirations both improve patient cotherapeutic intervention (i.e., acetazolamide, theophylline)
comfort (deep sedation is not a pre-requisite for this mode to “fix” these numbers, which has not been shown to improve
of ventilation) and also significantly contribute to increas- patient outcomes (Faisy 2016, Newsome 2018). Nor does the
ing minute ventilation, which mitigates the hypercapnia that associated respiratory acidosis require correction by sodium
may occur from the shortened expiratory times and longer bicarbonate infusion. Third, APRV is an option in cases
respiratory cycles. Without these spontaneous respirations, of refractory hypoxemic respiratory failure/ARDS (Maung
an appropriate minute ventilation is unlikely to be achieved. 2011, Alhurani 2016). Another therapy for refractory hypox-
Spontaneous breathing activity in APRV also has beneficial emia, NMBAs, and the associated sedation required, reduce
effects on airflow dynamics and lung recruitment that may be spontaneous respirations. Because spontaneous respira-
independently associated with its oxygenation improvements tions in APRV are associated with improved oxygenation, the

CCSAP 2021 Book 1 • Pulmonary and Endocrinology 15 Mechanical Ventilation and Pulmonary Procedures
possibility exists that this combination of therapies negates indicates worse oxygenation status. An FiO2 greater than 60%
positive effects of the other therapies. Neuromuscular block- is associated with O2 toxicity because of free radical forma-
ade has only been studied in ARDS in the setting of the VC tion, and more conservative oxygenation strategies may be
mode (Duan 2017, National Heart Blood Institute 2019). warranted (Tobin 2013, ICU-ROX Investigators 2020).
Positive end-expiratory pressure is the amount of pressure
Prone Positioning in the lungs at the end of expiration. Intrinsic PEEP is pres-
Prone positioning, in which the patient is placed face down ent in all lungs and is a physiologic adaptation to reduce the
to lay on the stomach, is a maneuver recommended by guide- work of breathing. (For active learning, one can breathe out
lines to improve oxygenation and mortality for refractory naturally and then force out a bit more breath–this “extra air”
ARDS and is often used in conjunction with MV. Globally, the is one’s intrinsic PEEP). A possible metaphor for understand-
prone position is used to recruit collapsed alveoli, improve ing PEEP is to imagine the difficulty in the first few breaths
secretion management/prevent atelectasis, and optimize ven- when blowing up a balloon; yet after those first few breaths,
tilation/perfusion matching by shifting perfusion to recruited it becomes much easier to stretch the rubber. With this
alveoli in the anterior lung regions. In general, patients in the action as those first few “balloon breaths” in the alveoli, PEEP
prone position are managed with PEEP levels based on the means that, with each successive breath, one is not required
ARDSNet protocol and low V T ventilation, as observed in the to reopen collapsed or derecruited alveoli. Standard PEEP
PROSEVA study (Guerin 2013). In the era of coronavirus dis- ranges are 5–10 cm H2O, with higher values indicating the
ease 2019, use of the prone position for nonintubated, awake need for alveolar recruitment or difficultly oxygenating (Tobin
patients has shown initial success in oxygenation improve- 2013). Of note, obese patients may require slightly higher
ment and prevention of intubation, a key strategy when levels of baseline PEEP because of obesity-related hypoven-
resources are limited (Ziehr 2020). These studies are gener- tilation and the impact of the chest wall on transpulmonary
ally case series and small retrospective studies, but prone pressure (Pirrone 2016). High levels of PEEP (greater than
positioning for awake patients does seem to improve oxy- 15 cm H2O) may be associated with spontaneous pneumo-
genation (potentially reducing further deterioration) and is a thorax. Thus, as a patient approaches relative maximums of
safe, inexpensive, and adaptable strategy (Caputo 2020, Sun FiO2 and PEEP, the clinician may predict the use of advanced
2020). The position may be paired with several supplemen- modes of ventilation and/or rescue therapies, such as an
tal O2 forms (e.g., nasal cannula, face mask) and used for a NMBA. However, the advent of patient-specific PEEP optimi-
few hours for more transient benefit versus longer periods zation techniques (e.g., driving pressure, esophageal balloon
(Scaravilli 2015, Elharrar 2020). To achieve the oxygenation manometry) identifies an optimal PEEP that may be signifi-
benefits, patients must be in the prone position for extended cantly higher than “usual” but one that ideally optimizes
periods—the protocol for PROSEVA was at least 16 hours individual respiratory mechanics without excess risk from
daily (Guerin 2013). Prone positioning does require specific these higher static pressures.
logistical considerations, including provision of advanced Regarding the interrelationships among ventilation vari-
cardiovascular life support and addressing the potential for ables, generally FiO2 and PEEP are directly associated with
pressure ulcers and ocular edema, as well as nutrition consid- oxygenation and are manipulated for PaO2. In contrast, min-
erations, and interprofessional protocols are helpful. ute ventilation (the product of RR and V T ) is inversely related to
CO2; for example, increasing the minute ventilation decreases
IPPV Monitoring Variables and Concepts CO2. As such, respiratory acidosis/alkalosis may be improved
Two RRs are shown on a ventilator monitor and should be through manipulations of minute ventilation on the machine
evaluated in patient review: the set respiratory rate and the (Tobin 2013).
patient’s actual respiratory rate. These two values are not nec- Low V T ventilation or protective lung ventilation remains
essarily the same. If the patient is “over-breathing” the pre-set the primary intervention to improve mortality in ARDS but
RR, in which the patient’s RR is greater than the pre-set RR, has been increasingly shown to reduce VILI in other disease
then the patient’s RR should be evaluated. Alternatively, the states as well. This type of ventilation targets lower V T (6–8
patient may be “riding the vent,” wherein every breath the mL/kg of predicted body weight) and thus targets lower over-
patient takes is secondary to machine initiation, which may all airway pressures, which reduces VILI (Brower 2000). In
occur for a variety of reasons, including altered mentation or addition, low V T ventilation has been incorporated as a com-
sedation. Furthermore, the patient and ventilator RR may be ponent in several pharmacist-driven quality-improvement
identical, but all of the breaths are patient initiated, which may protocols as a standard of care (Sutherasan 2014, Leguelinel-
indicate an optimal level of sedation and ventilator settings. Blache 2018).
The FiO2 of room air is about 21%. This ventilator vari- Plateau pressure is an IPPV monitoring variable in which
able is typically titrated to O2 saturation and/or PaO2 and is an inspiratory hold is performed manually, wherein the
generally set at 40%, although a setting of 30% may also be machine essentially causes the patient to hold the breath for
used. Higher FiO2, generally in conjunction with higher PEEP, about 0.5–1 second at the end of the inspiratory phase. If an

CCSAP 2021 Book 1 • Pulmonary and Endocrinology 16 Mechanical Ventilation and Pulmonary Procedures
inspiratory hold maneuver is performed, pressure will equil- respiratory drive (Nava 2009). In the critical care setting,
ibrate while flow has ceased and a plateau pressure can be NIPPV is used both as a final measure before intubation
obtained to assess for elevated pressures in the alveoli and and as a means of facilitating successful extubation in the
small airways, which is associated with barotrauma. In the appropriate clinical setting. The two most common modes
setting of high peak pressures but relatively normal plateau of NIPPV are continuous airway pressure (CPAP) and biphasic
pressure, common clinical findings are mucous plugging and positive airway pressure (BiPAP) (Garpestad 2007).
bronchoconstriction, for which mucolytics and bronchodi-
Indications for NIPVV
lators may be used, respectively. Evaluation for the patient
biting the endotracheal tube and need for a bite block may Timing and use of NIPPV remains an ongoing area of research.
be warranted. Elevated plateau pressures often indicate poor Notably, the use of NIPPV has resulted in preventing intuba-
compliance (“stiff” lungs), but may also indicate air trapping tion, achieving successful extubation, and other improved
and the need for evaluation of variables such as I:E ratio and outcomes in chronic obstructive pulmonary disease, asthma,
auto-PEEP (which may be especially appropriate in settings hypoventilation syndromes, trauma, congestive heart failure
of restrictive/obstructive airway disease). (Garpestad 2007).
Ventilator dyssynchrony is a common occurrence that
Biphasic Positive Airway Pressure
should be managed with both ventilator manipulations and
In BiPAP, the machine can be set to support all breaths or only
pharmacotherapy (e.g., analgosedation, NMBA). Types of
to provide support during machine-timed breaths. The two
ventilator dyssynchrony include but are not limited to the fol-
primary settings are inspiratory positive airway pressure and
lowing: ineffective efforts, wherein inspiratory muscle breath is
expiratory positive airway pressure. The difference, or “delta”,
not followed by a ventilator breath; double cycling, wherein the
between these airway pressures is directly related to the V T
inspiratory effort continues beyond the ventilator’s inspiratory
received, and this difference may be used in the setting of
time creating several “breaths” before exhalation; inspira-
hypercapnia to improve ventilation, such as exacerbation of
tory airflow dyssynchrony, wherein patient’s inspiratory effort
chronic obstructive pulmonary disease (Garpestad 2007).
creates concavity in the pressure tracing because of inade-
Figure 10 depicts the relationship of these pressure values
quate gas flow; and reverse triggering (de Haro 2019). Reverse
to continuous positive airway pressure and IPPV. Because
triggering occurs when ventilator insufflations trigger dia-
BiPAP requires the use of a face mask, patient cooperation
phragmatic muscle contractions, a form of patient-ventilator
is an important component—some patients will not tolerate
interaction called entrainment (Akoumianaki 2013, Bourenne
a tight-fitting mask with air being blown on them continu-
2019). This reverse triggering can promote VILI through the
ously. Further, BiPAP must be removed for meals and should
Pendelluft effect (literally “swinging air”) (Greenblatt 2014).
not be used in the setting of nausea/vomiting or questionable
In this phenomenon, nonhomogeneous inflation/deflation
airway protection, such as in ventilation cases as related to
creates regional pressure differences in the lung and airflow
mental status, because the risk of aspiration is high.
among them and thus increases regional Vt and transpul-
monary pressures. Reverse triggering has gained increased Continuous Positive Airway Pressure
attention because of its association with deep sedation and In continuous positive airway pressure, a continuous level of
potential to increase mortality in ARDS and as a potential positive pressure is applied throughout the entire respiratory
confounder in the conflicting results of the trials evaluat- cycle. In contrast to BiPAP, V T cannot be titrated as effec-
ing neuromuscular blockade in this population (ACURASYS tively; therefore, continuous positive airway pressure is often
and ROSE) (Park 2019, Slutsky 2019). Because of the highly used in association with disorders of oxygenation, such as
individual nature of the ventilator-patient interaction, the pos- obstructive sleep apnea, pulmonary edema (Garpestad 2007).
sibility exists that subtle manipulations may improve patient
comfort without adversely affecting oxygenation/ventilation O2 Delivery Systems
goals and may obviate the need for further analgesia/seda- Nasal cannula, high-flow nasal cannula, face masks, and
tion. Clinically, patients may tolerate one form of IPPV well other systems may all be used to deliver O2 and can be classi-
and not another. Within established safety guidelines, efforts fied as low- and high-flow devices, with circuits that contain
should be made to adapt the ventilator to the patient before a humidifier. The nasal cannula is a low-flow device for mild
adapting the patient to the ventilator. hypoxemia and is generally set between 1–6 L/minute. The
FiO2 increases about 4% for every 1 L increase in flow; 1–6 L/
minute correlates to 24%—40% FiO2. A simple face mask is a
FUNDAMENTALS OF NIPPV
low-flow device that can generally be set between 5–10 L/
Key Terminology and Concepts in NIPPV minute (35%–55% FiO2). A nonrebreather mask uses a reser-
Noninvasive positive pressure ventilation delivers positive voir bag and one-way valve to prevent inhalation of previously
pressure ventilation through an interface such as a nasal expired air to deliver higher concentrations of O2 (10–15 L/
mask, face mask, or nasal prongs to patients with preserved minute, 80%–95% FiO2). A venturi mask is a high-flow device

CCSAP 2021 Book 1 • Pulmonary and Endocrinology 17 Mechanical Ventilation and Pulmonary Procedures
PEEP ~ CPAP ~ EPAP
Pressure support ~ IPAP - EPAP

Inspiration Expiration

Pressure

Pressure
support
IPAP

EPAP

Time

Figure 10. BiPAP pressure curve. The EPAP can be conceptualized as about equal to both the CPAP pressure and
also PEEP in IPPV. Further, pressure support is the difference between EPAP and IPAP; thus the settings from a patient
previously receiving IPPV can be used to inform BiPAP settings.
BiPAP = biphasic positive airway pressure; CPAP = continuous positive airway pressure; EPAP = expiratory positive airway pressure;
IPAP = inspiratory positive airway pressure; IPPV = invasive positive pressure ventilation; PEEP = positive end-expiratory pressure.
Information from: Tobin MJ. Principles and practice of mechanical ventilation, 3rd ed. New York: McGraw-Hill Medical, 2013.

that allows for more precise measurement of O2 delivery. Use An appropriate reintubation rate is likely around 15%, with
of a high-flow nasal cannula allows for independent control lower values indicating that patients may have been intu-
of flow and FiO2, can deliver up to 60L/minute, and even pro- bated longer than necessary (Krinsley 2012, Thille 2013).
vides a low level (about 2–5 cm H2O) of PEEP (Brochard 1995). Extubation success rates remain an area of controversy and
ongoing research. Of interest, patients who self-extubate in
MV LIBERATION an unplanned fashion are often able to remain extubated, with
“success” rates ranging up to 70% (Penuelas 2011, Lin 2019).
Role of the Pharmacist
Pharmacotherapy and patient liberation from IPPV are linked,
and pharmacists should monitor how key pharmacotherapy, Spontaneous Awakening Trial/Spontaneous
such as analgesia, sedation, or diuretics, relates to weaning Breathing Trial
guidelines and overall spontaneous awakening trial (SAT)/ Both SATs and SBTs help decrease the time to extubation and
SBT performance. Evaluation of the level of respiratory sup- the ICU length of stay (Kress 2000, Girard 2008, Shi 2013). In
port, including mode, FiO2, and PEEP, all act as important particular, daily SBTs with pressure augmentation (5–8 cm
monitoring variables for therapy such as diuresis, analge- H2O) are recommended by the American Thoracic Society/
sia/sedation, and antibiotic therapy. For example, if a patient ACCP clinical practice guideline for liberation from MV of
becomes apneic on PS, oversedation is a common etiology critically ill adults (Fan 2017b). This use of pressure aug-
and evaluation of residual sedatives or current therapy is mentation in the SBT differentiates it from a more historical
warranted. Conversely, tachypnea may result from patient “T-piece” trial in which the patient breathes spontaneously
anxiety that can be mitigated through the judicious use of with no respiratory support. In this scenario, a patient must
anxiolytics (e.g., dexmedetomidine). overcome the narrowed airway from the endotracheal tube
A patient can be evaluated for liberation from MV by con- to succeed, likened to the difficulty of “breathing through a
sidering the following questions: (1) Has the initial indication straw.” Although the T-piece trial has a respectable positive
that warranted intubation been reversed? (2) Is the patient eli- predictive value, it does not have a good negative predic-
gible for an SBT? If not, why not? (3) Did the patient pass the tive value and is thus considered overly conservative (Fan
SBT? If not, why did the patient experience failure? Was the 2017a). Assessment of SBTs is highly variable, and phar-
SBT failure related to present pharmacotherapy or can it be macists should investigate what specific criteria led to the
addressed by an intervention such as fixing a cuff leak or cor- “failure” of SBT. This assessment includes the conditions of
recting fluid overload? the trial and evaluation of medication therapy that could have
Although extubation failure is associated with worse out- contributed to that failure. Of note, a successful SBT indi-
comes, a causal association has been difficult to determine. cates pulmonary readiness for extubation and appropriate

CCSAP 2021 Book 1 • Pulmonary and Endocrinology 18 Mechanical Ventilation and Pulmonary Procedures
respiratory drive; however, the presence of clinical scenarios a highly coordinated approach, guidelines recommend a
such as angioedema or tracheitis (with the associated thick protocolized and interprofessional approach to liberation
secretions and/or poor cough) may still preclude a patient (American Thoracic Society/European Respiratory Society
from being extubated. Finally, a common misconception is 2002, Devlin 2018).
the requirement for a patient to be able to follow commands
and lack the presence of delirium or altered mental status to
IPPV-RELATED ORGAN DYSFUNCTION
be extubated. Although protecting the airway is of primary
AND COMPLICATIONS
concern, this ability is only partially related to a patient’s
ability to successfully complete a delirium assessment cor- Infectious Complications of MV
rectly. Pharmacy-related considerations during the SAT/SBT Infectious complications of IPPV are relatively common and
process include evaluation of current analgesia/sedation associated with increased mortality and morbidity. Ventilator-
therapy and the use of neuromuscular blockade, as well as acquired pneumonia and ventilator-acquired sinusitis are
clinical conditions that may prohibit the safe performance of two of the most common infectious complications and are
a SAT/SBT, such as an elevated intracranial pressure requir- extensively reviewed in other settings (Kalil 2016). Prevention
ing treatment. of VAP through bundled care is a common approach and
one in which pharmacists can play a key role in implemen-
Weaning Parameters tation, although supportive evidence is variable (Wip 2009,
To assess readiness to liberate, an SBT is performed. An Bouadma 2010, Tokmaji 2015, Caroff 2016, Kalil 2016, Khan
SBT simulates the conditions after extubation (e.g., minimal 2016). For example, the Institute for Healthcare Improvement
vs. no ventilator support), evaluating whether this presently recommends the use of a particular ventilator bundle to
intubated patient can tolerate those conditions. In general, decrease VAP as follows: head of bed elevation between 30 and
minimal PS and PEEP settings are applied, such as PS/PEEP 45 degrees; daily sedative interruptions and assessment of
of 10/5 or 5/5, for about 30–60 minutes, although ranges and readiness to extubate; stress ulcer prophylaxis; and daily use
settings may vary. A combination of clinical evaluation and of chlorhexidine gluconate. Evidence of improvement has
objective indices are used to predict extubation success. been demonstrated with pharmacist involvement in each of
Clinical evaluation may include work of breathing, high RR, these areas (Resar 2005, Leguelinel-Blache 2018).
obvious signs of distress or agitation, and hemodynamic
stability. The RSBI or Tobin index is the most commonly used— Hemodynamic Changes and Assessment
and most validated—objective tool (Tobin 2013). Calculated The net hemodynamic effect of IPPV is the result of a com-
by dividing the respiratory rate by tidal volume (RR/V T ), an plex interplay of positive pressure with pre-existing cardiac
RSBI less than 105 breaths/minute/L is the general cut-off for function, volume status, presence of dynamic hyperinflation
successful extubation, with a 78% positive predictive value of the lungs, and effects specific to the disease state, such
and 95% negative predictive value (Yang 1991). In addition to as sepsis-mediated reductions in systemic vascular resis-
RSBI, a reasonable minute ventilation and overall oxygenation tance (Alviar 2018). For example, in a patient intubated for
must be maintained. Other indices studied have either been status asthmaticus and experiencing dynamic hyperinfla-
less effective, or their complexity limits clinical utility (Yang tion, the end-expiratory lung volume is increased, leading to
1991). A more historical weaning parameter called the nega- elevated intrathoracic pressures. These increased intratho-
tive inspiratory force is assessed by having the patient breathe racic pressures can then reduce the right ventricular venous
in maximally on a small device (low numbers denote weaker return and also the left-ventricular afterload; however, the net
respiratory muscles and predicted less ability to success- effect of these alterations depends largely on the patient’s
fully extubate); however, this device requires the coordinated pre-existing cardiac function (Figure 11). A patient with
effort of a patient who can follow directions, which may or reduced cardiac output may benefit from the reduced after-
may not be actually indicative of a patient’s readiness to load, whereas a patient with a normal cardiac output may
extubate (American Thoracic Society/European Respiratory have reduced function because of the reduced venous return
Society 2002, Fan 2017a, Fan 2017b). or functional preload (Tobin 2013). Sepsis guidelines recom-
An elevated RSBI (or failed SBT) can then be evaluated mend use of dynamic indices (i.e., pulse pressure variation
for medication-related etiologies, including but not limited or stroke volume variation) for assessment of fluid respon-
to fluid overload, agitation, or airway obstruction, which may siveness (Rhodes 2017). Because of alterations of the pulsus
be rectified by such interventions as diuretics, anxiolytics, paradoxus, IPPV interferes with this measurement, and these
or bronchodilators, respectively. Oversedation from opioids measurements have been largely validated in highly con-
or benzodiazepines, which may require discontinuation, trolled IPPV settings, specifically, V T of at least 8 mL/kg and
dose reduction, or more time to be cleared by the body, is machine-controlled respirations (Michard 2000, Marik 2013).
another common cause of SBT failure. Because assessment As such, the pharmacist must exercise caution when inter-
of weaning parameters in conjunction with SBTs requires preting values outside of these specific conditions. Globally,

CCSAP 2021 Book 1 • Pulmonary and Endocrinology 19 Mechanical Ventilation and Pulmonary Procedures
Figure 11. Hemodynamic effects of positive pressure ventilation.
↑ = increased; ↓ = decreased; CO = cardiac output; HR = heart rate; LV = left ventricle; MAP = mean airway pressure; RV = right
ventricle; SV = stroke volume; SVR = systemic vascular resistance.
Information from: Jones TW, Smith SE, Van Tuyl JS, et al. Sepsis with preexisting heart failure: management of confounding clinical
features. J Intensive Care Med 2020:885066620928299.

if a patient has a sudden change in hemodynamic status, effects of MV. Increased intrathoracic pressure from the use
evaluation is warranted of the clinical correlation with recent of positive pressure ventilation results in decreased venous
changes to IPPV settings, such as a blood pressure drop after return (and thus cardiac output) and leads to compensatory
PEEP was adjusted from 5 cm H2O to 15 cm H2O. Hypotension homeostatic responses, including neurohormonal alterations
after rapid sequence intubation (RSI) is a relatively common that increase intravascular volume. These responses include
occurrence resulting from the interplay of RSI medications, increased antidiuretic hormone secretion, reduced atrial
new application of positive pressure, and patient-specific natriuretic peptide secretion, up-regulation of the renin-an-
comorbidities (Lin 2008). Pharmacists should evaluate for giotensin-aldosterone system, and increased sympathetic
this occurrence and consider the RSI agent profile and the outflow (Frazier 1999). Alterations in intra-abdominal pressure
possible requirement for peri-RSI fluids or pressors, either may decrease hepatosplanchnic perfusion, with higher inspi-
push-dose or continuous infusion (Weingart 2015). ratory pressure, V T, and PEEP exaggerating this effect (Jakob
The use of PEEP can have variable effects on the cardiac 2010). Invasive positive pressure ventilation is associated
system based on right and left ventricular function, preload, with decreased urine output, sodium and water retention, and
and afterload, which is especially pronounced in the setting reduced renal perfusion, all contributing to reduced renal clear-
of heart failure. If a patient is in a preload-dependent state ance. This reduced renal function is thought to be mediated
(e.g., right ventricular dysfunction), moderate to high PEEP by renal vasoconstriction and ischemia from increased CO2
may decrease cardiac output; however, in an afterload-de- and decreased oxygen; the use of PEEP, resulting in reduced
pendent state (e.g., left ventricular dysfunction), moderate cardiac output and thus renal perfusion; inflammatory cyto-
to high PEEP may increase cardiac output. Cognizance of kines released from volutrauma or barotrauma; and, finally,
a patient’s cardiac function during changes in MV manage- neurohormonal alterations (Moore 1974, Priebe 1981, Kuiper
ment will improve hemodynamic assessment (Alviar 2018, 2005). In the complex interplay of critical illness that includes
Jones 2020). the potential for a hypermetabolic state, hepatic or renal dys-
function, use of renal replacement therapy, and altered volume
Altered Pharmacokinetics of distribution, IPPV is just one factor. As such, pharmacists
Renal and hepatic function, and thus drug pharmacokinet- should use therapeutic drug monitoring and drug dosing spe-
ics, may be altered by the hemodynamic and neurohormonal cific to critical illness whenever possible (Vitrat 2014).

CCSAP 2021 Book 1 • Pulmonary and Endocrinology 20 Mechanical Ventilation and Pulmonary Procedures
Stress-Related Mucosal Disease Pain, Agitation, and Delirium
The 1994 Cook et al. study identified IPPV as one of the Pain, agitation, and delirium management is a fluctuating tar-
two strongest risk factors for stress-related mucosal dis- get in patients receiving MV, with goals assessed daily for
ease; therefore, stress ulcer prophylaxis has remained a the individual patient. Core components of analgesia and
key component of most ventilator bundles (Cook 1994, sedation management include such tenets as first-line use
Barletta 2016). Risk of stress-related mucosal disease in the of analgosedation, use of nonbenzodiazepine agents (propo-
modern era of critical care management and agent selec- fol or dexmedetomidine), targeting light levels of sedation
tion remain ongoing areas of clinical debate and research. (Richmond Agitation-Sedation Scale 0 to —2), and use of delir-
Presently, both histamine-2 receptor antagonists and proton ium prevention, sedation interruption, and SBTs (Barr 2013,
pump inhibitors have demonstrated superiority over other Devlin 2018). Delirium prevention requires a multifaceted
acid-suppression agents and may be considered based on approach. Pharmacist-led optimization of analgosedation,
patient-specific factors (ASHP Commission 1999, Barletta especially combined with their familiarity of pharmacokinetic
2014, 2016, Alhazzani 2018, Krag 2018, Barbateskovic 2019, profiles, improves outcomes for these patients (Rudis 2000,
Marker 2019). Louzon 2017).

Post-Extubation Stridor Ventilator-Induced Lung Injury


Inflammation and edema around the endotracheal tube can Ventilator-induced lung injury is a preventable cause of lung
create a narrowed airway, resulting in a high-pitched sound, injury secondary to the use of MV and is a significant con-
or post-extubation stridor (PES), which can lead to increased tributor to morbidity and mortality for critically ill patients
rates of extubation failure (Epstein 1998). Pharmacologic (Slutsky 2013). One of the most definitive “positive” trials in
interventions for PES are efficacious when coordinated all of critical care medicine focuses on the mitigation of VILI
appropriately (Pluijms 2015). Guidelines recommend evalua- through the use of low V T ventilation and targeting lower pla-
tion for the risk of PES using a cuff leak test (CLT) for patients teau pressures (Brower 2000). Mediation of VILI is through
on MV who meet extubation criteria and are at high risk of alveolar overdistension and subsequent damage from vol-
PES, and the administration of systemic steroids at least utrauma, barotrauma, atelectrauma, shear strain, and
4 hours before extubation in patients for whom the CLT has inflammatory damage. Also, VILI appears to be increased
failed (Girard 2017). High-risk features include at least one with comorbid physiologic insults such as sepsis or trauma
of the following criteria: (1) traumatic intubation, (2) female (Beitler 2016). Efforts to mitigate VILI can lead to difficult
sex, (3) endotracheal intubation 6 days or more, (4) trauma trade-offs among adequate oxygenation and ventilation while
to upper airway anatomy, (5) reintubation after unexpected doing so with the minimum V T and positive pressure support
extubation, or (6) a large endotracheal tube (greater than 8 necessary (Beitler 2016). The use of PEEP, the prone position,
mm) (Jaber 2003, Wittekamp 2009, Girard 2017). The CLT and the more advanced modes of ventilation (e.g., APRV) all
compares the expiratory V T with cuff inflation and defla- aim to minimize the factors that cause VILI while still achiev-
tion; however, variation in measurement methods and cut-off ing appropriate ventilation targets.
values exist. An absolute volume less than 110 mL for fail-
ure is used most consistently (Miller 1996, Sandhu 2000, ICU-Acquired Muscle Weakness
Pluijms 2015, Smith 2018). Steroids are the primary pharma- Prolonged MV is a significant risk factor for ICU-acquired
cologic prevention and treatment, but many regimens have muscle weakness, which can lead to worse patient outcomes.
been proposed. Intravenous methylprednisolone has demon- Early mobility efforts, even in patients who are undergoing
strated the most promising results, administered as 20 mg MV, are an important consideration and recommended by
every 4 hours starting 12 hours before extubation, with sig- the MV liberation guidelines. Medication-oriented consider-
nificant reduction in laryngeal edema and reintubation rates ations include the shortest duration possible of paralytics,
(Francois 2007, Wittekamp 2009). Dexamethasone has also use of light sedation, and early nutrition and appropriate pro-
been studied, but results are inconsistent (Darmon 1992, tein intake (Hermans 2015, Fan 2017b).
Lee 2007). Protocolized management of mechanically ven-
tilated patients at risk of PES may be a reasonable method Obesity-Related Considerations
to coordinate patient identification, testing, treatment, and Obesity is a commonly encountered comorbidity in patients
extubation (Wittekamp 2009, Girard 2017, Smith 2018). undergoing MV and poses specific challenges (De Jong
Beyond prevention, management strategies vary widely 2017). Obesity causes atelectasis and decreased lung vol-
and are largely extrapolated from studies of patients with umes, increases airway resistance and work of breathing,
asthma. Treatment consistently includes steroids (e.g., intra- and limits the ability to ventilate (termed obesity hypoventi-
venous methylprednisolone 40 mg as a one-time dose) and lation). Overall, oxygenation decreases as weight increases
nebulized epinephrine (e.g., 2.25% racemic epinephrine because of both increased O2 consumption (about 1.5 times
0.5-mL solution as needed) (Pluijms 2015). that for the normal-weight individual) and increased work of

CCSAP 2021 Book 1 • Pulmonary and Endocrinology 21 Mechanical Ventilation and Pulmonary Procedures
breathing. Although patients with an extremely high BMI are hypothesized to be secondary to aspiration of CHG and devel-
excluded from the studies of low V T ventilation, lung protec- opment of ARDS (Kampf 2016). Many regimens have been
tive ventilation with 6–8 mL/kg using predicted body weight evaluated, but a commonly observed version is 0.12% chlor-
is often recommended, and, generally, obese patients seem hexidine gluconate administered as 15 mL, available in unit
to benefit from slightly higher levels of PEEP (De Jong 2017). dose cups, applied to the mouth twice daily throughout the
Furthermore, appropriate positioning and head of bed consid- period of MV (Munro 2009, Zuckerman 2016).
erations are of high importance because of the risk of obesity
supine death syndrome or the alteration of thoracic compli- Bronchodilators
ance secondary to the weight of the panniculus (Lemyze 2018). Bronchodilators are often used in patients receiving MV for
the proposed benefits of enhanced mucociliary clearance,
Refractory Hypoxemia Management optimization of lung mechanics, decreased work of breath-
Invasive positive pressure ventilation remains a core compo- ing, and reduced pulmonary edema. Anticholinergic agents
nent of refractory hypoxemia management with techniques are used to decrease mucous hypersecretion (Manocha 2006,
including recruitment maneuvers and advanced modes of Chang 2007, Ari 2012, Bassford 2012). Whereas patients with
ventilation (e.g., high-frequency oscillation ventilation, APRV) indications such as chronic obstructive pulmonary disease
in conjunction with rescue therapies such as NMBAs, inhaled or asthma have primary indications for the use of inhaled
pulmonary vasodilators, extracorporeal membrane oxygen- agents while receiving MV, IPPV as a primary indication for
ation, and higher hemoglobin goals. High-quality data are these agents remains uncertain. Although these agents
lacking, and management is characterized by widespread have been associated with metrics of improved mechanics,
variation (Duan 2017). these theoretical benefits have not translated to patient-ori-
ented outcomes such as ventilator-free days or reductions in
length of stay (Morina 1997, Perkins 2006, Acute Respiratory
MV ADJUNCTIVE THERAPIES
Distress Syndrome Clinical Trials Network 2011). Although β-2
General Overview of IPPV-Acquired Medication agonists do carry the theoretical risk of causing tachycardia
Interventions
and arrhythmias, no difference in tachycardia risk has been
In addition to many of the higher-level interventions discussed seen between albuterol and levalbuterol (Asmus 2000, Lam
for a patient receiving MV, routine interventions regarding 2003, Scott 2003, Khorfan 2011, Acute Respiratory Distress
medication administration logistics are a necessary role of Syndrome Clinical Trials Network 2011).
pharmacists. These types of interventions may include but
are not limited to evaluation and modification of medications Neuromuscular Blocking Agents
that cannot be crushed (e.g., extended release formulations, In combination with MV, an NMBA is used to improve oxy-
enteric coated, teratogenic medications, capsules); addition genation (PaO2/FiO2 ratios) and reduce VILI in patients with
of IPPV-related prophylaxis (e.g., chlorhexidine gluconate, refractory hypoxemia. An NMBA can reduce ventilator asyn-
stress ulcer prophylaxis); opportunities to incorporate fluid chrony. By improving synchrony, elevated airway pressures,
stewardship by means of concentrating or minimizing exces- which can lead to VILI and associated up-regulation of inflam-
sive fluid intake; promoting deresuscitation; and changing matory biomarkers, are avoided (Gainnier 2004, Forel 2006,
other variables or continuous infusion targets as needed. Murray 2016). All patients receiving an NMBA should be
treated based on the most recent guidelines, including the
Chlorhexidine Gluconate use of scheduled eye care with lubricating drops to avoid
Chlorhexidine gluconate (CHG), as either a 0.12% or 0.2% solu- corneal abrasions and exposure keratitis, and deep seda-
tion, applied to the mouth for oral hygiene up to four times tion (achieved before paralysis) with continuous infusions
daily throughout the period of intubation reduces VAP. A of opioid analgesics and amnestic sedatives (i.e., propofol,
meta-analysis evaluating 18 randomized controlled trials midazolam) (Murray 2016). In addition, pharmacists should
determined that the use of CHG reduced VAP with an impres- evaluate patients on an NMBA for signs of efficacy, defined as
sive number needed to treat of 17 (Hua 2016). Combined with improved oxygenation or ventilator synchrony; signs of over-
this efficacy, a small acquisition cost and a minimal adverse blockade, defined as train of four (less than 1–2 twitches of
effect profile (most commonly including dysgeusia and 4 twitches); and lack of appropriate analgesia and sedation,
mouth discoloration) have led to CHG being recommended in such as lacrimation and diaphoresis (Murray 2016). Of note
many VAP bundles (Andresen 1996). More recently, the role of regarding the train of four, although it is still often used in
CHG has come into question because it has been associated the ICU, it was not used in either the ACURASYS or ROSE tri-
with increased bacterial resistance (although the incidence als and has conflicting literature regarding its overall utility in
is thought to be low), and several meta-analyses have shown monitoring the safety and efficacy of neuromuscular block-
that CHG may increase the mortality risk (Bouadma 2018). ade (Baumann 2004, Papazian 2010, National Heart, Blood
The mechanism of increased mortality is unknown, but it is Institute 2019).

CCSAP 2021 Book 1 • Pulmonary and Endocrinology 22 Mechanical Ventilation and Pulmonary Procedures
Phosphate Replacement acetazolamide only be considered in patients with signifi-
Phosphorus replacement improves diaphragmatic activity cant alkalosis (defined as pH greater than 7.5) with clinically
and potentially reduces IPPV weaning failure (Agusti 1984, important correlates such as reduced respiration rate. This
Brunelli 2007, Miller 2020). Large, randomized studies that value may help reduce the inclination for arbitrary optimiza-
show the benefits of phosphate repletion for a specific goal tion of laboratory values without clinically meaningful effects
are lacking; however, the relationship of phosphorus concen- (i.e., “number fixing”) (Adamson 2017).
trations with both respiratory failure and worse outcomes in
critically ill patients makes phosphorus replacement a rea- Secretion Management
sonable intervention to support IPPV liberation. In general, Patients who are receiving MV are at risk of impaired secretion
concentrations greater than 3 mg/dL, using either intrave- removal and subsequent infection risk because of impaired
nous or oral replacement strategies, seem to show the most mucociliary removal from the endotracheal tube, atelectasis,
beneficial effects (Gravelyn 1988, Kraft 2005, Zhao 2016, impaired cough, and medications, such as ketamine-induced
Lemon 2017). hypersalivation (Branson 2007). Secretion management is
thus a routine part of care of patients undergoing IPPV. The
Acetazolamide primary treatments are adequate humidification and suc-
Acetazolamide has been proposed as an intervention to tioning. Mucolytic agents (e.g., saline, acetylcysteine) and
correct metabolic alkalosis, which has been associated anticholinergic agents (e.g., scopolamine, glycopyrrolate)
with hypoventilation and failure to wean. Through increas- have been used (Branson 2007, Kallet 2013). Anticholinergic
ing the urinary excretion of bicarbonate, acetazolamide agents are associated with delirium, and although they do
can help reverse metabolic alkalosis, but its benefit on clin- reduce volume of secretions, they can also lead to hyper-
ically meaningful outcomes, such as duration of MV, has viscosity, resulting in secretions that are harder to clear. As
yet to be definitively shown (Gallagher 1979, Mazur 1999, such, anticholinergic agents to reduce oropharyngeal secre-
Faisy 2010, 2016, Mishra 2016). Acetazolamide is a nonspe- tions (e.g., drooling) may be quite useful, but lower airway
cific carbonic anhydrase inhibitor having activity in both secretions may become harder to manage when thickened
renal and pulmonary sites, and thus its use may result in a through the use of these agents. Although various forms of
variety of physiologic alterations, especially in critically ill saline have been used to “loosen” thick secretions, mucus
patients (Heming 2012). One recent review recommends that generally does not easily absorb topically applied water. The

Patient Care Scenario


A 58-year-old man (weight 80 kg) with a history of asthma, minute, 200 mL V T ). Chest radiography shows diffuse pul-
stage 1 chronic kidney disease, and hypertension was monary edema. His results for the confusion assessment
intubated 4 days ago for acute respiratory failure second- method for the ICU are negative. A cuff leak is detected.
ary to pneumonia. On evaluation today, his weight today His blood gas values after a 30-minute SBT are pH 7.22,
is 89 kg. He undergoes discontinuation of fentanyl (150– partial pressure of CO2 50 mm Hg, PaO2 61 mm Hg, and
250 mcg/hour) and midazolam (2–4 mg/hour) for an SBT bicarbonate 20 mEq/L. Interpret his performance on his
with the following settings: RSBI 25 breaths/minute/L and SBT and the possible pharmacotherapeutic interventions
minute ventilation of 1 L/minute (average RR 5 breaths/ as next steps in his care.

ANSWER
His RSBI of 25 breaths/minute/L is below the cutoff of demonstrate greater than 10% weight gain since admis-
105 breaths/minute/L, which is associated with extuba- sion and he has pulmonary edema, which may indicate
tion success. However, his RR and V T are low and not likely the presence of fluid overload and the need for diuresis
associated with the ability to be extubated. The low RR or other fluid removal strategies. Perhaps most contrib-
and V T potentially explain his poor blood gas values, indi- utory was his high opioid and benzodiazepine doses over
cated by his acidosis, elevated CO2, and low oxygen. The the previous 4 days, which may be a causative factor for
presence of a cuff leak indicates that he likely will not have his low respiratory drive. He may need a sedation hol-
issues with post-extubation stridor. His negative results iday to allow these medications to wash out before his
on assessment for confusion in the ICU indicate that delir- extubation.
ium was not contributory to his SBT performance. He does

1. Fan E, Zakhary B, Amaral A, et al. Liberation from mechanical ventilation in critically ill adults. an official ATS/ACCP Clinical Practice
Guideline. Ann Am Thorac Soc 2017;14:441-3.
2. Stollings JL, Foss JJ, Ely EW, et al. Pharmacist leadership in ICU quality improvement: coordinating spontaneous awakening and
breathing trials Ann Pharmacother 2015;49:883-91.
3. Yang KL, Tobin MJ. A prospective study of indexes predicting the outcome of trials of weaning from mechanical ventilation N Engl J Med
1991;324:1445-50.

CCSAP 2021 Book 1 • Pulmonary and Endocrinology 23 Mechanical Ventilation and Pulmonary Procedures
mechanism of benefit is likely more because of the irritant
nature, causing bronchoconstriction rather than true mucol- Table 1. Light Criteria
ysis (Jelic 2008). Acetylcysteine breaks disulfide bonds in
Criteria Transudate Exudate
mucus, thereby reducing its viscosity; however, data are
extremely limited for its role in mucus clearance with several Pleural fluid <0.5 >0.6
key adverse effects (Jelic 2008). Acetylcysteine antagonizes protein/serum
both aminoglycosides and ß-lactam antibiotics in vitro and protein ratio
can cause false negative cultures for Pseudomonas aerugi- Pleural fluid <0.6 >0.5
nosa (Jelie 2008). Acetylcysteine is an irritant that can cause LDH/serum
bronchoconstriction and must generally be administered LDH ratio
with a ß2-agonist. Given little outcomes data and notable lim- Pleural fluid <2/3 ULN for serum >2/3 ULN for
itations, acetylcysteine should likely be avoided for routine LDH serum
use. Although more often used for secretion management in
Etiology • Congestive heart • Pneumonia
end-of-life care, scopolamine, atropine eye drops, and glyco-
failure • Cancer
pyrrolate may also reduce excess secretions through their • Cirrhosis • Trauma
anticholinergic properties (Clary 2009). In sum, these agents • Pulmonary • Systemic lupus
are symptomatic treatment only with limited outcomes data embolism erythematosus
and should be used sparingly in a patient-specific manner. • Nephrotic syndrome • Fluid overload

OTHER ICU PULMONARY LDH = lactate dehydrogenase; ULN = upper limit of normal.
PROCEDURES Light RW. Clinical practice. Pleural effusion. N Engl J Med
2002;346:1971-7.
Thoracentesis
Thoracentesis is a procedure that involves the insertion of a
needle into the pleural space to remove fluid either to diag- pneumothorax, from the needle puncturing the lung, is one of
nose the cause of a pleural effusion or to therapeutically the biggest safety concerns (Thomsen 2006, Pathak 2017).
drain larger effusions with potential to cause respiratory
compromise (Figure 11) (Thomsen 2006). The Light criteria Bronchoscopy
can be used to delineate whether the fluid is transudative or Bronchoscopy is a procedure that involves the insertion of a
exudative, which may help guide therapy, such as using anti- thin tube with a light and camera for visualization of the tra-
biotics for pneumonia (Light 2002). Criteria and etiology are cheobronchial tree (Figure 12). This procedure may be used
summarized in Table 1. As a minimally invasive procedure, both diagnostically and therapeutically (Table 2) (Esquinas
thoracentesis is generally considered safe, even in settings of 2013, La Combe 2016). Bronchoscopy can be associated with
coagulopathy, and the decision to administer platelets or anti- decreased respiratory and cardiac function intraprocedur-
coagulant reversal agents is highly patient specific. Tension ally (especially related to medications used for procedural

Lung Bronchoscope

Rib

Parietal pleura

Pleural space

Pleural
effusion Thoracentesis needle

Figure 12. Schematic of a bronchoscopy.


Figure 11. Schematic of a thoracentesis. Information from: Esquinas A, Zuil M, Scala R, et al.
Information from: Thomsen TW, DeLaPena J, Setnik GS. Bronchoscopy during noninvasive mechanical ventilation:
Videos in clinical medicine Thoracentesis. N Engl J Med a review of techniques and procedures. Arch Bronconeumol
2006;355:e16. 2013;49:105-12.

CCSAP 2021 Book 1 • Pulmonary and Endocrinology 24 Mechanical Ventilation and Pulmonary Procedures
Table 2. Indications for Bronchoscopy

Indication Visualization Biopsy Therapy


Lung
Aspiration ✓ ✓ ✓

Infection ✓ ✓

Atelectasis ✓ ✓ Intrapleural space


Airway ✓ ✓
Chest tube
management

Airway assessment, ✓ ✓
such as for burns Drainage container
Foreign body ✓ ✓

Hemorrhage ✓ ✓

sedation), and risks and benefits in hemodynamically unstable Figure 13. Schematic of a chest tube.
patients must be carefully weighed (Ergan 2018). For elective Information from: Miller KS, Sahn SA. Chest tubes. Indications,
bronchoscopy, holding warfarin/full anticoagulation and nor- technique, management and complications. Chest 1987;
91:258-64.
malized laboratory coagulation values are recommended; in
the case of emergency bronchoscopy, rapid reversal may be
warranted based on patient-specific risk factors (Douketis
2012, Pathak 2017, Youness 2017). Topical lidocaine or benzo- recurrent spontaneous pneumothorax and pleural effu-
caine have been used to alleviate patient discomfort; however, sions, often in the setting of malignancy (Mierzejewski 2019)
the risk of methemoglobinemia is present, and some studies (Figure 14). Pleurodesis may be achieved by direct injury to
have shown no difference as an addition to appropriate pro- the pleura using mechanical or chemical methods through
cedural sedation (Stolz 2005, Kwok 2008). administration of sclerosing agents, such as talc, bleomycin,
and tetracycline antibiotics (Mierzejewski 2019). Application
Chest Tubes of a sclerosing agent sets off an inflammatory process lead-
Insertion of chest tubes or thoracostomy tubes are a com- ing to activation of pleural cells, the coagulation cascade,
mon procedure with applications ranging from emergency fibrin chain formation, and fibrinogenesis, all leading to the
use to evacuate hemothorax to routine use for postoperative
chest drainage after elective surgery (Figure 13). Indications
include pneumothorax, penetrating chest trauma, blunt chest
trauma, chylothorax, pleural effusion, post-operatively, and
empyema. In general, a tube is placed into the intrapleural
space to facilitate breathing by draining fluid (Miller 1987,
Lung
Kwiatt 2014). Both digital and three-chamber (using wet or
dry suction) systems may be used to facilitate the drainage
process. The large bore nature of surgically placed chest
tubes makes them quite painful, thus they are listed in the Intrapleural space
guidelines for PADIS procedures–those that may cause pain,
agitation, delirium, immobility, or sleep disruption. Pre- and
periprocedural pain assessment and pre-emptive analgesia Pleurodesis
with the use of opioids is recommended. Strategies often
include both topical (e.g., lidocaine patches) and systemic Parietal pleura
analgesia (Miller 1987, Kwiatt 2014, Porcel 2018). Furthermore,
evaluation of fluid output is recommended to assess the
Figure 14. Pleurodesis schematic.
appropriateness of anticoagulation therapy.
Information from: Mierzejewski M, Korczynski P, Krenke R,
et al. Chemical pleurodesis—a review of mechanisms
Pleurodesis
involved in pleural space obliteration. Respir Res 2019;
Pleurodesis is a procedure conducted to create the symphy- 20:247.
sis of the parietal and visceral pleura and is used to prevent

CCSAP 2021 Book 1 • Pulmonary and Endocrinology 25 Mechanical Ventilation and Pulmonary Procedures
bonding of the pleura (Mierzejewski 2019). Talc has been
Practice Points
repeatedly shown to be the most efficacious agent in terms of
successful pleurodesis; however, universal acceptance has Clinical pharmacist face many challenges in their efforts
to optimize pharmacotherapy for patients receiving MV.
been slower because of its temporal association with ARDS.
As a result, guidelines/recommendations, best prac-
Therefore, despite lower efficacy rates, tetracyclines are com- tices, new indications for existing medications, and new
monly used (Webb 1992, Hartman 1993, Walker-Renard 1994, roles for pharmacists continue to evolve.
Viallat 1996). Pleurodesis is often painful, and procedural opi- • A working understanding of the principles of MV supports
oid/anxiolytic agents are warranted. In addition, intrapleural the development of timely and effective pharmacothera-
administration of lidocaine is recommended (Antunes 2003). peutic interventions. Furthermore, pharmacists play a key
role in quality improvement and protocol implementation
for critically ill patients on MV.
Tracheostomy
• When evaluating a patient’s respiratory support, the type of
A tracheostomy tube is placed surgically into the trachea MV (invasive vs. noninvasive), mode (e.g., PS, VC), and key
and can be either open to air or attached to a ventilator settings (e.g., FiO2, PEEP, RR) provide a clinical picture of
(Figure 15). A tracheostomy tube can be temporary or per- the patient’s status.
manent and facilitates airway protection and weaning from • The use of MV is associated with a host of consequences
that contribute both to prolonged duration of intubation
the ventilator. Tracheostomy can provide increased quality of
and prolonged ICU length of stay as well as other deleteri-
life because it provides the patient with the ability to eat and ous outcomes. Thus, reducing the duration of MV reduces
speak. A Passy Muir valve may ultimately be placed, which the associated complications and may improve patient
allows for the patient to speak (Engels 2009). Tracheostomy outcomes.
may be performed at the bedside and is considered to pose • Patients often experience failure on an SBT because of
a low bleeding risk. Depending on patient-specific factors, medication-related issues, including but not limited to
oversedation, agitation, fluid overload, or restrictive and
antiplatelet therapy and anticoagulation therapy may be con-
obstructed airways.
tinued. The decision to hold therapy is based on specific • Both MV and hemodynamic status are often intertwined,
assessment of bleeding and clotting risk and the ability to plan and careful hemodynamic assessment in patients undergo-
patient care around the procedure (Pathak 2017). Although ing changes in respiratory support is warranted.
endotracheal intubation avoids the surgical complications
associated with tracheostomy, such as bleeding, nerve/tra-
cheal wall injury, and wound infection, tracheostomy does tracheostomy becomes preferable. Although minor postoper-
offer specific benefits, including improved patient comfort, ative pain is expected and may be managed symptomatically,
easier oral care, reduced sedation/analgesia requirements, the pharmacist may expect reduced analgesia and sedation
reduced unplanned extubation, and faster rehabilitation and requirements in a clinical setting (Terragni 2014, Bice 2015).
ventilator weaning. Timing and patient selection remain areas
of ongoing discussion; however, for cases in which a patient REFERENCES
is predicted to have prolonged ventilator requirements, Acute Respiratory Distress Syndrome Clinical Trials
Network. Randomized, placebo-controlled clinical trial
of an aerosolized β₂-agonist for treatment of acute lung
injury. Am J Respir Crit Care Med 2011;184:561-8.
Adamson R, Swenson ER. Acetazolamide use in severe
chronic obstructive pulmonary disease. Pros and cons.
Esophagus Ann Am Thorac Soc 2017;14:1086-93.
Agusti AG, Torres A, Estopa R, et al. Hypophosphatemia as a
Trachea cause of failed weaning: the importance of metabolic fac-
tors. Crit Care Med 1984;12:142-3.
Tracheostomy tube Akoumianaki E, Lyazidi A, Rey N, et al. Mechanical venti-
lation-induced reverse-triggered breaths: a frequently
Inflatable cuff unrecognized form of neuromechanical coupling. Chest
2013;143:927-38.
Alhazzani W, Alshamsi F, Belley-Cote E, et al. Efficacy and
safety of stress ulcer prophylaxis in critically ill patients:
Figure 15. Tracheostomy schematic. a network meta-analysis of randomized trials. Intensive
Information from: Pathak V, Allender JE, Grant MW. Care Med 2018;44:1-11.
Management of anticoagulant and antiplatelet therapy in
Alhurani RE, Oeckler RA, Franco PM, et al. Refractory hypox-
patients undergoing interventional pulmonary procedures
emia and use of rescue strategies. A U.S. national survey
Eur Respir Rev 2017;26:170020.
of adult intensivists. Ann Am Thorac Soc 2016;13:1105-14.

CCSAP 2021 Book 1 • Pulmonary and Endocrinology 26 Mechanical Ventilation and Pulmonary Procedures
Alviar CL, Miller PE, McAreavey D, et al. Positive pressure Bouadma L, Deslandes E, Lolom I, et al. Long-term impact of
ventilation in the cardiac intensive care unit. J Am Coll a multifaceted prevention program on ventilator-associ-
Cardiol 2018;72:1532-53. ated pneumonia in a medical intensive care unit.
Clin Infect Dis 2010;51:1115-22.
American Thoracic Society/European Respiratory Society.
ATS/ERS statement on respiratory muscle testing. Bouadma L, Klompas M. Oral care with chlorhexidine:
Am J Respir Crit Care Med 2002;166:518-624. beware! Intensive Care Med 2018;44:1153-5.

Andresen M, Castillo L, Dougnac A, et al. Patients with Bourenne J, Guervilly C, Mechati M, et al. Variability of
acute adult respiratory distress syndrome: effects of reverse triggering in deeply sedated ARDS patients.
inhaled nitric oxide on gas exchange and hemodynamics. Intensive Care Med 2019;45:725-6.
Rev Med Chil 1996;124:813-9.
Branson RD. Secretion management in the mechani-
Antunes G, Neville E, Duffy J, Ali N, et al. BTS guidelines for cally ventilated patient. Respir Care 2007;52:1328-1342;
the management of malignant pleural effusions. Thorax discussion 1342-7.
2003;58 Suppl 2:ii29-38.
Brochard L, Mancebo J, Wysocki M, et al. Noninvasive ven-
Ari A, Fink JB, Dhand R. Inhalation therapy in patients tilation for acute exacerbations of chronic obstructive
receiving mechanical ventilation: an update. J Aerosol pulmonary disease. N Engl J Med 1995;333:817-22.
Med Pulm Drug Deliv 2012;25:319-32.
Brower RG, Matthay MA, Morris A, et al. Ventilation with
ASHP Commission. ASHP Therapeutic Guidelines on Stress lower tidal volumes as compared with traditional tidal
Ulcer Prophylaxis. ASHP Commission on Therapeutics and volumes for acute lung injury and the acute respiratory
approved by the ASHP Board of Directors on November 14, distress syndrome. N Engl J Med 2000;342:1301-8.
1998. Am J Health Syst Pharm 1999;56:347-79.
Brunelli SM, Goldfarb S. Hypophosphatemia: clinical
Asmus MJ, Hendeles L. Levalbuterol nebulizer solution: is it consequences and management. J Am Soc Nephrol
worth five times the cost of albuterol? Pharmacotherapy 2007;18:1999-2003.
2000;20:123-9.
Caputo ND, Strayer RJ, Levitan R. Early self-proning in
Barbateskovic M, Marker S, Granholm A, et al. Stress ulcer awake, non-intubated patients in the emergency depart-
prophylaxis with proton pump inhibitors or histamin-2 ment: a single ED’s experience during the COVID-19
receptor antagonists in adult intensive care patients: a pandemic. Acad Emerg Med 2020;27:375-8.
systematic review with meta-analysis and trial sequential
Caroff DA, Li L, Muscedere J, et al. Subglottic secretion
analysis. Intensive Care Med 2019;45:143-158.
drainage and objective outcomes: a systematic review and
Barletta JF, Bruno JJ, Buckley MS, et al. Stress ulcer prophy- meta-analysis. Crit Care Med 2016;44:830-40.
laxis. Crit Care Med 2016;44:1395-405.
Cawley MJ. Advanced modes of mechanical ventilation:
Barletta JF, Sclar DA. Use of proton pump inhibitors for the introduction for the critical care pharmacist. J Pharm
provision of stress ulcer prophylaxis: clinical and eco- Pract 2019;32:186-98.
nomic consequences. Pharmacoeconomics 2014;32:5-13.
Cawley MJ. Mechanical ventilation: a tutorial for pharma-
Barr J, Fraser GL, Puntillo K, et al. Clinical practice guide- cists. Pharmacotherapy 2007;27:250-66.
lines for the management of pain, agitation, and delirium
Cawley MJ. Mechanical ventilation: introduction for the phar-
in adult patients in the intensive care unit. Crit Care Med
macy practitioner. J Pharm Pract 2011;24:7-16.
2013;41:263-306.
Chang LH, Honiden S, Haithcock JA, et al. Utilization of bron-
Bassford CR, Thickett DR, Perkins GD. The rise and fall
chodilators in ventilated patients without obstructive
of beta-agonists in the treatment of ARDS. Crit Care
airways disease. Respir Care 2007;52:154-8.
2012;16:208.
Chant C, Dewhurst NF, Friedrich JO. Do we need a pharma-
Baumann MH, McAlpin BW, Brown K. A prospective random-
cist in the ICU? Intensive Care Med 2015;41:1314-20.
ized comparison of train-of-four monitoring and clinical
assessment during continuous ICU cisatracurium paraly- Clary PL, Lawson P. Pharmacologic pearls for end-of-life
sis. Chest 2004 126(4):1267-73. care. Am Fam Physician 2009;79:1059-65.
Beitler JR, Malhotra A, Thompson BT. Ventilator-induced Cook DJ, Fuller HD, Guyatt GH, et al., Canadian Critical Care
lung injury. Clin Chest Med 2016;37:633-46. Trials Group. Risk factors for gastrointestinal bleeding in
critically ill patients. N Engl J Med 1994;330:377-81.
Bice T, Nelson JE, Carson SS. To trach or not to trach: uncer-
tainty in the care of the chronically critically ill. Semin Daoud EG. Airway pressure release ventilation. Ann Thorac
Respir Crit Care Med 2015;36:851-58. Med 2007;2:176-9.
Bissell BD, Laine ME, Thompson Bastin ML, et al. Impact of Darmon JY, Rauss A, Dreyfuss D, et al. Evaluation of risk fac-
protocolized diuresis for de-resuscitation in the intensive tors for laryngeal edema after tracheal extubation in adults
care unit. Crit Care 2020;24:70. and its prevention by dexamethasone. A placebo-controlled,

CCSAP 2021 Book 1 • Pulmonary and Endocrinology 27 Mechanical Ventilation and Pulmonary Procedures
double-blind, multicenter study. Anesthesiology Fan E, Zakhary B, Amaral A, et al. Liberation from mechan-
1992;77:245-51. ical ventilation in critically ill adults an official ATS/
ACCP clinical practice guideline. Ann Am Thorac Soc
de Haro C, Ochagavia A, Lopez-Aguilar J, et al. Patient- 2017b;14:441-3.
ventilator asynchronies during mechanical ventilation:
current knowledge and research priorities. Intensive Care Forel JM, Roch A, Marin V, et al. Neuromuscular blocking
Med Exp 2019;7(suppl 1):43. agents decrease inflammatory response in patients pre-
senting with acute respiratory distress syndrome.
De Jong A, Chanques G, Jaber S. Mechanical ventilation in Crit Care Med 2006;34:2749-57.
obese ICU patients: from intubation to extubation.
Crit Care 2017;21:63. Francois B, Bellissant E, Gissot V, et al. 12-h pretreatment
with methylprednisolone versus placebo for prevention
Devlin JW, Skrobik Y, Gelinas C, et al. Clinical practice of postextubation laryngeal oedema: a randomised dou-
guidelines for the prevention and management of pain, ble-blind trial. Lancet 2007;369:1083-9.
agitation/sedation, delirium, immobility, and sleep
disruption in adult patients in the ICU. Crit Care Med Frazier SK. Neurohormonal responses during positive pres-
2018;46:e825-73. sure mechanical ventilation. Heart Lung 1999;28:149-65.

Douketis JD, Spyropoulos AC, Spencer FA, et al. Perioperative Frerk C, Mitchell VS, McNarry AF, et al. Difficult airway soci-
management of antithrombotic therapy: antithrombotic ety 2015 guidelines for management of unanticipated
therapy and prevention of thrombosis, 9th ed: American difficult intubation in adults. Br J Anaesth 2015;115:827-48.
College of Chest Physicians Evidence-Based Clinical Gainnier M, Roch A, Forel JM, et al. Effect of neuromuscular
Practice Guidelines. Chest 2012;141:e326S-50S. blocking agents on gas exchange in patients presenting
Duan EH, Adhikari NKJ, D’Aragon F, et al. Management of with acute respiratory distress syndrome. Crit Care Med
acute respiratory distress syndrome and refractory hypox- 2004;32:113-9.
emia: a multicenter observational study. Ann Am Thorac Gallagher TJ. Metabolic alkalosis complicating weaning
Soc 2017;14:1818-26. from mechanical ventilation. South Med J 1979;72:786-7.
Elharrar X, Trigui Y, Dols AM, et al. Use of prone positioning Garpestad E, Brennan J, Hill NS. Noninvasive ventilation for
in nonintubated patients with COVID-19 and hypoxemic critical care. Chest 2007;132:711-20.
acute respiratory failure. JAMA 2020;323:2336-8.
Girard TD, Alhazzani W, Kress JP, et al. An official American
Engels PT, Bagshaw SM, Meier M, et al. Tracheostomy: from Thoracic Society/American College of Chest Physicians
insertion to decannulation. Can J Surg 2009;52:427-33. clinical practice guideline: liberation from mechanical
ventilation in critically ill adults rehabilitation protocols,
Epstein SK, Ciubotaru RL. Independent effects of etiol-
ventilator liberation protocols, and cuff leak tests.
ogy of failure and time to reintubation on outcome for
Am J Respir Crit Care Med 2017;195:120-33.
patients failing extubation. Am J Respir Crit Care Med
1998;158:489-93. Girard TD, Kress JP, Fuchs BD, et al. Efficacy and safety
of a paired sedation and ventilator weaning protocol
Ergan B, Nava S. The use of bronchoscopy in critically ill
for mechanically ventilated patients in intensive care
patients: considerations and complications. Expert Rev
(Awakening and Breathing Controlled trial): a randomised
Respir Med 2018;12:651-63.
controlled trial. Lancet 2008;371:126-34.
Esquinas A, Zuil M, Scala R, et al. Bronchoscopy during Gravelyn TR, Brophy N, Siegert C, et al. Hypophosphatemia-
non-invasive mechanical ventilation: a review of techniques associated respiratory muscle weakness in a general
and procedures Arch Bronconeumol 2013;49:105-12. inpatient population. Am J Med 1988;84:870-6.
Faisy C, Meziani F, Planquette B, et al. Effect of acetazol- Greenblatt EE, Butler JP, Venegas JG, et al. Pendelluft in the
amide vs placebo on duration of invasive mechanical bronchial tree. J Appl Physiol (1985) 2014;117:979-88.
ventilation among patients with chronic obstructive
pulmonary disease: a randomized clinical trial. JAMA Guerin C, Reignier J, Richard JC, et al. Prone positioning in
2016;315:480-8. severe acute respiratory distress syndrome. N Engl J Med
2013;368:2159-2168.
Faisy C, Mokline A, Sanchez O, et al. Effectiveness of acet-
azolamide for reversal of metabolic alkalosis in weaning Habashi NM. Other approaches to open-lung ventilation:
COPD patients from mechanical ventilation. Intensive Care airway pressure release ventilation. Crit Care Med
Med 2010;36:859-63. 2005;33:S228-40.

Fan E, Del Sorbo L, Goligher EC, et al. An official American Hahn L, Beall J, Turner RS, et al. Pharmacist-developed seda-
Thoracic Society/European Society of Intensive Care tion protocol and impact on ventilator days. J Pharm Pract
Medicine/Society of Critical Care Medicine Clinical prac- 2013;26:406-8.
tice guideline: mechanical ventilation in adult patients
Hammond D, Flowers HJC, Meena, N, et al. Cost avoidance
with acute respiratory distress syndrome. Am J Respir Crit
associated with clinical pharmacist presence in a medical
Care Med 2017a;195:1253-63.
intensive care unit. J Am Coll Clin Pharm 2019;2:610-5.

CCSAP 2021 Book 1 • Pulmonary and Endocrinology 28 Mechanical Ventilation and Pulmonary Procedures
Hartman DL, Gaither JM, Kesler KA, et al. Comparison of Krag M, Marker S, Perner A, et al. Pantoprazole in patients at
insufflated talc under thoracoscopic guidance with stan- risk for gastrointestinal bleeding in the ICU. N Engl J Med
dard tetracycline and bleomycin pleurodesis for control 2018;379:2199-208.
of malignant pleural effusions. J Thorac Cardiovasc Surg
1993;105:743-7. Kress JP, Pohlman AS, O’Connor MF, et al. Daily interruption
of sedative infusions in critically ill patients undergoing
Hawkins WA, Smith SE, Sikora Newsome A, et al. Fluid mechanical ventilation. N Engl J Med 2000;342:1471-7.
Stewardship During Critical Illness: A Call to Action.
J Pharm Pract 2020; 33(6): 863-72. Krinsley JS, Reddy PK, Iqbal A. What is the optimal rate of
failed extubation? Crit Care 2012;16:111.
Heming N, Urien S, Faisy C. Acetazolamide: a second wind
for a respiratory stimulant in the intensive care unit? Kuiper JW, Groeneveld AB, Slutsky AS, et al. Mechanical
Crit Care 2012;16:318. ventilation and acute renal failure. Crit Care Med 2005;
33:1408-15.
Hermans G, Van den Berghe G. Clinical review: intensive care
Kwiatt M, Tarbox A, Seamon MJ, et al. Thoracostomy tubes:
unit acquired weakness. Crit Care 2015;19:274.
a comprehensive review of complications and related top-
Hua F, Xie H, Worthington HV, et al. Oral hygiene care for ics. Int J Crit Illn Inj Sci 2014;4:143-55.
critically ill patients to prevent ventilator-associated pneu-
Kwok S, Fischer JL, Rogers JD. Benzocaine and lidocaine
monia. Cochrane Database Syst Rev 2016;10:CD008367.
induced methemoglobinemia after bronchoscopy: a case
ICU-ROX Investigators. Conservative oxygen therapy during report. J Med Case Rep 2008;2:16.
mechanical ventilation in the ICU. N Engl J Med 2020
La Combe B, Messika J, Labbe V, et al. High-flow nasal
12;382(11):989-998.
oxygen for bronchoalveolar lavage in acute respiratory
Jaber S, Chanques G, Matecki S, et al. Post-extubation stri- failure patients. Eur Respir J 2016;47:1283-6.
dor in intensive care unit patients. Risk factors evaluation
Lam S, Chen J. Changes in heart rate associated with nebu-
and importance of the cuff-leak test. Intensive Care Med
lized racemic albuterol and levalbuterol in intensive care
2003;29:69-74.
patients. Am J Health Syst Pharm 2003;60:1971-5.
Jakob SM. The effects of mechanical ventilation on hepato-
Lat I, Pacuillo C, Daley MJ et al. Position paper on critical
splanchnic perfusion Curr Opin Crit Care 2010;16:165-8.
care pharmacy services: 2020 update. Crit Care Med 2020
Jelic S, Cunningham JA, Factor P. Clinical review: airway 49(9):e813-34.
hygiene in the intensive care unit Crit Care 2008;12:209.
Lee CH, Peng MJ, Wu CL. Dexamethasone to prevent postex-
Jones TW, Smith SE, Van Tuyl JS, et al. Sepsis with preex- tubation airway obstruction in adults: a prospective,
isting heart failure: management of confounding clinical randomized, double-blind, placebo-controlled study.
features. J Intensive Care Med 2020:885066620928299. Crit Care 2007;11:R72.

Kalil AC, Metersky ML, Klompas M, et al. Management of Lee H, Ryu K, Sohn Y, et al. Impact on patient outcomes of
adults with hospital-acquired and ventilator-associ- pharmacist participation in multidisciplinary critical care
ated pneumonia: 2016 clinical practice guidelines by the teams: a systematic review and meta-analysis. Crit Care
Infectious Diseases Society of America and the American Med 2019;47:1243-50.
Thoracic Society Clin Infect Dis 2016;63:e61-111. Leguelinel-Blache G, Nguyen TL, Louart B, et al. Impact
of quality bundle enforcement by a critical care phar-
Kallet RH. Adjunct therapies during mechanical ventilation:
macist on patient outcome and costs. Crit Care Med
airway clearance techniques, therapeutic aerosols, and
2018;46:199-207.
gases. Respir Care 2013;58:1053-73.
Lemon SJ, Zack SD, Voils SA. No difference in mechanical
Kampf G. Acquired resistance to chlorhexidine—is it time
ventilation-free hours in critically ill patients who received
to establish an ‘antiseptic stewardship’ initiative? J Hosp
intravenous, oral, or enteral phosphate replacement. J Crit
Infect 2016;94:213-27.
Care 2017;39:31-5.
Khan R, Al-Dorzi HM, Al-Attas K, et al. The impact of imple-
Light RW. Clinical practice. Pleural effusion. N Engl J Med
menting multifaceted interventions on the prevention of
2002;346:1971-7.
ventilator-associated pneumonia. Am J Infect Control
2016;44:320-6. Lin CC, Chen KF, Shih CP, et al. The prognostic factors of
hypotension after rapid sequence intubation. Am J Emerg
Khorfan FM, Smith P, Watt S, et al. Effects of nebulized
Med 2008;26:845-51.
bronchodilator therapy on heart rate and arrhythmias in
critically ill adult patients. Chest 2011;140:1466-72. Lin PH, Chen CF, Chiu HW, et al. Outcomes of unplanned
extubation in ordinary ward are similar to those in inten-
Kraft MD, Btaiche IF, Sacks GS, et al. Treatment of electrolyte
sive care unit: a STROBE-compliant case-control study.
disorders in adult patients in the intensive care unit.
Medicine (Baltimore) 2019;98:e14841.
Am J Health Syst Pharm 2005;62:1663-82.

CCSAP 2021 Book 1 • Pulmonary and Endocrinology 29 Mechanical Ventilation and Pulmonary Procedures
Louzon P, Jennings H, Ali M, et al. Impact of pharmacist Morina P, Herrera M, Venegas J, et al. Effects of nebulized
management of pain, agitation, and delirium in the inten- salbutamol on respiratory mechanics in adult respiratory
sive care unit through participation in multidisciplinary distress syndrome. Intensive Care Med 1997;23:58-64.
bundle rounds. Am J Health Syst Pharm 2017;74:253-62.
Munro CL, Grap MJ, Jones DJ, et al. Chlorhexidine, tooth-
Manocha S, Gordon AC, Salehifar E, et al. Inhaled beta-2 ago- brushing, and preventing ventilator-associated pneumonia
nist salbutamol and acute lung injury: an association with in critically ill adults. Am J Crit Care 2009;18:428-37.
improvement in acute lung injury. Crit Care 2006;10:R12.
Murray MJ, DeBlock H, Erstad B, et al. Clinical practice guide-
Manthous CA, Hall JB, Kushner R, et al. The effect of lines for sustained neuromuscular blockade in the adult
mechanical ventilation on oxygen consumption in critically critically ill patient. Crit Care Med 2016;44:2079-103.
ill patients. Am J Respir Crit Care Med 1995;151:210-4.
National Heart, Lung, and Blood Institute PETAL Clinical
Marik PE, Cavallazzi R. Does the central venous pressure Trials Network, Moss M, Huang DT, et al. Early neuromus-
predict fluid responsiveness? An updated meta-analy- cular blockade in the acute respiratory distress syndrome.
sis and a plea for some common sense. Crit Care Med N Engl J Med 2019;380:1997-2008.
2013;41:1774-81.
Nava S, Navalesi P, Carlucci A. Non-invasive ventilation.
Marini JJ, Slutsky AS. Physiological basis of ventilatory Minerva Anestesiol 2009;75:31-6.
support New York: Marcel Dekker, 1998.
Newsome A, Smith SE, Jones TW, et al. A survey of critical
Marker S, Perner A, Wetterslev J, et al. Pantoprazole pro- care pharmacists to patient ratios and practice charac-
phylaxis in ICU patients with high severity of disease: a teristics in intensive care units. J Am Coll Clin Pharm
post hoc analysis of the placebo-controlled SUP-ICU trial. 2020;3:68-74.
Intensive Care Med 2019;45:609-18.
Newsome AS, Chastain, DB, Watkins P, et al. Complications
Marshall J, Finn CA, Theodore AC. Impact of a clinical phar- and pharmacologic interventions of invasive positive pres-
macist-enforced intensive care unit sedation protocol on sure ventilation during critical illness. J Pharm Technol
duration of mechanical ventilation and hospital stay. 2018;34:153-70.
Crit Care Med 2008;36:427-33.
O’Croinin D, Ni Chonghaile M, Higgins B, et al. Bench-to-
Maung AA, Kaplan LJ. Airway pressure release ventilation bedside review: permissive hypercapnia. Crit Care
in acute respiratory distress syndrome. Crit Care Clin 2005;9:51-9.
2011;27:501-9.
Otis AB, Fenn WO, Rahn H. Mechanics of breathing in man.
Mazur JE, Devlin JW, Peters MJ, et al. Single versus multiple
J Appl Physiol 1950;2:592-607.
doses of acetazolamide for metabolic alkalosis in criti-
cally ill medical patients: a randomized, double-blind trial. Papazian L, Forel JM, Gacouin A. Neuromuscular blockers
Crit Care Med 1999;27:1257-61. in early acute respiratory distress syndrome N Engl J Med
2010 16;363(12):1107-16.
Michard F, Boussat S, Chemla D, et al. Relation between
respiratory changes in arterial pulse pressure and fluid Park S, Schmidt M. Early neuromuscular blockade in mod-
responsiveness in septic patients with acute circulatory erate to severe acute respiratory distress syndrome: do
failure. Am J Respir Crit Care Med 2000;162:134-8. not throw the baby out with the bathwater! J Thorac Dis
2019;11:E231-4.
Mierzejewski M, Korczynski P, Krenke R, et al. Chemical pleu-
rodesis—a review of mechanisms involved in pleural space Pathak V, Allender JE, Grant MW. Management of antico-
obliteration. Respir Res 2019;20:247. agulant and antiplatelet therapy in patients undergoing
Miller CJ, Doepker BA, Springer AN, et al. Impact of serum interventional pulmonary procedures. Eur Respir Rev
phosphate in mechanically ventilated patients with 2017;26:170020.
severe sepsis and septic shock. J Intensive Care Med Penuelas O, Frutos-Vivar F, Esteban A. Unplanned extubation
2020;35:485-93. in the ICU: a marker of quality assurance of mechanical
Miller KS, Sahn SA. Chest tubes. Indications, technique, ventilation. Crit Care 2011;15:128.
management and complications. Chest 1987;91:258-64. Perkins GD, McAuley DF, Thickett DR, et al. The beta-agonist
Miller RL, Cole RP. Association between reduced cuff lung injury trial (BALTI): a randomized placebo-controlled
leak volume and postextubation stridor. Chest clinical trial. Am J Respir Crit Care Med 2006;173:281-7.
1996;110:1035-40. Pirrone M, Fisher D, Chipman D, et al. Recruitment maneu-
Mishra S, Azim A, Baronia A. Acetazolamide and invasive vers and positive end-expiratory pressure titration in
mechanical ventilation for patients with COPD. JAMA morbidly obese ICU patients. Crit Care Med 2016;44:300-7.
2016;316:100.
Pluijms WA, van Mook WN, Wittekamp BH, et al.
Moore ES, Galvez MB, Paton JB, et al. Effects of positive Postextubation laryngeal edema and stridor resulting in
pressure ventilation on intrarenal blood flow in infant respiratory failure in critically ill adult patients: updated
primates. Pediatric research 1974;8:792-6. review. Crit Care 2015;19:295.

CCSAP 2021 Book 1 • Pulmonary and Endocrinology 30 Mechanical Ventilation and Pulmonary Procedures
Porcel JM. Chest tube drainage of the pleural space: a con- Stolz D, Chhajed PN, Leuppi J, et al. Nebulized lidocaine
cise review for pulmonologists. Tuberc Respir Dis (Seoul) for flexible bronchoscopy: a randomized, double-blind,
2018;81:106-15. placebo-controlled trial. Chest 2005;128:1756-60.

Prasanna D, Bhat S. Nasotracheal intubation: an overview. Sun Q, Qiu H, Huang M, et al. Lower mortality of COVID-19
J Maxillofac Oral Surg 2014;13:366-72. by early recognition and intervention: experience from
Jiangsu Province. Ann Intensive Care 2020;10:33.
Priebe HJ, Heimann JC, Hedley-Whyte J. Mechanisms of
renal dysfunction during positive end-expiratory pressure Sutherasan Y, Vargas M, Pelosi P. Protective mechanical ven-
ventilation. J Appl Physiol Respir Environ Exerc Physiol tilation in the non-injured lung: review and meta-analysis.
1981;50:643-9. Crit Care 2014;18:211.

Putensen C, Wrigge H. Clinical review: biphasic positive air- Terragni P, Faggiano C, Martin EL, et al. Tracheostomy in
way pressure and airway pressure release ventilation. mechanical ventilation. Semin Respir Crit Care Med
Crit Care 2004;8:492-7. 2014;35:482-91.

Resar R, Haraden C, Simmonds, et al. Using a bundle approach Thille AW, Richard JC, Brochard L. The decision to extubate
to improve ventilator care processes and reduce ventila- in the intensive care unit. Am J Respir Crit Care Med
tor-associated pneumonia. Jt Comm J Qual Patient Saf 2013;187:1294-302.
2005;31:243-8.
Thomsen TW, DeLaPena J, Setnik GS. Videos in clinical
Rhodes A, Evans LE, Alhazzani W, et al. Surviving sepsis medicine Thoracentesis. N Engl J Med 2006;355:e16.
campaign: international guidelines for management
of sepsis and septic shock: 2016. Intensive Care Med Tobin MJ. Principles and practice of mechanical ventilation,
2017;43:304-77. 3rd ed. New York: McGraw-Hill Medical, 2013.

Rudis MI, Brandl KM. Position paper on critical care phar- Tokmaji G, Vermeulen H, Muller MC, et al. Silver-coated
macy services. Society of Critical Care Medicine and endotracheal tubes for prevention of ventilator-associated
American College of Clinical Pharmacy Task Force pneumonia in critically ill patients. Cochrane Database
on Critical Care Pharmacy Services. Crit Care Med Syst Rev 2015:CD009201.
2000;28:3746-50. Uijtendaal EV, van Harssel LL, Hugenholtz GW, et al. Analysis
Sandhu RS, Pasquale MD, Miller K, et al. Measurement of potential drug-drug interactions in medical intensive
of endotracheal tube cuff leak to predict postextuba- care unit patients. Pharmacotherapy 2014;34:213-9.
tion stridor and need for reintubation. J Am Coll Surg Varpula T, Jousela I, Niemi R, et al. Combined effects of
2000;190:682-7. prone positioning and airway pressure release ventilation
Scaravilli V, Grasselli G, Castagna L, et al. Prone positioning on gas exchange in patients with acute lung injury.
improves oxygenation in spontaneously breathing nonin- Acta Anaesthesiol Scand 2003;47:516-24.
tubated patients with hypoxemic acute respiratory failure: Viallat JR, Rey F, Astoul P, et al. Thoracoscopic talc poudrage
a retrospective study. J Crit Care 2015;30:1390-4. pleurodesis for malignant effusions. A review of 360 cases.
Scott VL, Frazee LA. Retrospective comparison of nebulized Chest 1996;110:1387-93.
levalbuterol and albuterol for adverse events in patients Vincent JL. Give your patient a fast hug (at least) once a day.
with acute airflow obstruction. Am J Ther 2003;10:341-7. Crit Care Med 2005;33:1225-9.
Shi Z, Xie H, Wang P, et al. Oral hygiene care for critically Vincent WR, Hatton KW. Critically ill patients need “FAST
ill patients to prevent ventilator-associated pneumonia. HUGS BID” (an updated mnemonic). Crit Care Med
Cochrane Database Syst Rev 2013:CD008367. 2009;37:2326-7; author reply 2327.
Singh PM, Borle A, Trikha A. Newer nonconventional modes of Vitrat V, Hautefeuille S, Janssen C, et al. Optimizing antimi-
mechanical ventilation. J Emerg Trauma Shock 2014; crobial therapy in critically ill patients. Infect Drug Resist
7:222-7. 2014;7:261-71.
Slutsky AS, Ranieri VM. Ventilator-induced lung injury. Walker-Renard PB, Vaughan LM, Sahn SA. Chemical
N Engl J Med 2013;369:2126-36. pleurodesis for malignant pleural effusions. Ann Intern
Slutsky AS, Villar J. Early paralytic agents for ARDS? Yes, no, Med 1994;120:56-64.
and sometimes. N Engl J Med 2019;380:2061-3. Webb WR, Ozmen V, Moulder PV, et al. Iodized talc pleu-
Smith SE, Newsome AS, Hawkins WA. An argument for the rodesis for the treatment of pleural effusions. J Thorac
protocolized screening and management of post-extuba- Cardiovasc Surg 1992;103:881-5.
tion stridor. Am J Respir Crit Care Med 2018;197:1503-5. Weingart S. Push-dose pressors for immediate blood
Stollings JL, Foss JJ, Ely EW, et al. Pharmacist leadership pressure control. Clin Exp Emerg Med 2015;2:131-2.
in ICU quality improvement: coordinating spontaneous Wip C, Napolitano L. Bundles to prevent ventilator-associ-
awakening and breathing trials. Ann Pharmacother ated pneumonia: how valuable are they? Curr Opin Infect
2015;49:883-91. Dis 2009;22:159-66.

CCSAP 2021 Book 1 • Pulmonary and Endocrinology 31 Mechanical Ventilation and Pulmonary Procedures
Wittekamp BH, van Mook WN, Tjan DH, et al. Clinical review: Zhao Y, Li Z, Shi Y, et al. Effect of hypophosphatemia on
post-extubation laryngeal edema and extubation failure in the withdrawal of mechanical ventilation in patients with
critically ill adult patients. Crit Care 2009;13:233. acute exacerbations of chronic obstructive pulmonary
disease. Biomed Rep 2016;4:413-6.
Yang KL, Tobin MJ. A prospective study of indexes predict-
ing the outcome of trials of weaning from mechanical Ziehr DR, Alladina J, Petri CR, et al. respiratory pathophysiol-
ventilation. N Engl J Med 1991;324:1445-50. ogy of mechanically ventilated patients with COVID-19: a
cohort study. Am J Respir Crit Care Med 2020;201:1560-4.
Youness HA, Keddissi J, Berim I, et al. Management of oral
antiplatelet agents and anticoagulation therapy before Zuckerman LM. Oral Chlorhexidine use to prevent ventilator-
bronchoscopy. J Thorac Dis 2017;9:S1022-33. associated pneumonia in adults: review of the current lit-
erature. Dimens Crit Care Nurs 2016;35:25-36.

CCSAP 2021 Book 1 • Pulmonary and Endocrinology 32 Mechanical Ventilation and Pulmonary Procedures
Self-Assessment Questions
1. A 72-year-old man with community acquired pneumonia C. Sedation holiday to allow fentanyl/propofol to
(CAP) is intubated for hypoxemic respiratory failure and ‘wash-out’ from her system
increased work of breathing. In keeping with the three D. Albuterol for potential presence of restrictive airway
primary goals of mechanical ventilation, which one of disease
the following is best to recommend for this patient? 5. A 67-year-old man is intubated for acute respiratory dis-
A. Appropriate antibiotic therapy for CAP; mechanical tress syndrome. Despite high settings on PRVC (PEEP
ventilation to alleviate hypoxemia; minimizing 15 cmH2O, FiO2 100%), his oxygenation is critically low
duration of mechanical ventilation (PaO2/FiO2 ratio <100). Currently, the patient is sedated to
B. Inhaled bronchodilators for hypoxemia; achieving RASS -4 to -5 to facilitate oxygenation and is not initiating
appropriate peak pressures; reducing ventilator his own breaths. His care team is considering switching
induced lung injury (VILI) the patient from PRVC to APRV and asks about the best
C. Reversal of hypoxemia; resolution of tachypnea; course of action regarding sedation level when switching
avoiding patient discomfort between PRVC and APRV. Given how spontaneous breaths
D. Reducing work of breathing; bronchodilators for contribute to minute ventilation in APRV and PRVC, which
hypoxemia; minimizing patient-ventilator asynchrony one of the following is best to recommend for this patient?

Questions 2 and 3 pertain to the following case. A. Continue the current sedation level.
B. Change to RASS 0 to -2.
B.M. is a 64-year-old man with a medical history of hyperten-
C. Add cisatracurium.
sion, tobacco abuse, chronic obstructive pulmonary disease,
D. Decrease the respiratory rate.
and hypercholesterolemia. He is intubated for the following
arterial blood gas: pH 7.23 paCO2 58 paO2 68 HCO3 23. 6. A 65-year-old man is intubated for drug overdose and
aspiration pneumonia. On day 2, the team reports the
2. On the basis of his medical history, which one of the fol-
patient’s ABG reads as: pH 7.35 paCO2 45 paO2 106 HCO3
lowing is most likely to require B.M. to have mechanical
23. His current ventilator settings are PRVC with a set
ventilation for supportive care?
RR 14 breaths/minute, tidal volume 450 mL/breath (6
A. Hyperventilation
mL/kg), FiO2 40%, and PEEP 15 cm H2O. In an effort to
B. Ventilation-perfusion mismatch
manage worsening oxygenation and ARDS, the patient is
C. Increased fraction of inspired oxygen
switched from PRVC to APRV as a ventilator mode. After
D. Diffusion impairment
24 hours on APRV, his arterial blood gas values are: pH
3. On day 2 of intubation, B.M. begins to show signs of 7.29 paCO2 54 paO2 102 HCO3 22 with a PaO2/FiO2 128.
ventilator dyssynchrony (i.e., increased sedation require- The team discusses intervening on the paCO2. Which one
ments, high peak pressure alarms from the machine). On of the following is best to recommend for this patient?
the basis of his medical history, which one of the follow-
A. Initiate acetazolamide to correct acidosis.
ing assessments is the most likely to reveal the etiology
B. Sedate to RASS -3 to decrease spontaneous
of B.M.’s discomfort?
respirations.
A. Inspiratory hold C. Switch to PRVC for better paCO2 management.
B. Pressure support trial D. No intervention is necessary because permissive
C. Rapid shallow breathing index hypercapnia is acceptable.
D. T-piece trial
7. A 58-year-old man was intubated 48 hours ago for mixed
4. A 54-year-old woman with no contributory medical history hypoxic-hypercapnic respiratory failure. Today, his ABG
is intubated for airway protection. Today, she underwent reads as: pH 7.29 paCO2 47 paO2 48 HCO3 23. His cur-
discontinuation of fentanyl and propofol for an SBT with rent ventilator settings are pressure-regulated volume
the following RSBI 125 breaths/min/L. Chest radiography control with a RR 12 breaths/minute, tidal volume 400
is notable for diffuse pulmonary edema; her admission mL/breath (6mL/kg predicted body weight), FiO2 40%,
weight was 70 kg while today she weighs 82 kg. She was and PEEP 8 cm H2O. Which one of the following is best to
CAM-ICU negative. She has a cuff leak. Which one of the recommend for this patient?
following is best to recommend for this patient?
A. Increase both the FiO2 and PEEP.
A. The addition of dexmedetomidine to manage anxiety
B. Increase his minute ventilation.
and rapid respiratory rate
C. Increase his respiratory rate and tidal volume.
B. Fluid removal prior to extubation to manage the
D. Increase FiO2 but decrease the PEEP.
presence of fluid overload

CCSAP 2021 Book 1 • Pulmonary and Endocrinology 33 Mechanical Ventilation and Pulmonary Procedures
8. A 48-year-old woman was intubated 48 hours ago for 11. A woman with CHF (LVEF 25%) is intubated, and her
mixed hypoxic-hypercapnic respiratory failure. Today, PEEP is increased from 5 to 10 cm H2O to improve alveo-
her ABG reads as: pH 7.24 paCO2 68 paO2 98 HCO3 28. lar recruitment. The team notes decreased milrinone and
Her current ventilator settings are pressure-regulated norepinephrine requirements. Which one of the following
volume control with a RR 14 breaths/minute, tidal volume best evaluates this patient’s reaction to the increased
350 mL/breath (6mL/kg), FiO2 40%, and PEEP 5 cm H2O. positive pressure?
Which one of the following is best to recommend for this A. Reduced left ventricular afterload
patient? B. Reduced right ventricular output
A. Increase both the FiO2 and PEEP. C. Decreased stroke volume
B. Increase her respiratory rate but reduce the tidal D. Increased pulmonary vascular resistance
volume. 12. A 66-year-old woman who has been intubated for 8 days
C. Increase her minute ventilation. is being assessed for extubation. Which one of the fol-
D. Increase the FiO2 but decrease the PEEP. lowing is best to recommend for this patient?
A. A cuff leak test is not necessary.
Questions 9 and 10 pertain to the following case. B. Administer methylprednisolone 20 mg every 4 hours
P.O., a 20-year-old man (weight 73 kg), has a medical history x 3 doses.
of asthma for which he uses an albuterol inhaler as needed. C. Perform a cuff leak test and delay extubation for a
He is intubated after a motor vehicle collision and is managed value < 110 mL.
on continuous infusions of fentanyl and propofol. P.O.’s chest D. Perform a cuff leak test and delay extubation to give
radiography is clear except for some signs of bronchial wall steroids for a value < 110 mL.
thickening.
13. A 56-year-old man (BMI 45 kg/m2) is brought to the ED
9. On day 4 of admission, P.O. experiences failure of an SBT with respiratory distress of unknown etiology. He has
(RSBI 185 breaths/min/L with obvious signs of increased been placed on nasal cannula at 6 L/min and shows obvi-
work of breathing). Which one of the following is the ous work of breathing, and the team is concerned about
most likely cause of P.O.’s respiratory failure? having to intubate. Which one of the following oxygen
A. Delirium delivery systems is best to recommend for this patient?
B. Fluid overload A. Simple face mask at 6 L/min
C. Restrictive airway disease B. Non-rebreather mask at 4 L/min
D. Anxiety C. Non-rebreather mask at 8 L/min
10. P.O. has a spontaneous breathing trial performed, D. HFNC at 30 L/min providing 2 cm H2O PEEP
and the following parameters are recorded: RSBI 200 14. A 57-year-old woman experiences failure of her SBT.
breaths/min/L on a T-piece trial. Upon physical assess- Her laboratory parameters are: magnesium 1.8 mg/dL,
ment, the patient is euvolemic, able to follow commands potassium 3.7 mEq/L, Ca 8.3 mg/dL, Na 134 mEq/L, and
during SBT, and initiates his own breaths on PRVC. The phosphorus 2.2 mg/dL. Which one of the following is the
care team requests the pharmacist’s input regarding the best intervention to support extubation success and why
use of dexmedetomidine for extubation. Which one of it is justified in this patient?
the following is best to recommend for P.O.?
A. Phosphate replacement; diaphragmatic weakness
A. PS SBT has been shown to have a better negative B. Magnesium replacement; delirium
predictive value of extubation success compared C. Potassium replacement; cardiac instability
to T-piece trials, so an SBT using PS should be D. Sodium replacement; altered mental status
conducted first prior to initiation of any drug
15. A patient is about to undergo large-bore chest tube
therapy.
placement. Which one of the following is the best anal-
B. Because of the performance on the T-piece trial, the
gesia regimen to recommend for this patient?
patient is unlikely to benefit from dexmedetomidine
and likely will benefit from another day on the A. Pre-emptive IV acetaminophen and PRN opiate
ventilator. therapy post-procedure
C. The patient does not need dexmedetomidine B. Pre-emptive IV acetaminophen and PRN topical
because he is able to follow commands and have lidocaine post-procedure
appropriate respiratory drive. C. Pre-emptive IV opioid therapy with no PRN therapy
D. A T-piece trial should be repeated with post-procedure
dexmedetomidine to mitigate anxiety as a potential D. Pre-emptive IV opioid therapy and PRN opiate
reason for why the patient failed the T-piece trial. therapy post-procedure

CCSAP 2021 Book 1 • Pulmonary and Endocrinology 34 Mechanical Ventilation and Pulmonary Procedures
Learner Chapter Evaluation: Mechanical Ventilation and Pulmonary Procedures

As you take the posttest for this chapter, also evaluate the 8. The teaching and learning methods used in the chapter
material’s quality and usefulness, as well as the achievement were effective.
of learning objectives. Rate each item using this 5-point scale: 9. The active learning methods used in the chapter were
effective.
• Strongly agree
10. The learning assessment activities used in the chapter
• Agree
were effective.
• Neutral
• Disagree 11. The chapter was effective overall.
• Strongly disagree 12. The activity met the stated learning objectives.
13. If any objectives were not met, please list them here.
1. The content of the chapter met my educational needs.
2. The content of the chapter satisfied my expectations.
3. The author presented the chapter content effectively.
OTHER COMMENTS
4. The content of the chapter was relevant to my practice 14. Please provide any specific comments related to any
and presented at the appropriate depth and scope. perceptions of bias, promotion, or advertisement of
commercial products.
5. The content of the chapter was objective and balanced.
15. Please expand on any of your above responses, and/or
6. The content of the chapter is free of bias, promotion, and
provide any additional comments regarding this chapter:
advertisement of commercial products.
7. The content of the chapter was useful to me.

CCSAP 2021 Book 1 • Pulmonary and Endocrinology 35 Mechanical Ventilation and Pulmonary Procedures
COPD, Asthma, and PAH
By Paige Garber Bradshaw, Pharm.D., BCCCP; and Maria Guido, Pharm.D., BCPS

Reviewed by Chris Droege, Pharm.D., FCCM, FASHP, BCCCP; and Zachary Smith, Pharm.D., BCPS, BCCCP

LEARNING OBJECTIVES

1. Apply knowledge of disease-specific endotypes and their implications in the targeted treatment for chronic obstructive
pulmonary disease (COPD) and asthma.
2. Develop a comprehensive therapeutic plan for a patient experiencing an acute exacerbation of COPD.
3. Design an appropriate therapeutic plan for a patient presenting with life-threatening status asthmaticus.
4. Develop a therapeutic plan for pharmacologic management of pulmonary arterial hypertension (PAH) in critically ill
patients.
5. Design a therapeutic plan for managing care transitions of PAH patients who require additional or interchangeable
medication therapy.

COPD AND ASTHMA


ABBREVIATIONS IN THIS CHAPTER
Chronic obstructive pulmonary disease and asthma are classified
AATD Alpha-1 antitrypsin deficiency
as obstructive pulmonary diseases that primarily present as chronic
AECOPD Acute exacerbation of chronic
obstructive pulmonary disease inflammation affecting the respiratory tract function and structure,
COPD Chronic obstructive pulmonary with major differences in their etiology, symptom manifestation, and
disease overall disease progression. Although both disease states require
ETRA Endothelin receptor antagonist different combinations of inhaled bronchodilators and corticoste-
FEV1 Forced expiratory volume in roids for chronic management, these medications are initiated in a
1 second specific order and titrated to different treatment effects.
GINA Global Initiative for Asthma Despite pathophysiologic differences, COPD and asthma are often
GOLD Global Initiative for Chronic grouped together as chronic lower respiratory diseases in epidemi-
Obstructive Lung Disease ologic studies. The 2015 Global Burden of Disease study concluded
ICS Inhaled corticosteroid that overall respiratory disease burden is led by COPD and asthma,
LABA Long-acting β-agonist which was largely attributed to population growth, aging, and disease
LAMA Long-acting muscarinic antagonist prevalence (GBD 2017). In the United States, an estimated 13 million
mPAP Mean pulmonary arterial pressure and 19 million adults have a diagnosis of COPD and asthma, respec-
MV Mechanical ventilation tively. According to CDC National Center for Health Statistics, in 2017
NIV Noninvasive ventilation more than 160,000 deaths were attributed to COPD and asthma com-
PAH Pulmonary arterial hypertension bined; 1.6 million adult ED visits were attributed to asthma; and an
PDE5 Phosphodiesterase type 5 additional 923,000 ED visits listed COPD as the primary diagnosis
PH Pulmonary hypertension in the United States alone. At the severe stages of these diseases,
PVR Pulmonary vascular resistance episodes of acute respiratory failure often require hospital and ICU
REMS Risk evaluation and mitigation admission.
strategy
RHC Right heart catheterization Pathophysiology and Diagnosis
RV Right ventricular Chronic Obstructive Pulmonary Disease
SABA Short-acting β2-agonist Chronic obstructive pulmonary disease is a heterogeneous and pro-
SaO2 Oxygen saturation gressive respiratory disease characterized by irreversible airflow
limitation caused by airway and alveolar structural abnormalities.
Table of other common abbreviations.
Respiratory structural remodeling is caused by significant exposure

CCSAP 2021 Book 1 • Pulmonary and Endocrinology 37 COPD, Asthma, and PAH
to noxious particles or gases and is further influenced by expiration, and subsequent static and dynamic lung hyperin-
host factors, such as lung pathology or existing paren- flation. Furthermore, mucolytic dysfunction is directly related
chymal destruction. The persistent inflammatory burden to structural disruption of small airways and can lead to per-
associated with COPD promotes remodeling of small air- sistent production of thick sputum and chronic cough (Rabe
way compartments and loss of elastic recoil, resulting in a 2017).
progressive decline of FEV1, inadequate lung emptying on Diagnosis of COPD requires spirometry confirmation
(FEV1/forced vital capacity <0.7 after administration of a
bronchodilator); however, if patients older than 40 years have
the symptoms of dyspnea, chronic cough, or sputum pro-
BASELINE KNOWLEDGE STATEMENTS
duction, experience recurrent respiratory tract infections, or
Readers of this chapter are presumed to be familiar have a family history of COPD with or without the presence of
with the following: risk factors, a COPD diagnosis should be further investigated
• General knowledge of COPD and asthma disease (GOLD 2020). Risk factors for COPD include active or history
pathophysiology of tobacco smoking and occupational exposure to dusts,
• General knowledge regarding spirometry measure- vapors, fumes, or gases (Rabe 2017). Severity classification
ments, normal values, and their role in COPD and of airflow limitation in COPD uses post-bronchodilator FEV1
asthma diagnosis
thresholds correlated to the following grades: mild, defined as
• Drug knowledge of the pharmacologic agents used FEV1 ≥80% predicted (Grade 1); moderate, defined as 50% <
to treat stable COPD and their role in therapy
FEV1 <80% predicted (Grade 2); severe, defined as 30% < FEV1
• Drug knowledge of the pharmacologic agents used <50% predicted (Grade 3); and very severe, defined as FEV1 <
to treat adult asthma and their role in therapy using
30% predicted (Grade 4) (GOLD 2020).
guidelines for a stepwise approach to therapy
• Drug knowledge of the pharmacologic agents used Asthma
to treat PAH
Asthma is classified as a chronic, heterogeneous respira-
• Imaging and tests used to diagnose and monitor PAH tory disorder; however, it is more accurately characterized
• Pharmacology of agents for treatment of PAH by airflow obstruction, bronchial hyperresponsiveness, and
• Application of hemodynamic monitoring underlying inflammation (GINA 2020). Asthmatic airflow
• Management of volume status in the ICU obstruction is linked to the dysregulation of airway mast cells
• Use of vasopressors and inotropes in the ICU and T helper type 2 cells, which play a vital role in the release
of inflammatory mediators, recruitment and activation of
Table of common laboratory reference values.
inflammatory cytokines, and enhancement of circulating
eosinophils within the airway (Bush 2019, Wenzel 2012).
ADDITIONAL READINGS These complex cellular interactions are responsible for the
pathophysiologic features that variably present in patients
The following free resources have additional back-
ground information on this topic: with asthma, including severe bronchoconstriction, airway
edema, airway remodeling, and mucus hypersecretion.
• Global Initiative for Chronic Obstructive Lung Disease The presence of clinical respiratory symptoms, such as
(GOLD). 2020 GOLD Reports: Global Strategy for
the Diagnosis, Management and Prevention of wheezing, shortness of breath, chest tightness, and cough
COPD. that vary over time and in intensity, in combination with
• Global Initiative for Asthma (GINA). 2020 GINA Main expiratory airflow limitation, are common clinical manifes-
Report: 2020 GINA Report, Global Strategy for tations of asthma (Israel 2017). Confirmation of expiratory
Asthma Management and Prevention. airflow limitation (FEV1/forced vital capacity <0.75) is evalu-
• Galiè N, Humbert M, Vachiery JL, et al. 2015 ESC/ ated using either a bronchodilator reversibility test, variability
ERS guidelines for the diagnosis and treatment of in peak expiratory flow, or positive exercise challenge (GINA
pulmonary hypertension. Eur Heart J 2016;37:
2020). Asthma severity is assessed retrospectively based
67-119.
on the level of treatment required to control symptoms and
• Galié N, Channick RN, Frantz RP, et al. Risk exacerbations.
stratification and medical therapy of pulmonary
arterial hypertension. Eur Respir 2019;53: 1801889.
Disease Phenotypes and Endotypes
• Smith Z, Rangarajan K, Barrow J, et al. Development
of best practice recommendations for the safe use The terms phenotypes and endotypes have recently been
of pulmonary hypertension pharmacotherapies introduced to help clinicians identify and differentiate exist-
using a modified Delphi method. Am J Health ing subtypes of COPD and asthma. Phenotypes are observed
System Pharm 2019;76:153-65. clinical characteristics associated with a disease. Classified
endotypes are groups of patients who share observed

CCSAP 2021 Book 1 • Pulmonary and Endocrinology 38 COPD, Asthma, and PAH
characteristics because of shared underlying biology. The formal clinical trials to assess efficacy with conventional spi-
formal identification of an endotype implies the recognition rometric outcomes have not been undertaken. Observational
of several shared disease features, including clinical charac- studies suggest a reduction in spirometric decline and clin-
teristics, genetics, physiology, histopathology, epidemiology, ical improvement in treated versus non-treated patients,
and treatment response (Garudadri 2018). Clinicians may be predominantly in patients with a FEV1 35%–49% of predicted
able to provide a more specific and targeted treatment for (Chapman 2009). Because of the identification of AATD as
patients through identifying more specific endotypes. the inciting factor related to disease progression, more lib-
eral recommendations regarding patient candidacy have
COPD Endotypes
been published. Both COPD Foundation and GOLD recom-
The heterogeneity of COPD refers to the spectrum of phe- mend consideration of augmentation therapy for all patients
notypic components of chronic bronchitis and emphysema with AATD, although they emphasize stronger evidence for
present in patients with COPD. Chronic obstructive pulmonary treating those with an FEV1 ≤65% predicted than those with
disease serves as an umbrella term that includes emphysema better lung function. The therapy is administered by a weekly
and chronic bronchitis as a continuum of respiratory symp- intravenous infusion and is considered ongoing and lifelong.
toms, with many patients exhibiting mixed clinical symptoms. Annual costs for patients with AATD can exceed $120,000
Chronic bronchitis carries a high inflammatory burden and depending on the prescribed augmentation regimen; there-
is notable for overproduction and hypersecretion of mucus fore, the main limitations surrounding augmentation therapy
by goblet cells. The classic definition of chronic bronchitis is are cost and lack of wide availability (Sieluk 2019).
chronic cough and sputum production for at least 3 months
per year for 2 consecutive years. Patients with chronic bron- Eosinophilic COPD
chitis are prone to infection because of the overproduction
Eosinophilic COPD refers to another endotype that is char-
of thick mucus that impedes effective airway clearance.
acterized by increased concentrations of circulating
Conversely, emphysema is described as progressive alveo-
eosinophils, commonly diagnosed by a peripheral blood dif-
lar damage that reduces oxygen exchange surface area and
ferential containing eosinophil counts 2% or greater, which
increases air trapping, leading to incomplete exhalation and
equates to about 150–200 cells/mm3. Eosinophilic-mediated
persistent breathlessness. Although emphysema and chronic
inflammation may be present in up to 40% of COPD patients
bronchitis are widely accepted phenotypic subtypes of COPD,
and has been associated with higher rates of an acute exac-
the research focus is expanding to identify and treat newly
erbation (Celli 2019). Recent data have confirmed that higher
identified COPD endotypes. Alpha-1 antitrypsin deficiency
blood eosinophils concentrations are associated with better
and eosinophilic COPD are the only currently identified COPD
clinical response to ICS (Bafadhel 2018). The increasing evi-
endotypes that constitute treatable traits and are identifiable
dence linking blood eosinophils to beneficial ICS response
using biomarkers (Celli 2019).
in COPD strengthens their use as a convenient biomarker to
personalize targeted therapy. These findings have been rec-
Alpha-1 Antitrypsin Deficiency ognized in the GOLD recommendations with an alteration of
Alpha-1 antitrypsin deficiency is a heritable condition char- the influences affecting selection of COPD pharmacotherapy.
acterized by the relative deficiency of circulating alpha-1 Consideration for ICS therapy was previously reliant on the
antitrypsin, a major serine protease inhibitor responsible patient’s exacerbation history and clinical symptoms; how-
for balancing the protective effects of the neutrophil elas- ever, ICS therapy should now be targeted to patients who
tase. In its absence, elastase has increased and unopposed often experience exacerbations, have progressive symp-
enzymatic activity that causes increased host damage and toms, and have a blood eosinophil count of 300 cells/mm3or
inflammation. Pulmonary manifestations of AATD include greater. Further details regarding eosinophil thresholds used
chronic bronchitis, emphysema, and clinical bronchiecta- in escalation and de-escalation strategies for COPD are dis-
sis. This deficiency is believed to affect less than 1% of COPD cussed in the section on exacerbation prevention.
patients (Barnes 2018).
The recommendation by WHO is that all patients with a Asthma Endotypes
diagnosis of COPD should be screened once in their lifetime The heterogeneity of asthma encompasses its underly-
for AATD. If AATD is identified, augmentation therapy may ing complex biological network of distinct and interrelating
be initiated for targeted management. Augmentation therapy inflammatory pathways. Gene expression profiling of airway
refers to the supplementation of alpha-1 antitrypsin obtained samples from patients with asthma led to the broad classi-
from the plasma of healthy human donors for patients with a fication of type 2 and non-type 2 asthma endotypes. Type 2
diagnosis of both COPD and AATD. The treatment goal is to asthma refers to the extensive inflammation pathway regu-
slow or stop the progression of lung destruction by replac- lated predominately by the immune response and activity of
ing the deficient protein. However, therapy can neither restore T helper type 2 cells, a CD4+ cell subtype. The T helper type
lost lung function nor cure the disease. Because AATD is rare, 2 cells secrete the prototypical cytokines interleukin (IL)-4,

CCSAP 2021 Book 1 • Pulmonary and Endocrinology 39 COPD, Asthma, and PAH
IL-5 and IL-13, and stimulate type 2 immunity, which includes COPD EXACERBATIONS
high antibody titers and eosinophilia. Type 2 inflammation Despite optimal medical therapy, patients with COPD have
is prevalent in patients with uncontrolled persistent asthma episodic worsening of respiratory symptoms, termed exac-
and is related to higher disease burden (Bush 2019, Carr erbations. Acute exacerbations of COPD (AECOPD) are a
2018). Collectively, these discoveries have shifted existing significant cause of morbidity, mortality, hospital admission,
paradigms in tailoring novel therapies and standard asthma impaired health status, and increased cost burden (Rabe
management. 2017). Among patients who are hospitalized with a severe
AECOPD, the in-hospital mortality ranges from 5%–7%, and
Type 2 Asthma mortality at 1 year increases to 25%–40% (Crisafulli 2018).
Type 2 asthma may be referred to as eosinophilic asthma
because of the presence of eosinophil-mediated inflamma- Diagnosis and Assessment
tion. The immune response of type 2 asthma is primarily Acute exacerbations of COPD can be diagnosed using either
mediated by IL-4, IL-5, IL-13, and immunoglobulin E (Bush 2019, acute symptom-based or event-based criteria. Per GOLD
Carr 2018). Identification of patients with type 2 asthma relies guidelines, an acute exacerbation of COPD is defined as an
on laboratory evidence of a blood eosinophil count of 150 acute worsening of respiratory symptoms that results in
cells/mm3 or greater or sputum eosinophils 2% or greater, the need for additional therapy. The 2019 National Institute
clinical identification of allergen-driven asthma, or need for for Health and Care Excellence (NICE) guidelines further
maintenance oral corticosteroids to control symptoms (GINA define AECOPD as symptoms worsening from the patient’s
2020). For asthmatic clinical considerations, identification of usual stable state and acute in onset; common symp-
the type 2 immune response endotype has prompted investi- toms that necessitate a change in medication therapy are
gation for targeted biologic treatment options because of the worsening breathlessness, cough, increased sputum pro-
increasing refractory response to ICS in such patients. Many duction, and change in sputum color (Hopkinson 2019).
targeted treatments are in various stages of clinical devel- Both definitions emphasize the acute onset of increased
opment for patients with type 2 immune response-driven respiratory symptoms requiring an escalation in ther-
inflammation, including anti-IL-4/IL-13, anti-IL-4, anti-IL-5, apy. Although AECOPD may be triggered by both viral
and anti-immunoglobulin E antibodies, as well as selective and bacterial respiratory infections, pollution or ambient
prostaglandin D2 receptor antagonists (chemoattractant temperature exposure may also initiate or amplify exacer-
receptor-homologous molecule expressed on T helper type 2 bation symptoms (Crisafulli 2018).
cells, or CRTH2, antagonists). Related advances in the 2020 Differentiating AECOPD from other causes of worsen-
GINA recommendations include considerations for add-on ing respiratory symptoms is important to tailor therapy
type 2 biologic therapy for select patients as follows: anti-IL-4 appropriately. Common differentiating , and often competing,
receptor therapy (e.g., dupilumab) is now recommended as diagnoses with AECOPD include pneumonia, pneumotho-
add-on therapy for with severe type 2 asthma or oral corti- rax, pleural effusions, pulmonary embolism, atrial cardiac
costeroid-dependent asthma; anti-IL-5 (e.g., mepolizumab, arrhythmias, or pulmonary edema caused by cardiac-related
reslizumab) and anti-IL-5 receptor (benralizumab) therapy conditions (GOLD 2020). During the diagnostic work-up for
may be considered as add-on therapy for patients with severe AECOPD, assessing symptom severity, drawing arterial or
eosinophilic asthma (300 cells/mm3 or greater) that is uncon- venous blood gases, and analyzing chest radiograph results
trolled on steps 4–5 of guideline recommended treatment are often needed for diagnostic guidance and confirmation.
(GINA 2020). The treatment setting depends largely on symptom sever-
ity. Indications for respiratory or medical ICU admission focus
Non-Type 2 Asthma on inadequate response to emergency therapy, the need for
Non-type 2 asthma, commonly described as non-eosino- more invasive respiratory support, or signs of systemic insta-
philic asthma, is characterized predominantly by neutrophilic bility caused by profound hypoxia or altered hemodynamics
inflammation mediated by IL-1, IL-6, IL-17, and tumor necro- (Box 1).
sis factor (Bush 2019, Carr 2018). Although the mechanisms Patients may experience varying degrees of symptom
driving type 2 asthma inflammation have become more severity during an AECOPD. The severity grading defined by
widely recognized, less attention has been given to the the GOLD guidelines refers to the setting and intensity of
underlying mechanisms behind non-type 2 asthma, which treatment indicated for presenting symptoms (Table 1).
has led to a lack of available novel biologic therapies for this
patient population. Patients categorized as having non-type Pharmacologic Treatment
2 asthma may be considered for non-biologic add-on treat- Maintenance pharmacotherapy for patients with stable COPD
ment if not already trialed, such as tiotropium, leukotriene aims to improve clinical symptoms and the health-related
modifier, low-dose macrolide or oral corticosteroid therapy quality of life as well as to reduce the risk and frequency
(GINA 2020). of exacerbations. Inhaled bronchodilators are central to

CCSAP 2021 Book 1 • Pulmonary and Endocrinology 40 COPD, Asthma, and PAH
Patients presenting with AECOPD, however, require medi-
Box 1. Indications for Inpatient
cation changes from their baseline therapy. Pharmacologic
Management of Acute Exacerbation of
treatment of AECOPD relies on inhaled bronchodilator ther-
Chronic Obstructive Pulmonary Disease
apy, systemic corticosteroids, and assessment of antibiotic
Hospital Admission
need. Adjunctive agents have been evaluated for select popu-
• Severe respiratory symptoms: sudden worsening resting lations, but these agents lack robust evidence for routine use
dyspnea; tachypnea (respiratory rate >20 breaths/minute
with increased work of breathing); decreased SaO2 in all patients with AECOPD.
(<88%–90%)
• Acute respiratory failure: hypoxia (PaO2 <60 mm Hg or SaO2 Inhaled Bronchodilator Agents
<90%) or hypercapnia (PaCO2 >50 mm Hg) with symptoms Inhaled bronchodilators are the backbone of initial therapy for
of acute respiratory distress or new physical symptoms of
patients presenting with dyspnea associated with AECOPD.
cyanosis or peripheral edema
The preferred treatment is a SABA, with or without a short-act-
• Insufficient home support, such as lack of home safety or
social/medical connectivity ing anticholinergic (GOLD 2020). Although to date no clinical
• Worsening or new respiratory symptoms secondary trials have assessed the effectiveness of continuing mainte-
to failed medical management of a previous existing nance long-acting bronchodilators (e.g., LABA, LAMA, LABA/
exacerbation LAMA) with or without ICS during AECOPD, GOLD guidelines
ICU Admission recommend continuing these therapies during acute manage-
• Severe dyspnea with inadequate response to initial ment or to at least start these medications as soon as possible
emergency therapy: short-acting β2 agonist with or without before hospital discharge, the latter being a more common
a short-acting anticholinergic
practice, with a focus on short-acting bronchodilators during
• Changes in mental status, such as lethargy, confusion, and acute management and resumption of maintenance inhalers
coma
• Persistent or worsening hypoxemia (PaO2 <40 mm Hg) when patients have returned to baseline respiratory status.
and/or severe/worsening respiratory acidosis (PaCO2 >45 Short-acting β2-agonists, most commonly albuterol, should
mm Hg and arterial pH <7.35) be administered at increased doses and/or frequency from
• Need for invasive mechanical ventilation outpatient therapy; usual doses of albuterol are 2.5 mg by
• Hemodynamic instability, without response to fluids and nebulizer every 1–4 hours as needed or 4–8 puffs by metered
vasopressors
dose inhaler (MDI) with a holding chamber every 1–4 hours as
• PaO2 = partial pressure of arterial oxygen.
needed. Stimulation of β2-adrenergic receptors can produce
Information from: Global Initiative for Chronic Obstructive resting sinus tachycardia and has the potential to precipi-
Lung Disease (GOLD). 2020 GOLD Reports: Global Strategy for
tate cardiac rhythm disturbances in susceptible patients.
the Diagnosis, Management and Prevention of COPD. Fontana,
WI: GOLD, 2020. Therefore, it is recommended to monitor response to therapy
and deliver treatments for current symptoms, instead of con-
tinuing indefinite scheduled therapy if dyspneic symptoms
symptom management and are commonly administered as have resolved. Levalbuterol, the R-enantiomer of albuterol,
a single agent or as combination inhaler therapy to prevent has not demonstrated superiority to albuterol in terms of
or reduce symptoms. Recommended initial pharmacological patient tolerance. Because of formulary and cost-restric-
treatment for stable COPD correlates to baseline severity and tions, albuterol is commonly administered as the SABA of
exacerbation frequency (GOLD 2020). choice for patients with AECOPD.

Table 1. Classification of the Severity of Acute Exacerbation of Chronic Obstructive Pulmonary Disease

Severity Treatment Setting Clinical Course

Mild Outpatient management SABA monotherapy

Moderate Outpatient management SABA +/– oral corticosteroids +/– antibiotics

Severe Inpatient management; possible ICU SABA + ipratropium + intravenous or oral corticosteroids +/–antibiotics
if associated with acute respiratory Respiratory support, with noninvasive ventilation before mechanical
failure ventilation if appropriate

+ = with; – = without; SABA = short-acting β2 agonist.


Information from: Global Initiative for Chronic Obstructive Lung Disease (GOLD). 2020 GOLD Reports: Global Strategy for the
Diagnosis, Management and Prevention of COPD. Fontana, WI: GOLD, 2020.

CCSAP 2021 Book 1 • Pulmonary and Endocrinology 41 COPD, Asthma, and PAH
A systematic review assessing the route of delivery for days) was noninferior to a longer course (14 days) in AECOPD
short-acting bronchodilators found no significant differ- in a comparison of exacerbation relapse rates up to 6 months
ence in FEV1 between MDI or nebulizers. However, nebulizers (Leuppi 2013). Of note, less than 15% of the enrolled patients
may be more feasible for use in acutely ill patients who are required MV, and it is unclear how many of the patients were
unable to independently operate an MDI, including mechan- critically ill. Because of the nebulous interpretation of these
ically ventilated patients. In comparison with administration results for critically ill patients, varying ICU practice patterns
of an MDI, administering SABAs (e.g., albuterol, levalbuterol), still exist regarding corticosteroid duration in patients with
short-acting muscarinic antagonists (e.g., ipratropium), or a AECOPD. Long-term use of oral corticosteroids have numer-
SABA/short-acting muscarinic antagonist combination (e.g., ous adverse effects, including development of osteoporosis,
albuterol/ipratropium) as a nebulizer solution may provide adrenal insufficiency, myopathy, immune suppression, and
simplicity of administration augmented by more confident negative GI, hepatic, and ophthalmologic effects. Therefore,
drug delivery to the site of action without manipulation of the previous notion that longer durations of corticoste-
ventilator settings or coordination with spontaneous breaths roid therapy are indicated after exacerbation episodes has
by the patient (Van Geffen 2016). been strongly discouraged because of the lack of evidence
confirming benefit, and possible proposed harm. A recent
Systemic Corticosteroids observational study of more than 60,000 patients suggests
Systemic corticosteroids have been shown to improve FEV1 that longer courses of oral corticosteroids (greater than 10
oxygenation, shorten recovery time, and decrease hospital days or greater than 250 mg cumulative prednisolone dose)
length of stay for patients presenting with AECOPD (GOLD for AECOPD are associated with an increased risk of pneumo-
level of evidence A). Corticosteroid therapy has also been nia and mortality (Sivapalan 2019).
shown to reduce treatment failure and rate of relapse after To further tailor systemic corticosteroid therapy to patients
exacerbation episodes (Walters 2014). In brief, systemic corti- with the highest beneficial yield, recent studies suggest that
costeroids should be prescribed for most exacerbations, with corticosteroids for AECOPD may be less efficacious in patients
a larger benefit seen in moderate to severe AECOPD episodes. with lower eosinophil counts (Bafadhel 2018). A prospective,
An oral dose of 40 mg of prednisone once daily for 5 days multicenter randomized, controlled trial (CORTICO-COP) con-
is recommended (GOLD level of evidence B). If oral admin- cluded that eosinophil-guided corticosteroid-sparing therapy
istration is not an option, equivalent doses of intravenous (for which prednisolone therapy was withheld if the serum
hydrocortisone or methylprednisolone may be administered. eosinophil count was less than 300 cells/mm3) was noninfe-
Oral and intravenous steroid therapy are deemed equally rior compared with standard care for the number of days alive
effective. Controversy is ongoing surrounding the optimal and out-of-hospital, and reduced the duration of systemic
corticosteroid dosage, specifically in a critically ill popula- corticosteroid exposure for AECOPD treatment (Sivapalan
tion, with some evidence suggesting that higher doses of 2019). Critically ill patients were excluded from this trial, how-
1–2 mg/kg/day of prednisone dose equivalency with subse- ever. Further studies on steroid-sparing treatment regimens
quent rapid taper may be beneficial in regard to decreased for AECOPD are needed before this practice can be recom-
duration of MV and ICU length of stay (Dixit 2015). Contrary mended for other hospitalized patients.
to this point, study findings suggest that doses of methyl-
prednisolone greater than 240 mg/day are associated with Antibiotic Therapy
longer hospital and ICU length of stay, the need for insulin Many patients with AECOPD are treated with antibiotics. The
therapy, and fungal infections compared with doses of 240 benefit of antibiotic therapy remains uncertain because of
mg/day or lower in critically ill patients undergoing treatment conflicting results from systematic reviews, which enrolled
for AECOPD (Kiser 2014). Nebulized corticosteroids have broad ranges of exacerbation severity and assessed varying
been investigated in hospitalized patients with AECOPD as treatment effects. Antibiotics are included in the treatment
an alternative to systemic corticosteroids and provide similar differential because bacterial infections are a common
benefits to oral prednisone and intravenous methylprednis- exacerbation trigger, contributing to about 50%–70% of all
olone, specifically in non-acidotic exacerbation episodes episodes. Patients with infectious exacerbations have longer
(Pleasants 2018). However, these findings are not inclusive of hospitalizations and greater lung function impairment than
critically ill patients, and caution should be used with extrap- those with noninfectious exacerbations (Vollenweider 2018).
olating to such populations. Differentiating an infectious from a noninfectious AECOPD
Recommendations regarding the duration of steroid ther- cause can be difficult. Valid arguments against overuse of
apy explicitly state that therapy should not exceed 5–7 days antibiotics are development of multidrug resistance organ-
for AECOPD treatment (GOLD level of evidence A). The most isms, polypharmacy, potential adverse effects, and cost.
robust evidence supporting the recommendation for shorter Current recommendations do not support the use of
steroid duration is cited from the REDUCE study, which con- antibiotics for all exacerbations, but antibiotic therapy is
cluded that a shorter course of systemic corticosteroids (5 recommended for moderately or severely ill patients with

CCSAP 2021 Book 1 • Pulmonary and Endocrinology 42 COPD, Asthma, and PAH
AECOPD who exhibit cardinal symptoms, classified as dys- regimens were associated with an overall reduction in 30-day
pnea, increased sputum volume, and sputum purulence. hospital readmissions (7.3% vs. 8.8%, p<0.01), increased time
Antibiotic therapy should be reserved for patients who either to next all-cause (159 vs. 130 days, p<0.01) or AECOPD (200
exhibit all cardinal symptoms or have two of the cardinal vs. 175 days, p=0.03) readmission, and decreased hospital
symptoms, including purulent sputum, or for patients who costs, compared with non-macrolide regimens (Kiser 2019).
require either a noninvasive or invasive approach to venti- Antibiotics should be narrowed based on respiratory
lation—NIV or MV (GOLD 2020). The presence of purulent culture results and continued for 5–7 days (GOLD level of
changes or increases in sputum production carries a heavier evidence B). In support of shorter durations for antibiotic
weight in determining appropriate candidates for antibiotic courses, a meta-analysis of 21 studies concluded that a short
therapy, which is supported by the findings that observed course of antibiotic treatment (5 days or less) is as effec-
sputum purulence is 94.4% sensitive for correlation to high tive as the traditional longer treatment (more than 5 days) in
bacterial loads on culture (Stockley 2000). A 2018 Cochrane patients with mild to moderate AECOPD (Moussaoui 2007).
review concluded that antibiotic use for treatment of AECOPD Conclusive data in an ICU population regarding duration are
in ICU patients had a strong beneficial effect, which was pre- more difficult to find. It should be noted that patients admit-
dominantly powered by the results of one study that included ted to the ICU may have a competing diagnosis of pneumonia,
93 patients and showed a statistically significant benefi- largely differentiated on chest imaging, and may require anti-
cial effect of antibiotics on treatment failure (RR 0.19; 95% biotic therapy in accordance with national or institutional
CI, 0.08–0.45), mortality (OR 0.21; 95% CI, 0.06–0.72), and guidelines. It is reasonable to assess a patient’s clinical sta-
hospital length of stay (median –9.60 days; 95% CI, –12.84 tus, such as return to baseline respiratory status, at 5 days of
to –6.36) (Vollenweider 2018). These results further support antibiotic therapy to determine appropriate discontinuation
the use of antibiotics in patients requiring ICU admission for versus continuation to 7 days for AECOPD management.
AECOPD management with cardinal symptom criteria or MV. Specific biomarkers have been investigated as a tool to
Infectious causes of AECOPD are commonly attributed help guide appropriate antibiotic initiation or continuation
to pathogens that colonize the respiratory tract, includ- for patients presenting with AECOPD. Procalcitonin (PCT)
ing Haemophilus influenzae, Streptococcus pneumoniae, and monitoring has previously been used as biomarker for active
Moraxella catarrhalis (Vollenweider 2018). Atypical microor- bacterial infections. Limitations to its routine use include
ganisms, such as Mycoplasma pneumoniae and Chlamydia spp. expense and lack of wide availability, and data supporting its
have also been implicated in 5%–15% and 5%–10% of AECOPD use in AECOPD are conflicting. A recent systematic review
cases, respectively (Rabe 2017). For patients who often expe- found that PCT-guided algorithms can reduce unnecessary
rience exacerbations, severe airflow limitation (GOLD stages administration of antibiotics without increasing adverse out-
3 and 4), or the need for MV, respiratory cultures should be comes related to treatment failure in severe AECOPD (Ni 2019).
obtained to help ensure appropriate treatment. Resistant However, for patients requiring admission in the ICU, PCT has
gram-negative bacteria, including Pseudomonas spp., are poor diagnostic value. A recent randomized controlled trial
more prevalent in these patient populations and should be investigated the role of PCT algorithms for ICU patients with
considered when selecting appropriate empiric therapy. A severe AECOPD and found an increase in 3-month mortal-
recent observational study found that Pseudomonas aerugi- ity with no difference noted in antibiotic exposure compared
nosa colonization independently predicted an increased risk with controls (Daubin 2018). Monitoring of C-reactive protein
of hospitalization for exacerbation and all-cause mortality; (CRP) has also been investigated as a method for avoiding
therefore, ensuring appropriate empiric coverage for those at unnecessary antibiotic prescribing. In the outpatient setting,
highest risk is vital (Eklof 2019). withholding antibiotics in patients with CRP concentrations
The GOLD guidelines offer empiric monotherapy recom- less than 20 mg/L resulted in less antibiotic use with no evi-
mendations of an aminopenicillin with clavulanic acid, a dence of harm associated with withheld treatment (Butler
macrolide, or tetracycline. Reasonable intravenous options 2019). Monitoring of CRP in the ICU setting is not validated
include ceftriaxone, doxycycline, azithromycin, and ampicil- for this indication. At this time, neither PCT nor CRP should
lin/sulbactam. Initial antibiotics should be chosen on the basis be routinely used to assess antibiotic appropriateness for
of previous antibiotic use, known pathogen susceptibility, and AECOPD in the ICU setting, and emphasis should be placed
the local bacterial resistance patterns. If Pseudomonas spp. on evaluation of cardinal symptoms and the need for ventila-
are suspected, high-dose levofloxacin, cefepime, or piperacil- tion support.
lin/tazobactam should be initiated. Macrolides are often used
as adjunct therapy because of their activity against atypical Adjunctive Agents
bacteria as well as their intrinsic anti-inflammatory effects. Adjunctive treatment modalities to alleviate symptoms more
Further evidence to support the addition of macrolide ther- effectively and decrease the severity or duration of AECOPD
apy is from a matched cohort analysis of patients admitted to is an ongoing initiative because of the high morbidity and
the ICU for AECOPD management; macrolide-based antibiotic mortality attributed to such episodes. Unfortunately, there is

CCSAP 2021 Book 1 • Pulmonary and Endocrinology 43 COPD, Asthma, and PAH
a lack of current evidence to support the use of methylxan-
Box 2. Clinical Indications for Advanced
thines (e.g., aminophylline, theophylline), phosphodiesterase
Respiratory Support During Acute
type 4 inhibitor therapy (e.g., roflumilast), or mucolytic agents
Exacerbation of Chronic Obstructive
(e.g., acetylcysteine) in the management of AECOPD
Pulmonary Disease
symptoms.
Indications for Noninvasive Mechanical Ventilation (NIV)
At least one of the following:
Oxygen Therapy and Respiratory Support • Respiratory acidosis (PaCO2 >45 mm Hg and/or arterial
Supplemental oxygen therapy and respiratory support are pH ≤7.35)
often indicated during exacerbation episodes to treat breath- • Severe dyspnea with clinical signs suggestive of respiratory
muscle fatigue, such as respiratory accessory muscles,
lessness or persistent hypoxia. The goal for hypoxia treatment paradoxical motion of the abdomen, or retraction of
is a sustained SaO2 88%–92% or greater after intervention. intercostal spaces
Options for supplemental oxygen delivery include a simple • Persistent hypoxemia despite supplemental oxygen therapy
face mask, nonrebreather mask, reservoir cannula, and high-
Indications for Invasive Mechanical Ventilation
flow nasal or transtracheal devices. Confirming a patient’s
• Inability to tolerate or failure of NIV
baseline oxygen requirement is an important step in quanti- • Respiratory or cardiac arrest
fying the degree of hypoxia present as well as documenting • Diminished consciousness or inability to protect airway
patient-specific criteria for improvement or worsening of • Massive aspiration
clinical status. Repeat arterial blood gas measurements are • Persistent inability to remove respiratory secretions
recommended 30–60 minutes after initiating oxygen by any
• Severe hemodynamic instability without response to fluids
and vasoactive drugs
device to ensure the patient is not retaining carbon dioxide • Severe cardiac arrhythmias
and is oxygenating appropriately. Arterial sampling is more • Life threatening hypoxemia in patients unable to tolerate NIV
difficult and painful for patients than venous sampling. A
recent study comparing venous blood gas (VBG) values to Information from: Global Initiative for Chronic Obstructive
Lung Disease (GOLD). 2020 GOLD Reports: Global Strategy for
those from arterial draws for initial COPD assessment demon- the Diagnosis, Management and Prevention of COPD. Fontana,
strated that VBG accurately assessed bicarbonate and pH WI: GOLD, 2020.
compared with arterial readings. This approach allows for the
initial assessment of AECOPD to be based on VBG analysis
and pulse oximetry, simplifying the care pathway and improv- of invasive MV. The critical care team should make efforts
ing the patient experience (McKeever 2016). Additional data to discuss the short- and long-term implications of invasive
are needed to clarify the role of VBG values in acute respi- MV, including consideration of patients’ care directives and
ratory failure associated with AECOPD, especially in patients discussion regarding reversibility of the precipitating event.
with persistent hypoxic. Risks associated with invasive MV include ventilator-ac-
quired pneumonia, barotrauma, and prolonged weaning or
Noninvasive Ventilation failure to liberate to spontaneous ventilation. See Box 2 for
Noninvasive ventilation provides continuous positive airway the clinical indications for invasive MV.
pressure with the use of nasal prongs, face masks, or hel- The main objective of invasive MV during AECOPD is to
met devices. This type of ventilation should be the first mode reduce dynamic hyperinflation and air trapping. Optimized
used in COPD patients with acute respiratory failure who have expiratory times, low respiratory rates, high inspiratory flows,
no absolute contraindications, such as facial trauma, recent and consideration of the intrinsic positive end-expiratory
surgery on the upper airway, copious or unmanageable pressures used all aid in the ability for MV to provide ade-
secretions, or high aspiration risk (GOLD level of evidence A). quate gas exchange while minimizing lung injury (Ahmed
Indications for NIV are listed in Box 2. This approach to venti- 2015). Mechanical ventilation also increases oxygenation
lation improves gas exchange, reduces the work of breathing while minimizing work of breathing. Because of the patho-
and need for intubation, and decreases hospital length of physiologic components of COPD leading to chronic retention
stay while improving survival (GOLD level of evidence A). A of partial pressure of carbon dioxide, hypercapnia correction
systematic review confirmed the positive effects of NIV on is aimed at the patient’s baseline value at rest, not normo-
decreased mortality and estimated a 65% RR reduction in capnia. Permissive hypercapnia associated with low tidal
the need for intubation after NIV implementation for AECOPD volumes of less than 7 mL/kg per ideal body weight should be
(Osadnik 2017). used to promote controlled hypoventilation while minimizing
regional overdistention.
Invasive MV
Although NIV is preferred for first-line respiratory support in AECOPD Prevention
eligible patients, many patients may present with severe exac- After an AECOPD episode, appropriate measures for future
erbation symptoms requiring escalation to or immediate need exacerbation prevention should be initiated. Exacerbation

CCSAP 2021 Book 1 • Pulmonary and Endocrinology 44 COPD, Asthma, and PAH
Continued exacerbations LABA or LAMA monotherapy
Eo
sin
op
hil
s ≥3
00
ce
lls
/m
m 3

3
Eosinophils <300 cells/mm
LABA + LAMA LABA + ICS
Eo
Eo sin
sin op
op hil
hil s<3
s ≥1 00
00 ce
ce lls
lls /m
/m m 3
m3

LABA + LAMA + ICS

3
Eosinophils <100 cells/mm

Roflumilast Azithromycin
FEV1 < 50% and
Former smoker
chronic bronchitis

Figure 1. Escalation and de-escalation strategies for maintenance inhaler therapy in patients who experience
frequent AECOPD. For patients on LABA or LAMA monotherapy escalation to either combination LABA/LAMA or LABA/
ICS is recommended, with a preference for LABA/ICS in patients with blood eosinophil counts ≥300 cells/mm3 because
of greater predictive benefit. If patients were maintained on LABA/LAMA therapy at baseline with eosinophil count
≥100 cells/mm3, then therapy should be escalated to triple therapy (LABA/LAMA/ICS), whereas patients with blood
eosinophils <100 cells/mm3 should have ICS therapy withheld and consider the addition of novel therapies, such as
roflumilast or chronic macrolide therapy. Patients prescribed LABA/ICS at baseline should receive escalation to triple
therapy as appropriate. Roflumilast, chronic macrolide therapy, or cessation of ICS therapy should be considered in
patients who experience frequent AECOPD with triple therapy.
FEV1 = forced expiratory volume in 1 second; ICS = inhaled corticosteroid; LABA = long-acting β-agonist; LAMA = long-acting
muscarinic antagonist.
Information from: Global Initiative for Chronic Obstructive Lung Disease (GOLD). 2020 GOLD Reports: Global Strategy for the
Diagnosis, Management and Prevention of COPD. Fontana, WI: GOLD, 2020.

frequency increases with disease severity; therefore, imple- be regarded as estimates that can predict different probabil-
menting strategies and therapies that may slow disease ities of treatment benefit, rather than precise cut-off values.
progression may also decrease the incidence of AECOPD. Pharmacists can play a vital role in optimization of prescribed
maintenance therapy at discharge in compliance with guide-
Pharmacologic Secondary Prevention line recommendations, as well as actively identify and resolve
After AECOPD, appropriate measures for future preven- issues regarding medication access (e.g., prior authorization
tion should be initiated. Initial emphasis should be placed referrals, insurance formulary interchanges) (Gentene 2019).
on ensuring appropriate prescription of maintenance ther- In addition, education and emphasis regarding the impor-
apy. Details regarding maintenance therapy optimization for tance of proper inhaler technique and should be included in
patients with continued exacerbations (2 or more moderate discharge preparation for all patients with COPD.
exacerbations or at least 1 severe exacerbation requiring
hospitalization within the previous year) are listed in Figure 1. ICS Cessation
Notably, eosinophil cell count thresholds are now used to Discontinuation of ICS therapy may be considered if patients
help identify both patients with higher predictive benefit experience a lack of benefit attributed to therapy or experi-
from the addition of ICS therapy as well as considerations ence adverse effects, such as repeated pneumonia episodes.
for withdrawing ICS therapy in patients without response. These recommendations were largely motivated by a large
It is important to note that the discussed thresholds of less observational pharmacoepidemiologic study that found
than 100 cells/mm3 and greater than 300 cells/mm3 should similar effectiveness of LABA/LAMA and ICS/LABA, but

CCSAP 2021 Book 1 • Pulmonary and Endocrinology 45 COPD, Asthma, and PAH
a significantly higher risk of pneumonia in those treated 25-hydroxy vitamin D) results in a 50% reduction in AECOPD
with ICS/LABA (severe pneumonia of LABA/LAMA rela- episodes and hospital admission; however, such benefit was
tive to LABA/ICS, HR 0.66; 95% CI, 0.5–0.87) (Suissa 2019). not seen in patients with normal baseline concentrations
Furthermore, because of the recent advances in using blood (Jolliffe 2019). Therefore, it is recommended that all patients
eosinophil counts as a biomarker for estimating efficacy of hospitalized for AECOPD should be assessed and investi-
ICS, patients with eosinophil counts 300 cells/mm3 or greater gated for severe deficiency followed by supplementation if
have the highest risk of experiencing exacerbations if ICS required.
therapy is withdrawn and therefore should be monitored
closely if therapy is discontinued. Methylxanthines
Although methylxanthines do not have a role in acute man-
Prophylactic Antibiotics agement of AECOPD, they have been investigated as add-on
Prophylactic, chronic antibiotics have been investigated as therapy to decrease exacerbation frequency in stable COPD.
an intervention to decrease exacerbation frequency and have The TWICS trial investigated the effectiveness of adding low-
historically failed to show routine benefit. Chronic azithro- dose theophylline (200 mg once or twice daily to provide goal
mycin therapy (250 mg/day or 500 mg three times weekly) plasma concentrations of 1–5 mg/L) to ICS in adult patients
reduces the occurrence of AECOPD and may be best consid- with COPD who experienced at least 2 exacerbations in the
ered in nonsmoking patients optimized on triple therapy who previous year. Adjunctive theophylline failed to reduce the
still experience exacerbations (Han 2014, Albert 2011). Risks number of COPD exacerbations over a 1-year period, and a
associated with long-term antibiotic therapy should be con- higher percentage of patients experienced serious cardiac
sidered, such as development of resistant organisms, as well and GI adverse effects compared with placebo (Devereux
as drug specific toxicities, such as QTc prolongation, tinni- 2020). Methylxanthines should not be used for AECOPD pre-
tus, and GI adverse effects for azithromycin. No data show vention because of unproven efficacy and the high risk of
efficacy or safety of chronic azithromycin treatment to pre- adverse effects.
vent AECOPD beyond 1 year of treatment. Most recently, the
BACE trial concluded that the addition of azithromycin (500 Vaccination
mg/day for 3 days, then 250 mg every other day for 3 months) Ensuring appropriate vaccination in high-risk populations
to standard care during an infectious AECOPD requiring hos- remains a hallmark intervention for both primary and sec-
pitalization significantly reduced treatment failure when ondary disease prevention. The CDC recommendation is that
initiated within 48 hours of admission (Vermeersch 2019). patients with or at risk of COPD receive a yearly influenza vac-
A similar approach was investigated with ciprofloxacin for cine and the pneumococcal vaccine series for all patients 65
retreatment of incompletely recovered AECOPD and failed years and older. Annual influenza vaccination reduces seri-
to show beneficial effect on exacerbation frequency or time ous illness and death in patients with COPD (GOLD level of
to next exacerbation (Ritchie 2020). At this time, prolonged evidence B). The pneumococcal vaccine series includes the
antibiotic therapy beyond treatment of AECOPD is not rec- pneumococcal conjugate vaccine (PCV13) and the pneumo-
ommended for most patients. Weak evidence supports the coccal polysaccharide vaccine (PPSV23). Although current
current recommendations to consider addition of chronic recommendations focus on a COPD population older than
macrolide therapy for patients experiencing exacerbations 65 years receiving the pneumococcal vaccine series, PPSV23
despite being optimized on triple therapy. reduces the incidence of community-acquired pneumonia in
COPD patients younger than 65 years with a predicted FEV1
Phosphodiesterase Type 4 Inhibitor Therapy less than 40% and in those with comorbidities, predominately
Roflumilast has no role in management of AECOPD but is cardiovascular (GOLD level of evidence B). In addition, PCV13
indicated as a treatment to reduce the risk of future exacerba- has demonstrated significant efficacy in reducing bacte-
tions. The addition of roflumilast may be best considered in remia and serious invasive pneumococcal disease in the
patients with severe COPD (FEV1 50% or less than predicted) general population, and because of the lack of similar data
with chronic bronchitis features and have been hospitalized in a concentrated COPD population, such results have been
for an exacerbation within the previous year (GOLD 2020). extrapolated to existing high-risk COPD patients. Therefore,
patients may elect to receive the pneumococcal series before
Vitamin D age 65 years after discussion with a health professional. If a
Vitamin D contains immune-modulating activity and has patient received any doses of PPSV23 before age 65 years,
been implicated in the pathophysiology of AECOPD. As with then 1 final dose of the vaccine should be administered at
many chronic diseases, vitamin D concentrations are lower in age 65 or older, separated by at least 5 years from the previ-
COPD patients compared with a healthy population. Studies ous PPSV23 administration. In addition, PCV13 and PPSV23
have shown that supplementation in patients with severe vaccination should be separated by at least 1 year, regardless
deficiency (less than 10 ng/mL or less than 25 nmol/L serum of a patient’s age at the time of administration.

CCSAP 2021 Book 1 • Pulmonary and Endocrinology 46 COPD, Asthma, and PAH
Nonpharmacologic Secondary Prevention from programs lasting 6–8 weeks. Pulmonary rehabilita-
Aside from tailoring pharmacologic agents for long-term tion reduces hospital readmissions and mortality in patients
COPD management and exacerbation prevention, nonphar- who have had a recent exacerbation (2 weeks or less since
macologic interventions also carry high impact in decreasing the most recent hospitalization) and decreases disease-re-
AECOPD frequency in compliant patients. lated anxiety and depression (Gordon 2019, Puhan 2016).
Pulmonary rehabilitation should be considered part of inte-
Pulmonary Rehabilitation Programs grated patient management for patients across all grades of
Pulmonary rehabilitation is an effective multidisciplinary COPD severity.
treatment strategy to improve dyspnea, exercise tolerance,
and health-related quality of life for a broad population of Smoking Cessation
patients with chronic respiratory disease, including COPD, Tobacco smoking is associated in more than 80% of diag-
pulmonary hypertension, interstitial lung disease, and pul- nosed COPD cases. As a modifiable risk factor, smoking
monary fibrosis. Participation in pulmonary rehabilitation cessation is key to reducing progressive decline in lung
allows patient-tailored therapies, such as supervised exer- function, decreasing exacerbation frequency, and minimiz-
cise training, disease state education, and self-management ing smoking-related comorbidities, including lung cancer
behavioral interventions, to optimize patients’ physical and and cardiovascular disease. Despite the health implications,
psychological conditions. Optimal benefits are achieved an estimated 40% of COPD patients continue to smoke,

Patient Care Scenario


A 64-year-old man (weight 92 kg, no known drug aller- performed. Vital signs are blood pressure 122/84 mm
gies) is admitted to the ICU because of acute hypoxic Hg, heart rate 92 beats/minute, and temperature 99.8°F
respiratory failure secondary to a COPD exacerbation. The (37.6°C). Notable laboratory values include a serum cre-
patient required intubation in the ED after failure of NIV atinine of 1.15 mg/dL (baseline of 1 mg/dL) and WBC
because of persistent hypoxia. His medical history is sig- count 13.2 × 103 cells/mm3. His SaO2 is documented as
nificant for COPD (current smoker, 30 pack-years), type 92% on the following MV settings: fraction of inspired
2 diabetes mellitus, gout, and hypertension. He was not oxygen 60%, positive end-expiratory pressure 10 cm H2O.
accompanied by family to the ICU, but chart review shows Outpatient COPD medications include umeclidinium/
prescription of a 5-day course of azithromycin 10 days vilanterol, 1 inhalation by mouth daily; albuterol, 2 inha-
before admission. This COPD exacerbation is his sec- lations by mouth every 4 hours as needed for shortness of
ond requiring hospitalization within the past 3 months. breath. What initial management strategy is best to rec-
On admission to the ICU, enteral access is obtained by ommend for this patient’s COPD exacerbation?
feeding tube placement and repeat laboratory testing is

ANSWER

• A SABA, with or without a short-acting anticholinergic, a course of antibiotics, and because of his need for MV,
is the preferred bronchodilator therapy for COPD exacer- he should be covered for Pseudomonas spp. when select-
bations; it is reasonable to initiate albuterol/ipratropium ing empiric therapy, in addition to the other most com-
2.5–0.5 mg/3 mL of inhalation solution by nebulization mon COPD exacerbation pathogens (S. pneumoniae, H.
every 4 hours. influenzae, and M. catarrhalis). Therefore, it is reasonable
• Corticosteroids: It is reasonable to initiate prednisone to consider cefepime 2 g intravenously every 8 hours (in
40 mg orally by nasogastric tube with consideration to consideration of empiric coverage for institutional Pseu-
complete therapy after 5 days. If the feeding tube is not domonas susceptibility) for 5–7 days, pending clinical im-
cleared to use, then equivalent doses of intravenous hy- provement.
drocortisone or methylprednisolone may be administered. • COPD exacerbation prevention: Although not an initial in-
No consistent evidence suggests that the higher doses or tervention, special consideration should be made to ensure
longer duration of corticosteroids is beneficial for man- smoking cessation resources, pulmonary rehabilitation
agement of a COPD exacerbation. referrals, and appropriate vaccines are discussed before
• Antibiotics: Because the patient is intubated, noninvasive discharge. Because of the patient’s frequent exacerbation
respiratory cultures should be acquired and antibiotic history, he qualifies for escalation to triple therapy (LABA/
therapy should be ordered. The patient recently completed LAMA/ICS) for his maintenance inhaler regimen.

1. Global Initiative for Chronic Obstructive Lung Disease (GOLD). 2020 GOLD Reports: Global Strategy for the Diagnosis, Management and
Prevention of COPD. Fontana, WI: GOLD, 2020. Available at https://goldcopd.org/gold-reports/. Accessed 2020.
2. Leuppi JD, Schuetz P, Bingisser R, et al. Short-term vs conventional glucocorticoid therapy in acute exacerbations of chronic obstruc-
tive pulmonary disease: the REDUCE randomized clinical trial. JAMA 2013;309:2223-31.
3. Moussaoui R, Roede B, Speelman P, et al. Short-course antibiotic treatment in acute exacerbations of chronic bronchitis and COPD: a
meta-analysis of double-blind studies. Thorax 2018;63:415-22.

CCSAP 2021 Book 1 • Pulmonary and Endocrinology 47 COPD, Asthma, and PAH
emphasizing the paramount importance of initiating effective factor) cause epithelial damage and mucus plugging, which
smoking cessation interventions for this population (Rabe subsequently increases endothelial permeability and causes
2017). Smoking cessation is deemed essential intervention acute airway edema.
for all patients with COPD in Groups A–D (GOLD 2020).
Diagnosis and Classification
Official diagnosis of status asthmaticus relies primar-
STATUS ASTHMATICUS
ily on a patient’s persistent clinical deterioration despite
An asthma exacerbation, or otherwise termed acute severe administration of bronchodilator therapy. Classification of
asthma, is defined as acute or subacute changes in a patient’s asthma exacerbation severity is listed in Table 2. Factors
respiratory symptoms and lung function, commonly con- associated with increased likelihood of need for hospi-
sisting of worsening shortness of breath, cough, wheezing, tal admission include female sex, 8 or more treatments of
and chest tightness, or a combination of these manifesta- SABA therapy within the previous 24 hours, need for rapid
tions (GINA 2020). Acute decreases in expiratory airflow are medical intervention on arrival, RR greater than 22 breaths/
often responsible for worsening respiratory symptoms, and minute, FEV 1 50% or less than predicted, history of severe
changes in therapy are required to address them. exacerbations, or frequent unscheduled ED visits for
Asthma exacerbation episodes are often classified as asthma management.
either slow onset and sudden onset. Exacerbations that are
slow onset, or subacute, exhibit a progressive worsening of Pharmacologic Treatment
peak expiratory flow rate over days and are mostly associ- The goals of acute asthma management are to relieve bron-
ated with patient-related factors, such as inadequate inhaler chospasm, improve gas exchange, identify and treat the
technique, suboptimal compliance to ICS regimen, or a psy- underlying cause, and prevent complications. Depending
chological stressor. Conversely, acute onset exacerbations on the presenting severity, inhaled β-agonist therapy, sys-
present with severe deterioration within hours and often cor- temic steroids, and oxygen therapy are the initial treatment
relate with sudden massive exposure to external triggers, interventions for an acute asthma exacerbation (GINA 2020).
such as predisposed allergens, food articles, or sulfites. Other Adjunctive agents, including systemic β-agonists, magne-
common exacerbation triggers include viral respiratory infec- sium, and ketamine, may offer the largest benefit to patients
tions or seasonal changes. About 80% of asthma fatalities with severe or life-threatening symptoms whose symptoms
are classified as a slow-onset asthma exacerbation, perhaps are refractory to initial therapy.
reflecting inadequate disease control over time. Additional
factors that increase risk of asthma-related death include Inhaled Bronchodilator Agents
a previous asthma exacerbation requiring MV, hospitaliza- Bronchodilator agents, specifically SABAs and anticho-
tion for asthma management within the past year, recent linergics, are the first-line inhaled therapy for patients
oral corticosteroid use, overuse of prescribed SABA therapy, presenting with status asthmaticus. The SABAs work
poor adherence to asthma medications, and food allergies immediately to reduce bronchoconstriction and airflow
(GINA 2020). These patients should be identified early in their obstruction. Continuous SABA nebulization (1 nebuliza-
course of an exacerbation and prompted to seek urgent med- tion every 15 minutes or greater than 4 nebulizations/hour)
ical care. Acute management of patients presenting with an may be indicated for patients experiencing persistent air-
asthma exacerbation include immediate administration of flow obstruction despite aggressive treatment. Systematic
bronchodilator therapy. If a patient does not respond within reviews of intermittent versus continuous nebulized SABAs
1 hour of inhaled bronchodilators, the diagnostic criteria are in acute asthma offer conflicting results regarding hospital-
met for status asthmaticus. ization rates and responsive lung function between delivery
Status asthmaticus is considered a medical emergency methods (Camargo 2003). A reasonable approach to inhaled
requiring immediate, intensive treatment. Status asthmat- SABA therapy in exacerbation is to initially use continuous
icus is often characterized by hypoxemia, hypercarbia, and therapy, followed by intermittent on-demand therapy for hos-
secondary respiratory failure. All patients with asthma are at pitalized patients. An inhaled short-acting anticholinergic
risk of developing an acute episode with progressive severity (e.g., ipratropium bromide) should be added to SABA therapy
that is poorly responsive to standard therapeutic measures, for all patients presenting with acute asthma requiring ED
regardless of baseline disease severity or phenotypic variant. management or hospitalization. Specifically, for adults with
Physiologically, status asthmaticus consists of premature moderate to severe exacerbations, treatment in the ED with
airway closure during exhalation, which leads to severe air both SABA and ipratropium was associated with fewer hospi-
trapping and increased airflow resistance, both of which can talizations and greater improvement in peak expiratory flow
exacerbate ventilation/perfusion mismatch and hypoxemia. and FEV1 compared with SABA monotherapy (Kirkland 2017).
In addition, infiltrative proinflammatory cells and mediators Typical dosing strategies for initial bronchodilator therapy are
(e.g., eosinophils, neutrophils, mast cells, tumor necrosis listed in Table 3.

CCSAP 2021 Book 1 • Pulmonary and Endocrinology 48 COPD, Asthma, and PAH
Table 2. Severity Classification of Acute Asthma Exacerbations

Severity Clinical Signs and Symptoms Initial Peak Expiratory Flow Clinical Course

Mild or Talks in phrases FEV1 >50% predicted or personal SABA + ipratropium + oral
Moderate Prefers sitting rather than lying flat best corticosteroids

HR 100–120 beats/minute
SaO2 90%–95%

Severe Talks in singular words FEV1 ≤50% predicted or personal SABA + ipratropium + intravenous
Sits hunched forwards best or oral corticosteroids +/–
adjunctive therapies
Drowsiness or confusion
Respiratory support
Silent chest
RR >30 breaths/min, use of
respiratory accessory muscles
HR >120 beats/minute
SaO2 <90%

Status Lack of clinical response within 1 hr of bronchodilator therapy (SABA


asthmaticus +/– ipratropium bromide)

+ = with; – = without; FEV1 = forced expiratory volume in 1 second ; HR = heart rate;; RR = respiratory rate; SABA = short-acting β2
agonist.
Information from: Global Initiative for Asthma (GINA). 2020 GINA Main Report: 2020 GINA Report, Global Strategy for Asthma
Management and Prevention.

Table 3. Bronchodilator Therapy for Status Asthmaticus

Mechanism of
Drug Available Strength Action Adult Dose

Albuterol MDI 90 mcg/puff SABA 4–8 puffs every 20 min up to 4 hr, then
(aerosol) every 1–4 hr as needed

Albuterol sulfate 0.63 mg/3 mL SABA 2.5–5 mg every 20 min for 3 doses, then
(nebulization solution) 1.25 mg/3 mL 2.5–10 mg every 1–4 hr as needed

2.5 mg/3 mL (0.083%) OR

2.5 mg/0.5 mL (0.5%) 10–15 mg/hr continuously

Ipratropium bromide MDI 17 mcg/puff Anticholinergic 8 puffs every 20 min as needed up to 3 hr

Ipratropium bromide 0.5 mg/2.5mL (0.02%) Anticholinergic 0.5 mg every 20 min for 3 doses, then as
(nebulization solution) needed

Ipratropium with albuterol Each 3 mL vial contains 0.5 mg SABA/ 3 mL every 20 min for 3 doses, then as
(nebulization solution) ipratropium bromide and 2.5 mg anticholinergic needed
albuterol

MDI = metered-dose inhaler; SABA = short-acting β2 agonist.


Information from: Camargo C, Rachelefsky G, Schatz M. Managing asthma exacerbations in the emergency department: summary
of the national asthma education and prevention program expert panel report 3 guidelines for the management of asthma
exacerbations. J Emerg Med 2009;37:S6-17.

CCSAP 2021 Book 1 • Pulmonary and Endocrinology 49 COPD, Asthma, and PAH
Systemic Corticosteroids admission is in its use for patients with a FEV1 <25%–30%
Corticosteroids are considered a hallmark intervention predicted at presentation (GINA level of evidence A) or adults
for acute asthma and have demonstrated distinct benefit. who experience a failure of response to initial treatment who
Mechanistically, corticosteroids reduce airway inflammation have persistent hypoxemia (GINA level of evidence B).
and mucus production; these agents have also been pro-
posed to potentiate β-agonist activity on smooth muscles Ketamine
and reduce β-agonists tachyphylaxis in patients with severe Ketamine is known to lower airway resistance and increase
asthma. Corticosteroids hasten the resolution of an asthma lung compliance in the asthmatic patients. The bronchodila-
exacerbation and should be administered as soon as possi- tion effects of ketamine are thought to be promoted directly
ble, ideally within 1 hour of presentation. A meta-analysis of by the bronchial smooth muscles and indirectly by the release
30 randomized controlled trials concluded that the use of cor- of endogenous catecholamines, stimulation of the sym-
ticosteroids in the ED significantly reduces rates of hospital pathetic nervous system, and inhibition of catecholamine
admission and number of future relapses in the subsequent reuptake. The evidence supporting ketamine to manage
7–10 days for patients with severe asthma (Rowe 2001). Both asthma exacerbations in adult patients suggest that its use
oral and intravenous formulations are deemed acceptable should be reserved only for patients with asthma whose
and equivalent in treating acute asthma. Parenteral cortico- respiratory failure does not respond to initial standard ther-
steroids should be considered first-line therapy in critically apy. A review of ketamine use in adult patients with severe
ill patients with questionable enteral integrity, with impaired respiratory distress found clinical improvements, decreased
gastric absorption, or who are too dyspneic or altered for wheezing sounds, and improved SaO2 after administration.
safe oral administration. Different corticosteroid dosing reg- Furthermore, patients who were intubated showed improved
imens have been investigated for use in acute asthma. A peak inspiratory pressures, dynamic compliance of lungs, gas
dose of prednisolone at 1 mg/kg/day or equivalent, up to 50 exchange, and decreased oxygen requirements associated
mg/day, is currently supported by GINA (GINA level of evi- with ketamine (Goyal 2013). Its concomitant analgesic and
dence B). In addition, Littenberg and Gluck demonstrated a dissociative effects further promote its role as an adjunctive
significant reduction in hospitalization with a methylpred- agent in refractory disease, especially in agitated or mechan-
nisolone dose of 125 mg intravenously on presentation in ically ventilated patients requiring sedation. Optimal dosing
the ED, a common clinical practice for acute management has not been established; however, bolus doses of 0.1–2
of presenting patients with a known history of asthma exac- mg/kg and continuous infusion rates of 0.15–2.5 mg/kg/hour
erbations (Littenberg 1986). Oral corticosteroids should be have been described in the available literature (Goyal 2013).
continued for a 5- to 7-day course, which has been found to be Adverse effects of dysphoria, hallucinations, and increased
as effective as a 10- to 14-day course for acute asthma man- secretions were also noted in trials reviewed. Further well-de-
agement (Jones 2002). Replacing systemic corticosteroids signed studies are warranted to define the exact role of
with increased doses of ICSs is generally not recommended ketamine in therapy; however, the current lack of evidence
for status asthmaticus. This approach can be used if asthma coincides with the GINA guidelines making no recommenda-
exacerbation symptoms are discovered earlier and if the tion for or mention of ketamine and its use in treatment of
patient is stable enough for outpatient, observational treat- adult acute asthma.
ment using written action plans from an overseeing provider.
Evidence from studies in which all patients were taking main- Epinephrine and Terbutaline
tenance ICS after discharge suggests no benefit in tapering Epinephrine and terbutaline exhibit β2-agonist activity and
dose of oral corticosteroids, either short-term or over several have been proposed as adjunctive treatment for status asth-
weeks. matics in adults. As systemic therapy, use of these agents has
typically been avoided because of their adverse effect profiles
Magnesium and lack of evidence proving their superiority to inhaled SABA
Intravenous magnesium sulfate has been a useful adjunct therapy. Intramuscular epinephrine (0.3 mg) is indicated in
in patients with acute status asthmaticus who are refrac- addition to standard therapy for acute asthma associated
tory to β-agonists because of its bronchodilation effects with anaphylaxis or angioedema. Beyond anaphylaxis and
mediated by calcium antagonism and the counteraction of angioedema, subcutaneous epinephrine or terbutaline may be
calcium-mediated smooth muscle contraction. At the com- considered for severe asthma exacerbations if a SABA is not
monly prescribed dose of 2 g administered intravenously over available or if the patient does not respond to initial inhaled
20 minutes, adverse effects of hypotension or hyporeflexia therapy after several hours. Recommended adult dosing for
are uncommon. Because of the lack of robust and consistent epinephrine 1 mg/mL is 0.3–0.5 mg subcutaneously every
benefit seen in small clinical trials, intravenous magnesium 20 minutes for 3 doses or terbutaline 1 mg/mL 0.25 mg sub-
sulfate is not recommended for routine use in all asthma exac- cutaneously every 20 minutes for 3 doses. Neither agent has
erbations; however, the best support for reducing hospital been proven to be better than inhaled SABA therapy, and both

CCSAP 2021 Book 1 • Pulmonary and Endocrinology 50 COPD, Asthma, and PAH
are contraindicated in patients with coronary artery disease PULMONARY ARTERIAL
(National Asthma Education and Prevention Program 2007). HYPERTENSION

Other Adjunctive Agents Pulmonary hypertension is defined by a mean pulmo-


nary artery pressure greater than 20 mm Hg at rest. It is
Intravenous methylxanthines (e.g., aminophylline, theoph-
a chronic condition that is associated with high morbidity
ylline), mucolytics (e.g., acetylcysteine), and leukotriene
and mortality. The etiology of and risks of PH are vari-
receptor antagonists (e.g., montelukast, zafirlukast) have no
able. Pulmonary hypertension is classified into five groups
role in management of status asthmaticus, and use should
based on hemodynamics, underlying pathophysiology, risk,
be withheld or avoided. Furthermore, antibiotics should only
and therapy options to guide clinicians with diagnosis and
be considered based on evidence of concurrent infection,
treatment (Online Appendix) as follows: (1) pulmonary arte-
and these drugs are generally not recommended for acute
rial hypertension; (2) PH associated with left-sided heart
asthma exacerbations.
disease; (3) PH secondary to chronic hypoxemic conditions
Oxygen Therapy Respiratory Support
such as interstitial lung disease or COPD; (4) chronic throm-
boembolic PH; and (5) multifactorial PH. Current approved
Patients with severe asthma have a ventilation/perfusion
medication therapies for PAH target specific pathophysio-
mismatch and those presenting with acute asthma often ben-
logic pathways to improve treatment outcomes. Evidence
efit from supplemental oxygen therapy. The goal for SaO2 is
supports the use of some of these same treatments in
93%–95% or greater for adults. An estimated 3%–16% of hos-
Group 4; however, improvement of outcomes in Groups 2, 3,
pitalized adult patients with asthma progress to respiratory
and 5 rely on treatment of underlying disease states
failure requiring ventilatory support. Permissive hypercap-
(Parikh 2019).
nia is the recommended ventilator strategy for patients with
Hospitals and health systems that care for patients with
asthma because it provides adequate oxygenation and
PAH face specific challenges related to management of com-
ventilation while minimizing high airway pressures and
plex, targeted, medication therapy. Departments of pharmacy
subsequent barotrauma (National Asthma Education and
in centers that care for these patients regularly must ensure
Prevention Program 2007).
that patients have continued access to medication therapy
The evidence regarding the role of NIV in severe asthma is
while in the hospital. Policies, procedures, and/or guidelines
weaker than that for COPD. A systematic review of five stud-
must be in place at the institutional level, including education
ies assessed the effectiveness of NIV in patients with acute
and competency of interdisciplinary health care providers, to
asthma. Results were conflicting, with conclusions from
ensure safe and effective use of these medications within the
two studies stating no difference in the need for endotra-
acute care setting (Smith 2019). In hospitals or health sys-
cheal intubation in NIV versus placebo; however, one study
tems without these capabilities, consideration should be
did identify fewer hospital admissions in the NIV group (Lim
given to transfer critically ill patients with PAH to an expert
2012). Because of the small sample sizes and inconsistent
center (Hoeper 2019). This section will review the process for
study designs, no firm recommendations are offered for the
medication use and targeted management of PAH (Group 1)
role of NIV in status asthmaticus. However, NIV should not
in a critically ill population.
be attempted in agitated patients, and patients should not
Diagnosis of PAH requires that mPAP be measured during
receive sedation to facilitate NIV (GINA level of evidence D).
RHC (Online Appendix). Historically, to meet diagnostic crite-
Secondary Prevention
ria for PAH, the threshold mPAP has been considered to be
greater than 25 mm Hg. To exclude elevation of mPAP second-
As patients recover for an asthma exacerbation and near dis-
ary to left-sided heart disease, the pulmonary capillary wedge
charge, assessment of maintenance inhaler therapy should
pressure should be less than 15 mm Hg and PVR more than 3
be completed. All patients should be prescribed regular,
Woods units (Simonneau 2019, Galiè 2016 ). Other diagnostic
ongoing ICS-containing treatment in addition to rescue SABA
tests such as ECHO, CT scan, pulmonary function tests, and
therapy after an exacerbation (GINA 2020). Monotherapy
ventilation/perfusion scan are typically used to support the
with a SABA is no longer recommended for persistent
diagnosis and rule out other etiologies of PH.
asthma management, and ICS therapy significantly reduces
Despite increased screening based on chronic risk factors,
the risk of asthma-related death or hospitalization. Patients
newly diagnosed PAH in a critically ill patient can commonly
should be referred for asthma care follow up within 1–4
present as new-onset RV failure. Just as commonly, patients
weeks of discharge, educated regarding inhaler techniques,
with known PAH may decompensate because of volume over-
and reminded to avoid known allergens or irritants. Patients
load or other comorbid factors and require ICU admission and
with asthma should also receive an annual influenza vac-
interventions for underlying PAH.
cine; however, there is insufficient evidence to recommend
Decompensated PAH is merely one potential cause of
routine pneumococcal vaccination in all patients (GINA level
RV failure in ICU patients and should be differentiated from
of evidence D).

CCSAP 2021 Book 1 • Pulmonary and Endocrinology 51 COPD, Asthma, and PAH
Table 4. Etiology of Right Ventricular Failure

Hemodynamics
Cause CO CVP PVR Intervention

Volume depletion Decreased Decreased Decreased or unchanged Volume repletion

Worsening underlying PAH Decreased Increased Increased PAH-targeted


pharmacotherapy

Right ventricular dysfunction Decreased Increased Unchanged Inotropes

Vasodilation Increased or unchanged Increased or Increased or unchanged Vasopressors


unchanged

Volume overload Increased or unchanged Increased Increased or unchanged Diuretics

Compensated/chronic Unchanged Decreased Unchanged No specific therapy

CO = cardiac output; CVP = central venous pressure; PAH = pulmonary arterial hypertension; PVR = pulmonary vascular resistance.
Information from: Jentzer JC, Mathier MA. Pulmonary hypertension in the intensive care unit. J Intensive Care Med 2016;31:369-85.

other causes (Table 4). Noninvasive ECHO is an important ini- This monitoring may help guide diuresis and initiation of
tial screening tool in critically ill patients when RV failure is PAH-target therapies, both of which may positively impact
suspected. When PAH is the underlying etiology of RV fail- mortality independent of the impact of those interventions on
ure, typical findings include RV dilation, elevated estimated pulmonary hemodynamics (Huynh 2012). A pulmonary artery
systolic pulmonary artery pressure, and tricuspid regurgita- catheter can be used to measure the response to medical
tion. Acutely decompensated RV failure secondary to PAH interventions for RV failure, including measures of pressures
is caused by elevations in pulmonary artery pressure from in the right and left atrium, cardiac output, mixed venous
increased PVR or pulmonary venous pressures. The RV is SaO2, and PVR (Granton 2011).
unable to maintain normal cardiac output in the setting of ele- Non-targeted interventions in critically ill patients with PAH
vated right atrial pressure. Elevated RV pressure and reduced and RV failure include oxygenation, volume management,
RV output affects forward flow, and this effect in addition to vasopressor support, and anticoagulation. Supplemental
the compensatory tachycardia lead to decreased right cor- oxygen therapy increases SaO2and decreases mPAP and
onary perfusion and RV ischemia. This can be complicated PVR in patients with PAH (Roberts 2001). In some cases,
by and worsen coexisting hypotension and/or hypoxemia both noninvasive and invasive ventilatory support may be
(Jentzer 2016). Because the pathophysiology is distinct, required. Optimization of volume status in patients with PAH
RHC is required for formal diagnosis before initiation of PAH- is critical for RV function given its dependence on preload.
targeted pharmacotherapy. If central venous pressures and clinical indications support
With treatment advances, PAH mortality has improved. that fluid administration may improve cardiac output, large
However, the 5-year overall survival remains 60%–65% (27.2% volumes and rapid infusion rates should be avoided because
for those with functional class IV symptoms), and ICU mortal- this approach can worsen a decompensated RV. Smaller vol-
ity remains high, ranging from 26%–45% (6–24 months after umes (i.e., up to 250 mL) can be considered (Jentzer 2016). In
discharge) (Tejwani 2018, Farber 2015). Several risk factors a state of volume overload and RV failure, intravenous loop
have been associated with an increased mortality in patients diuretics should be used to restore hemodynamics and asso-
with PAH in the ICU, including treatment with prostacyclin ciated hypoxemia.
before admission, need for MV, or renal replacement therapy. In low cardiac output states, particularly when patients
Notably, those with a higher Acute Physiology and Chronic are hypotensive, inotropes such as dobutamine may be used
Health Evaluation (APACHE II) score and patients with Group to increase cardiac output and decrease PVR. At doses in
3 (chronic hypoxemic) disease also had increased risk of ICU- excess of 5 mcg/kg/minute, dobutamine may worsen PAH
and 6-month mortality (Rush 2017, Huynh 2012). Advanced by causing tachycardia or arrhythmia and decreasing dia-
age and poor baseline pulmonary hemodynamics may also stolic filling time. Milrinone also increases contractility of
predict mortality after admission to the ICU (Tejwani 2018). the myocardium and can be considered in low cardiac output
Early initiation (within 3 days of ICU admission) of invasive states, but it can cause arrhythmias and worsen hypoten-
hemodynamic monitoring by a pulmonary artery catheter sion because of its effect on systemic vascular resistance.
has been linked to a decrease in mortality in ICU patients. Norepinephrine is the preferred vasopressor when patients

CCSAP 2021 Book 1 • Pulmonary and Endocrinology 52 COPD, Asthma, and PAH
with RV failure require additional pressure support because exercise tolerance), PAH risk category (Online Appendix), and
of hypotension. Vasopressin can be considered either as patient readiness. In the setting of PAH and acute RV failure
adjunct or as a vasopressor-sparing agent; however, limited in need of rescue therapy, there are important considerations
information is available outside of experimental models and that should be evaluated when initiating specific agents.
this agent may cause vasoconstriction within the pulmonary
vasculature and decrease RV contractility, which can worsen Parenteral PAH Medications
underlying PH (Mishra 2016, Granton 2011, Leather 2002). Exogenous parenteral prostacyclins are indicated to replace
Microvascular thromboses are inherent to PAH pathophys- or supplement the decreased production of endogenous
iology; however, this is of minimal benefit as a medication prostacyclins in patients with PAH. Prostacyclins provide a
target (Ascha 2017, Preston 2014). Patients who are on chronic benefit by causing direct vasodilation of pulmonary arterial
anticoagulation in the outpatient setting should have ther- vascular beds and inhibition of platelet aggregation. Early
apy continued in the ICU. Based on factors of critical illness, initiation of intravenous prostacyclin is recommended in
modality of anticoagulation may need to be altered. In patients patients with newly diagnosed PAH and RV failure. In criti-
who were not on anticoagulation before admission, initiation cally ill patients with PAH already on a prostacyclin, dose
of empiric anticoagulation can be considered in absence of should be optimized. Intravenous epoprostenol is preferred
other indications (e.g., comorbid pulmonary embolism) for for RV rescue in prostacyclin-naive patients. A typical start-
idiopathic, heritable, and anorexigen-associated PAH. The ing dose of intravenous epoprostenol is 1 to 2 ng/kg/minute.
evidence on which this recommendation is based is limited Because of its short half-life (6 minutes) it can be quickly
(Galiè 2016, Olsson 2014) and the decision may be deferred increased by 0.5–1 ng/kg/minute every 30 minutes. However,
to the outpatient setting during follow-up care with an expert in patients who are less stable or require vasopressor sup-
provider in the management of PAH. The decision to initiate port, a more reasonable frequency of every 8–12 hours can
anticoagulation must be weighed against individual risks, be followed. Infusion rates are titrated to highest tolerated
including effects of prostacyclin therapy on platelet aggre- dose for optimal effect. In general, a reasonable in-hospital
gation. Historically, patients with PAH who were given oral initial goal dose is 6–10 ng/kg/minute with additional outpa-
anticoagulation received warfarin. Direct-acting oral antico- tient titration to a maintenance dose of 20–25 ng/kg/minute
agulants have not specifically been studied in either PAH or (Lombardi 2018).
chronic thromboembolic PH, but their well-known advantages Intravenous prostacyclins should optimally be infused
over warfarin make them an appealing therapeutic option for through a single-lumen central catheter to reduce risk of
this indication. Large-scale, prospective data to support use medication error (including therapy interruption because of
of any specific agent for anticoagulation in PAH are lacking. loss of intravenous access, co-infusion of incompatible med-
ications, and accidental bolus administration when flushing
the line), although emergent, temporary infusion through a
PAH-TARGETED THERAPEUTIC
peripheral intravenous is permissible. Consideration should
OPTIONS AND DRUG
be given to changing patients on subcutaneous trepros-
ADMINISTRATION tinil who are critically ill and/or in RV failure to intravenous
Before market approval of intravenous epoprostenol, man- administration because hypoperfusion in the setting of crit-
agement of PAH primarily consisted of supportive treatment ical illness may limit subcutaneous absorption. In addition,
including diuretics, digoxin, anticoagulation, and supplemen- a critically ill patient may not be able to independently exe-
tal oxygen. In patients with positive vasoreactivity testing cute all functions needed to safely operate a home infusion
during RHC, calcium channel blockers were used to improve device. Transition from subcutaneous to intravenous trepros-
mPAP, PVR, and symptoms of right heart failure (Rich 1987). tinil can occur without changes to dose (ng/kg/minute) and
These agents are still considered first-line therapy for patients can generally occur with no or minimal overlap between the
who respond to vasoreactivity testing during RHC (Galiè two methods of infusion (Torbic 2019).
2016). Intravenous epoprostenol was the first PAH-targeted All prostacyclin infusions should be administered using
therapy and is the only medication with proven impact on a patient-specific dosing weight. For patients who present
mortality (Barst 1996). Since its market approval in 1995, the with critical illness and are already on prostacyclin therapy,
survival rate has increased and additional PAH-targeted phar- all dosing parameters, including the dosing weight, should
macotherapies have been developed (Farber 2015). Several be verified with the patient’s specialty pharmacy. For new ini-
oral and parenteral agents exist to treat PAH within three tiations, a dosing weight should be chosen for the patient.
pathways that aim to restore the balance of vasodilation and Dosing weight should remain consistent to avoid resultant
vasoconstriction within the pulmonary vasculature—the nitric changes to prostacyclin dose in ng/minute. With careful con-
oxide, endothelin, and prostacyclin pathways. In a controlled sideration, long-term changes to dosing weight can occur
setting, once a diagnosis of PAH is confirmed by RHC, medica- with direct order from a provider with experience in man-
tions are initiated based on symptoms (WHO functional class, agement of these infusions. All weight changes should be

CCSAP 2021 Book 1 • Pulmonary and Endocrinology 53 COPD, Asthma, and PAH
reviewed for clinical appropriateness by a pharmacist and Because outpatient approval and management of paren-
communicated to the outpatient specialty pharmacy before teral prostacyclins is a complicated process, it is important
the hospital discharge. to ensure that therapy is initiated only when indicated based
Monitoring of the response to parenteral prostacyclin in on specific diagnosis of PAH. Use of parenteral prostacyclins
critically ill patients must also account for clinical worsen- in other groups of PH is not only of limited clinical benefit,
ing. In prostacyclin-naive patients, new onset hypoxemia or but also can cause significant complications. Patients who
pulmonary edema could indicate underlying left heart dis- are stabilized and ready to transition to home infusion ther-
ease or pulmonary veno-occlusive disease (PVOD). PVOD and apy will require management by a specialist provider who is
PAH are not mutually exclusive; rather, PVOD is a subgroup able to monitor this complex therapy long-term. In addition,
(Group 1.6) of PAH characterized by remodeling of pulmonary private and government insurance plans have strict criteria
venules (Simmoneau 2019). Its presentation closely resem- for payment based on diagnosis and clinical indicators. Even
bles PAH, and PVOD may account for up to 20% of cases when insurance plans approve payment, the cost to patients
initially diagnosed as PAH. If introduction and titration of may be significant and they require additional assistance in
exogenous prostacyclin causes worsening hypoxemia and/ the form of private grants, foundation assistance, or other
or pulmonary edema, PVOD may be suspected, especially means. Because of these concerns, careful multidisciplinary
with a potent parenteral vasodilator such as epoprostenol. consideration—including identification of an appropriate pro-
Although prostacyclin therapy can benefit some patients with vider to take responsibility of long-term management—should
PVOD, overwhelming vasodilation on the arterial side in the be given to initiation of these agents. Once the RV has been
setting of stiff and remodeled venous vasculature can cause stabilized, patients who are not ideal candidates for infusion
life-threatening pulmonary edema. Dose reduction or discon- device self-management, or those who choose not to remain
tinuation of prostacyclin therapy may be warranted if PVOD on a parenteral agent, may need to be transitioned to alter-
is suspected based on underlying disease pathophysiology native oral therapies before being discharged from the acute
(Montani 2009). care setting (Martiosov 2020).
In patients on ambulatory prostacyclin infusions who
present with RV failure, it is important to be mindful of Inhaled PAH Medications
underlying pathophysiology driving the acute decompensa- Inhaled vasodilators increase blood flow within to pulmo-
tion. Hypotension and hypoxemia should be managed using nary arteries to help improve ventilation and hypoxemia
supplemental oxygen, MV, vasodilators, inotropes, or vaso- without causing systemic hypotension. Inhaled iloprost and
pressors. Chronic pathophysiologic changes associated with treprostinil may be used as long-term treatment for patients
PAH necessitate continuation of medications targeting the with WHO functional class III or IV symptoms (Galiè 2016).
prostacyclin pathway, although these agents may contrib- Newly approved oral agents have limited evidence support-
ute to systemic vasodilation and hypotension. Prostacyclins ing their use in patients with functional class IV symptoms,
are designated as high-alert medications by the Institute and inhaled prostacyclins remain an option for use in patients
for Safe Medication Practices (ISMP). Abrupt decreases in who were unable to continue parenteral prostacyclin ther-
dose or complete withdrawal of these life-sustaining thera- apy (McLaughlin 2010, Olschewski 2002). Pulsed inhaled
pies can lead to rebound PH and significant disease-related nitric oxide improves oxygenation and PH associated with
morbidity and mortality. Ambulatory infusion pumps are interstitial lung disease, but no current data support its use
maintained by patients and family members in the outpatient long-term for PAH (Nathan 2020).
setting with ongoing education and support from specialty Use of inhaled treprostinil and iloprost is limited in the
pharmacies. In turn, a patient’s specialty pharmacy can be setting of critical illness because specialized outpatient neb-
contacted to verify a patient’s parenteral prostacyclin dos- ulizer devices are required for administration. Other inhaled
ing information and serve as an invaluable resource in the pulmonary vasodilators can be used temporarily in acute RV
absence of an on-site health care professional who is pro- failure when ability to optimize doses of PAH targeted ther-
ficient in use of these devices. Patients may present to the apy is limited by systemic mean arterial pressure despite
hospital emergently because of abrupt withdrawal of pros- addition of supportive agents such as dobutamine or nor-
tacyclin therapy because of infusion device malfunction epinephrine. Historically, inhaled nitric oxide was the agent
or unexpected lapse in medication supply. Key differences of choice, but more recent data (and a favorable cost profile)
exist in the pharmacokinetics of the two available parenteral have supported off-label use of intravenous epoprostenol in
prostacyclins and also in the logistics of management of the nebulized form administered by a MV circuit (Buckley 2011).
different ambulatory infusion devices (Table 5). Resumption Placement of a pulmonary artery catheter helps monitor the
of prostacyclin infusion is key to avoid PAH complications, patient’s response to initiation, dose titration, and weaning/
and a pharmacist is well-positioned as part of the multidis- discontinuation. Hemodynamic response includes decrease
ciplinary team to assist with management of this medical in mPAP, improvement in cardiac output, and decrease in cen-
emergency (Smith 2019). tral venous pressure.

CCSAP 2021 Book 1 • Pulmonary and Endocrinology 54 COPD, Asthma, and PAH
Table 5. Key Differences Between Parenteral Prostacyclin Products

Product Feature Epoprostenola Treprostinila

Brand name Flolan Veletri Remodulin

Route of Continuous IV infusion Continuous IV infusion,


administration Continuous SC infusion

Ambulatory CADD-Legacy1 CADD-Legacy1 (IV)


administration CADD-MS3 (SC)
devices

Compatible diluents Sterile diluent for Normal saline, IV: normal saline, sterile water for infusion, sterile diluent
Flolan, pH 12 sterile sterile water for Flolan, pH 12 sterile diluent for Flolan, sterile diluent for
diluent for Flolan for infusion Remodulin
SC administration as an undiluted solution

Special precautions Temperature unstableb, Light unstable Light unstable


light unstable

Administration Infuse through 0.22 micron in-line filter Infuse through 0.2 or 0.22 in-line micron filter

Pharmacokinetics Half-life 6 min Half-life 4 hr


Rapid plasma hydrolysis Hepatic metabolism by CYP2C8

Clinical end points Improved mortality in patient with PAH Improved exercise capacity in patient with PAH and WHO
and WHO functional class III or IV functional class II-IV symptoms; reduced rate of clinical
symptoms worsening in patients who required transition from epoprostenol

a
Generic formulation(s) also available.
b
Varies based on formulation. Prescribing information should be referenced for specific product.
IV = intravenous; PAH = pulmonary artery hypertension; SC = subcutaneous.

Limitations of inhaled pulmonary vasodilators include a A patient in RV failure may require temporary rescue ther-
short half-life, the requirement for specialized equipment, apy with a parenteral prostacyclin for a short time to restore
frequent dosing, and trained personnel for administration, pulmonary hemodynamics. It is important in a decompen-
although these factors vary by the specific therapy used. sated patient with PAH to continue targeted therapy despite
Hospitals must ensure that appropriate distributive and complications of critical illness or RV failure because these
administration protocols are in place to ensure safe medica- medications are important to reduce RV afterload. This strat-
tion use. egy becomes challenging, especially for providers who are
less familiar with pharmacokinetics and pharmacology of
Oral PAH Medications PAH-targeted medications (Table 6). Management strate-
In recent years, several new medications with oral routes gies include off-setting systemic vasodilation or reduced
of administration have been approved for use in PAH. Now cardiac output with vasoactive agents or inotropes, as pre-
oral medication options are available in each of the targeted viously mentioned. Although consideration may be given to
pathways and patients with moderate disease (WHO func- slight reductions in doses of PAH-targeted medications, it
tional class II or III) have additional medication options to is generally recommended that these agents be continued
improve disease-related outcomes. Long-term management throughout the ICU stay to avoid causing rebound PH and
of patients with advanced disease (WHO functional class worsening RV failure. Other confounders of critical illness
IV) often includes one or two oral agents from each of the may pose challenges for administration of certain oral med-
medication pathways in addition to an ambulatory parenteral ications. Whereas some classes have available alternative
prostacyclin infusion (McLaughlin 2019, Galiè 2016). methods of enteral administration (e.g., crushed adminis-
It is not uncommon that patients who have been main- tration through enteral tubes), others will require transition
tained on one or more oral agents to encounter significant to parenteral or inhaled therapy (Torbic 2019). In patients
acute disease progression, and hospitals that care for these for whom non-adherence to oral therapy is a cause of acute
patients should develop guidance for conversion to alter- decompensated PAH and RV failure, steps should be taken
native parenteral therapies when necessary (Smith 2019). to safely resume prescribed medications to optimize out-
comes (Narechania 2020). In treatment-naive patients who

CCSAP 2021 Book 1 • Pulmonary and Endocrinology 55 COPD, Asthma, and PAH
Table 6. Oral Medications for Treatment of Pulmonary Arterial Hypertension

Medication
distribution
Pharmacology Half-life elimination Administration Notable drug interactions requirements

PDE5 Inhibitor

Sildenafil 4 hr PO: tablet, liquid Nitrates, riociguat Specialty


(extemporaneous) IV pharmacy may
be required by
Tadalafil 35 hr PO: tablet, liquid
insurance
(extemporaneous)

Soluble Guanylate Cyclase Stimulator

Riociguat 12 hr PO: tablet; no information Nitrates, PDE5 inhibitors REMS programa;


available for alternative specialty
enteral administration pharmacy
(teratogenic) required

Endothelin receptor antagonist

Bosentan 5 hr PO: tablet; all medications CYP2C9 and CYP3A4 substrate REMS programa;
are teratogenic and should and inducer specialty
not be crushed at bedside Contraindicated: cyclosporine, pharmacy
for enteral administration; glyburide; gemfibrozil, required
extemporaneous hormonal contraception,
compounding under simvastatin, rifampin
appropriate conditions can
Ambrisentan 9 hr P-gp, OATP, CYP2C19 and
be considered
CYP3A4 substrate
Category D: cyclosporine

Macitentan 16 hr CYP2C19 and 3A4 substrate;


(active metabolite: avoid use with strong CYP3A4
48 hr) inhibitors/inducers

Prostacyclin and Prostacyclin Receptor Agonist

Treprostinil 4 hr PO: tablet (extended-release); CYP2C8 inhibitors Specialty


cannot be crushed pharmacy
required
Selexipag 0.8–2.5 hr PO: tablet; cannot be CYP2C8 inhibitors, rifampin
(active metabolite: 6.2– crushed.
13.5 hr)

a
Enrollment in an REMS program is required for female patients because of reproductive risk.
IV = intravenous; OATP = organic anion transporting polypeptide; PDE5 = phosphodiesterase type 5; P-gp = P-glycoprotein; PO =
oral; REMS = risk evaluation and mitigation strategy.

present in acute RV failure, providers may wish to start new Phosphodiesterase Type 5 Inhibitors
oral agents in addition to parenteral prostacyclins to tar- The cornerstone of most PAH regimens is an oral PDE5 inhib-
get additional pathways based on increasing evidence that itor. These medications inhibit breakdown of nitric oxide
suggests combination therapy may have benefit over mono- by PDE5 in the pulmonary vasculature, thereby decreasing
therapy (D’Alto 2020). In addition, some patients may require breakdown of guanosine 3’,5’-cyclic monophosphate (cGMP)
RV rescue with parenteral prostacyclin, but they may not be (or “cyclic GMP”) and promoting vasodilation. Sildenafil
candidates for long-term ambulatory infusion. If RV function is taken three times daily and is able to be introduced in
recovers, such patients may require careful transition from low doses (10 mg) to treatment-naive patients to evaluate
a parenteral to an oral agent to avoid rebound PH and allow effect on systemic blood pressure in the setting of RV fail-
transition into the post-acute care or outpatient settings ure/reduced cardiac output. Doses up to 80 mg three times
(Box 3). daily have been evaluated with limited data to support use of

CCSAP 2021 Book 1 • Pulmonary and Endocrinology 56 COPD, Asthma, and PAH
Box 3. Sample Transition Plans for PAH-Targeted Pharmacotherapy
Clinical scenario: Urgent transition from oral treprostinil to IV Clinical scenario: Planned transition from IV epoprostenol to
treprostinil in a patient who is unable to take oral medica- oral selexipag
tions
• Transition should occur over 24 to 72 hours based on
Step 1: Place dedicated, single-lumen, central IV access anticipated tolerability and adverse effects
for administration (peripheral IV can be used • Selexipag should be administered 30 to 60 minutes before
for infusion up to 24 hours if unable to quickly decreasing IV epoprostenol to allow adequate time for
obtain central IV access) absorption
Step 2: Calculate equivalent dose of IV treprostinil • Transition steps should occur every 12 to 24 hours with
Step 3: Choose a dosing weight based on the patient’s scheduled dose of selexipag
estimated dry weight Step 1: Administer selexipag 200 mcg and decrease IV
Step 4: Select appropriate IV treprostinil concentration epoprostenol to 75% of starting dose
for administration Step 2: Administer selexipag 400 mcg and decrease IV
Step 5: Start IV treprostinil at 50% of the calculated epoprostenol to 50% of starting dose
equivalent dose Step 3: Administer selexipag 600 mcg and decrease IV
Step 6: After 2 hours, increase IV treprostinil to 100% of epoprostenol to 25% of starting dose
the calculated equivalent dose Step 4: Administer selexipag 800 mcg and stop IV epo-
prostenol infusion
Additional increases in selexipag to 1600 mcg (or highest
tolerated dose) twice daily can occur in the inpatient or
outpatient setting.

IV = intravenous; PAH = pulmonary arterial hypertension.


Information from: Litwak K, Sargent T, Maga A, Hall S. Successful transition of IV epoprostenol to oral selexipag in severe pulmonary arte-
rial hypertension. Crit Care Med 2020;48:520.

higher doses (Rubin 2011, Galiè 2005). Transition from silde- period during transition; however, in patients with signifi-
nafil to tadalafil may be appropriate for patients who achieve cant RV failure this washout may be longer than desired and
a stable dose of PDE5 inhibitor while in the ICU if once-daily cause additional rise in pulmonary pressures. If a patient is
dosing is desired. In addition, although both of these medica- started on riociguat in the hospital, it is important to navigate
tions may be crushed or prepared as a liquid for alternative the insurance approval process before discharge (Martirosov
enteral administration if needed, multiple daily administra- 2020). Outpatient distribution is limited through specialty
pharmacy only; all patients must be enrolled in a manufac-
tion times of sildenafil may support the need to temporarily
turer-specific program; and female patients must be enrolled
reduce doses or titrate in a more on-demand fashion. There is
in a risk evaluation and mitigation strategy (REMS) program
no exact equivalent dose for conversion between PDE5 inhib-
because of reproductive risk. Inpatient pharmacies must also
itors; tadalafil 40 mg/day is generally considered to have a
be enrolled in REMS program and adhere to specific program
dose equivalent range to sildenafil of 20 to 40 mg three times
requirements.
daily (Frantz 2014).
Endothelin Receptor Antagonists
Soluble Guanylate Cyclase Stimulator
Endothelin receptor antagonists may be used as monother-
Riociguat is the single agent in the class of PAH targeted apy for patients with WHO functional class II or III symptoms
medications known as soluble guanylate cyclase stimulators, or as part of a multidrug regimen in patients with WHO func-
and it is labeled for treatment of both PAH and chronic throm- tional class IV symptoms (Galiè 2016). Although use of
boembolic PH (Group 4). This agent increases production of ETRAs in PAH is common, these drugs are most often used
cGMP by the nitric oxide– soluble guanylate cyclase–cGMP as a second agent as part of a multidrug regimen rather than
pathway and promotes vasodilation. Riociguat can be used in as monotherapy because of several disadvantages com-
place of a PDE5 inhibitor but it should not be used in combina- pared with PDE5 inhibitors. In treatment-naive patients,
tion because of the profound risk of hypotension (Galiè 2015a). initial combination therapy with tadalafil and an ETRA
Even as monotherapy, riociguat may contribute significantly (either ambrisentan or macitentan) has a greater impact on
to systemic hypotension during RV failure. Small temporary PAH clinical outcomes than either agent alone (Sitbon 2020,
dose reductions may be needed in addition to pressure sup- Galiè 2015b). It is important to appreciate the several poten-
port when indicated. If such dose reductions do not resolve tial barriers to medication access in the outpatient setting,
hypotension in addition to vasoactive therapy, consideration including the need to identify a responsible provider enrolled
can be given to changing therapy to a PDE5 inhibitor, such in drug-specific REMS program, the requirement for patient
as sildenafil. Product labeling suggests a 24-hour washout enrollment in a REMS program, and insurance authorization

CCSAP 2021 Book 1 • Pulmonary and Endocrinology 57 COPD, Asthma, and PAH
(Martirosov 2020). In addition, REMS programs also require PAH provider, as quickly as possible. Information guiding this
outpatient, and in some cases, inpatient pharmacies to be conversion is provided in the product labeling, and a pharma-
enrolled and to adhere to strict dispensing guidelines to cist should be involved with multidisciplinary team though
distribute medication. Because of these factors, it is com- the process.
mon to delay initiation of ETRA in patients to the outpatient Selexipag is a selective prostacyclin receptor agonist.
setting. Its chemical structure differs from treprostinil which allows
The contribution of ETRA to fluid retention can be signifi- selectivity for the prostacyclin receptor over other prosta-
cant enough to prompt a care team to evaluate discontinuing noid receptors. In comparison to oral treprostinil, selexipag
this class of medication in acute RV failure from volume over- offers several advantages to a patient, including a reduced
load. Compared with the medications in the other targeted
dose frequency, independence of absorption related to
pathways, ETRAs have less of a dose-response relationship
food, and improved tolerability (most notably for GI-related
and have the least overall impact in the disease process when
adverse effects). Selexipag can be started in the outpatient
used in combination with other therapies. If a patient’s ETRA
setting and should be titrated to 1600 mcg twice daily or to
is held, adjustments to doses of other therapies (e.g., PDE5
the highest tolerated dose. Although there are no associ-
inhibitor or prostacyclin) may help to reduce the impact-
ated REMS program or prescriber requirements, distribution
ful rise in pulmonary pressures. It is important to note that
is limited to specialty pharmacies and insurance authori-
ETRAs are considered hazardous medications and should
not be crushed at the bedside before administration, but this zation must be considered when initiating in the hospital
limitation does not preclude their use in patients with alter- setting. In patients with WHO class II or III symptoms taking
native enteral access. Enteral administration by non-oral either a PDE5 inhibitor, an ETRA, or both (32% of enrolled
routes in facility-compounded form can be considered, but patients), selexipag reduced the combination clinical end
this should be approached carefully under the provisions of point of death or worsening of PAH compared with placebo
the pharmacy department compounding and hazardous med- (Sitbon 2015). Although additional information is needed
ication processes (National Institute for Occupational Safety to support routine use as part of a multidrug regimen for
and Health 2016, Buck 2011). patients with WHO functional class IV symptoms, selexi-
pag is an appealing choice in patients whose disease may
Oral Prostacyclin Receptor Agonists benefit from activity in the prostacyclin pathway but are not
As monotherapy, oral treprostinil improves exercise capac- candidates for parenteral prostacyclin or oral treprostinil
ity (Jing 2013). Early studies showed no additional benefit in (McLaughlin 2019).
exercise capacity when added to either a PDE5 inhibitor or an Although its formulation is not extended release, the
ETRA, or a combination of the two (Tapson 2013, 2012). This manufacturer has specifically recommended that selexi-
finding may be result from the fact that generally lower doses pag not be crushed for enteral administration. Re-titration
of treprostinil were achieved in early trials relative to the for missed doses is not required unless a patient misses
equivalent parenteral dose. Newer data support improvement more than 3 successive days of therapy. However, in the set-
in pulmonary hemodynamics, patient risk status, and delayed ting of critical illness, this limitation may pose a challenge
time to clinical worsening when treprostinil was used in con- to the care team to avoid rebound PH because prolonged
junction with a PDE5 inhibitor, a soluble guanylate cyclase withdrawal of an agent within the prostacyclin path-
stimulator, or an ETRA (White 2020). way can cause or worsen RV failure. Based on pulmonary
High rates of adverse effects, notably headache and diar- hemodynamics, consideration should be given to transi-
rhea, prevailed across phase III trials. The manufacturer tioning a patient unable to take selexipag to an alternative
now supplies many tablet strengths of treprostinil (as low PAH-targeted pharmacotherapy. Consultation with an out-
patient provider for PH care PH provider may be necessary
as 0.125 mg) to help patients achieve a maximum tolerated
because transition to parenteral prostacyclin, particularly
dose. Because of its extended release mechanism, the tab-
in the setting of RV failure, may be warranted. There is no
let formulation should not be crushed, which limits ability
documented dose equivalent of either epoprostenol or tre-
for alternative enteral administration, including by feeding
prostinil to oral selexipag. A typical approach is to initiate a
or other enteral tubes. Doses should be taken with a meal
cross-titration between the two agents in a stepwise fash-
of at least 500 calories to promote absorption and minimize
ion (see Box 3). The multidisciplinary team should account
adverse effects, which could potentially limit administra-
for the following key factors when designing a transition
tion in a critically ill patient. The potential for rebound PAH plan: prostacyclin tolerability; urgency of transition, such as
is high even when only a single dose is missed; therefore, an time without active therapy, as in the case of unintentional,
inability to continue taking this medication by mouth should abrupt withdrawal of selexipag, leading to acute RV failure;
be considered a medical emergency. A patient who is unable and target parenteral prostacyclin dose. In general, a dose
to take oral treprostinil should be assessed for conversion of 8 to10 ng/kg/minute (with a consistent dosing weight
to the equivalent dose of intravenous treprostinil to prevent identified) can be targeted for urgent transition from selexi-
rebound PAH after consultation with the patient’s primary pag to epoprostenol.

CCSAP 2021 Book 1 • Pulmonary and Endocrinology 58 COPD, Asthma, and PAH
Albert RK, Connect J, Bailey WC, et al. Azithromycin for
Practice Points prevention of exacerbations of COPD. N Engl J Med
Clinical pharmacists face many challenges in optimiz- 2011;365:689-98.
ing pharmacotherapy for patients with AECOPD , status
asthmaticus, and PAH. The most difficult task is in Bafadhel M, Peterson S, De Blas MA, et al. Predictors of
knowing how to apply the recent literature and guide- exacerbation risk and response to budesonide in patients
lines to patients in the ICU: with chronic obstructive pulmonary disease: a post-hoc
analysis of three randomized trials. Lancet Respir Med
• The backbone of pharmacologic treatment for an AECOPD
2018;6:117-26.
in the ICU involves nebulized SABA with or without an
anticholinergic, intravenous or enteral corticosteroids, and Barnes PJ. Inflammatory endotypes in COPD. Allergy 2019;
antibiotic therapy. 74:1249-56.
• The first mode of ventilation used in COPD patients with
acute respiratory failure should be NIV, with the goal to Barst RJ, Rubin LJ, Long WA, et al., The Primary Pulmonary
avoid intubation. Hypertension Study Group. A comparison of continuous
• For AECOPD management, corticosteroids should be intravenous epoprostenol (prostacyclin) with conventional
administered for shorter durations (5 days or less) and therapy for primary pulmonary hypertension. N Engl J Med
antibiotics should be continued for 5–7 days if consistent 1996;334:296-302.
with clinical improvement.
• First-line therapy for status asthmaticus includes oxygen Buck ML. Use of bosentan in pediatric pulmonary hyperten-
supplementation, inhaled SABA plus ipratropium, and corti- sion. Pediatric Pharmacotherapy 2011;17:1-4.
costeroids. Adjunctive therapy (epinephrine or terbutaline,
intravenous magnesium, or ketamine) can be considered Bush A. Pathophysiologic mechanisms of asthma. Front
for patients who are unresponsive to initial therapy. Pediatr 2019;7:68.
• Preventive strategies for both AECOPD and asthma exac-
Butler CC, Gillespie D, White P, et al. C-reactive protein test-
erbation are not the initial focus of ICU care, but should be
ing to guide antibiotic prescribing for COPD exacerbations.
discussed and initiated as appropriate for the patient.
N Engl J Med 2019;381:111-20.
• In critically ill patients with RV failure, non-targeted
interventions such as oxygenation, volume management, Camargo CA Jr , Spooner C, Rowe BH. Continuous versus
pressure support, and anticoagulation should be optimized. intermittent beta-agonists for acute asthma. Cochrane
• Initiation of PAH-target pharmacotherapy requires formal Database Syst Rev 2003;4:CD001115.
diagnosis by RHC because the use of these agents in other
pathophysiologic states can worsen RV failure. Carr TF, Zeki AA, Kraft M. Eosinophilic and noneosinophilic
• If targeted PAH therapies are initiated while a patient is asthma. Am J Respir Crit Care Med 2018:197;22-37.
hospitalized, it is important to ensure access to therapies
before discharge and assignment of a PAH specialist to Celli BR, Wedzicha JA. Update on clinical aspects of chronic
follow-up post-discharge. obstructive pulmonary disease. N Engl J Med 2019;
• If targeted PAH therapies are continued in the hospital 381:1257-66.
from home, the pharmacist should communicate with the
patient’s PAH specialist and specialty pharmacy to ensure Chapman KR, Stockley RA, Dawkins C, et al. Augmentation
correct PAH regimens. therapy for alpha1 antitrypsin deficiency: a meta-analysis.
• Abrupt withdrawal of PAH-targeted pharmacotherapy in the COPD 2009;6:177-84.
setting of hypotension associated with RV failure should be
Crisafulli E, Barbeta E, Ielpo, et al. Management of severe
avoided, as this can cause rebound pulmonary hypertension.
acute exacerbations of COPD: an updated narrative review.
Multidiscip Respir Med 2018;13:36.

CONCLUSION D’Alto M, Badagliacca R, Argiento P, et al. Risk reduction and


right heart reverse remodeling by upfront triple combi-
Chronic obstructive pulmonary disease exacerbations, status
nation therapy in pulmonary arterial hypertension. Chest
asthmaticus, and PH are commonly encountered diseases in
2020;157:376-83.
the ICU. Pharmacists can play a key role in optimizing medi-
cal management and ensuring that safe and evidence-based Daubin C, Valette X, Thiolliere F, et al. Procalcitonin algorithm
therapies are used to minimize the morbidity and mortality to guide initial antibiotic therapy in acute exacerbation
of COPD admitted to the ICU: a randomized multicenter
attributed to these diseases.
study. Intensive Care Med 2018;44:428-37.

REFERENCES Devereux G, Cotton S, Fielding S, et al. Effect of theophylline


Ahmed AM, Athar M. Mechanical ventilation in patients with as adjunct to inhaled corticosteroids on exacerbations in
chronic obstructive pulmonary disease and bronchial patients with COPD. JAMA 2018;320:1548-59.
asthma. Indian J Anaesth 2015;59:589-98. Dixit D, Bridgeman MB, Andrews LB, et al. Acute exacerba-
Ascha M, Zhou X, Rao Y, et al. Impact on survival of warfarin tions of chronic obstructive pulmonary disease: diagnosis,
in patients with pulmonary arterial hypertension receiving management, and prevention in critically ill patients.
subcutaneous treprostinil. Cardiovasc Ther 2017;35:10. Pharmacotherapy 2015;35:631-48.

CCSAP 2021 Book 1 • Pulmonary and Endocrinology 59 COPD, Asthma, and PAH
Eklof J, Sorensen R, Ingebrigtsen TS, et al. Pseudomonas Goyal S, Agreawal A. Ketamine in status asthmaticus: a
aeruginosa and risk of death and exacerbations in patients review. Indian J Crit Care Med 2013;17:154-61
with chronic obstructive pulmonary disease: an observa-
Granton J, Hoeper M. Intensive care unit management of
tional cohort study of 22,053 patients. Clin Microbiol Infec
patients with severe pulmonary hypertension and right
2019;26:227-34.
heart failure. Am J Respire Crit Care Med 2011;184:1114-24.
Farber HW, Miller DP, Poms AD, et al. Five-year outcomes
Han MK, Tayob N, Murray S, et al. Predictors of chronic
of patients enrolled in the REVEAL Registry. Chest
obstructive pulmonary disease exacerbation reduction in
2015;148:1043-54.
response to daily azithromycin. Am J Respir Crit Care Med
Frantz RP, Durst L, Burger CD, et al. Conversion from silde- 2014;189:1503-8.
nafil to tadalafil: results from the sildenafil to tadalafil in Hoeper MM, Benza RL, Corris P, et al. Intensive care,
pulmonary arterial hypertension (SITAR) study. right ventricular support and lung transplantation in
J Cardiovasc Pharmacol Ther 2014;19:550-7. patients with pulmonary hypertension. Eur Respir J
Gaine S. Pulmonary hypertension. JAMA 2000;284:3160-8. 2019;53:1801906
Hopkinson NS, Molyneux A, Pink J, et al. Chronic obstructive
Galiè N, Ghofrani HA, Torbicki A, et al. Sildenafil citrate ther-
pulmonary disease: diagnosis and management: summary
apy for pulmonary arterial hypertension. N Engl J Med
of updated NICE guidance. BMJ 2019;366:l4486.
2005;353:2148-57.
Huynh TN, Weight SS, Sugar CA, et al. Prognostic factors
Galiè N, Müller K, Scalise AV, et al. PATENT PLUS: a blinded,
and outcomes of patients with pulmonary hypertension
randomised and extension study of riociguat plus silde-
admitted to the intensive care unit. J Crit Care 2012;
nafil in pulmonary arterial hypertension. Eur Respir J
27:739.e7-13.
2015a;45:1314-22.
Israel E, Reddel H. Severe and difficult-to-treat asthma in
Galiè N, Barbera JA, Frost AE. Initial use of ambrisentan plus adult patients. N Engl J Med 2017;377:965-76.
tadalafil in pulmonary arterial hypertension. N Engl J Med
2015b;373:834-44. Jentzer JC, Mathier MA. Pulmonary hypertension in the
intensive care unit. J Intensive Care Med 2016;31:369-85.
Galiè N, Humbert M, Vachiery JL, et al. 2015 ESC/ERS
guidelines for the diagnosis and treatment of pulmonary Jing ZC, Parikh K, Pulido T, et al. Efficacy and safety of oral
hypertension. Eur Heart J 2016;37:67-119. treprostinil monotherapy for the treatment of pulmonary
arterial hypertension a randomized, controlled trial.
Galié N, Channick RN, Frantz RP, et al. Risk stratification and Circulation 2013;127:624-33.
medical therapy of pulmonary arterial hypertension. Eur
Respir 2019;53;1801889. Jolliffe DA, Greenberg L, Hooper R, et al. Vitamin D to
prevent exacerbations of COPD: systematic review
Garudadri S, Woodruff P. Targeting Chronic Obstructive and meta-analysis of individual participant data from
Pulmonary Disease Phenotypes, Endotypes, and randomised controlled trials. Thorax 2019;74:337-45.
Biomarkers. Ann Am Thorac Soc 2018: S234-38.
Jones AM, Munavvar M, Vail A, et al. Prospective, place-
GBD 2015 Chronic Respiratory Disease Collaborators. Global, bo-controlled trial of 5 vs 10 days of oral prednisolone in
regional, and national deaths, prevalence, disability-ad- acute adult asthma. Respir Med 2002;96:950-4.
justed life years, and years lived with disability for chronic
Kirkland SW, Vandenberghe C, Voaklander B, et al. Combined
obstructive pulmonary disease and asthma, 1990-2015:
inhaled beta-agonist and anticholinergic agents for emer-
a systematic analysis for the Global Burden of Disease
gency management in adults with asthma. Cochrane
Study 2015. Lancet Respir Med 2017;5:691-706.
Database Syst Rev 2017;1:CD001284.
Gentene AJ, Guido MR, Woolf B, et al. Multidisciplinary team Kiser TH, Allen RR, Valuck RJ, et al. Outcomes associated
utilizing pharmacists in multimodal, bundled care reduce with corticosteroid dosage in critically ill patients with
chronic obstructive pulmonary disease hospital readmis- acute exacerbations of chronic obstructive pulmonary dis-
sion rates. J Pharm Pract 2019 Nov 26:897190019889440. ease. Am J Respir Crit Care Med 2014;189:1052-64.
Global Initiative for Chronic Obstructive Lung Disease Kiser TH, Reynolds PM, Moss M, et al. Impact of mac-
(GOLD). 2020 GOLD Reports: Global Strategy for the rolide antibiotics on hospital readmissions and other
Diagnosis, Management and Prevention of COPD. clinically important outcomes in critically ill patients
Fontana, WI: GOLD, 2020. with acute exacerbations of chronic obstructive pulmo-
nary disease: a propensity score–matched cohort study.
Global Initiative for Asthma (GINA). 2020 GINA Main
Pharmacotherapy 2019;39:242-52.
Report: 2020 GINA Report, Global Strategy for Asthma
Management and Prevention. Fontana, WI: GINA, 2020. Klinger JR, Elliot CG, Levine DJ, et al. Therapy for pulmonary
arterial hypertension in adults. Chest 2019;155:565-86.
Gordon CS, Waller JW, Cook RM, et al. Effect of pulmonary
rehabilitation on symptoms of anxiety and depression Leather HA et al. Effects of vasopressin on RV function in an
in COPD: a systematic review and meta-analysis. Chest experimental model of acute pulmonary hypertension. Crit
2019;156:80-91. Care Med 2002;30:2548-52.

CCSAP 2021 Book 1 • Pulmonary and Endocrinology 60 COPD, Asthma, and PAH
Leuppi JD, Schuetz P, Bingisser R, et al. Short-term vs con- National Asthma Education and Prevention Program,
ventional glucocorticoid therapy in acute exacerbations Third Expert Panel on the Diagnosis and Management
of chronic obstructive pulmonary disease: the REDUCE of Asthma. Expert Panel Report 3: Guidelines for the
randomized clinical trial. JAMA 2013;309:2223-31. Diagnosis and Management of Asthma [Clinical Practice
Guidelines]. Report No. 07-4051. Bethesda, MD: National
Lim WJ, Mohammed A, Cason KV, et al. Non-invasive positive Heart, Lung, and Blood Institute, 2007.
pressure ventilation of respiratory failure due to severe
acute exacerbations of asthma. Cochrane Database Syst National Institute for Occupational Safety and Health
Rev 2012;12:CD004360. (NIOSH). Preventing Occupational Exposure to
Antineoplastic and Other Hazardous Drugs in Health Care
Littenberg B, Gluck E. A controlled trial of methylpredniso- Settings. DHHS (NIOSH) Publication Number 2016-161
lone in the emergency treatment of acute asthma. N Engl (supersedes 2014-138). Washington, DC: NIOSH, 2016.
J Med 1986;314:150-2.
Olschewski H, Simonneau G, Galiè N, et al. Inhaled iloprost
Lombardi S, Kingman M, Duncan M, et al. Titration of pulmo- for sever pulmonary hypertension. N Engl J Med 2002;
nary arterial hypertension therapeutics: experience-based 347:322-9.
recommendations. Respir Med 2018;143:139-46.
Olsson KM, Delcroix M, Ghofrani HA, et al. Anticoagulation
Martirosov AL, Smith ZR, Hencken L, et al. Improving transi- and survival in pulmonary arterial hypertension:
tions of care for critically ill adult patients on pulmonary results from the Comparative, Prospective Registry of
arterial hypertension medications. Am J Health Syst Newly Initiated Therapies for Pulmonary Hypertension
Pharm 2020;77:958-65. (COMPERA). Circulation 2014;129:57-65.

McKeever TM, Hearson G, Housley G, et al. Using venous Osadnik CR, Tee VS, Carson-Chahhoud KV, et al. Non-
blood gas analysis in the assessment of COPD exacerba- invasive ventilation for the management of acute
tions: a prospective cohort study. Thorax 2016;71:210-15. hypercapnic respiratory failure due to exacerbation
of chronic obstructive pulmonary disease. Cochrane
McLaughlin VV, Benza RL, Rubin LJ, et al. Addition of inhaled Database Syst Rev 2017;7:CD004104.
treprostinil to oral therapy for pulmonary arterial hyper-
tension: a randomized controlled clinical trial. J Am Coll Parikh V, Bhardwaj A, Nair A. Pharmacotherapy for pulmo-
Cardiol 2010;55:1915-22. nary arterial hypertension. J Thorac Dis 2019;11:S1767-81.

McLaughlin VV, Channick R, DeMarco T, et al. Results of Pleasants RA, Wang T, Xu X, et al. Nebulized corticoste-
an expert consensus survey on the treatment of pulmo- roids in the treatment of COPD exacerbations: systematic
nary arterial hypertension with oral prostacyclin pathway review, meta-analysis, and clinical perspective. Respir
agents. Chest 2019;157:955-65. Care 2018;63:1302-10.

Mishra A, Kumar B, Dutta V, et al. Comparative effect of levo- Preston RJ, Roberts KE, Miller DP, et al. Effect of warfarin
simendan and milrinone in cardiac surgery patients with treatment on survival of patients with pulmonary arterial
pulmonary hypertension and left ventricular dysfunction. hypertension (PAH) in the Registry to Evaluate Early and
J Cardiothorac Vasc Anesth 2016;30:639-46. Long-Term PAH Disease Management (REVEAL). Am J
Respir Crit Care Med 2014;189:A2464.
Montani D, Price L, Dorfmuller L, et al. Pulmonary veno-
Puhan MA, Gimeno-Santos E, Cates CJ, et al. Pulmonary
occlusive disease. Eur Resp J 2009;33:189-200.
rehabilitation following exacerbations of chronic pulmo-
Moussaoui R, Roede B, Speelman P, et al. Short-course nary obstructive disease. Cochrane Database Syst Rev
antibiotic treatment in acute exacerbations of chronic 2016;12:CD005305.
bronchitis and COPD: a meta-analysis of double-blind
Rich S, Brundage BH. High-dose calcium channel-blocking
studies. Thorax 2018;63:415-22.
therapy for primary pulmonary hypertension: evidence for
Ni W, Bao J, Yang D, et al. Potential of serum procalcitonin long-term reduction in pulmonary arterial pressure and
in predicting bacterial exacerbation and guiding anti- regression of right ventricular hypertrophy. Circulation
biotic administration in severe COPD exacerbations: a 1987;76:135-41.
systematic review and meta-analysis. Infect Dis (Lond) Rabe KF, Watz H. Chronic obstructive pulmonary disease.
2019;51:639-50. Lancet 2017;389:1931-40.
Nathan SD, Flaherty KR, Glassberg MK, et al. A random- Ritchie AI, Brill AE, Vlies BH, et al. Targeted retreatment of
ized, double-blind, placebo-controlled study of pulsed, incompletely recovered COPD exacerbations with cipro-
inhaled nitric oxide in subjects at risk of pulmonary floxacin: a double-blind, randomised, placebo-controlled,
hypertension associated with pulmonary fibrosis. Chest multicentre phase III trial. Am J Respir Crit Care Med
2020;158:637-45. 2020;202:549-57.
Narechania S, Torbic H, Tonelli AR. Treatment Roberts DH, Lepore JJ, Maroo A, et al. Oxygen therapy
Discontinuation or Interruption in Pulmonary Arterial improves cardiac index and pulmonary vascular resis-
Hypertension. J Cardiovasc Pharmacol Ther. 2020; tance in patients with pulmonary hypertension. Chest
25(2):131-141. 2001;120:1547-55.

CCSAP 2021 Book 1 • Pulmonary and Endocrinology 61 COPD, Asthma, and PAH
Rowe BH, Spooner C, Ducharme F, et al. Early emergency Tapson, VF, Torres F, Kermeen F, et al. Oral treprostinil for
department treatment of acute asthma with systemic the treatment of pulmonary arterial hypertension in
corticosteroids. Cochrane Database of Syst Rev patients on background endothelin receptor antagonists
2001;1:CD002178. and/or phosphodiesterase type 5 inhibitor therapy (the
FREEDOM-C study): A Randomized Controlled Trial. Chest
Rubin LJ, Badesch DB, Fleming TR, et al. Long-term 2012;142:1383-90.
treatment with sildenafil citrate in pulmonary arterial
hypertension: the SUPER-2 study. Chest 2011;140:1274-83. Tapson, VF, Jing ZC, Xu KF, et al. Oral treprostinil for the
treatment of pulmonary arterial hypertension in patients
Rush B, Biagioni BJ, Berger L, McDermid R. Mechanical ven- receiving background endothelin receptor antagonist
tilation outcomes in patients with pulmonary hypertension and phosphodiesterase type 5 inhibitor therapy (the
in the United States: a national retrospective cohort analy- FREEDOM-C2 study): a randomized controlled trial. Chest
sis. J Intensive Care Med 2017;32:588-92. 2013;144:952-8.
Sieluk J, Levy J, Sandhaus RA, et al. Costs of medical Tejwani V, Patel DC, Zein J, et al. Survival after an ICU
care among augmentation therapy users and non-users hospitalization for pulmonary hypertension. Chest
with alpha-1 antitrypsin deficiency in the United States. 2018;154:229-31.
Chronic Obstr Pulm Dis 2019;6:6-16.
Torbic H. Management of pulmonary arterial hypertension in
Simonneau G, Montani D, Celermajer DS, et al. Haemodynamic the ICU. J Pharm Pract 2019;32:303-13.
definitions and updated clinical classification of pulmonary
hypertension. Eur Respir J 2019;53:1801913. Van Geffen WH, Douma WR, Slebos DJ, et al. Bronchodilators
delivered by nebulizer versus pMDI with spacer or DPS
Sitbon O, Channick R, Chin K. Selexipag for the treat- for exacerbations of COPD. Cochrane Database Syst Rev
ment of pulmonary arterial hypertension. N Engl J Med 2016;8:CD011826.
2015;317:2522-33.
Vermeersch K, Gabrovsk M, Aumann J, et al. Azithromycin
Sitbon O, Cottin V, Canuet M. Initial combination therapy of during acute chronic obstructive pulmonary disease exac-
macitentan and tadalafil in pulmonary arterial hyperten- erbations requiring hospitalization (BACE). A multicenter,
sion. Eur Resp J 2020;56:2000673. randomized, double-blind, placebo-controlled trial. Am J
Sivapalan P, Ingebrigsten TS, Rasmussen, et al. COPD exac- Respir Crit Care Med 2019;200:857-68.
erbations: the impact of long versus short courses of oral Vollenweider DJ, Frei A, Steurer-Stey A, et al. Antibiotics for
corticosteroids on mortality and pneumonia: nationwide exacerbations of chronic obstructive pulmonary disease.
data on 67,000 patients with COPD followed for 12 months. Cochrane Database Syst Rev 2018:CD010257.
BMJ Open Respir Res 2019;6:e000407.
Walters JA, Tan DJ, White CJ, et al. Systemic corticosteroids
Smith Z, Rangarajan K, Barrow J, et al. Development of best for acute exacerbations of chronic obstructive pulmonary
practice recommendations for the safe use of pulmonary disease. Cochrane Database Syst Rev 2014;9:CD0071288.
hypertension pharmacotherapies using a modified Delphi
method. Am J Health Syst Pharm 2019;76:153-65. Wenzel SE. Asthma phenotypes: the evolution from clinical
to molecular approaches. Nature Medicine 2012;18:716-25.
Stockley RA, O’Brien C, Pye A, et al. Relationship of sputum
color to nature and outpatient management of acute exac- White RJ, Pulido T, Jerjes-Sanchez C, et al. Treatment with
erbations of COPD. Chest 2000;117:1638-45. oral treprostinil improves hemodynamics in partici-
pants with PAH. Abstract presented at: PVRI [Pulmonary
Suissa D, Dell’Aniello S, Ernst P. Comparative effectiveness Vascular Research Institute] 14th Annual World Congress
and safety of LABA-MALA vs LABA-ICS treatment of COPD on PVD; January 30–February 2, 2020; Lima, Peru.
in real-world clinical practice. Chest 2019;155:1158-65.

CCSAP 2021 Book 1 • Pulmonary and Endocrinology 62 COPD, Asthma, and PAH
Self-Assessment Questions
16. A 23-year-old woman with uncontrolled type-2 asthma B. Add nebulized budesonide 2 mg every 6 hours to
is maintained on albuterol and mometasone 200 mcg/5 aclidinium/formoterol.
mcg formoterol (2 inhalations twice daily) and pred- C. Use ipratropium and albuterol nebulization (0.5 mg
nisone 10 mg daily. Her most recent labs reveal blood ipratropium bromide/2.5 mg albuterol nebulized
eosinophil count of 230 cells/mm3. She has experienced every 1–4 hours as needed.
four acute exacerbations within the past 12 months. D. Use albuterol 4 puffs by metered dose inhaler (MDI)
Which one of the following is best to recommend regard- with holding chamber every 1–4 hours as needed.
ing add-on biologic therapy for this patient?
19. Which one of the following corticosteroid strategies is
A. No biologic therapy best to recommend for J.W.’s severe AECOPD?
B. Dupilumab
A. Prednisone 40 mg orally once daily for 5 days
C. Mepolizumab
B. Methylprednisolone 125 mg intravenously for 1 dose
D. Benralizumab
C. Nebulized budesonide 2 mg every 6 hours for 7 days
17. A 59-year-old man complains of persistent breathless- D. Prednisone 1 mg/kg/day orally divided twice daily
ness and chronic cough. He has a 35 pack-year smoking for 10 days
history and consulted his doctor regarding his symp-
20. J.W. improves during her hospital course and is now back
toms. He is referred for pulmonary function testing and
to her baseline respiratory status. The medicine team
results show a post-bronchodilator forced expiratory vol-
consults you to discuss COPD exacerbation preven-
ume in 1 second (FEV1)/forced vital capacity of 0.63 and
tion strategies. J.W.’s serum eosinophil count results as
FEV1 61%. Which one of the following best evaluates this
247 cells/mm3. Which one of the following is best to
patient’s presentation?
recommend to decrease the incidence of future AECOPD
A. Grade 1 (mild) chronic obstructive pulmonary
in J.W.?
disease (COPD)
B. Grade 2 (moderate) COPD A. Add azithromycin 500 mg three times weekly.
C. Grade 3 (severe) COPD B. Escalate to triple therapy (long-acting β-agonist
D. Adult-onset asthma [LABA]/long-acting muscarinic antagonist [LAMA]/
inhaled corticosteroid [ICS]).
Questions 18–20 pertain to the following case. C. Discontinue LABA and add an ICS.
J.W. is 68-year-old woman (weight 70 kg), with a medical his- D. Add prednisone 10 mg daily indefinitely.
tory of schizophrenia, COPD (25 pack-years – current smoker),
21. A 59-year-old man presents with a severe AECOPD requir-
diabetes, and hypertension. Two days ago, J.W had worsen-
ing hospital admission. His main complaint is worsening
ing dyspnea for which she was prescribed prednisone 20 mg
dyspnea, but he denies changes to his sputum volume or
by mouth daily at an outpatient clinic. Today she presents to
color and is stable on oxygen 2 L by nasal cannula with a
the ED with profound dyspnea, increased sputum production
SaO2 of 97%. He has no prior exacerbation history. Which
(thick and purulent), and increased confusion. Her son is her
one of the following is best to recommend for manage-
caretaker and corroborates that J.W.’s dyspnea has progres-
ment of this patient’s severe AECOPD?
sively worsened over the last 24 hours and is currently not at
her baseline respiratory function. Baseline COPD medications A. Albuterol 2.5 mg nebulization every 4 hours as
include aclidinium 400 mcg/formoterol 12 mcg (1 inhalation) needed + prednisone 40 mg orally daily for 10 days
twice daily therapy and a rescue albuterol inhaler every 4 hours + amoxicillin/clavulanate 875/125 mg orally twice
as needed for shortness of breath. J.W has been hospitalized daily for 10 days
twice within the past 6 months for severe COPD exacerbations. B. Ipratropium/albuterol 0.5/2.5 mg nebulization every
Vitals on presentation include: blood pressure 134/91 mm Hg; 4 hours as needed + prednisone 40 mg orally daily
heart rate 98 beats/minute; respiratory rate 22 breaths/minute; for 5 days
oxygen saturation (SaO2) 87%; and temperature 98.9°F (37.2°C). C. Ipratropium/albuterol 0.5/2.5 mg nebulization every
6 hours as needed + prednisone 40 mg orally daily
18. Which one of the following is best to recommend
for 10 days
regarding initial bronchodilator therapy for J.W.’s acute
D. Ipratropium/albuterol 0.5/2.5 mg nebulization every
exacerbation of COPD (AECOPD)?
4 hours as needed + prednisone 40 mg orally daily
A. Double the dose of maintenance aclidinium/ for 5 days + amoxicillin/clavulanate 875/125 mg
formoterol inhaler (2 inhalations twice daily). orally twice daily for 5 days

CCSAP 2021 Book 1 • Pulmonary and Endocrinology 63 COPD, Asthma, and PAH
22. A 54-year-old man with COPD, hypertension, coronary 25. C.D.’s symptoms progress despite initial treatment. She
artery disease, and heart failure with reduced ejection reports ongoing chest tightness and continues to strug-
fraction is recently hospitalized for AECOPD. The med- gle to talk in full sentences. Repeat vital signs show a
icine team is counseling the patient regarding future respiratory rate of 30 breaths/minute, SaO2 89%, blood
exacerbation prevention. In addition to an annual influ- pressure 152/96 mm Hg, and heart rate of 124 beats/min-
enza vaccine, which one of the following is best to ute. Which one of the following is best to recommend for
recommend regarding appropriate vaccines for this C.D.’s progressive symptoms?
patient?
A. Theophylline 200 mg orally daily
A. Pneumococcal conjugate vaccine (PCV13) and B. Methylprednisolone 125 mg intravenously
pneumococcal polysaccharide vaccine (PPSV23) C. Epinephrine 0.3 mg intramuscularly
separated by at least 1 year with PPSV23 D. Ketamine 90 mg intravenous push
revaccination after age 65 years
26. A 27-year-old woman (weight 52 kg) with asthma pres-
B. PCV13 and PPSV23 separated by at least 1 year with
ents to the ED with wheezing and dyspnea. Her SaO2 is
PCV13 revaccination after age 60 years
94% on room air and she is able to speak in full phrases.
C. PCV13 and PPSV23 separated by at least 5 years
Which one of the following is best to recommend for this
with PPSV23 revaccination after age 60 years
patient?
D. PCV13 and PPSV23 separated by at least 5 years
with no revaccination necessary A. Ipratropium/albuterol 0.5/2.5 mg nebulization every
20 minutes for 3 doses, prednisone 50 mg daily for
5 days, magnesium sulfate 2 g intravenously over
Questions 23–25 pertain to the following case.
20 minutes
C.D. is a 37-year-old woman (weight 109 kg) who presents
B. Albuterol nebulization 10–15 mg/hour continuously,
to the ED dyspneic, diaphoretic, and with chest tightness.
terbutaline 1 mg/mL 0.25 mg subcutaneously every
She can talk using single words and sits hunched forward to
20 minutes for 3 doses
lessen the labor of breathing. C.D.’s medical history is signif-
C. Ipratropium/albuterol 0.5/2.5 mg nebulization every
icant for anxiety, coronary artery disease, and uncontrolled
20 minutes for 3 doses, prednisone 50 mg daily for
asthma with frequent exacerbations. Her initial set of vitals
5 days
on arrival include a respiratory rate 34 breaths/minute, SaO2
D. Albuterol nebulization 10–15 mg/hour continuously,
88%, blood pressure 148/98 mm Hg, and heart rate of 122
prednisone 20 mg daily for 10 days
beats/minute. C.D.’s estimated initial peak expiratory flow is
30% of predicted prior to any medication administration.
Questions 27 and 28 pertain to the following case.
23. Which one of the following best classifies C.D.’s asthma
T.T., a 56-year-old woman with idiopathic pulmonary arterial
exacerbation severity?
hypertension (PAH), is admitted to the ICU after being found
A. Moderate down in her home. Her PAH-targeted drugs include riociguat
B. Severe 2.5 mg three times daily, macitentan 10 mg daily, and trepros-
C. Status asthmaticus tinil continuous intravenous infusion. T.T.’s vital signs upon
D. Mild admission include blood pressure 76/44 mm Hg, heart rate
124 beats/minute, and SaO2 of 95% on 6 L high-flow nasal can-
24. Which one of the following is best to recommend regard-
nula (home requirement of 2 L). An initial evaluation in the ED
ing initial bronchodilator therapy for C.D.’s asthma
reveals a urinalysis suggestive of infection, with white blood
exacerbation?
cell elevated to 17 × 103 cells/mm3, BUN 46 mg/dL, SCr 2.1 mg/
A. Magnesium sulfate 2 g intravenously over dL (baseline 0.8 mg/dL), and lactic acid 6 mmol/L. Antibiotics
20 minutes were started in the ED for sepsis secondary to a urinary tract
B. Ipratropium 0.5 mg nebulization every 20 minutes infection. T.T. is awake, but not alert, and confused, oriented
for 3 doses then as needed only to person (Glasgow Coma Scale score 11). Her husband is
C. Ketamine continuous infusion with a starting dose at bedside and confirms that they changed the supply of tre-
of 0.5 mg/kg/hour prostinil in her infusion pump this morning. The ambulatory
D. Albuterol nebulization 10–15 mg/hour continuously infusion pump appears to be functioning and has a volume
+ ipratropium 0.5 mg nebulization every 20 minutes remaining to be infused of 90 mL (from an initial volume of
for 3 doses 100 mL). The pharmacist calls the specialty pharmacy pro-
vided by T.T.’s husband to verify the infusion pump settings.

CCSAP 2021 Book 1 • Pulmonary and Endocrinology 64 COPD, Asthma, and PAH
27. Which one of the following is the best initial intervention (unchanged from baseline RHC at diagnosis 6 months
to recommend for T.T.’s hypotension? prior). The patient does not wish to continue on paren-
teral therapy at discharge. Which one of the following is
A. Rapidly administer 1 L intravenous crystalloid
best to recommend for transitioning this patient into the
bolus and replace crystalloid to a total goal of 30
outpatient setting?
mL/kg for volume resuscitation; hold intravenous
treprostinil. A. Continue sildenafil, discontinue ambrisentan,
B. Slowly administer 250 mL intravenous crystalloid and transition intravenous epoprostenol to oral
bolus; hold intravenous treprostinil. treprostinil.
C. Insert pulmonary artery catheter for hemodynamic B. Transition sildenafil back to tadalafil, continue
monitoring; continue intravenous treprostinil. ambrisentan, and transition intravenous
D. Slowly administer 250 mL intravenous crystalloid epoprostenol to selexipag.
bolus; continue intravenous treprostinil at home C. Continue sildenafil, continue ambrisentan, and
dose. transition intravenous epoprostenol to inhaled
epoprostenol.
28. Which one of the following is best to recommend as
D. Transition sildenafil back to tadalafil, discontinue
the initial intervention for T.T.’s oral PAH-targeted
ambrisentan, and transition intravenous
medications?
epoprostenol to inhaled epoprostenol.
A. Continue macitentan 10 mg daily and hold riociguat.
30. A 56-year-old woman with a history of scleroderma has
B. Reduce macitentan to 5 mg daily and continue
had a rapid progression of dyspnea and is admitted to
riociguat 2.5 mg three times daily.
the ICU with RV failure and WHO functional class IV
C. Hold macitentan and hold riociguat.
symptoms. A vasodilator challenge was not completed
D. Continue macitentan 10 mg daily and reduce
in the setting of acute RV failure, but RHC measurements
riociguat to 2 mg three times daily.
include mean pulmonary arterial pressure 35 mm Hg, pul-
29. A 43-year-old woman with idiopathic PAH (WHO func- monary capillary wedge pressure 13 mm Hg, pulmonary
tional class III symptoms at baseline) has had a 21-day vascular resistance 6 Woods units, right atrial pressure
ICU stay for volume overload and acute right ventricu- 17 mm Hg. N-terminal pro b-type natriuretic peptide is
lar (RV) failure. She reported dietary indiscretion and elevated (1650 ng/L). Which one of the following is best
nonadherence to her home diuretic regimen for 2 weeks to recommend for this patient’s newly-diagnosed PAH?
prior to admission but was taking her tadalafil 40 mg
A. Riociguat 2.5 mg three times daily plus oral
daily and ambrisentan 10 mg once daily. While in the ICU,
treprostinil 2.5 mg three times daily
she is transitioned to sildenafil 40 mg three times daily
B. Riociguat 2.5 mg three times daily plus macitentan
and ambrisentan is continued. After stabilization with
10 mg once daily
IV epoprostenol in the ICU, the patient is approaching
C. Sildenafil 20 mg three times daily plus IV
discharge readiness. A repeat right heart catheteriza-
epoprostenol titrated to 8–10 ng/kg/minute
tion (RHC) shows mean pulmonary arterial pressure 52
D. Sildenafil 20 mg three times daily plus inhaled
mm Hg with pulmonary capillary wedge pressure 12 mm
treprostinil
Hg and pulmonary vascular resistance 5 Woods units

CCSAP 2021 Book 1 • Pulmonary and Endocrinology 65 COPD, Asthma, and PAH
Learner Chapter Evaluation: COPD, Asthma, and PAH

As you take the posttest for this chapter, also evaluate the 25. The learning assessment activities used in the chapter
material’s quality and usefulness, as well as the achievement were effective.
of learning objectives. Rate each item using this 5-point scale: 26. The chapter was effective overall.

• Strongly agree 27. The activity met the stated learning objectives.
• Agree 28. If any objectives were not met, please list them here.
• Neutral
• Disagree OTHER COMMENTS
• Strongly disagree
29. Please provide any specific comments related to any
16. The content of the chapter met my educational needs. perceptions of bias, promotion, or advertisement of
17. The content of the chapter satisfied my expectations. commercial products.
18. The author presented the chapter content effectively. 30. Please expand on any of your above responses, and/or
provide any additional comments regarding this chapter:
19. The content of the chapter was relevant to my practice
and presented at the appropriate depth and scope.
20. The content of the chapter was objective and balanced. Questions 31–33 apply to the entire learning module.

21. The content of the chapter is free of bias, promotion, and 31. How long did it take you to read the instructional materi-
advertisement of commercial products. als in this module?

22. The content of the chapter was useful to me. 32. How long did it take you to read and answer the assess-
ment questions in this module?
23. The teaching and learning methods used in the chapter
were effective. 33. Please provide any additional comments you may have
regarding this module:
24. The active learning methods used in the chapter were
effective.

CCSAP 2021 Book 1 • Pulmonary and Endocrinology 66 COPD, Asthma, and PAH
Pulmonary and Endocrinology II
Pulmonary and Endocrinology II Panel

Series Editors: Pneumonia in the ICU


Bradley A. Boucher, Pharm.D., FCCP, FNAP, MCCM, BCPS
Author
Professor of Clinical Pharmacy and Translational Science
G. Christopher Wood, Pharm.D., FCCP, FCCM, BCCCP
Associate Dean for Strategic Initiatives and Operations
College of Pharmacy Professor
University of Tennessee Health Science Center Department of Clinical Pharmacy
Memphis, Tennessee and Translational Science
University of Tennessee Health Science
Curtis E. Haas, Pharm.D., FCCP
Center College of Pharmacy
Chief Pharmacy Officer Memphis, Tennessee
University of Rochester Medical Center
Rochester, New York
Reviewers
Faculty Panel Chair: Michael A. Peters, BS, BCCCP
Alexander H. Flannery, Pharm.D., FCCM, BCCCP, BCPS Clinical Manager ICU Specialist
Department of Pharmacy
Assistant Professor
Henry Ford Hospital
University of Kentucky College of Pharmacy
Detroit, Michigan
Lexington, Kentucky
Poorvi Shah, Pharm.D., BCCCP
Clinical Pharmacy Specialist, MICU
Drug-Induced Pulmonary Disorders and Department of Pharmacy
Pulmonary Drug Delivery Advocate Christ Medical Center
Oak Lawn, Illinois
Authors
Heather P. May, Pharm.D., FCCM, BCCCP, BCPS
Assistant Professor of Medicine and Pharmacy The American College of Clinical Pharmacy and the authors
Department of Pharmacy thank the following individuals for their careful review of the
Mayo Clinic Pulmonary and Endocrinology II chapters:
Rochester, Minnesota
Ishaq Lat, Pharm.D., MBA, FCCP, FCCM, BCCCP
Andrea M. Nei, Pharm.D., BCPS, BCCCP Director of Pharmacy
Critical Care Pharmacist Shirley Ryan Ability Lab
Department of Pharmacy (Formerly the Rehabilitation Institute of Chicago)
Mayo Clinic Chicago, Illinois
Rochester, Minnesota
Marisel Segarra-Newnham, Pharm.D., MPH, FCCP,
BCPS, BCIDP
Reviewers
Clinical Pharmacy Specialist, Infectious Diseases/HIV
Joseph M. Swanson, Pharm.D., FCCM, FCCP
Antimicrobial Stewardship Program Pharmacy Director
Professor Veterans Affairs Medical Center
Department of Clinical Pharmacy West Palm Beach, Florida
and Translational Science Clinical Assistant Professor of Pharmacy Practice
University of Tennessee College of Pharmacy University of Florida College of Pharmacy
Memphis, Tennessee Gainesville, Florida
Debbie Liang, Pharm.D., BCCCP
Clinical Pharmacy Specialist, Cardiac Critical Care
Department of Pharmacy
UNC Rex Healthcare
Raleigh, North Carolina
DISCLOSURE OF POTENTIAL CONFLICTS OF INTEREST
Consultancies: Stephanie Bass (Entasis); Mitchell S. Buckley (Wolters Kluwer); Chris Droege (Deloitte Consulting LLP); Eric Johnson
(Slayback Pharmaceuticals); Patrick M. Wieruszewski (La Jolla Pharmaceutical Company); G. Christopher Wood (ASHP)

Stock Ownership:

Royalties:

Grants: Brittany Bissell (ASHP, University of Kentucky); Chris Droege (ACCP); Alexander H. Flannery (ACCP, American Society
of Nephrology, La Jolla Pharmaceutical Company); Anthony Hawkins (University of Georgia College of Pharmacy); Heather P.
May (ACCP, Mayo Clinic); Andrea Sikora Newsome (NIH); Joseph M. Swanson (Cubist/Merck)

Honoraria: Abby Bailey (ACCP EM PRN); Joseph M. Swanson (United Arab Emirates – Ministry of Health)

Other:

Nothing to disclose: Mahmoud A. Ammar, Paige Garber Bradshaw, Jennifer Falvey, Maria Guido, Leslie A. Hamilton, Sara Jordan
Hyland, Heather A. Johnson, Debbie Liang, Brian P. Murray, Andrea M. Nei, Jaimini S. Patel, Michael A. Peters, Michael A. Rudoni,
Poorvi Shah, Zachary R. Smith, Jaclyn Stoffel, Kyle A. Weant, Maggie Zhao

ROLE OF BPS: The Board of Pharmacy Specialties (BPS) is an autonomous division of the American Pharmacists Association
(APhA). To maintain its strict, independent standards for certification, BPS does NOT endorse or provide review information,
preparatory courses, or study guides for Board Certification Examinations. The Board, through its specialty councils, is respon-
sible for specialty examination content, administration, scoring, and all other aspects of its certification programs. BPS is totally
separate and distinct from ACCP. CCSAP has been approved by BPS for use in BCCCP recertification. Information about the BPS
recertification process is available online.

Questions regarding BCCCP recertification should be directed to:

Board of Pharmacy Specialties


2215 Constitution Avenue NW
Washington, DC 20037
(202) 429-7591
CONTINUING PHARMACY EDUCATION
AND RECERTIFICATION INSTRUCTIONS
Continuing Pharmacy Education Credit: The American College of Clinical Pharmacy is accredited by the Accreditation
Council for Pharmacy Education (ACPE) as a provider of continuing pharmacy education (CPE).

CCSAP Target Audience: The target audience for Pulmonary and Endocrinology is not only ICU and ED pharmacists across the
spectrum of care but also any pharmacist caring for acutely ill patients whose management may be complicated by these
challenging scenarios.

Available CPE credits: Purchasers who successfully complete all posttests for CCSAP 2021 Book 1 (Pulmonary and Endocrinology)
can earn 22.5 contact hours of continuing pharmacy education credit. The universal activity numbers are as follows: Pulmonary
and Endocrinology I, 0217-0000-21-030-H01-P, 5.5 contact hours; Pulmonary and Endocrinology II, 0217-0000-21-031-H01-P,
5.0 contact hours; Pulmonary and Endocrinology III, 0217-0000-21-032-H01-P, 5.5 contact hours; and Pulmonary and Endocrinology IV,
0217-0000-21-033-H01-P, 6.5 contact hours. You may complete one or all available modules for credit. Tests may not be submitted
more than one time.

TO EARN CPE CREDITS FROM THIS CCSAP BOOK

Posttest access: Go to www.accp.com and sign in with your e-mail address and password. Technical support is available from 8 a.m.
to 5 p.m. (Central) weekdays by calling (913) 492-3311. CCSAP products are listed under My Products on your My Account page.

BCCCP test deadline: 11:59 p.m. (Central) on September 15, 2021.


ACPE test deadline: 11:59 p.m. (Central) on March 15, 2024.

BCCCP Recertification Credit: To receive BCCCP recertification CPE credit, a CCSAP posttest must be submitted
within the 6-month period after the book’s release (see above). Only completed tests are eligible for credit; no partial
or incomplete tests will be processed. You may complete one or all available modules for credit. Tests may not be
submitted more than one time.

The passing point to earn BCCCP recertification credit is based on an expert analysis of the assessment items in each posttest
module. Any posttest submitted before the BCCCP test deadline that meets this passing point will earn BCCCP recertification
credits. These credits will be assigned as of the date of test submission and reported within 48 hours to BPS. For statements of
recertification credit, visit www.bpsweb.org.

Questions regarding the number of hours required for BCCCP recertification should be directed to BPS at (202) 429-7591 or
www.bpsweb.org. The ACCP Recertification Dashboard is a free online tool that can track recertification credits as they are
earned through ACCP and schedule new opportunities for credits from upcoming ACCP professional development programs.

ACPE CPE Credit: To receive ACPE CPE credit for a CCSAP module, a posttest must be submitted within 3 years after the book’s
release (see above). Only completed tests are eligible for credit; no partial or incomplete tests will be processed. You may complete
one or all available modules for credit. Tests may not be submitted more than one time.

Any posttest submitted before the ACPE deadline that scores 50% or greater will be awarded the appropriate CPE. These
credits will be assigned as of the date of test submission and reported within 48 hours. For statements of CPE credit, visit
www.mycpemonitor.net.

Posttest answers: The explained answers—with rationale and supporting references—will be posted 2 weeks after the BCCCP
test deadline and will be available to anyone who has either (1) submitted a posttest or (2) waived the right to receive credit from
a posttest (see below). Go to www.accp.com and sign in with your e-mail address and password. Click the CCSAP book on your
My Account page and you will see a link to the explained answers.

Test Waivers: To access the explained answers without submitting a posttest, sign in to your My Account page, select the
CCSAP book, and click on the waiver link for that module. By completing the waiver form for a module, you waive the opportunity
to receive CPE credit for that module. After you submit a waiver, you will see a link to the PDF file that contains the answers for
the module you waived. Answers will be available starting 2 weeks after the BCCCP test deadline.
Drug-Induced Pulmonary Disorders
and Pulmonary Drug Delivery
By Heather P. May, Pharm.D., FCCM, BCCCP, BCPS; and Andrea M. Nei, Pharm.D., BCPS, BCCCP

Reviewed by Joseph M. Swanson, Pharm.D., FCCM, FCCP; and Debbie Liang, Pharm.D., BCCCP

LEARNING OBJECTIVES

1. Evaluate the potential for drug-induced lung injury as part of the differential diagnosis.
2. Distinguish among the various causes of drug-induced lung injury.
3. Design an appropriate treatment plan for different types of drug-induced lung injuries.
4. Assess the use of adjunctive inhaled/intrapulmonary mediations.
5. Develop a treatment plan for use of adjunctive inhaled/intrapulmonary medications.

INTRODUCTION
ABBREVIATIONS IN THIS CHAPTER
Available evidence describing safe and effective treatments for
BAL Bronchoalveolar lavage
drug-induced pulmonary disorders and pulmonary drug delivery is
DAH Diffuse alveolar hemorrhage
growing, but a standard of care has yet to be established in many
DNase Deoxyribonuclease
areas. Pharmacists play a key role in the following aspects of care
EVALI E-cigarette, or vaping, product
use-associated lung injury for these patients: (1) making pulmonary disease management
HAP Hospital-acquired pneumonia recommendations; (2) identifying appropriate opportunities for
intrapulmonary medications; (3) optimizing safety and efficacy in
ICI Immune checkpoint inhibitor
drug dosing and administration; and (4) ensuring appropriate moni-
ILD Interstitial lung disease
toring for adverse effects and complications. This chapter describes
IQR Interquartile range
existing knowledge as well as new insights in the areas of drug-
MDR Multidrug resistant
induced pulmonary disorders and pulmonary drug delivery that
rFVIIa Recombinant activated factor VII
ensure pharmacists can maximize their impact in each of these
THC Tetrahydrocannabinol
functions.
tPA Tissue plasminogen activator
TXA Tranexamic acid
DRUG-INDUCED INTERSTITIAL LUNG
VAP Ventilator-acquired pneumonia
DISEASE
Table of other common abbreviations. Interstitial lung disease encompasses a heterogeneous group of lung
diseases characterized by fibrotic and/or inflammatory changes of
the lung parenchyma (Faverio 2018). This type of lung disease can
be caused by drugs, connective tissue and autoimmune diseases, or
environmental exposures, or it may be idiopathic. Patients may pres-
ent to the ICU with acute respiratory failure resulting from de novo
ILD or exacerbation of underlying ILD. A wide range of clinical presen-
tations and radiologic and histopathologic changes associated with
drug-induced ILD are documented in the literature, many of which are
nonspecific (Skeoch 2018). Diagnosis often relies on the following:
careful exclusion of other causative factors; establishing a temporal
association between introduction of the drug in question and symp-
tom onset; improvement after discontinuation of the offending agent;
and recurrence of symptoms following its reintroduction. Broadly,

CCSAP 2021 Book 1 • Pulmonary and Endocrinology 73 Drug-Induced Pulmonary Disorders


several risk factors for ILD have been described, including been described, including malaise, cough, fever, and pleu-
extremes of age, male sex, select ethnicities (Japanese and ritic chest pain. Imaging most often reveals bilateral patchy
African American, in some cases), underlying lung disease opacities and evidence of pneumonitis, but amiodarone-re-
(e.g., chronic obstructive pulmonary disease), and radiation lated ILD can also mimic acute respiratory distress syndrome
exposure (Jackevicius 2011). This chapter discusses select and bronchiolitis obliterans organizing pneumonia (Camus
causes of drug-induced ILD that may prompt ICU admission. 2020, Skeoch 2018, Ashrafian 2001). Although peak onset
Many other possible drug causes of ILD are discussed in is considered to be within 6–12 months of treatment ini-
detail at www.pneumotox.com, an online data repository of tiation, symptoms may occur within a few days or, in some
reported patterns and causes of ILD. This resource was cre- cases, after years of therapy (Camus 2004). An estimated
ated and is maintained by leading experts in drug-induced 3%–6% of patients treated with amiodarone may develop
and iatrogenic respiratory diseases. related ILD (also referred to as amiodarone pulmonary toxic-
ity or amiodarone lung). Because the presentation may mimic
Amiodarone other common respiratory illnesses such as acute respi-
Amiodarone is one of the more well-described medications ratory distress syndrome, amiodarone-related ILD may be
to which ILD has been attributed, with more than 200 cases under-recognized (Dhokarh 2012, Ashrafian 2001). The risk
documented in the literature. This effect is thought to be appears to be dose-dependent (Jackevicius 2011), with the
caused in part by high concentrations of metabolites, such strongest association seen with high doses (e.g., minimum
as toxic oxygen radicals, accumulating in the lung paren- 400 mg/day) and long durations of exposure (e.g., 200 mg
chyma, although the exact mechanism of injury is unknown daily for more than 2 years) (Skeoch 2018).
(Ashrafian 2001). A broad spectrum of symptoms has Discontinuation of amiodarone therapy is the first step in
treatment. Avoiding oxygen delivery at high concentrations
(e.g., greater than 50% fraction of inspired oxygen) is recom-
mended because of concern for perpetuating lung injury from
toxic oxygen radicals. Steroids have also been used in many
BASELINE KNOWLEDGE STATEMENTS
cases of amiodarone-related ILD, despite a lack of compara-
Readers of this chapter are presumed to be familiar tive effectiveness trials or other evidence showing definitive
with the following: benefit (Camus 2004). Case reports describe starting ste-
roids in response to a lack of significant improvement with
• General knowledge of the pathophysiology and
possible manifestations of interstitial lung disease drug discontinuation alone. Available data are from nonran-
• Drugs included in various chemotherapeutic domized studies that lack pre-specified selection criteria,
classes conferring selection bias, and fail to describe detailed or pro-
• Classification of pleural effusion as exudative or tocolized treatment regimens. A wide variety of doses are
transudative reported in the available case-reports, ranging from 40–500
• Guideline recommendations for antimicrobial mg methylprednisolone equivalents (Skeoch 2018), although
selection for hospital-acquired and ventilator- the most common starting dose is 0.5–1 mg/kg. Many reports
acquired pneumonia describe symptomatic worsening if steroids are tapered
• Fundamental medication dosing in adult cardiac too quickly, which some interpret as support for their place
arrest in the treatment of amiodarone-induced ILD. Steroid tapers
are often prolonged to at least 3 months, with symptomatic
Table of common laboratory reference values.
recovery ensuing over a median of 36 months. In some cases,
this strategy may be guided by radiographic and symptom-
ADDITIONAL READINGS atic improvement.
The following free resources have additional back-
ground information on this topic: Chemotherapy- and Radiation-Induced Lung
Injury
• Pneumotox Online. The Drug-Induced Respiratory
Disease Website. Chemotherapeutic agents are the leading cause of drug-in-
duced ILD reported in the literature, accounting for
• Lara AR, Schwarz MI. Diffuse alveolar hemorrhage.
Chest 2010;137:1164-71 25%–50% of described cases (Skeoch 2018, Dhokarh 2012).
• Crotty Alexander LE, Ware LB, Calfee CS, et al. The most commonly cited offending agents are bleomycin,
E-cigarette or vaping product use associated lung gemcitabine, methotrexate, epidermal growth factor recep-
injury (EVALI): developing a research agenda. An tor-targeted agents (e.g., tyrosine kinase inhibitors), and
NIH workshop report. Am J Respir Crit Care Med mammalian target of rapamycin inhibitors. Although sys-
2020;202:795-802 tematic study of ILD associated with each drug is difficult,
several specific features have been iteratively described and

CCSAP 2021 Book 1 • Pulmonary and Endocrinology 74 Drug-Induced Pulmonary Disorders


are available for review at www.pneumotox.com. A popula- contributor, but there may be other causes as well (Aberegg
tion-based cohort study of ICU patients with a diagnosis of 2020, Blount 2020). It is challenging to identify specific
drug-induced ILD revealed that amiodarone is commonly causes of EVALI given the heterogenous nature of devices,
used in patients who have previously received these che- methods of delivery, and variety of substances used (Table 1).
motherapeutic agents (Dhokarh 2012). In addition, affected One study found more than 500 chemicals in the tested
patients may have previous or concurrent exposure to radia- medical-grade and counterfeit cartridges (Muthumalage 2020).
tion. Both of these conditions are hypothesized to potentiate Characteristics associated with higher odds of EVALI include
the risk of ILD (Skeoch 2018) and may increase the clinical younger age, male sex, use of THC-containing products, and
suspicion for drug-induced ILD if other causes of respiratory obtaining products from an illicit source (Aberegg 2020).
failure have been sufficiently ruled out. Given the variety of Clinical features of EVALI are noted in Table 2. A spectrum
drug exposures often experienced by patients with cancer, of vaping-associated lung diseases has been suggested,
pharmacists are encouraged to complete a comprehensive including acute respiratory distress syndrome, lipoid pneu-
medication history that includes previous chemotherapy monia, acute eosinophilic pneumonia, diffuse alveolar
regimens to determine if exposure to these agents, or other hemorrhage, and respiratory bronchiolitis-ILD, among oth-
drugs indicated in drug-induced ILD, are present. The website ers. Radiologic findings include bilateral pulmonary opacities
at www.pneumotox.com provides a complete listing. on chest radiography or CT. Less commonly, patients also
After discontinuing the suspected chemotherapeutic present with pneumomediastinum or pneumothorax, which
agent, steroids have been used for patients in whom symp- are possibly secondary to puff mechanics during vaping or
toms fail to resolve in the subsequent 24–48 hours (Camus mechanical disruption of the lung parenchyma.
2004). As with amiodarone, evidence in support of their ben- In the absence of specific diagnostic tests, EVALI is often
efit is lacking, and publication bias is a concern because of considered a diagnosis of exclusion, with features overlap-
under-reporting of negative results, although experts gen- ping that of infectious pneumonia or other inflammatory lung
erally agree this treatment is beneficial (Camus 2004). In diseases (Crotty Alexander 2020, Aberegg 2020). Clinical
particular, patients with suspected radiation-induced ILD testing for infectious causes, including bacterial, viral, fun-
have responded quickly to steroid therapy in published case gal, and Pneumocystis jirovecii, is warranted depending on
reports. If initiating steroid treatment, doses of 1 mg/kg patient risk factors. Empiric antibiotics for likely pathogens
methylprednisolone equivalents may be considered based according to patient risk factors are often instituted first.
on published reports (Skeoch 2018), followed by a prolonged Other inflammatory diagnoses may be considered with pre-
taper. Insufficient data or expert recommendations are avail- disposition to alternative diagnoses or presence of atypical
able to guide steroid tapering in this setting. Small, stepwise symptoms. The role of bronchoscopy in diagnosis of EVALI
dose reductions (e.g., 10 mg/day) on a weekly basis, com- is individualized, taking into account the likelihood of alter-
bined with close monitoring of respiratory symptoms, have native diagnoses and the risks of the procedure. Lipid-laden
been observed in current clinical practice. It is also advisable macrophages are a common finding but may not be spe-
to avoid oxygen delivery at high concentrations to avoid per- cific to EVALI and may be a marker of vaping; however, their
petuating further lung injury. absence is uncharacteristic of EVALI and may warrant fur-
ther investigation into other diagnoses. In addition, BAL
OTHER CAUSES OF DRUG-INDUCED cytology reflects macrophage and neutrophil predominance
with scant eosinophils, which is nonspecific. Detection of
LUNG INJURY
vitamin E acetate in the BAL sample may have a future role
E-Cigarette, or Vaping, Product Use-Associated
in diagnosis, but currently it is only investigational. Reported
Lung Injury
histopathologic findings are variable, with findings globally
Electronic nicotine delivery devices, or e-cigarettes, were first described as diffuse lung injury and inflammatory patterns
introduced to the market in 2007, and their use has increased (Aberegg 2020).
over the past few years with expanded use beyond nicotine Treatment of EVALI is largely supportive. Corticosteroids
products, including THC and cannabidiol. A World Health may be considered, but have not been formally studied;
Organization report documented 41 million e-cigarette users, therefore, the decision to administer should be individual-
or vapers, worldwide in 2018. The term EVALI refers to e-ciga- ized. Evidence to date is only in the form of case reports or
rette, or vaping, product use–associated lung injury, which is cohort studies (Choe 2020). In general, corticosteroids have
a respiratory illness linked to vaping that can be severe and been used across the spectrum of severity, from outpatient
life-threatening. The EVALI outbreak was recognized by the settings to the ICU, with reported improvement in symp-
CDC in August 2019; as of January 2020, more than 2700 cases toms in most cases (Blagev 2019, Layden 2020, Heinzerling
and 60 deaths have been reported across the United States. 2020). Intravenous methylprednisolone may be initiated first
The exact causes for EVALI are still under investigation. depending on illness severity, with transition to prednisone
Recent evidence implicates vitamin E acetate as a potential for completion of therapy. Moderate doses and short courses

CCSAP 2021 Book 1 • Pulmonary and Endocrinology 75 Drug-Induced Pulmonary Disorders


Table 1. Etiologic Factors in EVALI

Devices • Patients often use several different devices


• Individual devices and tools used specifically for inhalation of concentrated THC (e.g., dab, wax, honey)
• Differences in configuration and output for electrical heating power, which affects emissions of toxicants
• Certain vaping device and liquid combinations may generate toxic exposures
Vaping liquids • Patients often use several liquids and more than one active agent
and chemicals • No single type or brand has been linked with cases
• Unclear if chronic inhalation of certain nicotine constituents (PG or VG) change the inflammatory state of
the lung to create a higher propensity for reacting to constituents in CBD or THC

Solvents • Nicotine contains hydrophilic solvents (PG or VG)


• THC requires extraction with butane or carbon dioxide, then distillation/purification, and finally addition of
PG/VG, terpenes, and lyophilic compounds (termed cutting agents, including vitamin E acetate, coconut oil
and medium chain triglycerides)
• Mixing of substances may occur
• Thermal stability of solvents is variable and may generate toxic byproducts

Flavors • Hundreds of chemicals identified and under investigation

Vaping behaviors • Puff topography and breath holding


| Direct mechanical effects

| Change in aerosol particle properties

| Alter exposure levels

• May be used in addition to conventional cigarette and marijuana smoking

CBD = cannabidiol; EVALI = e-cigarette, or vaping, product use-associated lung injury; PG = propylene glycol; THC = tetrahydrocannab-
inol; VG = vegetable glycerin.
Information from: Crotty Alexander LE, Ware LB, Calfee CS, et al. E-cigarette or vaping product use associated lung injury (EVALI):
developing a research agenda. An NIH workshop report. Am J Respir Crit Care Med 2020;202:795-802.

are often reported. One case series reported a median ini- days (IQR 10–34) and 7 days (IQR 5–11) for patients in the ICU
tial dose of either intravenous methylprednisolone at 125 and inpatient ward, respectively. Patients should also be coun-
mg/day (IQR 120–240) or prednisone at 50 mg/day (IQR seled on nicotine and/or THC cessation because resumption
40–60) (Blagev 2019). Median duration of steroids was 15 may cause disease relapse (Crotty Alexander 2020).

Table 2. Clinical Features of EVALI

Symptoms Respiratory: dyspnea, chest or pleuritic pain, cough, hemoptysis


GI: nausea, emesis, abdominal pain, diarrhea
Constitutional: fever, fatigue, weight loss, chills
Duration of Symptoms Commonly acute: hours to days
Subacute also reported: weeks to months

Clinical Findings Hypoxemia


Tachypnea
Tachycardia

Laboratory Findings Elevated inflammatory markers: erythrocyte sedimentation rate or C-reactive protein
Elevated transaminases
Leukocytosis with neutrophilic predominance

EVALI = E-cigarette, or vaping, product use-associated lung injury.


Information from: Crotty Alexander LE, Ware LB, Calfee CS, et al. E-cigarette or vaping product use associated lung injury (EVALI):
developing a research agenda. An NIH workshop report. Am J Respir Crit Care Med 2020;202:795-802; Aberegg SK, Maddock SD,
Blagev DP, et al. Diagnosis of EVALI. Chest 2020;158:820-7.

CCSAP 2021 Book 1 • Pulmonary and Endocrinology 76 Drug-Induced Pulmonary Disorders


Immune Checkpoint Inhibitor-Pneumonitis One irAE encountered in the ICU is ICI-pneumonitis. This
Immune checkpoint inhibitors are a growing class of mono- condition is occasionally referred to as another cause of
clonal antibodies revolutionizing the treatment of a wide drug-induced ILD; however, most clinicians and research-
variety of solid tumor and hematologic neoplasms. As of ers treat it as a specific typology of lung disease. Although
mid-2020, seven ICIs have received FDA label approval for 16 early clinical investigations reported this complication in only
different indications and an exponentially growing number 2%–9% of patients (Da 2020, Weber 2017, Naidoo 2015), newer
of registered clinical trials are using these agents (Table 3). studies, particularly those conducted in patients with non-
The mechanism of action is to block intrinsic down-regula- small cell lung cancer, cite an incidence of up to 19%–34%
tors of immunity, such as programmed cell death 1 (PD1) or (Peters 2019, Suresh 2019). A lack of clear diagnostic criteria
its ligand (PD-L1) and cytotoxic T-lymphocyte associated and a broad differential diagnosis lead to the concern that ICI-
protein 4. In T cell signaling, two important connections pneumonitis is often under-recognized and underdiagnosed
are made between a T cell and antigen-presenting cell: the (Larsen 2019). Despite these challenges, ICI-pneumonitis
first signal is between the T cell receptor and antigen; the should be suspected in select patients receiving ICI therapy.
second signal determines if a T cell becomes activated or Although the risk factors have yet to be fully elucidated, higher
inactivated and involves co-stimulatory or co-inhibitory rates are reported in patients with non-small cell lung cancers
domains. When an ICI blocks co-inhibitory domains (e.g., who are receiving combination ICI therapy or concurrent radi-
PD1), T cells become activated, enhancing the ability of the ation and being re-challenged with an ICI after experiencing
immune system to destroy neoplastic cells. Unfortunately, a previous irAE (Simonaggio 2019, Sears 2019). Symptoms
this mechanism also gives rise to an array of immune-re- may present as mild, subclinical disease or more acutely and
lated adverse events (irAEs), which are generally attributed include shortness of breath or dyspnea on exertion, low-grade
to overactivation of the immune system (Postow 2018). In fever, fatigue, nonproductive cough, and chest pain (Larsen
some cases, however, irAEs may also represent true phar- 2019, Naidoo 2015). The timing of presentation varies from
macologic adverse effects (Sears 2019). As ICI use becomes 9 days to 19 months after initiation of or rechallenge with an
increasingly prevalent, pharmacists in critical care settings ICI, with some cases reported after administration of a single
must be able to recognize and recommend appropriate care dose (Dolladille 2020, Larsen 2019, Simonaggio 2019, Naidoo
strategies for irAEs. 2017). Given this wide range of time to presentation, obtaining

Table 3. Immune Checkpoint Inhibitors and Approved Indications

Pharmacologic Target Approved Indicationsa

Programmed cell death 1

Nivolumab Hodgkin disease; hepatocellular carcinoma; melanoma; mesothelioma; colorectal and


urothelial cancer; small cell and non-small cell lung cancer; head, neck and esophageal
cancer; renal cell carcinoma

Pembrolizumab Bladder, cervical, colorectal cancer; endometrial carcinoma; esophagogastric


adenocarcinoma; gastric cancer; head and neck cancer; Hodgkin disease; hepatocellular
carcinoma; melanoma; Merkel cell carcinoma; small cell and non-small cell lung cancer;
large B-cell lymphoma; renal cell carcinoma; squamous cell carcinoma

Cemiplimab Squamous cell carcinoma of the skin

Programmed cell death ligand 1

Atezolizumab Small cell and non-small cell lung cancer; hepatocellular carcinoma; melanoma; urothelial
carcinoma; breast cancer

Avelumab Merkel cell carcinoma; urothelial carcinoma; renal cell carcinoma

Durvalumab Small cell and non-small cell lung cancer; urothelial carcinoma

Cytotoxic T-lymphocyte associated protein 4

Ipilimumab Hepatocellular carcinoma; melanoma; colorectal cancer; non-small cell lung cancer; renal
cell carcinoma

a
Current as of August 2020; excludes other Orphan Drug designations and investigational uses.

CCSAP 2021 Book 1 • Pulmonary and Endocrinology 77 Drug-Induced Pulmonary Disorders


a thorough medication history including all anti-cancer ther- Diffuse Alveolar Hemorrhage
apies received within the past 1–2 years is an important task Bleeding from the lungs can originate from bronchial vessels
for pharmacists. Radiographic evidence commonly shows (e.g., bronchiectasis), pulmonary vessels (e.g., systemic vas-
bilateral ground glass and/or nodular consolidative opacities culitis) or the microcirculation, as is the case in DAH. The term
and occasionally pleural effusion. Histopathologic findings diffuse alveolar hemorrhage broadly refers to accumulation of
include organizing pneumonia and non-necrotizing granulo- intra-alveolar red blood cells from the alveolar capillaries, and
mas in most cases, with diffuse alveolar damage and severe the clinical syndrome includes some combination of hemop-
lung injury evident in select cases (Larsen 2019, Naidoo 2017). tysis, anemia, diffuse pulmonary infiltrates on radiography,
It is important to rule out other potential causes of present- and hypoxemic respiratory failure. Although hemoptysis is
ing symptoms, most notably infection, disease progression, considered the cardinal symptom, it can be absent in 33%
or other drug- or treatment-related adverse effect (e.g., radia- of cases, particularly in the setting of hematopoietic cell
tion pneumonitis). Distinguishing between bacterial or fungal transplant. Universally accepted criteria for diagnosis do not
infection and other causes, including ICI-pneumonitis, has exist; however, progressively bloody return from BAL fluid in
important implications for clinical management, primarily for consecutive aliquots is typically considered diagnostic for
the role of steroids. DAH. Hemosiderin-laden macrophages accumulate in the
The first step in treatment of ICI-pneumonitis is hold- alveoli as a result of the red blood cell and fibrin deposition
ing further doses of the ICI. After ruling out infection, and a threshold of greater than 20% in BAL fluid should also
steroids may be started in symptomatic patients for whom raise suspicion for DAH. These findings take 48–72 hours to
ICI-pneumonitis is a likely diagnosis. Unfortunately, no sys- develop, however, so the timing of BAL relative to DAH onset
tematic studies have investigated the optimal timing, dose, may influence the ability to make a diagnosis based on this
and duration of steroid therapy. Practice guidelines recom- factor alone. It is important for pharmacists to have a basic
mend prednisone or methylprednisolone 1–2 mg/kg/day, understanding of each of the numerous histologic subtypes
with a slow taper by 5–10 mg/week beginning once symp- and etiologies (Table 4) because evidence related to treat-
toms improve. Patients may receive bursts of high-dose ment options may not be generalizable across the spectrum
steroids (e.g., methylprednisolone 1 g/day) if symptoms of DAH. Causes may be broadly categorized as immune medi-
worsen despite steroid initiation at lower doses. If symp- ated or non-immune mediated. Another important distinction
toms improve after 3–5 days of high-dose steroids, most to make is that DAH is not synonymous with acute respiratory
are decreased to 1 mg/kg/day to begin tapering. Tapers typ- distress syndrome, although patients with DAH may often
ically conclude after a total of 4–6 weeks (Brahmer 2018). meet these criteria.
Patients who do not experience improvement after 48 hours The rarity of DAH and heterogenic underlying etiologies
of high-dose steroids may be considered steroid non-respon- contributes to the limited evidence in support of the treat-
sive and candidates for other systemic immunosuppression ment options listed in Table 4. Most evidence is derived from
like mycophenolate mofetil 1g twice daily, infliximab 5 case series and little data published in recent years relates
mg/kg, a 5-day course of intravenous immune globulin, or to treatment. Systematic evaluation has been most often
cyclophosphamide. Limited data are available to guide sec- performed in the population undergoing hematopoietic cell
ond-line therapy and is therefore chosen based on provider transplant. Initially, a historical cohort study found that treat-
preference or a concomitant irAE, if present. Occasionally ment with methylprednisolone greater than 30 mg/day was
symptoms will rebound while tapering steroids, prompting associated with an increased chance of 100-day survival
an increase in dose until improvement is noted and overall (Metcalf 1994). However, this study was affected by many
extending steroid exposure (Brahmer 2018). Consequences limitations, including non-standardized steroid doses and
of immunosuppressant use are still being evaluated. tapering schedules, prescribing bias, and inconsistencies in
Immunosuppression and prolonged steroid use in patients data reporting. The authors state that steroid regimens typi-
with ICI-pneumonitis significantly increased risk of infec- cally consisted of methylprednisolone 500–1000 mg/day for
tion such as pneumonia and bacteremia (Del Castillo 2016) 3 days followed by a subsequent taper over 2–4 weeks based
and appropriate preventive measures such as Pneumocystis on respiratory symptoms. This strategy has commonly been
prophylaxis should be considered (Brahmer 2018). Limited used ever since these data were reported. Although no sta-
retrospective data also indicate steroid use may decrease tistically significant difference was noted in the incidence
the anti-tumor activity of ICI therapy and result in worse out- of infection between study groups, the small number of
comes (Arbour 2018), although results are conflicting (Weber patients (63) likely limited the statistical power to detect any-
2017, Horvat 2015). Although significantly more research is thing but a very large effect size, and evaluation for infection
needed to best understand how to diagnose and treat this was not standardized. Although subsequent case reports or
complication of ICI, the increasing prevalence of this therapy series note perceived response to steroid therapy (Raptis
requires pharmacists in critical care settings to familiarize 1999), the initial positive results have not been reproduced
themselves with the evolving data. in more recent studies. Another historical cohort study of

CCSAP 2021 Book 1 • Pulmonary and Endocrinology 78 Drug-Induced Pulmonary Disorders


Table 4. Select Etiologies and Histologic Subtypes of Diffuse Alveolar Hemorrhage

Etiology Histology Treatment Options

Systemic lupus erythematosus a


Pulmonary capillaritis Steroids + cyclophosphamide

Microscopic polyangiitisa Pulmonary capillaritis Steroids + cyclophosphamide, plasmapheresis

Antineutrophil cytoplasmic antibody- Pulmonary capillaritis Steroids + cyclophosphamide, plasmapheresis,


associated granulomatous vasculitisa rituximab

Connective tissue diseasea Pulmonary capillaritis Steroids +/–cyclophosphamide, rituximab


Rheumatoid arthritis
Polymyositis
Antiphospholipid syndrome

Anti-glomerular basement membrane Bland pulmonary hemorrhage Steroids + cyclophosphamide, rituximab


diseasea

Hematopoietic cell transplantb Diffuse alveolar damage Supportive care, steroids

Lung allograft rejection b


Diffuse alveolar damage Steroids

Infection b
Diffuse alveolar damage Supportive care, treatment of the underlying
infection

a
Immune-mediated.
b
Non-immune-mediated.
Information from: Martínez-Martínez MU, Oostdam DAH van, Abud-Mendoza C. Diffuse alveolar hemorrhage in autoimmune
diseases. Curr Rheumatol Rep 2017;19:27; Lara AR, Schwarz MI. Diffuse alveolar hemorrhage. Chest 2010;137:1164-71.

patients with infectious and noninfectious DAH compared High-dose methylprednisolone is the cornerstone for
patients who received protocolized high-dose steroid treat- treatment of various types of immune-mediated DAH (see
ment to those who did not (Majhail 2006). The decision Table 4) (Mirouse 2020, Martínez-Martínez 2017, Lara 2010).
to use steroids was at the discretion of the treating pro- Collectively, case series and small clinical trials report daily
vider. No difference in 60-day survival was noted. A more doses of 1000 mg daily for a minimum of 2–5 days before
recent historical cohort study compared patients receiving a taper to 1–2 mg/kg/day is considered. Steroids may be
methylprednisolone as low-dose (less than 250 mg/day), combined with cyclophosphamide for some conditions.
medium-dose (250–1000 mg/day), or high-dose (greater Cyclophosphamide dosing strategies vary by indication and
than 1000 mg/day), with or without aminocaproic acid as a treatment should be designed in consultation with experts
4 g intravenous bolus followed by a 1 g/hour infusion (Rathi from rheumatology or other specialists with experience pre-
2015). Although only 119 patients were studied and only 37 scribing and dosing, including hematology pharmacists.
received steroids alone, the low-dose steroid group had sig- Plasmapheresis has been used in refractory cases but does
nificantly lower ICU and hospital mortality. The addition of not appear to improve survival (Martínez-Martínez 2017).
aminocaproic acid did not result in a difference in mortal- Recombinant activated factor VII has been investigated for
ity, ventilator days, or ICU or hospital length of stay. Taken the treatment of DAH from several etiologies (Ahya 2017, Lara
together, results from published studies must be carefully 2010). Use of rFVIIa failed to improve outcomes and increased
interpreted because of significant methodologic limitations. thrombotic events when given intravenously; therefore, this
The failure to reproduce positive results and the potential agent is not routinely recommended. Intrapulmonary delivery
harm associated with high doses and long durations of ste- of rFVIIa is discussed in further detail later in the chapter.
roid exposure mandate caution when considering the role for
this treatment in patients with DAH undergoing hematopoi- INHALED MEDICATIONS
etic cell transplant. Steroids may be considered for patients
Inhaled Thrombotics
with severe and progressive symptoms despite high-quality
supportive care. When used, data support consideration of Tranexamic Acid
lower doses (e.g., methylprednisolone 250 mg/day) and an Interest has been increasing in the use of inhaled or intrapul-
effort should be made to evaluate for a positive response monary delivery of TXA for patients with hemoptysis. As an
before a committing to long-term treatment. antifibrinolytic agent, TXA has been effective in treatment

CCSAP 2021 Book 1 • Pulmonary and Endocrinology 79 Drug-Induced Pulmonary Disorders


Patient Care Scenario
A 55-year-old man with a medical history of metastatic pulmonary disease exacerbation, and fluid overload. On
non-small cell lung cancer, hypertension, and smoking rounds the multidisciplinary team is discussing whether
presents to the ICU with hypoxemic respiratory failure. any other potential diagnoses should be considered. What
The critical care team begins to work-up potential diag- strategy is best to recommend for this patient?
noses including bacterial pneumonia, chronic obstructive

ANSWER:
It is important to perform a thorough medication history pharmacist should investigate the temporal association
to determine what drugs the patient has been treated with between introduction of the drug in question and symp-
over the past year. The pharmacist should make specific tom onset, while continuing to exclude other causative
inquiries about any chemotherapy, immunotherapy, and factors. Discontinuation of the drug in question may be
radiation that have been used to treat his cancer. This infor- recommended if a specific drug-induced pulmonary dis-
mation may help identify whether there is a likely cause of ease is suspected and disease-specific monitoring or
drug-induced pulmonary disease contributing to his cur- therapy should be considered. Progression of the underly-
rent condition, such as ICI-pneumonitis or drug-induced ing malignancy, radiation-induced lung injury, and fungal
ILD. Primary literature and resources such as pneumotox or viral infection are among those factors that should be
.com may be used to identify potential offending agents. considered in the context of an immunocompromised
If drug-induced pulmonary disease is suspected, the patient.

1. Faverio P, De Giacomi F, Sardella L, et al. Management of acute respiratory failure in interstitial lung diseases: overview and clinical
insights. BMC Pulm Med 2018;18:70.
2. Larsen BT, Chae JM, Dixit AS, et al. Clinical and histopathologic features of immune checkpoint inhibitor-related pneumonitis. Am J
Surg Pathol 2019;43:1331-40.

and prevention of bleeding when used orally, intravenously, inhaled TXA 250–500 mg every 6–12 hours in 18 of 19 chil-
and topically. Several case reports described resolution of dren (O’Neil 2020). No association was observed between
hemoptysis with nebulized TXA doses ranging from 250– days of bleeding, TXA dose or TXA treatment duration and
1000 mg delivered every 6–8 hours. Hemoptysis was most survival. No major adverse events attributed to the therapy
often attributed to bronchiectasis or cancer and a response were described. Initial research suggests a potential role for
to TXA was often seen after a single dose (Komura 2018, inhaled TXA in the treatment of hemoptysis; however, the
Calvo 2016, Solomonov 2009). These successes prompted validity and generalizability of these results should be care-
a randomized controlled trial wherein adults with moderate fully considered. In addition, safety has yet to be thoroughly
volume hemoptysis were randomized to nebulized TXA 500 evaluated because the collective number of patients reported
mg/5 mL of normal saline or placebo three times daily for to have received this treatment remains small and results are
up to 5 days (Wand 2018). Patients with respiratory or hemo- likely influenced by publication bias. Pharmacists should be
dynamic instability and kidney or liver dysfunction were aware of the doses and delivery methods that have been used
excluded. Most patients had hemoptysis caused by lung successfully and may consider recommending this therapy
disease (primarily bronchiectasis) or malignancy, and more in patients with persistent, moderate-volume hemoptysis
than 50% were on anticoagulation and/or antiplatelet ther- who are at imminent risk of clinical deterioration. Examples
apy. Resolution of bleeding by day 5 of treatment occurred may include patients on the verge of intubation or those for
in 96% and 50% of the TXA and placebo groups, respectively whom avoidance of a more invasive procedure is preferred.
(p<0.005), and TXA was associated with a significant reduc- Consideration should be given to the likelihood that patients
tion in expectorated blood between days 2 and 5 (p<0.01). may tolerate the nebulization without further worsening of
Fewer interventional procedures to control bleeding were per- their clinical condition. Patients must be able to maintain
formed in the TXA group, and hospital length of stay appeared adequate oxygenation during the nebulization treatment and
shorter. No adverse reactions were reported, including bron- should have a low baseline risk of bronchospasm.
chospasm, which had been reported in a previous case.
Despite promising results, concerns have been raised about Recombinant Activated Factor VII
the statistical power and methodologic rigor with which Intrapulmonary rFVIIa has also been investigated as a mech-
the study was conducted (Messika 2019). This randomized anism to control bleeding specifically associated with DAH.
controlled trial was followed by a historical cohort study The rationale for its use stems from the presence of tis-
in children that evaluated the effect of dose and treatment sue-factor inhibitors in inflamed lungs. Inhibition of the
duration of nebulized TXA on survival. Results describe ces- FVIIa-tissue factor formation may prevent factor X activation,
sation of pulmonary hemorrhage within 48 hours of starting a necessary step toward restoration of hemostasis. The two

CCSAP 2021 Book 1 • Pulmonary and Endocrinology 80 Drug-Induced Pulmonary Disorders


largest adult case series include 12 combined patients with
Box 1. Factors Affecting Use of Inhaled
DAH secondary to several etiologies, such as hematopoi-
Antibiotics
etic cell transplant, autoimmune disorders, infection (Baker
2016, Heslet 2006). Findings were resolution of intrapul- Antibiotic Factors
monary bleeding and improvement in partial pressure of • Optimal delivery/efficacy
○ Concentration-dependent killing
oxygen/fraction of inspired oxygen ratios after instillation
○ Not degraded by delivery process
of 30–50 mcg/kg/dose delivered once or twice on consec- ○ Penetrate sputum and lung secretions
utive days. For these studies rFVIIa was diluted in 50 mL ○ Particle size of 3–5 microns
of normal saline, divided, and distilled in each lung during ○ Remains active in lung
bronchoscopy. In addition, two case series describe use of • Safety/adverse effects
rFVIIa in children with DAH who were refractory to previous ○ Minimal systemic absorption
therapies, including inhaled TXA in some cases (Park 2015, ○ Not injurious to airway surface
○ Preservative free
Bafaqih 2015). Doses were similar to those used in adults,
○ pH, tonicity, and osmolality adjusted
and results describe successful resolution of bleeding in
10 of 12 patients. No thromboembolic events after intrapul- Nebulizer Factors
monary rFVIIa were reported in published literature. The • Nebulizer device: variation in particle delivery size, tempera-
ture/stability of antibiotic solution and amount of residual
heterogeneity of reported cases using intrapulmonary rFVIIa solution after delivery completion
and the absence of a controlled trial or thorough examination ○ Jet
for adverse events make it difficult to determine the appro- ○ Ultrasonica
priate place in therapy for treatment of DAH. The potential ○ Vibrating mesha
to hasten resolution of bleeding and improve oxygenation • Nebulizer positioning relative to Y-piece of ventilator:
location depends on synchronized versus continuous de-
must be weighed against possible adverse effects and asso-
livery and nebulizer size; improper positioning may impact
ciated drug costs. It may be reasonable for pharmacists to delivery
consider recommending intrapulmonary rFVIIa in patients • Synchronized versus continuous delivery: synchronization
with persistent DAH and poor oxygenation despite the best with inspiration may optimize delivery
supportive care and other proven treatments. The reported Other Factors
successes with doses as low as 30–35 mcg/kg/dose make • Degree of underlying airway and parenchymal disease
this a reasonable approach for balancing the potential risks impact lung deposition
and benefits. • Type of pneumonia (consolidated vs. bronchopneumonia):
antibiotic penetration and lung deposition
Inhaled Antibiotics • Ventilator settings that impact flow turbulence and overall
delivery/deposition: tidal volume, mode of ventilation, hu-
Inhaled antibiotics have been considered as adjunctive treat- midification, expiratory time, and patient synchrony
ment options for pneumonia for many years, particularly for
patients with HAP or VAP and those with MDR organisms. a
Preferred device.
Advances in delivery devices and the understanding of phar- Information from: Niederman MS. Adjunctive nebulized anti-
biotics: what is their place in ICU infections? Front Med
macokinetic delivery of antibiotics, in combination with (Lausanne) 2019;6:99.
increasing antimicrobial resistance, have led to continued
study of this method of antimicrobial delivery. The interest in
use of inhaled antibiotics is founded on the ability to achieve
a higher concentration of antibiotic at the site of infection familiar with risks associated with administration, including
and limiting systemic adverse effects while optimizing cure. respiratory complications such as worsening hypoxemia,
In addition, many factors influence the ability to achieve ade- bronchospasm, and occlusion of the nebulizer device (mesh
quate dosing of antimicrobials in critically ill patients, such nebulizers).
as volume resuscitation and fluid shifts, concurrent inotropes Previous evidence supporting use of inhaled antibiot-
and vasopressors, augmented renal clearance and extra- ics suggested potential benefit for clinical resolution in
corporeal therapies. These complicated factors, in addition patients with resistant pathogens and potential preven-
to a need to achieve higher concentrations for MDR organ- tion of the emergence of resistance. However, this evidence
isms, support the rationale for local pulmonary delivery of was variable and often differed between observational and
antimicrobials. randomized studies. In addition, past studies are hindered
A fundamental understanding of factors impacting nebu- by lack of standardization for aerosol delivery method. The
lized antimicrobial delivery is vital to providers instituting this 2016 Infectious Diseases Society of America Clinical Practice
therapy (Box 1). Standardization of antimicrobial selection, Guidelines for the Treatment of Adults with HAP and VAP
dosing, the nebulizer device, and the process of adminis- recommend consideration of inhaled therapy as adjunctive
tration is important. In addition, pharmacists should be treatment for resistant organisms (Kalil 2016). Specifically

CCSAP 2021 Book 1 • Pulmonary and Endocrinology 81 Drug-Induced Pulmonary Disorders


Table 5. Recent Publications for Inhaled Antibiotics in Gram-Negative VAP

Study Feature IASIS trial Hassan et al. INHALE trial

Design Multicenter, randomized, double- Single center, randomized, Multicenter, randomized, double-blind,
blind, placebo-controlled open-label placebo-controlled

Sample size 143 133 725 randomized (508 primary analysis)

Intervention 300 mg amikacin base with Amikacin 300 mg nebulized Amikacin 400 mg nebulized BID or
120 mg fosfomycin or placebo; BID or intravenous amikacin placebo for 10 days
nebulized BID for up to 10 days 20 mg/kg daily
(or time of extubation)

Nebulizer eFlow Inline System; vibrating Mechanically ventilated: simple Mechanically ventilated: vibrated
device plate electronic nebulizer pneumatic nebulizer mesh nebulizer with synchronized
Non-ventilated: ultrasonic inhalation system
nebulizer Non-ventilated: self-administered
handheld nebulizer with chamber

Systemic Meropenem or imipenem Piperacillin/tazobactam Standard-of-care intravenous antibiotics


antibiotics empirically empirically for gram-negative pneumonia

Pertinent • Mechanically ventilated • Positive culture confirming • Mechanically ventilated


inclusion • Diagnosis of pneumonia MDR gram-negative bacilli • Clinical suspicion of pneumonia
• Presence or high suspicion susceptible to amikacin, • Documentation of ≥2 risk factors for
of gram-negative organisms requiring dual antibiotic MDR gram-negative organisms
by gram stain or respiratory therapy
cultures within previous 7 days • Clinical suspicion of HAP or VAP

Findings • No difference in change in CPIS • Improved clinical cure, time • No difference in survival or other
score, clinical cure or mortality to clinical cure, duration of clinical outcomes
• Less positive tracheal cultures ventilation, ICU length of stay • No overall difference in adverse
AFIS vs. placebo and duration of amikacin in effects
• More ventilator-free days in nebulized vs. intravenous arm
placebo group

AFIS = amikacin-fosfomycin inhalation system; CPIS = Clinical Pulmonary Infection Score; HAP = hospital-acquired pneumonia;
MDR = multi-drug resistant; VAP = ventilator-acquired pneumonia.
Information from: Kollef MH, Ricard JD, Roux D, et al. A randomized trial of the amikacin fosfomycin inhalation system for the adjunctive
therapy of gram-negative ventilator-associated pneumonia: IASIS trial. Chest 2017;151:1239-46; Hassan NA, Awdallah FF, Abbassi MM,
et al. Nebulized versus intravenous amikacin as adjunctive antibiotic for hospital and ventilator-acquired pneumonia postcardiac
surgeries: a randomized controlled trial. Crit Care Med 2018;46:45-52; Niederman MS, Alder J, Bassetti M, et al. Inhaled amikacin
adjunctive to intravenous standard-of-care antibiotics in mechanically ventilated patients with gram-negative pneumonia (INHALE):
a double-blind, randomised, placebo-controlled, phase 3, superiority trial. Lancet Infect Dis 2020;20:330-40.

recommended are adjunctive inhaled colistin in addition to placebo for treatment of gram-negative bacterial VAP. All
intravenous polymyxins for carbapenem-resistant pathogens patients received meropenem or imipenem for a minimum of
in HAP/VAP and adjunctive inhaled therapy in VAP caused by 7 days, and antibiotics could be adjusted or added accord-
gram-negative bacilli susceptible only to aminoglycosides or ing to susceptibilities, clinical failure, allergy, or adverse
polymyxins; however, these are both weak recommendations event. The study drug was initiated within 72 hours of intra-
with low-quality evidence. Recently, larger randomized stud- venous antimicrobial initiation for treatment of pneumonia.
ies have provided insight to the place in therapy for inhaled The most common gram-negative organisms isolated were
antibiotics. Despite these important publications, however, Acinetobacter baumannii (93% carbapenem and colistin resis-
many questions remain for optimal use of this therapy in crit- tant) and Pseudomonas aeruginosa. No differences were
ically ill patients (Table 5). observed in the primary study end point of change in Clinical
In 2017, the IASIS trial was published (Kollef 2017). This Pulmonary Infection Score (CPIS) at day 10 or any of the sec-
phase II study compared safety and efficacy of a special- ondary end points, including mortality or clinical cure. Also,
ized amikacin-fosfomycin inhalation system (AFIS) and no differences were observed in the subsets of patients with

CCSAP 2021 Book 1 • Pulmonary and Endocrinology 82 Drug-Induced Pulmonary Disorders


MDR organisms (23 AFIS, 16 placebo). In patients with pan- Overall, no difference in survival was noted between groups
drug-resistant bacteria (all A. baumannii; 9 AFIS, 4 placebo) a (191 [75%] vs. 196 [77%] for amikacin vs. placebo; OR 0.841;
trend toward improved clinical cure rates and ventilator-free 95% CI 0.554–1.227; p=0.43). Median duration of systemic
days was observed, but this assessment is limited by sample antibiotics was 18 days (IQR 13–28) in the amikacin group
size. Patients in the AFIS group had a significant reduction and 18 days (IQR 11–30) in the placebo group. Secondary end
in rate of positive tracheal cultures at day 3 and 7. Overall, points, including early clinical response, days on mechanical
this study found no benefit to adjunctive AFIS for treatment of ventilation, and days in ICU, were not statistically different.
gram-negative VAP. One important limitation was prolonged In addition, no difference overall in adverse effects was
exposure to antibiotics for nonpulmonary indications before observed. Treatment effect in this study may be hindered
initiation of aerosolized therapy (median AFIS, 6 days vs. pla- because of the uncommon occurrence of MDR organisms
cebo, 4 days), potentially masking some benefit. and because no pathogens were reported to be sensitive to
The study by Hassan and colleagues provided a more only aminoglycosides or a polymyxin.
positive perspective on inhaled antibiotics for treatment of With new evidence it is clearer that inhaled antibiotics for
HAP/VAP postcardiac surgery (Hassan 2018). This study routine adjunctive treatment of HAP/VAP from suspected
compared efficacy and safety of nebulized amikacin versus or documented gram-negative organisms is not warranted.
intravenous amikacin for double coverage of gram-negative However, these studies do not specifically address patients
HAP or VAP. Piperacillin/tazobactam was started empirically with MDR organisms and it may still be reasonable to consider
and adjusted according to culture results, but dual therapy inhaled adjunctive therapies in difficult-to-treat pulmonary
was maintained after culture results were finalized. Duration infections from MDR organisms or for cases in which antimi-
of therapy was dependent on patient response; amikacin crobial options are restricted to those with poor pulmonary
was discontinued once a patient achieved clinical improve- penetration. In addition, studies for use of inhaled antibiotics
ment and piperacillin/tazobactam was discontinued after in clinical scenarios such as relapse or failure of antimicro-
achieving clinical cure. Clinical cure was higher in the nebu- bial monotherapy are currently lacking. Future studies are
lized group versus systemic group (79 [91.8%] vs. 33 [70.2%]; warranted in this population and to obtain a better under-
p=0.002); length of ICU stay; duration of mechanical venti- standing of the optimal administration to ensure delivery of
lation (median duration of amikacin 7 days [IQR 5–9] vs. medication to site of infection.
8 days [IQR 7–10]; p=0.002); and median days to clinical cure
(10 days [IQR 7–11] vs. 11 days [IQR 5–12]; p=0.001) all pos- Intrapleural Therapies for Pleural Infection
itive findings favored the nebulized amikacin arm. Fewer Pleural infection (complicated parapneumonic effusion and
patients experienced acute kidney injury (increase in creat- empyema) remains an important complication of bacterial
inine of 0.3 mg/dL during treatment) in the nebulized versus pneumonia, despite widespread use of antibiotics and pneu-
intravenous arm (6 [6.97%] vs. 9 [10.4%]; p=0.0232). Study lim- mococcal vaccines (Shen 2017). This infection is associated
itations include use of a simple pneumatic delivery system in with significant morbidity, mortality, and health care costs.
patients on mechanical ventilation with limited details on the About 1 million patients are hospitalized for pneumonia each
administration procedures and approach to dual systemic year; 20%–40% of these patients develop parapneumonic
antibiotic therapy. effusion with 5%–10% progressing to empyema. Despite
The largest study to date, the INHALE trial, compared advances in medical diagnostic and therapeutic strategies,
inhaled amikacin and placebo in mechanically ventilated pleural infection is associated with a 15% mortality, and
patients with pneumonia and documented or suspected 30% of patients require surgical intervention for definitive
gram-negative MDR organism (Niederman 2020). Patients management.
were excluded if systemic antibiotic treatment for gram-neg- Parapneumonic effusions occur as a result of pneumo-
ative pneumonia was administered for more than 48 hours nia-associated inflammation causing release of pleural
before study drug administration (unless the infection was proinflammatory cytokines within the pleural space, such as
resistant to previous antibiotics). Systemic antibiotics were interleukin-6, interleukin-8, monocyte chemoattractant pro-
administered at least until the end of nebulized therapy, with tein, tumor necrosis factor-α, and vascular endothelial growth
the final duration decided by the hospital service after discon- factor (Corcoran 2015). Table 6 includes the definitions for
tinuation of study drug. The primary outcome was survival parapneumonic effusion classifications. The persistent
at late follow-up visit (day 28–32) in patients who had cul- release of inflammatory cytokines leads to a pro-fibrotic state
ture-confirmed gram-negative infection. Safety analysis was secondary to a rise in fibrinolysis inhibitors (plasminogen
completed in all patients who received any study drug (712 activator inhibitor) and reduction of endogenous pleural tPA.
patients). The primary efficacy group included 76% of the This excess fibrin deposition divides the space into separate
study patients with a HAP or VAP diagnosis. P. aeruginosa pockets. Bacterial and leukocytic cell death and lysis leads
and A. baumannii were the most commonly isolated patho- to purulence and pus in the pleural space. If no intervention
gens, and 47% of patients did not have resistant organisms. is performed, the infection can progress to a chronic stage,

CCSAP 2021 Book 1 • Pulmonary and Endocrinology 83 Drug-Induced Pulmonary Disorders


Table 6. Parapneumonic Fluid Classifications

Classification Definition

Parapneumonic effusion An accumulation of pleural fluid associated with pneumonia, lung abscess, or bronchiectasis;
thought to develop secondary to increased permeability of visceral pleural membranes and
leakage of interstitial fluid in response to lung parenchyma inflammation

Uncomplicated Exudative, predominately neutrophilic effusions; typically sterile and resolve with appropriate
parapneumonic effusion antibiotic treatment of pneumonia

Complicated Effusion with exudative properties; infected with bacteria or other microorganisms or biochemical
parapneumonic effusion properties suggestive of recent infection (increased neutrophils, pH <7.20, glucose <40 mg/dL,
elevated lactic dehydrogenase >1000 IU/L); requires drainage for resolution

Empyema thoracis Frank pus in the pleural space; infected with bacteria or other microorganisms or biochemical
properties suggestive of recent infection (increased neutrophils, pH <7.20, glucose <40 mg/dL,
elevated lactic dehydrogenase >1000 IU/L)

Information from: Corcoran JP, Wrightson JM, Belcher E, et al. Pleural infection: past, present, and future directions. Lancet Respir
Med 2015;3:563-77; Light RW. A new classification of parapneumonic effusions and empyema. Chest 1995;108:299-301.

characterized by fibroblast proliferation and pleural thicken- that intrapleural streptokinase should generally be avoided
ing, ultimately impacting lung expansion and compliance. in pleural infection but could not rule out a benefit regard-
The cornerstone of managing complex parapneumonic ing reduction in volume of infected pleural fluid collections
effusion and empyema includes appropriate antibiotics and or macroscopic in vivo lysis of fibrin adhesions. In addition,
adequate drainage. Pleural fluid can often be difficult to drain authors suggested the potential role of concomitant DNase
because of fibrous septations and high fluid viscosity (Piccolo therapy. Lastly, it is unclear if fibrinolytic agents used for this
2015). Thus, intrapleural fibrinolytic and deoxyribonuclease indication differ in efficacy.
therapy is of particular interest to augment percutaneous A subsequent trial published in 2011, MIST-2, was a dou-
chest tube drainage of infected fluid collections and poten- ble-blind, placebo-controlled, four-group study evaluating
tially avoid the need for surgical intervention. Fibrinolytics the effectiveness of recombinant tPA (alteplase) and DNase
convert plasminogen to active plasmin, which degrades fibrin combination therapy, tPA monotherapy, DNase monotherapy,
into soluble products. The net effect when administered in or placebo in 210 patients with pleural infection (Figure 1)
the pleural space is lysis of fibrinous septations, aiding in (Rahman 2011). The primary outcome, change in pleural opac-
drainage of the effusion, and reduction in fibrous deposition ity (percentage of hemithorax occupied by effusion) on chest
and thickening. Human recombinant DNase acts as a mucol-
ytic agent, decreasing the burden of deoxyribonucleoprotein
content (cell degradation products), a major contributor to
viscosity in the pleural space. The proposed mechanism of
benefit for combination therapy is likely greater than the sim-
ple mechanisms just listed, but the additional mechanisms
of impact on mesothelial and inflammatory cells are not well
understood.
Early investigations of treatment for pleural infection stud-
ied fibrinolytic therapy primarily as monotherapy. Multicenter
Intrapleural Sepsis Trial (MIST)-1 evaluated intrapleural strep-
tokinase 250,000 international units twice daily for 3 days Figure 1. The MIST-2 intrapleural therapy treatment
protocol. Therapy was administered twice daily for 3
compared with placebo in 427 patients with pleural infection
days for a total of 6 doses.
(Maskell 2005). This study showed no benefit to streptokinase
administration in terms of mortality, rate of surgery, radio- DNase = deoxyribonuclease; MIST = Multicenter
Intrapleural Sepsis Trial.
graphic outcomes, or hospital length of stay. Streptokinase
Information from: Rahman NM, Maskell NA, West A, et al.
administration was associated with numerically more serious
Intrapleural use of tissue plasminogen activator and DNase
adverse effects, including chest pain, fever, or allergy (7% vs.
in pleural infection. N Engl J Med 2011;365:518-26.
3%; RR 2.49; 95% CI 0.98–6.36; p=0.08). Authors concluded

CCSAP 2021 Book 1 • Pulmonary and Endocrinology 84 Drug-Induced Pulmonary Disorders


radiography on day 7, was greater in the tPA–DNase group In addition to the controversy of when to give fibrinolytics
compared with placebo (–29.5±23.3% vs. –17.2±19.6% [dif- are the many questions related to the therapy itself, includ-
ference in pleural opacity of –7.9%]; 95% CI –13.4 to –2.4; ing optimal tPA dose (including for patients at high risk of
p=0.005). Combination therapy was also associated with bleeding), method of administration, duration of therapy, and
lower surgical referral at 3 months (4% [2 of 48 patients] vs. indications for intrapleural fibrinolytics outside of pleural
16% [8 of 51]; OR 0.17; 95% CI 0.03–0.87; p=0.03) and reduc- infection. In an effort to reduce risk of pleural bleeding and
tion in hospital length of stay (difference of –6.7 days; 95% CI cost of therapy, case reports and case series have published
–12.0 to –1.9; p=0.006). Treatment with DNase alone or tPA on the success of reduced-dose tPA regimens ranging from
alone was ineffective for the primary outcome, and DNase 0.5–5 mg per dose (most commonly 5 mg), with or without
alone was associated with an increase in surgical referral at reductions in DNase (Hart 2019, Lan 2019, Innabi 2018, Lai
3 months (39% [18 of 46 patients]; OR 3.56; 95% CI 1.30–9.75; 2017, Popowicz 2017). The regimen used in MIST-2 is highly
p=0.01). Frequency of adverse events did not differ signifi- labor intensive, and it is unclear if coadministration—separate
cantly among the groups. A recent economic evaluation of syringes, administered concomitantly in pleural space—of
MIST-2 suggested tPA-DNase was most cost effective of all tPA/DNase results in similar outcomes. Three retrospective
four groups, despite added medication costs; a prospective, and one prospective observational studies from two insti-
formal economic evaluation was recommended to further tutions have reported on coadministration of tPA/DNase
elucidate cost-effectiveness of combination therapy given together and have reported success, with one study finding
limitations of the retrospective analysis (Luengo-Fernandez no difference compared with sequential administration (Kheir
2019). Conversely, a recent study in pediatric patients with 2018, Bishwakarma 2017, Majid 2017, 2016). Intrapleural tPA-
pleural empyema found the addition of DNase (5 mg/day) to DNAse has also been reported to be beneficial in empyema
tPA (4 mg/day) for 3 days versus placebo was not superior from intra-abdominal origin and for evacuation of traumatic
to tPA alone for duration of hospital stay or other second- hemothorax in an effort to avoid surgical intervention. One
ary clinical outcomes (Livingston 2020). Limitations of the case series of 17 patients with empyema secondary to
study include use of an alternate dosage regimen (once daily inflammation of intra-abdominal structures reported treat-
instead of twice daily, but aligned with standard practice ment success in 88.2% of patients with a median of 2 doses
in pediatric patients), exclusion of children with long-term of tPA/DNase using standard dosing (Majid 2017). Alteplase
comorbidities, wide standard deviation in length of stay, and monotherapy has been evaluated for treatment of traumatic
use of a pragmatic approach limiting conclusion on lack of retained hemothorax. In comparison to treatment of empy-
effect attributed to DNase or systematic use of cointerven- ema, reported doses for alteplase in traumatic retained
tions in each group. hemothorax tend to be higher (6–100 mg), with one review
The most recent 2017 American Association for Thoracic recommending doses of 25–50 mg of alteplase in 50–100 mL
Surgery guidelines state that intrapleural fibrinolytics should of normal saline (Holsen 2019, Hendriksen 2019). Additional
not be used routinely for complicated pleural effusion and studies are needed to definitely evaluate tPA for retained
empyema, citing heterogeneity among the literature and the hemothorax.
unknown effect of therapy on clinical or long-term outcomes In conclusion, many questions remain regarding optimal
(Shen 2017). In addition, only a subset of patients with pleu- use of intrapleural tPA and DNase. Future studies should
ral infection experience treatment failure with antibiotics focus on identifying ideal dosing and method of admin-
and chest tube drainage (MIST placebo groups: 14% [32 of istration. Large database studies may be helpful to guide
211] MIST-1 patients and 16% [8 of 51] MIST-2 patients). Risk treatment recommendations related to alternative dosing
stratification of patients who benefit from intrapleural ther- regimens in select patients or for method of administration,
apy or early surgery remains unclear. One study identified for which randomized studies may be difficult to design or
the presence of pleural thickening or presence of an abscess execute. Randomized studies should identify patient factors
or necrotizing pneumonia to predict failure of intrapleu- most likely to benefit from therapy and compare intrapleural
ral tPA/DNase therapy, but more prospective studies are therapy with early surgical management. A randomized feasi-
needed given the retrospective nature and small sample size bility study, MIST-3, is currently under way and is assessing
of this study (Khemasuwan 2018). Finally, although compli- feasibility of randomizing patients with intrapleural infection
cation rates appear to be low, administration of intrapleural to standard care (antibiotics and drainage), early intrapleural
fibrinolytic therapy can be associated with significant chest tPA and DNase, or early video-assisted thoracoscopic sur-
pain and local bleeding within the pleural cavity, with pleu- gery (clinical trial registration number ISRCTN18192121).
ral bleeding reported between 1.8 and 12% (Shen 2017,
Popowicz 2017). One should also consider absolute con- Endotracheal Medications in Emergency
traindications for intrapleural fibrinolytic therapy including Response
bronchopleural fistula and relative contraindication of high Intravenous administration is typically the route of choice
bleeding risk. in emergency situations; in some cases, however, providers

CCSAP 2021 Book 1 • Pulmonary and Endocrinology 85 Drug-Induced Pulmonary Disorders


and pharmacists may have limited options and require con- the expense of a decrease in partial pressure of arterial oxy-
sideration of alternate routes, including the endotracheal genation. Data and volume recommendations for multiple
administration route. Certain drugs may be rapidly absorbed doses of medications are lacking. Selection of diluent also
by this route; however, based on a general lack of evidence impacts the medication delivery. Both 0.9% saline and ster-
for efficacy and optimal dosing, endotracheal administration ile water may be used as diluents in adults. Sterile water
is usually reserved for prehospital or cardiac arrest settings may deliver a greater amount of drug compared with 0.9%
for cases in which the preferred intravenous or intraosseous saline because of the hypotonic nature of sterile water being
routes of administration are not available. absorbed quicker from the alveoli to the intravascular space
Medications that are appropriate for endotracheal admin- (Naganobu 2000). Conversely, 0.9% saline may produce
istration based on animal and human data include naloxone, less pulmonary dysfunction compared with sterile water
atropine, vasopressin, epinephrine and lidocaine—the NAVEL (Greenberg 1982). Recommendations for pediatric and neo-
mnemonic. Literature on dosing of medications by endotra- natal populations differ with respect to volume and diluent
cheal route is controversial and at times contradictory. The (see Table 7).
optimal dosing of medications for endotracheal tube admin- A variety of techniques may be used for administration
istration is unknown, but the total dosage is likely greater than of endotracheal medications. The following are proposed
that for the intravenous route. Animal cardiac arrest studies methods of administration: direct instillation into the
have shown that about 3 to 10 times the intravenous dose proximal end of the endotracheal tube; administration
of epinephrine is required to achieved similar concentrations by a catheter that extends beyond the tip of the endotra-
by the endotracheal route (Behar 2019). However, because cheal tube; deep endobronchial administration with a long
endotracheal medication administration can be associated catheter; administration by endotracheal tube monitor-
with a depot effect, which may contribute to post-resuscita- ing ports; use of equipment specifically for endotracheal
tion dysrhythmias, hypertension, and tachycardia in the case atomized drugs; and direct injection through the side of
of epinephrine, lower dosing with repeat dosing as needed is the endotracheal tube with a needle. Some studies show
recommended. American Heart Association guidelines rec- that absorption with proximal instillation into the endo-
ommend 2 to 2.5 times the intravenous dose for medications tracheal tube is variable (Quinton 1987, McDonald 1985).
for adult advanced cardiac life support (Neumar 2010). Despite American Heart Association guidelines recommend injec-
the risk of depot effect, there are no recommendations for tion of the drug directly into the endotracheal tube, but use
extending the duration between endotracheal drug adminis- of a catheter may also be considered, if available.
trations. The safety and efficacy of endotracheal epinephrine Although the endotracheal route is a possible option for
in neonates has not been evaluated. Recommendations for medication administration in emergencies, intravenous or
endotracheal drug dosing in pediatric patients and neonates intraosseous routes are preferred because of more predict-
differ from adults (Table 7). able absorption and medication pharmacokinetics. Many
The volume of diluent for endotracheal medication uncertainties exist regarding dosing and administration, but
administration has significant effects on achievement of pharmacists must be prepared to make appropriate medica-
medication concentrations. Higher volume (more dilute) tion dose, diluent, and volume recommendations as needed
results in increased blood concentrations of drugs, but at in an emergency setting.

Table 7. American Heart Association Guidelines for Endotracheal Drug Administration

Guideline Adult Pediatric Neonatal

Medication dose 2–2.5 times the recommended Epinephrine 10 times the IV dose (0.1 mg/kg Epinephrine (1:10,000)
IV dose of 1 mg/mL [1:1000] solution) 0.05–0.1 mg/kg (class
Atropine 0.04–0.06 mg/kg indeterminate)
Lidocaine 2–3 mg/kg

Total volume 10 mL 5 mL 1 mL

Diluent 0.9% saline or distilled water 0.9% saline 0.9% saline

Reprinted with permission from: Behar S. Alternative methods of drug administration. In: Roberts JR, Custalow CB, Thomsen TW, eds.
Robert and Hedges’ Clinical Procedures in Emergency Medicine and Acute Care, 7th ed. Philadelphia, PA: Elsevier, 2019:476-81.

CCSAP 2021 Book 1 • Pulmonary and Endocrinology 86 Drug-Induced Pulmonary Disorders


Bafaqih H, Chehab M, Almohaimeed S, et al. Pilot trial
Practice Points of a novel two-step therapy protocol using nebulized
These practices will aid pharmacists in effectively tranexamic acid and recombinant factor VIIa in children
achieving the following: 1) making pulmonary disease with intractable diffuse alveolar hemorrhage. Ann Saudi
management recommendations; 2) identifying appro- Med 2015;35:231-39.
priate opportunities for intrapulmonary medications; 3)
Baker MS, Diab KJ, Graham Carlos W, et al. Intrapulmonary
optimizing safety and efficacy in drug dosing and
recombinant factor VII as an effective treatment for dif-
administration; and 4) ensuring appropriate monitoring
fuse alveolar hemorrhage: a case series. J Bronchol Interv
for adverse effects and complications.
Pulmonol 2016;23:255-8.
• Pharmacists should regularly perform a thorough medica-
tion history that considers all medication or unregulated Behar S. Alternative Methods of drug administration. In:
drug use, treatments, or behaviors that may contribute to Roberts JR, Custalow CB, Thomsen TW, eds. Robert and
pulmonary disease. Hedges’ Clinical Procedures in Emergency Medicine and
• Diagnosis of drug-induced pulmonary disease often relies
Acute Care, 7th ed. Philadelphia, PA: Elsevier, 2019:476-81.
on careful exclusion of other causative factors, establish- Bishwakarma R, Shah S, Frank L, et al. Mixing it up: coad-
ing a temporal association between introduction of the ministration of tPA/DNase in complicated parapneumonic
drug in question and symptom onset, and monitoring for pleural effusions and empyema. J Bronchol Interv
improvement after discontinuation of the offending agent Pulmonol 2017;24:40-7.
and/or recurrence of symptoms after its reintroduction
(if appropriate). Blagev DP, Harris D, Dunn AC, et al. Clinical presenta-
• The etiology and understanding of EVALI is evolving. tion, treatment, and short-term outcomes of lung injury
Management is primarily supportive and corticosteroids associated with e-cigarettes or vaping: a prospective
may be considered in select cases. observational cohort study. Lancet 2019;394:2073-83.
• Pharmacists must carefully consider safety in each patient-
Blount BC, Karwowski MP, Shields PG, et al. Vitamin E
specific context when weighing the risks and benefits of
acetate in bronchoalveolar-lavage fluid associated with
unproven or experimental treatments, particularly those
EVALI. N Engl J Med 2020;382:697-705.
that have been minimally described in the literature.
• The place in therapy for inhaled antibiotics in HAP or VAP Brahmer JR, Lacchetti C, Schneider BJ, et al. Management
remains unclear. Large, randomized studies do not support of immune-related adverse events in patients treated with
routine use in gram-negative HAP or VAP, but therapy may immune checkpoint inhibitor therapy: American Society of
be considered in select cases caused by MDR or difficult- Clinical Oncology Clinical practice guideline. J Clin Oncol
to-treat organisms. 2018;36:1714-68.
• Intrapleural dornase and alteplase is often used for
Calvo GS, De Granda-Orive I, Padilla DL. Inhaled tranexamic
treatment of complicated parapneumonic effusion and em-
acid as an alternative for hemoptysis treatment. Chest
pyema and improves chest imaging, surgical referral, and
2016;149:604.
hospital length of stay. Optimal dose and administration,
place in therapy relative to video-assisted thoracoscopic Camus P, Fanton A, Bonniaud P, et al. Interstitial lung
surgery and indications outside of bacterial pleural infec- disease induced by drugs and radiation. Respiration
tion are still being elucidated. 2004;71:301-26.
• Because of variability in pharmacokinetics, a general lack
of literature on dosing, and challenges in drug delivery, Choe J, Chen P, Falk JA, et al. A case series of vaping-associ-
administration of medications by the endotracheal route ated lung injury requiring mechanical ventilation. Crit Care
should be reserved only for emergency situations in which Explor 2020;2:e0079.
intravenous or intraosseous access is not available. Corcoran JP, Wrightson JM, Belcher E, et al. Pleural infection:
past, present, and future directions. Lancet Respir Med
2015;3:563-77.

REFERENCES Crotty Alexander LE, Ware LB, Calfee CS, et al. NIH workshop
Aberegg SK, Maddock SD, Blagev DP, et al. Diagnosis of EVALI. report: e-cigarette or vaping product use associated lung
Chest 2020;158:820-7. injury (EVALI): developing a research agenda. Am J Respir
Crit Care Med 2020;202:795-802.
Ahya VN. Noninfectious acute lung injury syndromes early
after hematopoietic stem cell transplantation. Clin Chest Da L, Teng Y, Wang N, et al. Organ-specific immune-related
Med 2017;38:595-606. adverse events associated with immune checkpoint inhib-
itor monotherapy versus combination therapy in cancer:
Arbour KC, Mezquita L, Long N, et al. Impact of baseline a meta-analysis of randomized controlled trials. Front
steroids on efficacy of programmed cell death-1 and Pharmacol 2020;10:1671.
programmed death-ligand 1 blockade in patients with
Del Castillo M, Romero FA, Argüello E, et al. The spectrum
non–small-cell lung cancer. J Clin Oncol 2018;36:2872-8.
of serious infections among patients receiving immune
Ashrafian H, Davey P. Is amiodarone an underrecognized checkpoint blockade for the treatment of melanoma.
cause of acute respiratory failure in the ICU? Chest 2001; Clin Infect Dis 2016;63:1490-3.
120:275-82.

CCSAP 2021 Book 1 • Pulmonary and Endocrinology 87 Drug-Induced Pulmonary Disorders


Dhokarh R, Li G, Schmickl CN, et al. Drug-associated acute Kheir F, Cheng G, Rivera E, et al. Concurrent versus sequen-
lung injury: a population-based cohort study. Chest 2012; tial intrapleural instillation of tissue plasminogen activator
142:845-50. and deoxyribonuclease for pleural infection. J Bronchol
Interv Pulmonol 2018;25:125-31.
Dolladille C, Ederhy S, Sassier M, et al. Immune checkpoint
inhibitor rechallenge after immune-related adverse events Khemasuwan D, Sorensen J, Griffin DC. Predictive Variables
in patients with cancer. JAMA Oncol 2020;6:865-71. for failure in administration of intrapleural tissue plas-
minogen activator/deoxyribonuclease in patients with
Faverio P, De Giacomi F, Sardella L, et al. Management of complicated parapneumonic effusions/empyema.
acute respiratory failure in interstitial lung diseases: over- Chest 2018;154:550-6.
view and clinical insights. BMC Pulm Med 2018;18:70.
Kollef MH, Ricard JD, Roux D, et al. A randomized trial of the
Greenberg MI, Baskin SI, Kaplan AM, et al. Effects of endo- amikacin fosfomycin inhalation system for the adjunctive
tracheally administered distilled water and normal saline therapy of gram-negative ventilator-associated pneumo-
on the arterial blood gases of dogs. Ann Emerg Med nia: IASIS Trial. Chest 2017;151:1239-46.
1982;11:600-4.
Komura S, Rodriguez RM, Peabody CR. Hemoptysis? Try
Hart JA, Badiei A, Lee YCG. Successful management of inhaled tranexamic acid. J Emerg Med 2018;54:e97-9.
pleural infection with very low dose intrapleural tis-
sue plasminogen activator/deoxyribonuclease regime. Lai C, Mohammad N, Hassan TM, et al. Low-dose intrapleu-
Respirol Case Reports 2019;7. ral alteplase (without deoxyribonuclease) in complicated
parapneumonic effusion: case series and literature
Hassan NA, Awdallah FF, Abbassi MM, et al. Nebulized reviews. BMJ Case Rep 2017;2017:bcr2017221239.
versus IV amikacin as adjunctive antibiotic for hos-
pital and ventilator-acquired pneumonia postcardiac Lan NSH, Vekaria S, Sidhu C, et al. Very low-dose intrapleural
surgeries: a randomized controlled trial. Crit Care Med tPA for indwelling pleural catheter-associated symptom-
2018;46:45-52. atic fluid loculation. Respirol Case Reports 2019;7:e00457.

Heinzerling A, Armatas C, Karmarkar E, et al. Severe lung Lara AA, Schwarz M. Diffuse alveolar hemorrhage. Chest
injury associated with use of e-cigarette, or vaping, prod- 2010;137:1164-71.
ucts—California, 2019. JAMA Intern Med 2020;180:861-9.
Larsen BT, Chae JM, Dixit AS, et al. Clinical and histopatho-
Hendriksen BS, Kuroki MT, Armen SB, et al. Lytic therapy for logic features of immune checkpoint inhibitor-related
retained traumatic hemothorax: a systematic review and pneumonitis. Am J Surg Pathol 2019;43:1331-40.
meta-analysis. Chest 2019;155:805-15. Layden JE, Ghinai I, Pray I, et al. Pulmonary illness related to
Heslet L, Nielsen JD, Levi M, et al. Successful pulmonary e-cigarette use in Illinois and Wisconsin—final report.
administration of activated recombinant factor VII in N Engl J Med 2020;382:903-16.
diffuse alveolar hemorrhage. Crit Care 2006;10:R177. Livingston MH, Mahant S, Connolly B, et al. Effectiveness
Holsen MR, Tameron AM, Evans DC, et al. Intrapleural tissue of intrapleural tissue plasminogen activator and dornase
plasminogen activator for traumatic retained hemothorax. alfa vs tissue plasminogen activator alone in children
Ann Pharmacother 2019;53:1060-6. with pleural empyema: a randomized clinical trial. JAMA
Pediatrics 2020;174:3320-40.
Horvat TZ, Adel NG, Dang TO, et al. Immune-related adverse
events, need for systemic immunosuppression, and Luengo-Fernandez R, Penz E, Dobson M, et al. Cost-
effects on survival and time to treatment failure in effectiveness of intrapleural use of tissue plasminogen
patients with melanoma treated with ipilimumab at activator and DNase in pleural infection: evidence from
Memorial Sloan Kettering Cancer Center. J Clin Oncol the MIST2 randomised controlled trial. Eur Respir J
2015;33:3193-8. 2019;54:1801550.

Innabi A, Surana A, Alzghoul B, et al. Rethinking the doses Majhail NS, Parks K, Defor TE, et al. Diffuse alveolar hem-
orrhage and infection-associated alveolar hemorrhage
of tissue plasminogen activator and deoxyribonucle-
ase administered concurrently for intrapleural therapy following hematopoietic stem cell transplantation: related
for complicated pleural effusion and empyema. Cureus and high-risk clinical syndromes. Biol Blood Marrow
Transplant 2006;12:1038-46.
2018;10:e2214.
Majid A, Kheir F, Folch A, et al. Concurrent intrapleural
Jackevicius CA, Tom A, Essebag V, et al. Population-level
instillation of tissue plasminogen activator and DNase
incidence and risk factors for pulmonary toxicity associ-
for pleural infection a single-center experience. Ann Am
ated with amiodarone. Am J Cardiol 2011;108:705-10.
Thorac Soc 2016;13:1512-8.
Kalil AC, Metersky ML, Klompas M, et al. Management
Majid A, Sebastian O, Sumit C, et al. Safety and efficacy of
of adults with hospital-acquired and ventilator-associ-
tissue plasminogen activator and DNase for complicated
ated pneumonia: 2016 clinical practice guidelines by the
pleural effusions secondary to abdominal pathology.
Infectious Diseases Society of America and the American
Ann Am Thorac Soc 2017;14: 342–46.
Thoracic Society. Clin Infect Dis 2016;63:e61-111.

CCSAP 2021 Book 1 • Pulmonary and Endocrinology 88 Drug-Induced Pulmonary Disorders


Martínez-Martínez MU, Oostdam DAH van, Abud-Mendoza Piccolo F, Popowicz N, Wong D, et al. Intrapleural tissue plas-
C. Diffuse alveolar hemorrhage in autoimmune diseases. minogen activator and deoxyribonuclease therapy for
Curr Rheumatol Rep 2017;19:27. pleural infection. J Thorac Dis 2015;7:999-1008.
Maskell NA, Davies CWH, Nunn AJ, et al. U.K. Controlled Popowicz N, Bintcliffe O, De Fonseka D, et al. Dose de-esca-
trial of intrapleural streptokinase for pleural infection lation of intrapleural tissue plasminogen activator therapy
2005;352:865-74. for pleural infection the alteplase dose assessment for
pleural infection therapy project. Ann Am Thorac Soc
McDonald JL. Serum lidocaine levels during cardiopulmo- 2017;14:929-36.
nary resuscitation after intravenous and endotracheal
administration. Crit Care Med 1985;13:914-5. Postow MA, Sidlow R, Hellmann MD. Immune-related
adverse events associated with immune checkpoint block-
Messika J, Prat D, Sztrymf B. Tranexamic acid inhalations ade. N Engl J Med 2018;378:158-68.
in nonmassive hemoptysis: a word of caution. Chest
2019;155:876. Quinton DN, O’Byrne G, Aitkenhead AR. Comparison of
endotracheal and peripheral intravenous adrenaline in
Metcalf JP, Rennard SI, Reed EC, et al. Corticosteroids as cardiac arrest. Is the endotracheal route reliable? Lancet
adjunctive therapy for diffuse alveolar hemorrhage asso- 1987;1:828-29.
ciated with bone marrow transplantation. Am J Med
1994;96:327-34. Rahman NM, Maskell NA, West A, et al. Intrapleural use of
tissue plasminogen activator and DNase in pleural infec-
Mirouse A, Parrot A, Audigier V, et al. Severe diffuse alveolar tion. N Engl J Med 2011;365:518-26.
hemorrhage related to autoimmune disease: a multicenter
study. Crit Care 2020;24:231. Raptis A, Mavroudis D, Suffredini AF, et al. High-dose cor-
ticosteroid therapy for diffuse alveolar hemorrhage in
Muthumalage T, Friedman MR, McGraw MD, et al. Chemical allogeneic bone marrow stem cell transplant recipients.
constituents involved in e-cigarette, or vaping product Bone Marrow Transplant 1999;24:879-83.
use-associated lung injury (EVALI). Toxics 2020;8:25.
Rathi NK, Tanner AR, Dinh A, et al. Low-, medium- and high-
Naganobu K, Hasebe Y, Uchiyama Y, et al. A comparison of dose steroids with or without aminocaproic acid in adult
distilled water and normal saline as diluents for endobron- hematopoietic SCT patients with diffuse alveolar hemor-
chial administration of epinephrine in the dog. Anesth rhage. Bone Marrow Transplant 2015;50:420-6.
Analg 2000;91:317-21.
Sears CR, Rivera MP, Camus P, et al. Knowledge gaps and
Naidoo J, Page DB, Li BT, et al. Toxicities of the anti-PD-1 research priorities in immune checkpoint inhibitor–
and anti-PD-L1 immune checkpoint antibodies. Ann Oncol related pneumonitis: an official American Thoracic
2015;26:2375-91. Society Research Statement. Am J Respir Crit Care Med
2019;200:E31-43.
Naidoo J, Wang X, Woo KM, et al. Pneumonitis in patients
treated with anti-programmed death-1/programmed death Shen KR, Bribriesco A, Crabtree T, et al. The American
ligand 1 therapy. J Clin Oncol 2017;35:709-17. Association for Thoracic Surgery consensus guidelines
for the management of empyema. J Thorac Cardiovasc
Neumar RW, Otto CW, Link MS, et al. Part 8: adult advanced car- Surg 2017;153:e129-46.
diovascular life support: 2010 American Heart Association
guidelines for cardiopulmonary resuscitation emergency Simonaggio A, Michot JM, Voisin AL, et al. Evaluation of
cardiovascular care. Circulation 2010;122:S729-67. readministration of immune checkpoint inhibitors after
immune-related adverse events in patients with cancer.
Niederman MS, Alder J, Bassetti M, et al. Inhaled amikacin JAMA Oncol 2019;5:1310-7.
adjunctive to intravenous standard-of-care antibiotics
in mechanically ventilated patients with gram-negative Skeoch S, Weatherley N, Swift A, et al. Drug-induced inter-
pneumonia (INHALE): a double-blind, randomised, place- stitial lung disease: a systematic review. J Clin Med
bo-controlled, phase 3, superiority trial. Lancet Infect Dis 2018;7:356.
2020;20:330-40. Solomonov A, Fruchter O, Zuckerman T, et al. Pulmonary
hemorrhage: a novel mode of therapy. Respir Med
O’Neil ER, Schmees LR, Resendiz K, et al. Inhaled tranexamic
2009;103:1196-200.
acid as a novel treatment for pulmonary hemorrhage in
critically ill pediatric patients: an observational study. Suresh K, Psoter KJ, Voong KR, et al. Impact of check-
Crit Care Explor 2020;2:e0075. point inhibitor pneumonitis on survival in NSCLC patients
receiving immune checkpoint immunotherapy. J Thorac
Park JA, Kim BJ. Intrapulmonary recombinant factor VIIa
Oncol 2019;14:494-502.
for diffuse alveolar hemorrhage in children. Pediatrics
2015;135:e216-20. Wand O, Guber E, Guber A, et al. Inhaled tranexamic acid
for hemoptysis treatment: a randomized controlled trial.
Peters S, Felip E, Dafni U, et al. Safety evaluation of nivolumab Chest 2018;154:1379-84.
added concurrently to radiotherapy in a standard first
line chemo-radiotherapy regimen in stage III non-small Weber JS, Hodi FS, Wolchok JD, et al. Safety profile of
cell lung cancer—the ETOP NICOLAS trial. Lung Cancer nivolumab monotherapy: a pooled analysis of patients
2019;133:83-7. with advanced melanoma. J Clin Oncol 2017;35:785-92.

CCSAP 2021 Book 1 • Pulmonary and Endocrinology 89 Drug-Induced Pulmonary Disorders


Self-Assessment Questions
1. Which one of the following patients, if initiated on 3. Which one of the following most supports a diagnosis
amiodarone for treatment of atrial fibrillation, is at high- of e-cigarette, or vaping, product use-associated lung
est risk for drug-induced interstitial lung disease (ILD)? injury (EVALI) for G.R.?
A. A 45-year-old man with rheumatoid arthritis treated A. Thrombocytosis
with weekly, low-dose methotrexate B. Fever
B. A 67-year-old woman with a remote history of C. Acute kidney injury
breast cancer in her 40s treated with paclitaxel and D. Wheezing
gemcitabine
4. G.R.’s care team discusses his nicotine and THC use in
C. A 55-year-old woman with chronic obstructive
more detail. Which one of the following puts G.R. at high-
pulmonary disease receiving levofloxacin and who
est risk of EVALI?
is intubated with a fraction of inspired oxygen (FiO2)
of 40% A. Use of both nicotine and THC products
D. An 82-year-old man with newly diagnosed non-small B. Use of a mint-flavored nicotine product
cell lung cancer undergoing treatment with radiation C. Mixing e-liquids containing THC from an illicit source
and who intubated with an FIO2 of 70% D. Use of both e-cigarette devices and regular
cigarettes
2. Five days ago, a 50-year-old woman presented to your
ICU with a low-grade fever, non-productive cough, and
Questions 5 and 6 pertain to the following case.
hypoxemic respiratory failure. She was intubated on hos-
B.G. is a 54-year-old man recently diagnosed with non-small
pital day 3 and over the past 48 hours has been unable to
cell lung cancer. He is receiving his first treatment cycle with
be weaned beyond 60% FIO2. Infection, pulmonary embo-
pembrolizumab, carboplatin, and pemetrexed. B.G. is main-
lism, and pulmonary edema have been ruled out, and you
tained on home oxygen at 2 L by nasal cannula. He presents
begin to suspect amiodarone-induced ILD because she
to the hospital with worsening shortness of breath, a non-pro-
has been taking amiodarone 400 mg twice daily for the
ductive cough, and reports having a fever prior to admission.
past 9 months for treatment of atrial fibrillation following
Chest radiography reveals worsening bilateral ground glass
an aortic valve replacement. Which one of the following
opacities compared to prior imaging studies and new nod-
is best to recommend for this patient?
ular infiltrates that the radiologist suggests could be due
A. Discontinue amiodarone and monitor for clinical to infection or worsening disease. On Day 1 of admission
improvement. B.G. is started on cefepime 2 g intravenous every 12 hours
B. Start methylprednisolone 1 mg/kg/day followed by a and vancomycin 20 mg/kg every 12 hours; these are dosed
slow taper when she has clinically improved. appropriately for the indication of pneumonia and his kid-
C. Continue to rule out other causes including chronic ney function, and blood and sputum cultures were obtained.
obstructive pulmonary disease, cancer, and On hospital day 3, B.G. is transferred to the ICU for emergent
pulmonary artery hypertension. non-invasive ventilation due to increasing oxygen require-
D. Discontinue amiodarone and start ments overnight. Culture data remains negative and he is
methylprednisolone 1 g intravenous daily for 3 days hemodynamically stable and afebrile.
followed by a rapid taper.
5. Which one of the following is best to recommend for B.G.?
A. Change antimicrobials to meropenem 1 g
Questions 3 and 4 pertain to the following case. intravenous every 8 hours and linezolid 600 mg
G.R. is a 21 year-old-man (weight 80 kg [176 lb]) with a history intravenous twice daily because of concern for
of nicotine and tetrahydrocannabinol (THC) use. He presents resistant or breakthrough bacterial infection.
to the ED with fever, chills, nausea and dyspnea for 3 days. B. Check a procalcitonin and β-d-glucan and start
On examination G.R. is noted to be in acute distress, wheez- caspofungin 70 mg intravenous once followed by
ing, tachypneic and tachycardic. Chest radiography shows 50 mg intravenous daily because of concern for
bilateral ground-glass opacities with left-sided retrocar- fungal infection.
diac opacification. Notable lab findings include acute kidney C. Start methylprednisolone 1 mg/kg/day because
injury, leukocytosis, thrombocytosis, mild elevation in trans- of concern for immune checkpoint inhibitor
aminases, and elevated C-reactive protein. Because of severe (ICI)-pneumonitis.
hypoxemia, G.R. is admitted to the ICU for high-flow oxygen D. Start methylprednisolone 1 g intravenous daily
therapy. because of concern for ICI-pneumonitis.

CCSAP 2021 Book 1 • Pulmonary and Endocrinology 90 Drug-Induced Pulmonary Disorders


6. B.G. improves after receiving a complete course of diffuse patchy infiltrates bilaterally that could represent
broad-spectrum antibiotics and starting methylprednis- infection, fluid, or inflammation. Over the next 12 hours the
olone 1.5 mg/kg/day, and he now has been extubated patient’s oxygen requirements increase and she is intubated.
to 4L of oxygen by nasal cannula. Which one of the fol- Consecutive aliquots show progressively bloody returns, and
lowing is best to recommend for B.G. before he transfers fluid analysis reports 16% hemosiderin-laden macrophages.
back to the general care floor? S.H.’s clinical status worsens and she requires more ventila-
tor support and red blood cell transfusions.
A. Begin tapering the steroid dose by 50% every 3 days
and start sulfamethoxazole/trimethoprim and 8. Which one of the following is best to recommend for
pantoprazole. S.H.?
B. Begin tapering the steroid dose by 25% every 7 days
A. Administer steroids because they are strongly
and start sulfamethoxazole/trimethoprim.
indicated as first-line therapy for treatment of DAH.
C. Continue steroid at the current dose and refer him to
B. Recombinant activated factor VII (rFVIIa) because
his primary oncologist for follow-up in the outpatient
it is strongly indicated, and will not increase her risk
setting.
for adverse events.
D. Decrease the steroid dose to 1 mg/kg/day and
C. Do not administer steroids; they are strongly
monitor symptoms, refer him to his primary
contraindicated because of infectious concerns,
oncologist for follow-up in the outpatient setting,
the absence of hemoptysis, and less than 20%
and start sulfamethoxazole/trimethoprim and
hemosiderin-laden macrophages.
pantoprazole.
D. Provide best practice supportive care and close
7. A previously healthy 29-year-old man presents to the ICU monitoring at this time; consider steroids if
after a 2-week history of fever, malaise, and arthralgias. symptoms continue to worsen.
The patient also complains of new skin lesions across
9. S.H.’s critical care and transplant teams would like to
the bridge of his nose and on his cheeks. He has new-on-
start pharmacotherapy for DAH. Which one of the follow-
set kidney failure and diffuse alveolar hemorrhage (DAH)
ing is best to recommend for S.H.?
requiring intubation and confirmed by progressively
bloody returns from bronchoalveolar lavage (BAL) fluid. A. Start methylprednisolone 1 g intravenous daily for
Diagnostic work-up has ruled out infection, and labora- 3 days and aminocaproic acid 4 g intravenous bolus
tory tests suggest a new diagnosis of systemic lupus followed by a 1 g/hour infusion.
erythematosus. He remains clinically unstable with B. Start methylprednisolone 1 g intravenous daily for
progressively worse kidney function now requiring dial- 3 days and give rFVII 90 mcg/kg intravenous once.
ysis and he has not been weaned from the ventilator. C. Start methylprednisolone 500 mg intravenous daily
Which one of the following is best to recommend for this for 3 days and give intra-pulmonary rFVII 30 mcg/kg
patient? once.
D. Start methylprednisolone 30 mg intravenous daily
A. Continue with best practice supportive care
and give intra-pulmonary rFVIIa 90 mcg/kg once.
for his end-organ dysfunction and consider
plasmapheresis. 10. A 64-year-old man with endobronchial cancer and mas-
B. Start methylprednisolone 1 mg/kg intravenous daily sive hemoptysis is intubated for airway protection and
and consult rheumatology for further evaluation. going for angiographic embolization within the hour. His
C. Start methylprednisolone 1 g intravenous daily and care team asks about the role of nebulized TXA for the
cyclophosphamide 500 mg intravenous today and treatment of hemoptysis. Which one of the following is
repeat in 1 month. best to share with the care team regarding the role of
D. Start methylprednisolone 1 g intravenous daily and TXA in this patient?
consult rheumatology for further evaluation and A. It should be given because we can be certain there
cyclophosphamide dosing. are no associated risks.
B. It has not been studied in the setting of massive
Questions 8 and 9 pertain to the following case. hemoptysis requiring embolization.
S.H., a 38-year-old woman, presents to your ICU with increas- C. It should be used in combination with rFVIIa
ing respiratory distress, low-grade fever, and declining because of massive hemoptysis.
hematocrit. Twenty-one days ago she underwent an alloge- D. It is only effective for patients with hemoptysis who
neic hematopoietic cell transplant for acute myelogenous are on anticoagulation.
leukemia. S.H. is started on empiric antibiotics for pneu- 11. Which one of the following patients is most likely to ben-
monia, cultures are obtained, and chest radiography shows efit from nebulized TXA?

CCSAP 2021 Book 1 • Pulmonary and Endocrinology 91 Drug-Induced Pulmonary Disorders


A. A patient with recurrent moderate hemoptysis on A. Transudative effusion
bilevel positive airway pressure (BiPAP) who is B. Uncomplicated parapneumonic effusion
unable to go without oxygen supplementation to C. Complicated parapneumonic effusion
swallow pills D. Empyema thoracis
B. A patient with a single episode of small-volume
14. Despite thoracentesis and chest tube placement, chest
hemoptysis whose respiratory status and
imaging shows T.J. has a persistent loculated pleural
coagulation parameters are normal
effusion. Which one of the following best evaluates the
C. A patient with recurrent episodes of moderate-
use of intrapleural alteplase or dornase for T.J.?
volume hemoptysis, increasing oxygen requirements
through nasal cannula, and a decrease in hematocrit A. Intra-pleural alteplase should not be used because
D. A patient with a single episode of massive bleeding risk in the setting of cirrhosis is an
hemoptysis who is intubated for airway protection absolute contraindication to therapy
and a decrease in hematocrit B. Early video-assisted thoracoscopic surgery is
preferred over intra-pleural alteplase and dornase
12. A 61-year-old man (weight 89 kg) has had a prolonged
because of a history of diabetes.
hospital course for influenza-associated acute respi-
C. Intra-pleural dornase monotherapy is preferred
ratory distress syndrome requiring extracorporeal
because of its association with decreased need for
membrane oxygenation support and tracheostomy. The
surgical intervention, compared with combination
patient is found to have Acinetobacter baumannii ventila-
therapy with alteplase.
tor-associated pneumonia and is started on meropenem.
D. Intra-pleural alteplase and dornase should be
Which one of the following best evaluates the use of
considered because combination therapy has been
inhaled aminoglycosides for this patient?
shown to improve chest imaging and decrease
A. Adjunct inhaled aminoglycosides have not been hospital length of stay.
shown to improve survival in large randomized
15. A 79-year-old man (weight 98 kg [216 lb]) presents to
studies in patients with ventilator-acquired
the ED while undergoing active cardiopulmonary resus-
pneumonia (VAP) and documented multidrug
citation for pulseless electrical activity cardiac arrest.
resistant (MDR) organisms.
He was intubated in the field and attempts at achiev-
B. Adjunct inhaled aminoglycosides have been
ing adequate intravenous or intraosseous access have
associated with increased acute kidney injury
been unsuccessful. The team wants to administer endo-
compared to placebo.
tracheal epinephrine while working on obtaining better
C. Inhaled aminoglycosides have been shown to be
access. Which one of the following is best to recommend
effective only if administered via a jet-nebulizer
for this patient?
device.
D. Current guidelines recommend against A. Epinephrine should not be given via the
inhaled aminoglycosides in patients with VAP endotracheal route due to inadequate absorption.
secondary to a gram-negative susceptible only to B. Epinephrine should be administered as 2.5 mg of the
aminoglycosides, due to lack of evidence. 1:10,000 (0.1 mg/mL) concentration.
C. Epinephrine should be administered as 2 mg of the
1:1,000 (1 mg/mL) concentration, diluted in 3 mL of
Questions 13 and 14 pertain to the following case.
dextrose 5% in water.
T.J. is a 75-year-old man with a medical history that includes
D. Epinephrine should be administered as 2 mg of the
type 2 diabetes, coronary artery disease, cirrhosis, and hyper-
1:1,000 (1 mg/mL) concentration, diluted in 8 mL of
tension. He is admitted to the ICU with severe community
normal saline.
acquired pneumonia requiring intubation. A thoracentesis of
his left-sided loculated pleural effusion shows frank pus, and
pleural fluid analysis shows pH 6.8, glucose less than 20 mg/
dL, lactate dehydrogenase (LDH) 3106 U/L (serum 430 U/L,
reference 100–200 U/L), protein 4.5 g/dL (serum 6 g/dL), amy-
lase less than 30 U/L.
13. Which one of the following best classifies T.J.’s pleural
effusion?

CCSAP 2021 Book 1 • Pulmonary and Endocrinology 92 Drug-Induced Pulmonary Disorders


Learner Chapter Evaluation: Drug-Induced Pulmonary Disorders and Pulmonary
Drug Delivery

As you take the posttest for this chapter, also evaluate the 8. The teaching and learning methods used in the chapter
material’s quality and usefulness, as well as the achievement were effective.
of learning objectives. Rate each item using this 5-point scale: 9. The active learning methods used in the chapter were
effective.
• Strongly agree
10. The learning assessment activities used in the chapter
• Agree
were effective.
• Neutral
• Disagree 11. The chapter was effective overall.
• Strongly disagree 12. The activity met the stated learning objectives.
13. If any objectives were not met, please list them here.
1. The content of the chapter met my educational needs.
2. The content of the chapter satisfied my expectations.
3. The author presented the chapter content effectively.
OTHER COMMENTS
4. The content of the chapter was relevant to my practice 14. Please provide any specific comments related to any
and presented at the appropriate depth and scope. perceptions of bias, promotion, or advertisement of
commercial products.
5. The content of the chapter was objective and balanced.
15. Please expand on any of your above responses, and/or
6. The content of the chapter is free of bias, promotion, and
provide any additional comments regarding this chapter:
advertisement of commercial products.
7. The content of the chapter was useful to me.

CCSAP 2021 Book 1 • Pulmonary and Endocrinology 93 Drug-Induced Pulmonary Disorders


Pneumonia in the ICU
By G. Christopher Wood, Pharm.D., FCCP, FCCM, BCCCP

Reviewed by Michael A. Peters, BS, BCCCP; and Poorvi Shah, Pharm.D., BCCCP, BCPS

LEARNING OBJECTIVES

1. Design appropriate empiric therapies for likely organisms causing hospital-acquired pneumonia/ventilator-associated
pneumonia (HAP/VAP) and severe community-acquired pneumonia (CAP) in high- and low-risk patients.
2. Develop definitive therapy plans for HAP/VAP and severe CAP, including drug selection/de-escalation, pharmacokinetic
optimization, dosing, and therapy duration.
3. Evaluate options for patients who experience therapy failure or recurrent pneumonia.
4. Justify the use of antiviral and corticosteroid therapy for patients with severe CAP.

INTRODUCTION
ABBREVIATIONS IN THIS CHAPTER
Pneumonia is the most common life-threatening infection in ICU
BAL Bronchoalveolar lavage
patients and continues to drive antibiotic use in hospitalized
CAP Community-acquired pneumonia
patients. Use of antibiotics for pneumonia encompasses inpatients
CRE Carbapenem-resistant
Enterobacteriaceae who develop hospital-acquired pneumonia (HAP [i.e., new pneu-
EA Endotracheal aspirate monia developing more than 48 hours after hospital admission]),
EI Extended infusion patients who develop ventilator-associated pneumonia (VAP [i.e.,
HAP in a patient on mechanical ventilation]), and patients admit-
ESBL Extended-spectrum β-lactamase
ted to the ICU with severe community-acquired pneumonia (CAP).
HAP Hospital-acquired pneumonia
In the past several years, however, there have been several relevant
MDR Multidrug-resistant (organism)
treatment updates, including new guideline revisions for HAP/VAP
PCT Procalcitonin
and CAP, new antibiotic approvals, and important clinical trials. This
RCT Randomized controlled trial
chapter focuses on updates in the treatment of HAP/VAP and CAP for
VAP Ventilator-associated pneumonia
patients in the ICU.
Table of other common abbreviations.
EPIDEMIOLOGY
HAP/VAP Infections
Hospital-acquired pneumonia/ventilator-associated pneumonia is
the most common hospital-acquired infection after UTIs. However,
because most UTIs are not life threatening, HAP/VAP is the most com-
mon life-threatening infection in ICU patients. This chapter focuses
on VAP because about 90% of patients with HAP are mechanically
ventilated, and because the current guidelines for empiric treatment
of HAP and VAP are essentially identical. Almost 10% of mechanically
ventilated ICU patients develop VAP, though this varies widely in dif-
ferent types of ICU patients. For instance, critically ill trauma patients
(particularly those with a severe traumatic brain injury) seem to have
a higher rate of VAP than medical ICU patients (Kalil 2016).
Longer durations of mechanical ventilation and lengths of ICU and
hospital stays, all by up to 7–10 days, are consequences related to
developing HAP/VAP (Kalil 2016). Raw mortality in patients with VAP

CCSAP 2021 Book 1 • Pulmonary and Endocrinology 95 Pneumonia in the ICU


is high (around 30%–50%), depending on the patient popula- year. This chapter focuses on the subset of patients with
tion, but attributable mortality is more difficult to determine severe CAP who are admitted to the ICU. Patients are consid-
and has been reported as 5%–15%, depending on the patient ered to have severe CAP if they have either one major criterion
population (Kalil 2016). Some studies have shown no attrib- (septic shock or requiring mechanical ventilation) or three
utable mortality related to VAP; however, others have shown or more minor criteria, which include worrisome signs and
a substantial increase. The severity and type of underlying symptoms such as leukopenia, hypothermia, hypotension
illnesses in patients largely affect mortality; however, most requiring aggressive fluids, impaired oxygenation, multi-lobar
critical care clinicians agree that some patients die of VAP. infiltrates, or mental status changes (Metlay 2019).
In addition, HAP/VAP is a large driver of antibiotic use in
the hospital (around 50% of antibiotic use) and serves as an
PATHOPHYSIOLOGY
important target for antimicrobial stewardship (Kalil 2016).
These efforts include informed empiric antibiotic selection, HAP/VAP Infections
high-quality diagnostics, and streamlining/discontinuing All infections require pathogenic organisms and a suscep-
antibiotics appropriately. Finally, VAP poses a high finan- tible host. Mechanically ventilated patients in the ICU meet
cial burden to the health care system, primarily because of both criteria. Organisms can reach the lungs from overt aspi-
increased length of stay (Kalil 2016). ration, or micro-aspiration during mechanical ventilation, of
naso-oropharyngeal or stomach secretions. Supine posi-
Community-Acquired Pneumonia tioning, sedation, neuromuscular blockade, and possibly
Community-acquired pneumonia is a common cause of hos- increased stomach pH aid in micro-aspiration. The endo-
pitalization, accounting for about 1.5 million admissions tracheal tube increases the risk of VAP in general ways by
annually. In contrast to HAP/VAP, for which attributable mor- (1) bypassing natural swallowing defenses for organisms
tality can be difficult to determine, CAP is the most common to enter the lung; (2) making it more difficult to mobilize
cause of infection-related deaths in the United States each secretions from the lung, which creates an attractive envi-
ronment for bacterial growth; and (3) providing a foundation
for bacterial microfilm growth. In addition, alterations in
the immuno-inflammatory phases occur such that an initial
BASELINE KNOWLEDGE STATEMENTS proinflammatory response from illness or injury results in
pulmonary endothelial damage, followed by an anti-inflam-
Readers of this chapter are presumed to be familiar
with the following: matory phase in which the immune response is somewhat
suppressed (Kalil 2016).
• General knowledge of the epidemiology and
pathophysiology of HAP/VAP and severe CAP
Community-Acquired Pneumonia
• General recommendations on diagnosis of HAP/
VAP and severe CAP Severe CAP develops from some of the same general con-
• Likely organisms causing HAP/VAP and CAP and cepts as HAP/VAP with a few differences. Patients with
general empiric therapies CAP often have impaired lung function, clearance of mucus,
• General issues regarding definitive therapy for and and epithelial function as the result of asthma, chronic
prevention of HAP/VAP and severe CAP obstructive pulmonary disease (COPD), or smoking. Age- or
disease-related impairment in swallowing can contribute to
Table of common laboratory reference values.
microaspiration of organisms. Finally, immune function can
be weakened with age, diabetes, and immunosuppressive
ADDITIONAL READINGS states (Metlay 2019).

The following free resources have additional back-


ground information on this topic: RISK FACTORS AND PREVENTION
• Metlay JP, Waterer GW, Long AC, et al. Diagnosis Risk factors for HAP/VAP can be broadly divided into mod-
and treatment of adults with community-acquired
pneumonia. Am J Respir Crit Care Med 2019; ifiable and nonmodifiable (Table 1). Several modifiable risk
200:e45-e67. factors are also targeted by pharmacists and other clinicians
for potential VAP prevention. In many ICUs, VAP prevention
• Kalil AC, Metersky ML, Klompas M, et al.
Management of adults with hospital-acquired and modalities have been packaged into pneumonia prevention
ventilator-associated pneumonia: 2016 clinical “bundles” (Klompas 2014) or more expansive bundles aimed
practice guidelines by the Infectious Diseases at ICU liberation (i.e., discontinuation of mechanical ventila-
Society of America and the American Thoracic tion and ICU discharge) (SCCM 2020). Because the need for
Society. Clin Infect Dis 2016;63:e61-e111.
mechanical ventilation is perhaps the strongest risk factor for
developing VAP, many prevention modalities for VAP are in

CCSAP 2021 Book 1 • Pulmonary and Endocrinology 96 Pneumonia in the ICU


Table 1. Common Risk Factors for HAP/VAP and CAP and Associated Prevention Measures

Modifiable Risk Factors Prevention Measure

Sedation and opioids Pain/agitation/delirium protocols

Neuromuscular blockade Optimize pain/agitation/ventilator to minimize use

Enteral nutrition (aspiration risk) Appropriate monitoring/management

Hyperglycemia Appropriate glucose management (goal < 180 mg/dL)

Blood transfusions Appropriate blood product management

Lack of immunizations Pneumococcal, influenza, and COVID-19 vaccinations, as appropriate

Nonmodifiable Risk Factors Prevention Measure (if possible)

Age

Lung disease (asthma, COPD, smoking)

Need for mechanical ventilation Minimize using the measures listed earlier
Elevate head of bed 30–45 degrees
Use endotracheal tubes with subglottic suctioning in selected patients (e.g., longer
duration of ventilation)

Diabetes/cardiovascular disease

Alcohol abuse

Trauma-related risk factors (e.g., traumatic


brain injury, chest trauma)

CAP = community-acquired pneumonia; COPD = chronic obstructive pulmonary disease; HAP = hospital-acquired pneumonia; VAP =
ventilator-associated pneumonia.

fact techniques used to minimize the duration of mechanical HAP/VAP Infections


ventilation. For example, the current SCCM ABCDEF bundle The current guidelines still recommend that abnormalities
focuses on (A) assessing, preventing, and managing pain; in several traditional signs and symptoms such as tem-
(B) spontaneous awakening and breathing trials; (C) choos- perature, WBC, tachycardia, tachypnea, sputum production,
ing appropriate sedatives and sedation goals; (D) delirium or oxygenation should trigger a lower respiratory tract cul-
assessment, prevention, and management; (E) early mobil- ture. However, only around 50% of patients with traditional
ity; and (F) family engagement. Other commonly included signs and symptoms of HAP/VAP actually have the infection.
modalities include elevating the head of the patient’s bed Thus, an appropriate microbiologic test is required to con-
(“E”), administering oral care, optimizing enteral nutrition firm a HAP/VAP diagnosis in patients with clinical suspicion.
(“E”), minimizing antibiotics (“A”), and providing stress ulcer However, many clinicians still do not follow up on signs and
prophylaxis, when possible. Several therapies such as opi- symptoms with an appropriate microbiologic test or initiate
oid analgesics, sedatives, neuromuscular blockers, enteral empiric antibiotics.
nutrition, and stress ulcer prophylaxis that increase gastric A major change in the 2016 guidelines was a move away
pH have been associated with VAP development. Thus, many from recommending more invasive, quantitative cultures
VAP prevention modalities help mitigate or optimize these such as bronchoscopic bronchoalveolar lavage (BAL) or bron-
therapies and are amenable to pharmacist intervention. choscopic protected specimen brush cultures in favor of the
less-invasive technique of endotracheal aspirate (EA) with
semiquantitative cultures (Kalil 2016). This change was based
DIAGNOSIS on a lack of clear outcome benefits for more invasive cultures
Optimal diagnostic tests for HAP/VAP and CAP remain elu- when the literature was thoroughly reviewed. Endotracheal
sive. Two primary issues are the difficulty in obtaining a aspirate cultures are collected by inserting a sterile suction
high-quality respiratory culture sample and the low spec- catheter through the patient’s endotracheal tube into the
ificity of traditional signs and symptoms such as fever and proximal lung space, and secretions are suctioned into a ster-
leukocytosis. ile container for quantitative or semiquantitative analysis.

CCSAP 2021 Book 1 • Pulmonary and Endocrinology 97 Pneumonia in the ICU


Each technique has pros and cons. Compared with EA, bron- elevated PCT concentrations over time can help decrease
choscopic BAL is more invasive, requires trained personnel antibiotic duration. The pneumonia severity index and CURB-
and specialized equipment, is costly, and can have signifi- 65 tools are useful in less severe CAP to determine the need
cant adverse events. However, bronchoscopic BAL is more for hospitalization but are not recommended to determine
accurate than EA, especially at identifying patients without whether patients require ICU admission (Metlay 2019).
VAP, which can be a powerful antimicrobial stewardship tool
(CCCTG 2006). This is especially true when using quantita- LIKELY ORGANISMS AND EMPIRIC
tive culture results from BAL, which have rather clear cutoff THERAPY
values for safely discontinuing antibiotics in patients without
HAP/VAP Infections
VAP in a variety of ICU patient populations. Similar data are
lacking for semiquantitative cultures from EA. As such, the Low vs. High Risk of MDR Organisms
guidelines recommend that centers that use more invasive The current guidelines have provided some major changes in
procedures continue to do so (Kalil 2016). how to stratify patients by low or high risk of having pneu-
Because of the complexity of obtaining pulmonary cul- monia caused by multidrug-resistant (MDR) organisms (Kalil
tures, composite tools of signs and symptoms such as the 2016). The previous version of the guidelines introduced the
clinical pulmonary infection score (CPIS) have been devel- term health care–associated pneumonia (HCAP) to identify
oped to improve the specificity of parameters such as fever patients who had previous significant interactions with the
and leukocytosis. However, after an initial surge in popularity, health care system that placed them at higher risk of MDR
the CPIS has largely been abandoned because subsequent organisms and thus the need for broader-spectrum empiric
studies showed that it did not perform well at diagnosing antibiotics. These criteria were largely based on expert opin-
VAP. More recently, a system of surveillance tools under the ion. In the intervening years, higher-quality data identified
umbrella term ventilator-associated events was created to pro- patient populations at higher risk of MDR organisms (Kalil
vide more objective tracking of adverse outcomes, including 2016). The current guidelines dropped the HCAP designation
possible and probable pneumonia. However, these tools are in favor of a simple low- or high-risk determination for these
not meant to be diagnostic criteria for the bedside treatment pathogens (see Table 1). However, the new guidelines some-
of patients, and their use in that role is uncertain. A poten- what confusingly separated the treatment of HAP and VAP.
tially helpful result of research in this area is that worsening The empiric antibiotic recommendations are almost identical
oxygenation status is the most useful of the clinical signs and for both states, and, as stated previously, I will focus on VAP.
symptoms associated with VAP. For patients with none of the MDR risk factors (i.e.,
Adding biomarkers such as procalcitonin (PCT) to more low-risk patients), likely organisms include Haemophilus
rapidly diagnose VAP has largely failed, and they are not rec- influenzae, Streptococcus pneumoniae (and other species),
ommended as diagnostic tools. Modern rapid diagnostics methicillin-sensitive S. aureus (MSSA), and more sensitive
have focused far more on other types of infections (e.g., bac- Enterobacteriaceae. Atypical bacteria such as Mycoplasma
teremia, Clostridioides [formerly Clostridium] difficile infection) pneumoniae are not common causes of HAP/VAP and are
than HAP/VAP. Recent data suggest that meaningful results not covered empirically. The previous guidelines recom-
can be obtained by testing pulmonary secretions, an exciting mended non-antipseudomonal agents such as ceftriaxone
area to observe over the next several years (Pickens 2020). for such patients; however, the new guidelines (Kalil 2016)
have expanded their recommendation to provide coverage
Community-Acquired Pneumonia for Pseudomonas and MSSA. Using this expanded recommen-
Optimal CAP diagnosis has a related set of complexities, dation provides effective coverage of more organisms but
including the difficulty in obtaining a high-quality sputum risks the overuse of antipseudomonal agents in ICUs when
sample in a large percentage of patients. Patients with severe Pseudomonas is rare in low-risk patients. Ideally, ICUs should
CAP should have sputum and blood cultures obtained. It is generate annual antibiograms according to the Clinical &
unclear whether more invasive procedures such as broncho- Laboratory Standards Institute recommendations and track
scopic BAL or protected specimen brush improve patient organisms that cause VAP in their units to provide the best-ed-
outcomes, and these procedures are not routinely recom- ucated selection for empiric therapy for every unit. Tracking
mended. Unlike HAP/VAP, there are rapid diagnostic tests for organisms that cause VAP is a step that anecdotally is rarely
urinary pneumococcal and Legionella urinary antigens, which added to the antibiogram but offers valuable data that critical
are recommended in severe CAP. Alternatively, a PCR-based care pharmacists are well situated to provide.
test for Legionella can be performed on lower respiratory Each of the recommended agents has advantages and dis-
tract secretions. A nasal PCR test for methicillin-resistant advantages. The first main antipseudomonal β-lactam option
Staphylococcus aureus (MRSA) is also recommended to help is cefepime, which provides enhanced activity against pen-
guide the need for anti-MRSA therapy. As with HAP/VAP, PCT icillin-resistant pneumococcus (similar to ceftriaxone) as
is not recommended as a diagnostic tool, but resolution of well as expanded activity against some gram-negative bacilli

CCSAP 2021 Book 1 • Pulmonary and Endocrinology 98 Pneumonia in the ICU


compared with the other primary agents. However, over the estimate the AUC. Published data have shown an association
past decade, neurotoxicity with cefepime has been of con- between AUC values of 400–600 and improved outcomes in
cern, especially in patients with renal dysfunction when doses MRSA infections. It is less clear whether the currently recom-
are not adjusted. The second main antipseudomonal β-lac- mended trough goal of 15–20 mg/dL for severely ill patients
tam option is piperacillin/tazobactam, which adds coverage or life-threatening infections is associated with improved
for Bacteroides fragilis; although B. fragilis is not a concern in outcomes. Nephrotoxicity associated with vancomycin con-
VAP, it may be a concern in patients with other relevant infec- tinues to cause concern. The latest trend in the data suggests
tion sites (e.g., intra-abdominal). However, the main concern that vancomycin can be nephrotoxic when trough concentra-
with piperacillin/tazobactam over the past 5 years has been tions are greater than 20 mg/dL or especially when combined
an association with nephrotoxicity. Many observational stud- with other nephrotoxins (e.g., loop diuretics, intravenous con-
ies suggest that piperacillin/tazobactam is more nephrotoxic trast dye); however, the nephrotoxicity rate without these
than cefepime (Bellos 2020). Nephrotoxicity rates have been additional factors is probably low. Similarly, linezolid has
higher in studies with or without vancomycin therapy in both limitations such as neuropathies (potentially optic nerve
groups (Bellos 2020). The third main antipseudomonal β-lac- damage), anemias (most notably thrombocytopenia), and
tam option is imipenem/cilastatin or meropenem. This is serotonin syndrome if given with serotonergic drugs. The
generally the broadest spectrum of the three classes and may FDA has raised its level of concern about SSRI interactions
be excessive for the organisms causing VAP in many low-risk in the past several years. Regarding efficacy, some clinicians
ICU patients. Relative to cefepime and piperacillin/tazobac- prefer linezolid because of the results of the largest random-
tam, imipenem/cilastatin or meropenem has coverage for ized controlled trial (RCT) to date comparing vancomycin and
penicillin-resistant S. pneumoniae, B. fragilis, and, importantly linezolid for MRSA VAP (Wunderink 2012). Linezolid was more
for VAP, extended-spectrum β-lactamase (ESBL)-positive effective; however, there are concerns about the trial, includ-
Enterobacteriaceae and Acinetobacter. ing possible differences between groups at baseline and poor
For patients with a severe β-lactam allergy, there is a vancomycin concentrations, which may have affected the
renewed focus on aztreonam as an antipseudomonal option. results. The guidelines do not specify a preference between
However, aztreonam requires a drug for gram-positive organ- vancomycin and linezolid for MRSA coverage. Of interest,
isms concurrently (e.g., vancomycin) and may have worse although telavancin is FDA approved for MRSA VAP, the FDA
Pseudomonas activity than the primary agents used in cer- recommends it only as a second-line agent because of worse
tain ICUs. Over the past decade, there has been a greater outcomes in patients with renal dysfunction.
understanding of “penicillin allergy cross-reactivity” with For gram-negative coverage, the primary β-lactam drugs
non-penicillin β-lactams. The general theme is that carbapen- were discussed earlier except for ceftazidime. Ceftazidime
ems, as well as aztreonam, have very low cross-reactivity. In has fallen out of favor in most centers but is currently avail-
addition, later-generation cephalosporins such as cefepime able and has maintained good antipseudomonal activity.
have very low cross-reactivity and can generally be used A secondary antipseudomonal, a fluoroquinolone such as
in a non–life-threatening allergy. In severe-life threatening ciprofloxacin or levofloxacin, or an aminoglycoside, is rec-
allergies, levofloxacin and ciprofloxacin are non–β-lactam ommended for patients with one or more MDR risk factors to
options. Ciprofloxacin may have better Pseudomonas activity, prevent inadequate empiric antibiotic therapy, which occurs
but the minimal MSSA activity necessitates a second drug when the organism is resistant to the primary β-lactam
(e.g., vancomycin). Levofloxacin has variable activity against drug. Inadequate empiric antibiotic therapy has been asso-
Pseudomonas in some centers, so clinicians should confirm ciated with higher mortality in some studies. Ciprofloxacin
the susceptibility rates in their units. Disadvantages of quino- and levofloxacin have disadvantages as outlined earlier.
lones include neuropathy, tendon rupture, dysglycemia, and Aminoglycosides require individualized weight-based dos-
perhaps a higher induction of bacterial resistance and an ing and therapeutic drug monitoring. Once-daily dosing can
association with C. difficile than with other drug classes. decrease the risk of nephrotoxicity, but these agents remain
For patients with one or more MDR risk factors, triple nephrotoxic and are also ototoxic. These risks increase with
therapy with an anti-MRSA agent and two antipseudomo- longer therapy and higher serum concentrations but can also
nal agents from different drug classes is recommended. occur at therapeutic (i.e., normal) concentrations.
Vancomycin or linezolid is recommended for MRSA coverage. Pharmacists can help capture a critical point that clini-
However, each has limitations. Vancomycin requires individu- cians often miss by providing local susceptibility patterns
alized, weight-based dosing with therapeutic drug monitoring. for the secondary agent to determine its likelihood of cov-
After more than 10 years of trough-only recommended mon- ering organisms that are resistant to the first drug. Ideally,
itoring, the newly updated MRSA treatment guideline this should require reviewing the sensitivity report for all
revision recommends AUC monitoring (Rybak 2020). This gram-negative organisms associated with VAP and tracking
will be somewhat of a “back-to-the-future” scenario in which which secondary option would provide appropriate cover-
two concentrations (e.g., a peak and trough) are needed to age. For instance, in the unit where I work, organisms that

CCSAP 2021 Book 1 • Pulmonary and Endocrinology 99 Pneumonia in the ICU


are resistant to cefepime (our empiric VAP antibiotic) are also not RCTs, which sometimes have exaggerated outcome ben-
resistant to ciprofloxacin, gentamicin, and tobramycin. The efits because of likely type I errors. Other studies have not
only secondary agent that would provide additional coverage shown improved outcomes with continuous infusion/EI dos-
is amikacin, which is generally reserved for highly resistant ing. Continuous infusion/EI dosing appears to be safe and
organisms. I recommend performing this type of analysis to may improve the efficacy of β-lactams. Main disadvantages
make an informed decision in every ICU regarding the role of a include additional intravenous access as a result of pro-
secondary antipseudomonal agent (Luu 2020). Empiric treat- longed infusions (e.g., potential incompatibilities) and the
ment in patients with a severe β-lactam allergy can become logistics of implementation.
difficult if it is determined that no β-lactams can be used. In
such cases, an antipseudomonal quinolone plus an amino- Community-Acquired Pneumonia
glycoside is probably the most reasonable option, though The most common bacterial organisms causing severe CAP
with limited data. in ICU patients continue to be S. pneumoniae, H. influenzae,
and atypical bacteria (M. pneumoniae, Chlamydia pneumoniae,
Dosing Considerations Legionella pneumoniae). Empiric antibiotic recommendations
Finally, dosing is critical in HAP/VAP because of the sever- for severe CAP have not generally changed in the most recent
ity of the infection, and certain agents such as vancomycin, guideline update (Metlay 2019). There is a stronger empha-
β-lactams, and aminoglycosides do not adequately penetrate sis on determining whether patients have risk factors for
into lung secretions. The 2005 guidelines highlighted this MRSA or Pseudomonas. For patients with severe CAP without
issue; therefore, VAP doses for many agents differ from typi- risk factors for MRSA or Pseudomonas, either an appropriate
cal doses (Table 2). β-lactam plus an intravenous macrolide (azithromycin) or a
Another area of increased research over the past decade β-lactam plus a respiratory quinolone is still recommended.
has been the use of continuous infusions, or easier 3- to Some data suggest that patients have lower mortality with
4-hour extended infusions (EIs) of β-lactams to take advan- β-lactam/macrolide therapy, but this evidence was not con-
tage of the time over MIC pharmacodynamic killing profile for sidered strong enough to prioritize such regimens. The most
these drugs. Meropenem, cefepime, and piperacillin/tazobac- widely used β-lactam is ceftriaxone because of its enhanced
tam have been the most-studied agents, with mixed results. activity against penicillin-resistant S. pneumoniae and con-
Of importance, no study shows that patients have worse out- venience of once-daily dosing. Ampicillin/sulbactam is also
comes with continuous or EI dosing. Some studies show listed as an option; however, it lacks activity against penicil-
dramatically improved outcomes with continuous infusion/ lin-resistant S. pneumoniae. Levofloxacin and moxifloxacin are
EI dosing. However, these are typically observational studies, the most common “respiratory” quinolones because of their

Table 2. Suggested HAP/VAP Dosing for Common IV Agentsa

Agent Suggested Dosage

Amikacin 15–20 mg/kg/day

Aztreonam 2 g q6–q8hr (for life-threatening infections)

Cefepime 2 g q8–q12hr (2 g q8hr used in most recent VAP studies)

Ciprofloxacin 400 mg q8hr (HAP/VAP dose)

Gentamicin/tobramycin 5–7 mg/kg/day

Imipenem/cilastatin 1 g q6–q8hr (3–4 g/day recommended for life-threatening infections)

Levofloxacin 750 mg q24hr (CAP dose)

Meropenem 1–2 g q8hr (no VAP dose, 1 g q8hr indicated for pseudomonal infections, 2 g q8hr
recommended for meningitis)

Piperacillin/tazobactam 4.5 g q6hr (HAP/VAP dose with standard intermittent infusion)

a
Doses are for normal renal function. Dose adjustments in renal impairment may be required.
IV = intravenous(ly); q = every.
Information from: American Thoracic Society; Infectious Diseases Society of America. Guidelines for the management of adults
with hospital-acquired, ventilator-associated, and healthcare-associated pneumonia. Am J Respir Crit Care Med 2005;171:388-416.

CCSAP 2021 Book 1 • Pulmonary and Endocrinology 100 Pneumonia in the ICU
activity against penicillin-resistant S. pneumoniae as well as however, many studies have not shown improved outcomes
ceftriaxone-resistant pneumococcus. Some clinicians pre- for treating Pseudomonas (or other gram-negative organ-
fer to avoid quinolones, when possible, because of the issues isms) with two agents. The main exception is in patients with
described earlier. However, azithromycin has an increased risk septic shock, for which some data suggest better outcomes
of cardiac arrhythmias, which may limit its use in certain situ- with dual agents (Ibrahim 2020). The guidelines suggest that
ations. Doxycycline is a third-line agent for atypical coverage patients with Pseudomonas VAP who are in septic shock (or at
after azithromycin and respiratory quinolones for less-severe high risk of death) should receive two antipseudomonals as
hospitalized CAP; however, it is not recommended for severe definitive therapy. In patients without septic shock, the cur-
CAP because of the lack of data in these patients. Finally, there rent guidelines recommend discontinuing the second agent
is no need to add anaerobic coverage in patients with aspira- once it is known that the organism is being covered by the pri-
tion pneumonia because evidence is minimal that anaerobic mary agent (Kalil 2016).
organisms cause severe CAP (Metlay 2019).
A major change of emphasis in the guidelines is assess- Community-Acquired Pneumonia
ing risk factors for MRSA or Pseudomonas and broadening Similar to HAP/VAP, previous studies in addition to the guide-
empiric coverage, as necessary, to target these organisms. lines recommend that MRSA and antipseudomonal therapies
The guidelines recommend that centers validate risk factors be de-escalated to more traditional regimens when possible
for MRSA and Pseudomonas CAP in their patient populations; according to the culture and sensitivity report. This serves
however, this level of research may not be feasible in many as another stewardship opportunity that may be suitable in
centers. In general, prior isolation of MRSA or Pseudomonas many medical centers.
(especially from the respiratory tract) or recent hospital-
ization and exposure to intravenous antibiotics are the Therapy Duration
best-known risk factors for those pathogens. For patients at Another stewardship opportunity for pharmacists is in lim-
high risk who have MDR risk factors, empiric therapy is the iting the duration of definitive therapy for VAP. The current
same as for HAP/VAP. guidelines recommend 7 days of treatment for patients with
a good clinical response to therapy. However, the landmark
study of this question found that nonfermenting gram-neg-
DEFINITIVE THERAPY
ative bacilli (almost exclusively Pseudomonas) had a higher
HAP/VAP Infections recurrence rate with 8 days of therapy than with 15 days
As mentioned in the Diagnosis section, one advantage of (Chastre 2003). Thus, the guideline recommendation is some-
quantitative invasive cultures (e.g., bronchoscopic) is the what unclear regarding the recommended treatment duration
support of observational studies that suggest it is safe to for Pseudomonas. In real-world clinical practice, the average
discontinue empiric antibiotics in patients who do not have duration of definitive therapy for VAP is often much longer
bacterial growth above recognized thresholds (e.g., 10,000 than 7 days. This may be because of clinician reluctance to
or 100,000 CFU/mL for BAL) and who have had infection treat for a short time. An instructive example are the data on
ruled out at other sites. This is a tremendous antimicrobial using decreases in serial PCT concentrations to aid in the dis-
stewardship opportunity for discontinuing empiric antibiot- continuation of antibiotics for VAP. Most studies show that
ics in patients without VAP. However, this is also probably adding PCT monitoring can result in shorter therapy for VAP.
the most difficult step to gain acceptance from physicians. However, using PCT monitoring may often reduce the VAP
The recommendation to use EA with semiquantitative cul- therapy duration to where it should have been originally (i.e.,
tures effectively makes this result a qualitative proposition about 7 days). Thus, PCT can still be an antimicrobial stew-
because there are no high-quality data on the difference in ardship tool by encouraging ICU teams to follow the clinical
patient outcomes (i.e., infection vs. colonization) between the guidelines that they could otherwise be reluctant to follow.
four gradations (e.g., 1+, 2+, 3+, 4+). Thus, the only opportu- Using PCT monitoring may better pinpoint patients who
nity to discontinue empiric antibiotics is with a fully negative require longer therapy, though that remains unanswered.
test. A middle option is to use quantitative results with EA Recommended therapy duration for inpatient CAP is 5–7
samples at a diagnostic threshold of 1,000,000 CFU/mL; how- days. The guidelines note that most treatment duration stud-
ever, there are fewer data with EA thresholds than with BAL. ies for CAP did not include many patients with severe CAP;
For patients with a definitive diagnosis of VAP, another however, they still suggest 5–7 days in patients who are
major antimicrobial stewardship opportunity is streamlin- responding well to treatment. The guidelines recommend that
ing the regimen, when possible. An evolving question over MRSA or Pseudomonas CAP be treated for 7 days. However,
the past decade is the appropriate duration of the second- as with HAP/VAP, many centers routinely treat patients for
ary antipseudomonal agent to treat a gram-negative VAP. more than 7 days, resulting in another opportunity for stew-
In past years, many clinicians continued the second agent ardship. The experience with PCT is also similar between CAP
when treating Pseudomonas. Some clinicians still do this; and HAP/VAP, where it may decrease the average therapy

CCSAP 2021 Book 1 • Pulmonary and Endocrinology 101 Pneumonia in the ICU
duration. Another issue leading to longer therapy is the rel- option for MRSA coverage. Omadacycline and eravacycline
atively high percentage of patients in whom an organism is are new fluorocycline agents (similar to tetracyclines) that
never identified. Most clinicians continue to treat the CAP have activity against MRSA but are not indicated for VAP.
definitively on the basis of clinical presentation. Omadacycline has similar indications to ceftaroline: CAP
(excluding MRSA) and acute bacterial skin and skin structure
Treating MDR Organisms/Newer Treatment infections (including MRSA). Delafloxacin has MRSA activity
Options but also lacks an indication for HAP/VAP.
Over the past decade, some new promising agents have been The development of gram-negative agents has been more
developed for the treatment of MDR VAP (Table 3). For MRSA promising. Ceftolozane/tazobactam has a potential niche as
coverage, telavancin is a secondary option, as discussed ear- a drug for MDR Pseudomonas with resistance to other β-lac-
lier. Ceftaroline has been used for MRSA VAP; however, there tams. Another potential use for ceftolozane/tazobactam is
are no high-quality data for this indication. Similar to other as a carbapenem-sparing agent for ESBL infections when
cephalosporins that are widely used for CAP and HAP/VAP, carbapenem restriction is desirable because of resistance
ceftaroline has a similar safety profile but offers an additional issues. However, ceftolozane/tazobactam does not have good

Table 3. Newer Agents for Multidrug-Resistant Organisms

Typical
Activity Against: Dosinga Comments
Extended- Carbapenem-
Spectrum Resistant
Agent MRSA Pseudomonas β-Lactamases Enterobacteriaceae

Ceftaroline Yes No No No 600 mg IV Has CAP indication


(Teflaro) q12hr (but not MRSA CAP)

Ceftolozane/ No Yes Yes No 1.5 g IV q8hr


tazobactam
(Zerbaxa)

Ceftazidime/ No Yes Yes Yes 2.5 g IV q8hr Has HAP/VAP indication


avibactam
(Avycaz)

Meropenem/ No Yes Yes Yes 4 g IV q8hr


vaborbactam
(Vabomere)

Imipenem/ No Yes Yes Yes 1.25 g IV q6hr


cilastatin/
relebactam
(Recarbrio)

Eravacycline Yes No Yes Yes 1 mg/kg IV Increase dose to 1.5 mg/kg


(Xerava) q12hr with CYP3A inducers

Omadacycline Yes No Yes Yes 200 mg IV on Has CAP indication. Oral


(Nuzyra) day 1; then option available (dosing
100 mg IV daily different from IV)

Delafloxacin Yes Yes No No 300 mg IV Oral option available


(Baxdela) q12hr (dosing different from IV)

Plazomicin No Yes Yes Yes 15 mg/kg IV


(Zemdri) daily

a
Doses are for normal renal function. Dose adjustments in renal impairment may be required for all agents except eravacycline and
omadacycline.
CAP = community-acquired pneumonia; HAP = hospital-acquired pneumonia; MRSA = methicillin-resistant Staphylococcus aureus;
VAP - ventilator-acquired pneumonia.

CCSAP 2021 Book 1 • Pulmonary and Endocrinology 102 Pneumonia in the ICU
activity against carbapenem-resistant Enterobacteriaceae who require supplemental oxygen (including mechanical ven-
(CRE). Fortunately, ceftazidime/avibactam, meropenem/ tilation or extracorporeal membrane oxygenation [ECMO]). Of
vaborbactam, and imipenem/cilastatin/relebactam are note, some clinicians may alternatively use a dexamethasone
highly active against CRE organisms and MDR Pseudomonas. (or other steroid) regimen developed for acute respiratory dis-
These agents are replacing colistin in many centers, primarily tress syndrome (ARDS) as opposed to COVID-19 such as the
because of the risk of nephrotoxicity, questionable efficacy as CoDEX trial dosing (dexamethasone 20 mg for 5 days, then 10
monotherapy, uncertainties regarding optimal combination mg for 5 days) (Tomazini 2020). Remdesivir can be added to
therapy, and difficulty with sensitivity testing. Some hospitals dexamethasone in patients requiring supplemental oxygen;
are not equipped to provide timely sensitivity testing for colis- however, it is no longer recommended in mechanically venti-
tin. The new agents likely provide a better safety profile and lated patients or patients receiving ECMO because of unclear
faster sensitivity testing in many cases. However, they are benefits in these groups. For patients who cannot receive
expensive, with retail acquisition costs of as much as $1000 dexamethasone, the currently available anti-inflammatory
per day. Plazomicin is a new aminoglycoside with improved arthritis agent baricitinib has received an emergency use
activity against some gram-negative isolates that are resis- authorization from the FDA for use in combination with rem-
tant to other aminoglycosides. Although not FDA approved desivir. Two antibody agents (bamlanivimab and casirivimab/
for VAP, plazomicin may be an attractive secondary option in imdevimab) have received emergency use authorizations from
very limited situations. Of interest, omadacycline and erava- the FDA but are only to be used in non-hospitalized patients.
cycline have activity against some CRE organisms but lack Convalescent plasma is not currently recommended because
good-quality clinical data in VAP. Regarding tigecycline, the of a lack of data. Similarly, no special anticoagulation regi-
inferior efficacy observed in severe infections warrants con- mens for venous thromboembolism (VTE) prophylaxis are
sideration when evaluating its role. indicated, despite the apparent high risk of VTE associated
A major lack in the treatment spectrum for the new β-lac- with COVID-19.
tams is Acinetobacter. Because MDR Acinetobacter isolates
often use several mechanisms of resistance, the new agents Role of Corticosteroids
do not significantly improve coverage against most isolates Most research in this area has been in severe CAP. The
compared with traditional agents. Treatment of MDR (i.e., guidelines do not recommend routine use of low doses of
carbapenem resistant) Acinetobacter continues to include corticosteroids (e.g., hydrocortisone 200–300 mg/day) for
primarily ampicillin/sulbactam, colistin, or doxycycline/ severe CAP caused by bacteria or an influenza virus. Two
minocycline. Stenotrophomonas maltophilia can be problem- RCTs and subsequent meta-analyses have shown mortality
atic in some units and is generally treated with high-dose benefits from steroids in CAP (Metlay 2019). However, these
trimethoprim/sulfamethoxazole (e.g., 12–15 mg/kg/day of positive studies had significant design flaws, including differ-
trimethoprim) or a second-line option such as fluoroquinolo- ences in patient groups at baseline and a suspiciously high
nes. S. maltophilia sensitivities are not reliable in automated mortality difference. Other studies have also failed to show
testing; therefore, E-tests or disk diffusion tests are needed. improved mortality.
Intravenous trimethoprim/sulfamethoxazole has issues, For HAP/VAP, the HYPOLYTE study of trauma patients in
including the need for appropriate weight-based dosing, large the ICU (Roquilly 2011) reported that low-dose corticoste-
volumes for infusion, limited stability after mixing, and sev- roids decreased the incidence of VAP, though this practice
eral serious adverse events such as life-threatening skin is not currently recommended, largely because of the lack
reactions, anemias, and hepato- and nephrotoxicity. of confirmatory data and concerns of some clinicians about
corticosteroid use in surgical patients (e.g., impaired wound
Antiviral Therapies healing).
Influenza is a primary cause of severe CAP and can easily
be diagnosed with a rapid test. The guidelines recommend OPTIONS FOR NONRESPONDING/
administering oseltamivir to patients who test positive for RECURRENT INFECTIONS
influenza regardless of the symptom duration before the Despite appropriate antibiotic therapy, initial therapy fails for
diagnosis. Oseltamivir should optimally be initiated within 2 about one-third of patients in VAP RCTs. About one-half of the
days of the start of illness, but lower mortality and shorter failures are deaths and the other half are other failures such
duration of ICU stay may still occur if initiated later. as relapses with the same (often more resistant) organism or
As of writing of this chapter (late 2020), the COVID-19 pan- persistent symptoms after therapy. The text that follows pro-
demic is still active. Treatment guidelines have changed vides three options for optimizing treatment in these patients.
rapidly; see the current treatment guidelines available at the
NIH website (NIH 2020). Some major treatment recommen- Option 1: Optimize Current Therapy
dations include dexamethasone 6 mg daily (intravenous or If the team decides to continue with the current antibiotic (and
enteral) for 10 days or until hospital discharge for patients the sensitivity panel supports that choice), ensure the dose

CCSAP 2021 Book 1 • Pulmonary and Endocrinology 103 Pneumonia in the ICU
Patient Care Scenario
A 25-year-old man (height 69 inches, weight 80 kg) is of inspired oxygen (Fio2) of 40% and positive end-expira-
admitted to the trauma ICU after a motor vehicle crash. He tory pressure (PEEP) of 5 mm Hg. The new trauma fellow
is unconscious and mechanically ventilated with a severe wants to empirically start 7 days of vancomycin 1000 mg
traumatic brain injury and several rib fractures. He has intravenously every 12 hours, piperacillin/tazobactam
no significant medical history. On ICU day 3, he develops 3.375 g intravenously every 6 hours, and tobramycin 300
a fever, leukocytosis, and worsening infiltrates on chest mg every 24 hours. What therapy is best to recommend
radiography. He is hemodynamically stable, has normal for this patient?
renal function, and is still oxygenating well on a fraction

ANSWER
This patient has signs and symptoms of VAP (fever, leu- in their unit and the sensitivity patterns of these organ-
kocytosis, infiltrates), but they are nonspecific and isms to guide the best choice for empiric therapy in a
could simply have been caused by his injuries. He is also given unit. For example, in my ICU, VAP cases in patients
oxygenating well on low ventilator settings, remains hemo- without MDR risk factors are overwhelmingly caused by
dynamically stable, and does not currently appear to have Streptococcus spp., MSSA, and H. influenzae, and as such,
sepsis or septic shock. First, recommend a good-quality we can use narrower empiric therapy for these patients
culture to definitively diagnose VAP (at least a semiquan- (e.g., ampicillin/sulbactam or ceftriaxone). However,
titative EA culture). Because around 50% of suspected standard antibiograms only give sensitivity information
VAP cases are not confirmed by cultures, this provides for organisms from all sites pooled together and do not
a potential opportunity to discontinue antibiotics if the report the causative organisms for each type of infection.
culture is negative, rather than empirically treating it for Nonetheless, such information can be useful in select-
7 days. ing empiric therapy if there is a clear advantage for one of
Second, evaluate the patient’s risk of MDR organisms, the primary antipseudomonals (piperacillin/tazobactam,
and select appropriate empiric therapy. This patient has cefepime, imipenem/meropenem) for activity against
none of the five risk factors for MDR organisms in VAP Pseudomonas and other expected VAP pathogens.
(see Table 1). Therefore, he does not require MRSA cov- There is an educational opportunity regarding antibi-
erage or a second drug for Pseudomonas. Piperacillin/ otic dosing in VAP. This is a 25-year-old with a normal SCr.
tazobactam or cefepime is a standard choice. Imipenem However, he is likely to have a large volume of distribution
or meropenem is technically an option, but each usually because of his injuries and could have augmented renal
provides coverage that is too broad for patients without clearance. Thus, he needs aggressive dosing of antibi-
MDR risk factors. The HAP/VAP guidelines recommend otics. The VAP dose of piperacillin/tazobactam is 4.5 g
that clinicians track which organisms cause HAP/VAP intravenously every 6 hours.

1. Kalil AC, Metersky ML, Klompas M, et al. Management of adults with hospital-acquired and ventilator-associated pneumonia:
2016 clinical practice guidelines by the Infectious Diseases Society of America and the American Thoracic Society. Clin Infect Dis
2016;63:e61-e111.
2. Luu Q, Vitale K, Shan G, et al. Evaluation of guideline recommendations for dual antipseudomonal therapy in hospitalized adults with
pneumonia using combination antibiograms. Pharmacotherapy 2020;40:1089-98.

of the drug is optimized for HAP/VAP dosing, and consider Option 3: Add a Second Agent from a Different
EI/continuous infusion dosing. However, this is probably the Class
least likely course for a patient whose therapy is failing and Where I work, options 2 and 3 are most widely used for
will certainly not be used if the organism becomes resistant nonresponding or recurrent VAP. The question of which sec-
to the currently used agent. ondary agent to add is often answered by the sensitivity
report and, at times, by reviewing the number of dilutions
Option 2: Change the Primary Agent between the reported MIC and the breakpoint for each drug.
Various primary β-lactams have variable activity against com- Overall, there is no clear advantage between quinolones or
mon gram-negative VAP pathogens because of differences in aminoglycosides as a secondary choice. Another option for
sites of action (i.e., binding proteins) and resistance mech- aminoglycosides is aerosolized rather than intravenous admin-
anisms. These differences can be exaggerated by a course istration. The guidelines used to recommend that clinicians
of therapy with one agent that can up-regulate certain resis- consider adding aerosolized aminoglycosides in highly resis-
tance mechanisms. In general, it makes sense to compare the tant or nonresponding VAP. However, this was mainly based
reported MIC with the breakpoint for each drug and choose an on observational literature suggesting that aerosolized ami-
agent that is the furthest dilution from its breakpoint, though noglycosides or colistin improved the outcomes of patients
there are no clear outcome benefits for this practice. with MDR VAP. However, two pivotal RCTs have since been
published that did not show improved outcomes when aero-
solized aminoglycosides were added as initial therapy for VAP

CCSAP 2021 Book 1 • Pulmonary and Endocrinology 104 Pneumonia in the ICU
provide promising alternatives for highly resistant organisms
Practice Points
such as CRE.
HAP/VAP and severe CAP are common in critically ill
patients and offer several opportunities to optimize
individual patient antibiotic regimens as well as oppor- REFERENCES
tunities to improve antimicrobial stewardship at the Bellos I, Karageorgiou V, Pergialiotis V, et al. Acute kidney
policy level: injury following the concurrent administration of
• Guidelines for diagnosing HAP/VAP have changed to favor antipseudomonal β-lactams and vancomycin: a network
less-invasive EA cultures that have various advantages meta-analysis. Clin Microbiol Infect 2020;26:696-705.
and disadvantages compared with more invasive methods
(i.e., bronchoscopic BAL), which can have implications for Canadian Critical Care Trials Group (CCCTG). A randomized
drug therapy. trial of diagnostic techniques for ventilator-associated
• Identifying causative organisms in severe CAP continues to pneumonia. N Engl J Med 2006;355:2619-30.
be difficult, with many patients having negative cultures.
Chastre J, Wolff M, Fagon JY, et al. Comparison of 8 vs
• De-escalating (discontinuing or streamlining) empir-
15 days of antibiotic therapy for ventilator-associated
ic antibiotics on the basis of culture results is a major
pneumonia in adults. JAMA 2003;290:2588-98.
opportunity to decrease antibiotic use.
• Appropriate aggressive dosing for pneumonia and consid- Ibrahim D, Jabbour JF, Kanj SS. Current choices of antibiotic
eration of EIs of β-lactams are recommended. treatment for Pseudomonas aeruginosa infections. Curr
• HAP/VAP prevention bundles revolve around mitigating Opin Infect Dis 2020;33:464-73.
modifiable risk factors such as minimizing sedation,
controlling hyperglycemia, and shortening the duration of Kalil AC, Metersky ML, Klompas M, et al. Management
mechanical ventilation, when possible. of adults with hospital-acquired and ventilator-associ-
• Guidelines for HAP/VAP and severe CAP urge clinicians ated pneumonia: 2016 clinical practice guidelines by the
to track pneumonia pathogens in their ICUs to improve Infectious Diseases Society of America and the American
targeted empiric therapy for their patient populations. This Thoracic Society. Clin Infect Dis 2016;63:e61-e111.
is a major stewardship opportunity.
• General empiric antibiotic selection recommendations have Klompas M, Branson R, Eichenwald EC, et al. Strategies to
been updated for HAP/VAP to include new risk factors for prevent ventilator-associated pneumonia in acute care
MDR organisms. Similarly, updated recommendations for hospitals: 2014 update. Infect Control Hosp Epidemiol
CAP highlight the need to assess for the risk of MRSA and 2014;35:915-36.
Pseudomonas, providing another stewardship opportunity.
• The recommended durations of definitive therapy for HAP/ Kollef MH, Ricard JD, Roux D, et al. A randomized trial of the
VAP and CAP are often shorter than in practice and create amikacin fosfomycin inhalation system for the adjunctive
an additional opportunity to decrease antibiotic use. Serial therapy of gram-negative ventilator-associated pneumo-
PCT monitoring may help ICU teams adopt shorter therapy nia. Chest 2017;151:1239-46.
durations.
• New antibiotics provide options for treating highly resistant Luu Q, Vitale K, Shan G, et al. Evaluation of guideline rec-
organisms, including CRE. ommendations for dual antipseudomonal therapy in
hospitalized adults with pneumonia using combination
antibiograms. Pharmacotherapy 2020;40:1089-98.

Metlay JP, Waterer GW, Long AC, et al. Diagnosis and treat-
(Niederman 2020; Kollef 2017). The role of aerosolized amino-
ment of adults with community-acquired pneumonia.
glycosides in salvage/recurrent therapy is unclear, and some Am J Respir Crit Care Med 2019;200:e45-e67.
clinicians still use them in an attempt to maximize pulmonary
drug concentrations and minimize toxicity. If clinicians use Niederman MS, Alder J, Bassetti M, et al. Inhaled amika-
aerosolized antibiotics for VAP, it is important to understand cin adjunctive to intravenous standard of care antibiotics
in mechanically ventilated patients with gram-negative
good practices for preparation of doses and administration.
pneumonia (INHALE): a double-blind, randomised, place-
bo-controlled, phase 3, superiority trial. Lancet Infect Dis
CONCLUSION 2020;20:330-40.
Hospital-acquired pneumonia/VAP and severe CAP continue
National Institutes of Health (NIH). Coronavirus Disease
to be common infections and common causes of death. They 2019 (COVID-19) Treatment Guidelines. 2020.
also present opportunities to improve antibiotic use and
stewardship. Major updates for HAP/VAP treatment include Pickens C, Wunderink RG, Qi C, et al. A multiplex poly-
merase chain reaction assay for antibiotic stewardship
recommended culture technique, MDR risk assessment, and
in suspected pneumonia. Diagn Microbiol Infect Dis
empiric antibiotic selection. For severe CAP, updates include 2020;98:115179.
enhanced risk assessment for MRSA and Pseudomonas and
related empiric antibiotic selections. New antibiotics may Roquilly A, Mahe PJ, Seguin P, et al. Hydrocortisone therapy
for patients with multiple trauma: the randomized con-
trolled HYPOLYTE study. JAMA 2011;305:1201-9.

CCSAP 2021 Book 1 • Pulmonary and Endocrinology 105 Pneumonia in the ICU
Rybak MJ, Le J, Lodise TP, et al. Therapeutic monitor- Tomazini BM, Maia IS, Cavalcanti AB, et al. Effect of dexa-
ing of vancomycin for serious methicillin-resistant methasone on days alive and ventilator-free in patients
Staphylococcus aureus infections: a revised consensus with moderate or severe acute respiratory distress syn-
guideline and review by the American Society of Health- drome and COVID-19. The CoDEX randomized clinical trial.
System Pharmacists, the Infectious Diseases Society of JAMA 2020;324:1307-16.
America, the Pediatric Infectious Diseases Society, and
the Society of Infectious Diseases Pharmacists. Wunderink RG, Niederman MS, Kollef MH, et al. Linezolid in
Clin Infect Dis 2020;71:1361-4. methicillin-resistance Staphylococcus aureus nosocomial
pneumonia: a randomized, controlled study. Clin Infect Dis
Society of Critical Care Medicine. https://www.sccm.org/ 2012;54:621-9.
ICULiberation/ABCDEF-Bundles. Accessed February 18,
2021.

CCSAP 2021 Book 1 • Pulmonary and Endocrinology 106 Pneumonia in the ICU
Self-Assessment Questions
16. A 65-year-old man is admitted to the ICU with suspected B. Doxycycline 100 mg intravenously every 12 hours
severe community-acquired pneumonia (CAP). The C. Moxifloxacin 400 mg intravenously daily
patient is not yet mechanically ventilated. In addition to D. Colistin 2.5 mg/kg intravenously every 12 hours
blood cultures, which one of the following would best
20. A 63-year-old woman (height 64 inches, weight 65 kg) in
help evaluate this patient?
the general ICU develops VAP on hospital day 7 caused
A. Sputum culture alone unless mechanically ventilated by multidrug-resistant (MDR) Acinetobacter bauman-
B. Endotracheal aspirate (EA) with semiquantitative nii. On the initial automated testing panel, the isolate
culture is sensitive to ampicillin/sulbactam and amikacin and
C. Bronchoscopic bronchoalveolar lavage (BAL) with resistant to meropenem, cefepime, piperacillin/tazo-
quantitative culture bactam, and ciprofloxacin. Pertinent laboratory values
D. Sputum culture, pneumococcal/Legionella urinary include SCr 1.8 mg/dL (was normal on admission). The
antigen test, and nasal MRSA patient’s medical history includes diabetes, COPD, and a
17. A 55-year-old woman in the general ICU on mechanical severe penicillin allergy (e.g., anaphylaxis). Which one of
ventilation day 5 develops new-onset fever, leukocytosis, the following is best to recommend for this patient?
a new infiltrate on chest radiography, and worsening oxy- A. Amikacin
genation status requiring increased ventilator settings. B. Minocycline
The ICU team suspects ventilator-associated pneumo- C. Meropenem/vaborbactam
nia (VAP). On the basis of the IDSA/ATS guidelines for D. Ampicillin/sulbactam
diagnosing VAP, which one of the following is best to rec-
21. A 72-year-old man (height 69 inches, weight 90 kg) in the
ommend for this patient?
surgical ICU develops VAP caused by Enterobacter cloa-
A. EA with semiquantitative culture
cae on ICU day 10. The initial automated sensitivity report
B. Bronchoscopic BAL with quantitative culture
shows the isolate is resistant to cefepime, piperacillin/
C. Bronchoscopic protected specimen brush with
tazobactam, meropenem, ciprofloxacin, and all amino-
quantitative culture
glycosides. Pertinent laboratory values include SCr 2.4
D. Sputum culture with semiquantitative result
mg/dL (normal on admission), and he is receiving con-
18. A 42-year-old woman is admitted to the medical ICU with tinuous renal replacement therapy. While additional test
suspected severe CAP. Her rapid test for influenza is results are pending for the following antibiotic choices,
positive. The patient’s family reports that her symptoms which one is best to recommend for this patient?
began 4 or 5 days ago and have progressively worsened.
A. Ceftolozane/tazobactam
She requires supplemental oxygen but is not mechani-
B. Eravacycline
cally ventilated. The ICU team is debating whether to use
C. Colistin
oseltamivir. Which one of the following is best to recom-
D. Ceftazidime/avibactam
mend as regards oseltamivir use for this patient?
22. Your ICU has a new medical director with whom you are
A. It only benefits patients within 2 days of infection
meeting to review current clinical pathways that you
and should not be used.
have helped develop with the medical team. She has dis-
B. It works best within 2 days of infection but should
covered that the hospital’s laboratory can perform serum
still be used.
procalcitonin (PCT) assays. She asks you to help imple-
C. Its efficacy is not affected by the start date relative
ment the use of PCT concentrations in the CAP pathway.
to symptom onset.
The average antibiotic therapy duration for CAP in your
D. It should combined with hydrocortisone when
unit is 10 days. Which one of the following is best to rec-
initiated after day 2.
ommend regarding the use of PCT in the diagnosis and
19. A 27-year-old man (height 70 inches, weight 80 kg) in the
management of hospital-acquired pneumonia (HAP)/
trauma ICU develops VAP on ICU day 15. His laboratory
VAP in this ICU?
test results show normal renal and hepatic function. The
patient has no contributory medical history except for a A. Serial measurements to help determine the therapy
rash to sulfonamides. A BAL culture shows that his VAP duration
is caused by S. maltophilia. Which one of the following is B. Baseline measurement to help diagnose infection
best to recommend for this patient? C. Measurement at therapy day 7 to determine whether
antibiotics can be discontinued
A. Trimethoprim/sulfamethoxazole 4 mg/kg intravenously
D. Baseline and serial measurements for diagnosis and
every 8 hours (based on trimethoprim)
determining the therapy duration

CCSAP 2021 Book 1 • Pulmonary and Endocrinology 107 Pneumonia in the ICU
23. A 67-year-old man (height 68 inches, weight 100 kg) in the Pertinent laboratory values on admission include SCr 3.5
medical ICU is being treated for CAP caused by MRSA. mg/dL and WBC 15.5 × 103 cells/mm3. A sputum culture
He is hemodynamically stable but requires mechanical has been sent to the microbiology laboratory. Which one
ventilation. On day 5 of linezolid treatment, his fever, leu- of the following is best to recommend as empiric therapy
kocytosis, and oxygenation status have improved from for this patient?
the onset of CAP. Which one of the following is best to
A. Ceftriaxone, azithromycin
recommend as the duration for this patient’s antibiotic
B. Linezolid, ceftriaxone, levofloxacin
therapy?
C. Vancomycin, piperacillin/tazobactam, tobramycin
A. 5 days D. Linezolid, cefepime, ciprofloxacin
B. 7 days
28. A man is admitted to the trauma ICU with pulmonary
C. 10 days
contusions and several orthopedic injuries after a motor
D. 14 days
vehicle crash. On hospital day 3, he develops signs and
24. A patient in the surgical ICU is being treated for VAP symptoms of VAP. The patient is hemodynamically sta-
caused by Pseudomonas aeruginosa. According to a 2003 ble, has a normal SCr, and is oxygenating well on baseline
landmark study comparing 8 days with 15 days of antibi- ventilator settings (SIMV [synchronized intermittent
otic therapy for VAP in adults, which one of the following mandatory ventilation] rate 14 beaths/minute, Fio2 40%,
organisms would most likely require more than 7 days of PEEP 5 mm Hg). Which one of the following is best to rec-
treatment to avoid recurrence of VAP in this patient? ommend as empiric therapy for this patient?
A. MRSA A. Cefepime
B. Klebsiella pneumoniae B. Levofloxacin
C. Enterobacter aerogenes C. Ceftriaxone plus azithromycin
D. P. aeruginosa D. Vancomycin, piperacillin/tazobactam, and
ciprofloxacin
25. At a meeting of your hospital’s pharmacy and therapeutics
committee, the ICU director asks about the advantages 29. A 74-year-old woman in your ICU requires mechanical
and disadvantages of implementing extended infusions ventilation for suspected CAP. Influenza and COVID-19
(EIs) of β-lactams for severe infections in the ICU. Which tests are negative. Her medical history includes con-
one of the following is best to recommend regarding EI gestive heart failure and treatment of ventricular
dosing? arrhythmias. Her care team determines that the patient
has no risk factors for MRSA or Pseudomonas but asks
A. Implement because studies consistently show
your opinion on empiric antibiotic therapy. Which one of
improved outcomes.
the following is best to recommend for this patient?
B. Implement because it is easier to administer.
C. Clinical data are mixed, but some studies show a A. Ceftriaxone plus levofloxacin
benefit. B. Ceftriaxone plus azithromycin
D. Generally avoid because of intravenous line C. Ampicillin/sulbactam plus doxycycline
incompatibility issues. D. Moxifloxacin
26. A 24-year-old woman is in the trauma ICU with a severe 30. A 28-year-old man is admitted to the trauma ICU with
traumatic brain injury. On day 10, she develops sus- a severe traumatic brain injury after a 20-ft fall from a
pected VAP. Pertinent laboratory values include SCr 0.8 ladder. On hospital day 3, the patient develops signs
mg/dL and WBC 17.9 × 103 cells/mm3, but the patient has and symptoms of VAP and an increase in SCr from 0.8
a history of a life-threatening penicillin allergy. Which mg/dL to 1.8 mg/dL. Your ICU antibiogram shows that in
one of the following is best to recommend as empiric patients without MDR risk factors in your unit, Pseudomo-
therapy for this patient? nas causes 1% of VAP cases. Which one of the following
is best to recommend as empiric therapy for this patient?
A. Aztreonam
B. Linezolid, imipenem/cilastatin A. Cefepime
C. Vancomycin, cefepime, gentamicin B. Piperacillin/tazobactam
D. Linezolid, ciprofloxacin, tobramycin C. Ceftriaxone
D. Vancomycin plus meropenem
27. A 47-year-old woman is admitted to the medical ICU with
CAP. About 1 month ago, she was treated as an inpatient
with intravenous antibiotics for a diabetic foot infection.

CCSAP 2021 Book 1 • Pulmonary and Endocrinology 108 Pneumonia in the ICU
Learner Chapter Evaluation: Pneumonia in the ICU

As you take the posttest for this chapter, also evaluate the 25. The learning assessment activities used in the chapter
material’s quality and usefulness, as well as the achievement were effective.
of learning objectives. Rate each item using this 5-point scale: 26. The chapter was effective overall.

• Strongly agree 27. The activity met the stated learning objectives.
• Agree 28. If any objectives were not met, please list them here.
• Neutral
• Disagree OTHER COMMENTS
• Strongly disagree
29. Please provide any specific comments related to any
16. The content of the chapter met my educational needs. perceptions of bias, promotion, or advertisement of
17. The content of the chapter satisfied my expectations. commercial products.
18. The author presented the chapter content effectively. 30. Please expand on any of your above responses, and/or
provide any additional comments regarding this chapter:
19. The content of the chapter was relevant to my practice
and presented at the appropriate depth and scope.
20. The content of the chapter was objective and balanced. Questions 31–33 apply to the entire learning module.

21. The content of the chapter is free of bias, promotion, and 31. How long did it take you to read the instructional materi-
advertisement of commercial products. als in this module?
22. The content of the chapter was useful to me. 32. How long did it take you to read and answer the assess-
23. The teaching and learning methods used in the chapter ment questions in this module?
were effective. 33. Please provide any additional comments you may have
24. The active learning methods used in the chapter were regarding this module:
effective.

CCSAP 2021 Book 1 • Pulmonary and Endocrinology 109 Pneumonia in the ICU
Pulmonary and Endocrinology III
Pulmonary and Endocrinology III Panel

Series Editors: Immune-Mediated Reactions


Bradley A. Boucher, Pharm.D., FCCP, FNAP, MCCM, BCPS
Authors
Professor of Clinical Pharmacy and Translational Science
Abby M. Bailey, Pharm.D., BCCCP
Associate Dean for Strategic Initiatives and Operations
College of Pharmacy Coordinator, Emergency Medicine
University of Tennessee Health Science Center Department of Pharmacy
Memphis, Tennessee University of Kentucky HealthCare
Lexington, Kentucky
Curtis E. Haas, Pharm.D., FCCP
Jaclyn M. Stoffel, Pharm.D., BCPS
Chief Pharmacy Officer
University of Rochester Medical Center Clinical Pharmacist—Emergency Medicine
Rochester, New York Department of Pharmacy
Baylor University Medical Center
Faculty Panel Chair: Dallas, Texas
Alexander H. Flannery, Pharm.D., FCCM, BCCCP, BCPS
Reviewers
Assistant Professor
University of Kentucky College of Pharmacy Kyle A. Weant, Pharm.D., FCCP, BCPS, BCCCP
Lexington, Kentucky Clinical Assistant Professor
Department of Clinical Pharmacy and Outcome Sciences
University of South Carolina College of Pharmacy
Endocrinology in the ICU Columbia, South Carolina
Author Heather Johnson, Pharm.D., MS, BCCP, BCCCP
Brittany D. Bissell, Pharm.D., Ph.D., BCCCP Clinical Pharmacy Manager
Methodist Hospital and Methodist Children’s Hospital
Assistant Professor
San Antonio, Texas
Division of Pulmonary, Critical Care, and Sleep
College of Medicine
Department of Pharmacy Practice and Science The American College of Clinical Pharmacy and the authors
College of Pharmacy thank the following individuals for their careful review of the
University of Kentucky
Pulmonary and Endocrinology III chapters:
Lexington, Kentucky
Joseph S. Bubalo, Pharm.D., BCPS, BCOP
Reviewers Oncology Clinical Pharmacy Specialist
Leslie A. Hamilton, Pharm.D., FCCP, FCCM, BCPS, BCCCP Assistant Professor of Medicine
Associate Professor Oregon Health and Science University Hospital and Clinics
Department of Clinical Pharmacy Portland, Oregon
and Translational Science Michael C. Thomas, Pharm.D., FCCP, BCPS
University of Tennessee Health Science Professor, Pharmacy Practice 
Center College of Pharmacy McWhorter School of Pharmacy
Knoxville, Tennessee Samford University
Michael A. Rudoni, Pharm.D., BCPS, BCCCP Birmingham, Alabama
Critical Care Specialist—Medical ICU
Department of Pharmacy
Cleveland Clinic
Cleveland, Ohio
DISCLOSURE OF POTENTIAL CONFLICTS OF INTEREST
Consultancies: Stephanie Bass (Entasis); Mitchell S. Buckley (Wolters Kluwer); Chris Droege (Deloitte Consulting LLP); Eric Johnson
(Slayback Pharmaceuticals); Patrick M. Wieruszewski (La Jolla Pharmaceutical Company); G. Christopher Wood (ASHP)

Stock Ownership:

Royalties:

Grants: Brittany Bissell (ASHP, University of Kentucky); Chris Droege (ACCP); Alexander H. Flannery (ACCP, American Society
of Nephrology, La Jolla Pharmaceutical Company); Anthony Hawkins (University of Georgia College of Pharmacy); Heather P.
May (ACCP, Mayo Clinic); Andrea Sikora Newsome (NIH); Joseph M. Swanson (Cubist/Merck)

Honoraria: Abby Bailey (ACCP EM PRN); Joseph M. Swanson (United Arab Emirates – Ministry of Health)

Other:

Nothing to disclose: Mahmoud A. Ammar, Paige Garber Bradshaw, Jennifer Falvey, Maria Guido, Leslie A. Hamilton, Sara Jordan
Hyland, Heather A. Johnson, Debbie Liang, Brian P. Murray, Andrea M. Nei, Jaimini S. Patel, Michael A. Peters, Michael A. Rudoni,
Poorvi Shah, Zachary R. Smith, Jaclyn Stoffel, Kyle A. Weant, Maggie Zhao

ROLE OF BPS: The Board of Pharmacy Specialties (BPS) is an autonomous division of the American Pharmacists Association
(APhA). To maintain its strict, independent standards for certification, BPS does NOT endorse or provide review information,
preparatory courses, or study guides for Board Certification Examinations. The Board, through its specialty councils, is respon-
sible for specialty examination content, administration, scoring, and all other aspects of its certification programs. BPS is totally
separate and distinct from ACCP. CCSAP has been approved by BPS for use in BCCCP recertification. Information about the BPS
recertification process is available online.

Questions regarding BCCCP recertification should be directed to:

Board of Pharmacy Specialties


2215 Constitution Avenue NW
Washington, DC 20037
(202) 429-7591
CONTINUING PHARMACY EDUCATION
AND RECERTIFICATION INSTRUCTIONS
Continuing Pharmacy Education Credit: The American College of Clinical Pharmacy is accredited by the Accreditation
Council for Pharmacy Education (ACPE) as a provider of continuing pharmacy education (CPE).

CCSAP Target Audience: The target audience for Pulmonary and Endocrinology is not only ICU and ED pharmacists across the
spectrum of care but also any pharmacist caring for acutely ill patients whose management may be complicated by these chal-
lenging scenarios.

Available CPE credits: Purchasers who successfully complete all posttests for CCSAP 2021 Book 1 (Pulmonary and Endocrinology)
can earn 22.5 contact hours of continuing pharmacy education credit. The universal activity numbers are as follows: Pulmonary
and Endocrinology I, 0217-0000-21-030-H01-P, 5.5 contact hours; Pulmonary and Endocrinology II, 0217-0000-21-031-H01-P,
5.0 contact hours; Pulmonary and Endocrinology III, 0217-0000-21-032-H01-P, 5.5 contact hours; and Pulmonary and Endocrinology IV,
0217-0000-21-033-H01-P, 6.5 contact hours. You may complete one or all available modules for credit. Tests may not be submitted
more than one time.

TO EARN CPE CREDITS FROM THIS CCSAP BOOK

Posttest access: Go to www.accp.com and sign in with your e-mail address and password. Technical support is available from 8 a.m.
to 5 p.m. (Central) weekdays by calling (913) 492-3311. CCSAP products are listed under My Products on your My Account page.

BCCCP test deadline: 11:59 p.m. (Central) on September 15, 2021.


ACPE test deadline: 11:59 p.m. (Central) on March 15, 2024.

BCCCP Recertification Credit: To receive BCCCP recertification CPE credit, a CCSAP posttest must be submitted
within the 6-month period after the book’s release (see above). Only completed tests are eligible for credit; no partial
or incomplete tests will be processed. You may complete one or all available modules for credit. Tests may not be
submitted more than one time.

The passing point to earn BCCCP recertification credit is based on an expert analysis of the assessment items in each posttest
module. Any posttest submitted before the BCCCP test deadline that meets this passing point will earn BCCCP recertification
credits. These credits will be assigned as of the date of test submission and reported within 48 hours to BPS. For statements of
recertification credit, visit www.bpsweb.org.

Questions regarding the number of hours required for BCCCP recertification should be directed to BPS at (202) 429-7591 or
www.bpsweb.org. The ACCP Recertification Dashboard is a free online tool that can track recertification credits as they are
earned through ACCP and schedule new opportunities for credits from upcoming ACCP professional development programs.

ACPE CPE Credit: To receive ACPE CPE credit for a CCSAP module, a posttest must be submitted within 3 years after the book’s
release (see above). Only completed tests are eligible for credit; no partial or incomplete tests will be processed. You may complete
one or all available modules for credit. Tests may not be submitted more than one time.

Any posttest submitted before the ACPE deadline that scores 50% or greater will be awarded the appropriate CPE. These
credits will be assigned as of the date of test submission and reported within 48 hours. For statements of CPE credit, visit
www.mycpemonitor.net.

Posttest answers: The explained answers—with rationale and supporting references—will be posted 2 weeks after the BCCCP
test deadline and will be available to anyone who has either (1) submitted a posttest or (2) waived the right to receive credit from
a posttest (see below). Go to www.accp.com and sign in with your e-mail address and password. Click the CCSAP book on your
My Account page and you will see a link to the explained answers.

Test Waivers: To access the explained answers without submitting a posttest, sign in to your My Account page, select the
CCSAP book, and click on the waiver link for that module. By completing the waiver form for a module, you waive the opportunity
to receive CPE credit for that module. After you submit a waiver, you will see a link to the PDF file that contains the answers for
the module you waived. Answers will be available starting 2 weeks after the BCCCP test deadline.
Critical Care Endocrinology
By Brittany D. Bissell, Pharm.D., Ph.D., BCCCP

Reviewed by Leslie A. Hamilton, Pharm.D., FCCP, FCCM, BCPS, BCCCP; and Michael A. Rudoni, Pharm.D., BCPS, BCCCP

LEARNING OBJECTIVES

1. Assess patients for critical illness–related changes in normal endocrine physiology.


2. Justify recommendations to implement corticosteroid therapy.
3. Distinguish between anti-diuretic hormone issues occurring in the ICU.
4. Develop a monitoring plan for thyroidal supplementation therapy in the setting of thyroid insufficiency.
5. Evaluate current evidence for glycemic control recommendations in the general ICU population.
6. Distinguish between acute adrenal abnormalities and chronic critical illness related disorders.

ENDOCRINE PHYSIOLOGY AND CRITICAL


ABBREVIATIONS IN THIS CHAPTER
ILLNESS PATHOPHYSIOLOGY
ACTH Adrenocorticotropic hormone
Introduction to the Endocrine System in Critical
ADH Anti-diuretic hormone
Illness
AI Adrenal insufficiency
CIRCI Critical illness–related corticoste- The endocrine system is most simply described as the chemical
roid insufficiency messenger system within the human body, responsible for hormone
CRH Corticotropin-releasing hormone production regulating a vast array of functions, such as metabolism.
DI Diabetes insipidus With critical illness, many hormonal changes arise secondary to a mul-
HPA Hypothalamic–pituitary–adrenal titude of stressors. These responses occur in context of the greater
HPT Hypothalamic–pituitary–thyroid syndrome of general adaptation, better known as the stress response
rT3 Reverse triiodothyronine (Selye 1998). The stress response seeks to restore bodily homeostasis
SIADH Syndrome of inappropriate antidi- through alterations in the HPA axis and the sympathoadrenal system,
uretic hormone secretion with several downstream effects (Marik 2007). Typically, the stress
T3 Triiodothyronine response is thought to occur in three phases, all of which have effects
T4 Thyroxine within the endocrine system. The first is the acute phase of critical ill-
TRH Thyrotropin-releasing hormone ness, in which organ damage is imminent and therapeutic measures
TSH Thyroid-stimulating hormone may often prevent further organ damage. Changes in the thymus, GI
system, and adrenal glands are seen in this phase (Selye 1998). The
Table of other common abbreviations. second phase ensues once organ failure occurs. Current therapies in
this phase typically seek to support normal physiologic adaption mea-
sures and prevent iatrogenic harm. At this stage, thyroidal changes
may occur, as well as changes in the function of reproductive organs
and/or function (Selye 1998). The third and final phase of the stress
response is the repair and recovery phase, ideally in which endocrine
function returns to normal and the acuity resolves. However, some
authors classify the third phase as the exhaustive phase, in which
return to normal function does not occur despite continued stress and
need for stress response. (Cuesta 2012). What challenges both prac-
titioners and researchers in intensive care is the degree to which the
stress response is supportive or antagonistic to overall recovery. For
example, catecholamine surges are commonly thought of as benefi-
cial and necessary for the restoration of hemodynamic compromise

CCSAP 2021 Book 1 • Pulmonary and Endocrinology 117 Critical Care Endocrinology
in shock; however, sympathomimetics are known to cause neuropeptides responsible for neuronal synaptic transmis-
physiologic harm to many organ systems (Ferreira 2018). The sion, and catecholamines from the adrenal medulla. (Eiden
same principle holds true for endocrine adaptation in critical 2018). Cytokine action increases the production of ACTH.
illness—although some physiologic responses may be consid- Adrenal cortex responses are mediated through activation of
ered normal, delineation of the point at which these responses the HPA axis (Figure 1). With stress, release of arginine vaso-
become maladaptive is an important consideration for thera- pressin and CRH is triggered by the paraventricular nucleus,
peutic management. This chapter focuses on thyrotropic and which resides within the hypothalamus. Its parvocellular neu-
adrenotropic changes as a consequence of critical illness. The rosecretory cells are primarily responsible for all hormonal
primary causes for critical illness are also discussed. functions, with paraventricular nucleus neurons projecting
directly onto the posterior and anterior pituitary. Vasopressin
HPA Axis in Critical Illness is a vasoactive peptide, resulting in vasoconstriction through
HPA Responses During Acute Critical Illness its actions on the vascular smooth muscle cells mediated by
During the acute phase of critical illness, activation of the the vasopressin V1a receptor (Demiselle 2020). Regarding
sympathoadrenal system directly stimulates the release of ACTH production, vasopressin is a secretagogue of corticotro-
cytokines, chemokines responsible for immune cell migration, pin with relatively weak action alone, but it has a synergistic

BASELINE KNOWLEDGE STATEMENTS

Readers of this chapter are presumed to be familiar


with the following:
• Historical knowledge of corticosteroid recommen-
dations in septic shock
• Understanding of the pathophysiology and diagnostic
criteria of diabetic ketoacidosis and hyperglycemic
hyperosmolar state
• Drug knowledge of pharmacologic agents used for
thyroidal and adrenal crises
• Pharmacotherapeutic background on insulin therapy
in the management of diabetes mellitus
• Consequences of poor glycemic control on critical
care outcomes
• Normal adrenal physiology and function
Table of common laboratory reference values.

ADDITIONAL READINGS

The following free resources have additional back-


ground information on this topic:
• Rhodes A, Evans LE, Alhazzani W, et al. Surviving
Sepsis Campaign: international guidelines for
management of sepsis and septic shock: 2016.
Intensive Care Med 2017;4:304-77.
• Jonklaas J, Bianco AC, Bauer AJ, et al. Guidelines
for the treatment of hypothyroidism. Thyroid 2014;
24:16701-1751.
• Jacobi J, Bircher N, Krinsley J, et al. Guidelines for
the use of an insulin infusion for the management
Figure 1. The HPA Axis. Types of adrenal insuffi-
of hyperglycemia in critically ill patients. Crit Care
ciency and primary location of pathophysiology in
Med 2012;40:3251-76.
diamonds.
• Kitabchi AE, Umpierrez GE, Miles JM, et al. 1= primary AI; 2= secondary AI; 3=tertiary AI; ACTH=
Hyperglycemic crises in adult patients with
diabetes. Diabetes Care 2009;32:1335-43. Adrenocorticotropic hormone; CRH = Corticotropin-
releasing hormone

CCSAP 2021 Book 1 • Pulmonary and Endocrinology 118 Critical Care Endocrinology
effect in combination with CRH. Production of ACTH by the cortisol concentration greater than 34 mcg/dL at baseline or
anterior pituitary is further stimulated by CRH, thereby result- a change in cortisol after a stimulation test of 9 mcg/dL or
ing in cortisol production by ACTH action on the zona less were independent predictors of mortality (Annane 2000).
fasciculata portion of the adrenal cortex. Notably, cortisol is However, pooled data demonstrate a broad range of mortality
not stored within the adrenals—instead, an increase in corti- rates across baseline cortisol concentrations of 21.75–47.125
sol synthesis is seen secondary to ACTH control (Marik 2017). mcg/dL (Sakharova 2007). Furthermore, cortisol assays vary
Cortisol thereafter exerts various responses across the body. in sensitivity, specificity, accuracy, and precision, yielding dis-
Cortisol, a glucocorticoid, is thought to increase in criti- similar results with different assays and preventing ubiquitous
cal illness because of its physiologic effects. The effects of reference ranges across institutions. In addition, whether the
co rtisol are mediated by intracellular glucocorticoid recep- diurnal variation in cortisol typically seen in healthy individu-
tors within the cytoplasm. Cortisol increases blood pressure als has an effect in the ICU is also unknown. The half-life of
and cardiac output by several mechanisms. Glucocorticoids cortisol is 90 minutes; unsurprisingly, hourly variations have
enhance catecholamine action in vascular and cardiac tis- been demonstrated (Dalegrave 2012).
sue and prolong the action by inhibition of catecholamine Given the limitations of plasma cortisol, several alternatives
uptake. Cortisol can also increase sensitivity to catechol- have been considered. The cortisol stimulation test—admin-
amines by increasing the affinity of β-adrenergic receptors istration of 250 mcg of exogenous cosyntropin—evaluates
in arterial smooth muscle and increasing their binding adrenal cortical reserve by evaluation of cortisol concentra-
capacity (Sapolsky 2000). In addition to adrenergic activ- tions at baseline and 30–60 minutes after dose administration.
ity, cortisol upregulates angiotensin II receptors, almost However, a normal response to this test does not rule out adre-
exclusively angiotensin type 1 (Ullian 1999). Cortisol is also nal suppression, given the supraphysiologic dose of ACTH
thought to inhibit prostaglandin synthesis, preventing vaso- administered (Rai 2004). With this lack of clinical relevance,
dilation by this mechanism. Regarding immune function, guidelines for the management of sepsis no longer recom-
glucocorticoids are believed to prevent a hyperinflammatory mend use of the cortisol stimulation test (Rhodes 2017). Given
state and overly exaggerated immune response, particularly that cortisol-binding protein fluctuates in critical illness, free
when induced during the stress response. Synergistically plasma cortisol has received the most attention in the past 5
with catecholamines and glucagon, glucocorticoids assist years. Although some data have shown a correlation between
with mediation of the metabolic actions of the stress measured free and total cortisol concentrations, no impact on
response, stimulating lipolysis and proteolysis, increasing clinical outcomes has been evaluated (Dichtel 2019; Molenaar
free fatty acids, and increasing glycogenolysis and gluco- 2015). A study by Molenaar et al. that calculated free cortisol
neogenesis. Corticosteroids prevent the use of glucose and concentrations was limited by bias and imprecision, with the
inhibit peripheral transport, resulting in an overall increase authors concluding that such calculations remain inappro-
in blood glucose (Sapolsky 2000). priate in the ICU (Molenaar 2015). Other tests, including the
urinary cortisol, plasma ACTH , insulin tolerance, metyrapone,
Diagnostic Considerations and CRH stimulation tests, all continue to be uncommon in
Typically the degree to which the HPA axis is activated directly practice either because of their technical difficulty or poten-
correlates to illness severity; however, several factors may tial adverse effects, such as those secondary to induction of
modify or dampen the HPA axis response (as discussed in hypoglycemia with the insulin tolerance test (Rai 2004).
the Adrenal Disorders chapter). The lack of a defined mecha-
nism causing an inadequate HPA response creates ambiguity
HPT Axis in Critical Illness
for distinguishing patients most at risk of adequate versus
inadequate HPA response. Unfortunately, identifying objec- HPT Responses During Acute Critical Illness
tive indicators of HPA function proves just as difficult. First, Less understood and much less studied is the impact of crit-
there is no validated cortisol value to define a “normal” stress ical illness on the HPT axis and overall acute response. The
response. Reported plasma cortisol concentrations vary sequence of physiologic changes is less well described, with
greatly, with studies demonstrating a range of 14.50–50.75 current timelines relying on chronologic changes in plasma
mcg/dL in critically ill patients throughout the day, in contrast hormone concentrations. What appears to occur are changes
to normal human cortisol concentrations peaking in the morn- to both thyroid hormones as well as deiodinases, the enzymes
ing, typically between 4–19 mcg/dL (Dorin 2015; Rai 2004). that remove an iodine from the thyroidal hormones. Under nor-
Thresholds suggested for insufficient adrenal response in crit- mal physiologic circumstances, hypothalamic release of TRH
ical illness range between 10–34 mcg/dL (Sakharova 2007). from its paraventricular nucleus stimulates the synthesis of
Further complicating the use of cortisol concentration to TSH and release from the anterior pituitary. During critical ill-
define a normal stress response is a lack of correlation between ness, the response of TSH to TRH release is blunted (Nylen
these values and clinical outcomes. A benchmark study seek- 2004). Furthermore, TRH secretion is decreased secondary
ing to provide cortisol-based prognostication found that a to several factors as follows: enhanced type 2 deiodinase, the

CCSAP 2021 Book 1 • Pulmonary and Endocrinology 119 Critical Care Endocrinology
Diagnostic Considerations

D1 D1 Similar to the difficulties for interpretation of adrenal alter-


D2 T3
D3 ations, the clinical implications and diagnostic relevance
of thyroidal changes remain unknown. Thyroxine-binding
T4 T2 globulin concentrations decrease and its binding affinity is
reduced, so free hormone concentrations should be used
D3 rT3 D1 instead when possible for diagnosis (Khardori 2012). Free
D1 D2 T4 is considered to be the most trusted measurement of thy-
roid function; however, heparin is known to increase free T4
concentrations, secondary to displacement from plasma pro-
Figure 2. Thyroid hormones and alterations in
teins. Furthermore, free T4 concentrations in context of high
critical illness. Enzymes and hormones are shown in
circles and boxes, respectively: those that increase protein concentrations may underestimate free T4, thereby
in acute illness are bolded and underlined; those that complicating interpretation (Sakharova 2007). A third-gen-
decrease are italicized. eration assay should be used to evaluate TSH, although
D1–3 = type 1–3 deiodinase; rT3 = reverse triiodothyronine; concentrations below the limit of detection by this assay (less
T2 = 3, 5-diiodothyronine; T3 = triiodothyronine; T4 = thyroxine. than 0.01 mU/L) are common in patients with primary hyper-
thyroidism. Patients with overt primary hypothyroidism have
TSH concentrations greater than 10 mU/L, but concentrations
predominate activating enzyme; activity at the level of the may be low in the setting of secondary hypothyroidism (Nylen
hypothalamus, resulting in increased local T3; and inaccurate 2004). Treatment decisions must consider these factors as
negative feedback (Khardori 2012). Plasma concentrations of well as the timing of critical illness and patient-specific con-
both free and total T3 decrease while concentrations of rT3, siderations, as discussed later in this chapter.
a hormone with no biologic activity, increase secondary to
increased conversion and decreased degradation (Boonen
ADRENAL DISORDERS
2014). Although rT3 appears to have no effects on the body,
an increase of this hormone in critically ill patients does Adrenal Excess
appear to be associated with increased mortality (Peeters Hyperfunction of the adrenal gland, commonly known as
2005). It is suspected that this change occurs secondary to Cushing syndrome, can be either ACTH-dependent or ACTH-
a decrease in the activity of type 1 deiodinase, responsible independent (Dang 2014). Cushing disease, distinct from the
for peripheral T4 conversion to T3, with an increase in type 3 syndrome, is specifically the result of an ACTH-secreting pitu-
deiodinase activity, which is the major inactivating thyroidal itary tumor and is the most common cause of adrenal excess.
enzyme (Figure 2). Although a decrease in T4 concentration Etiologies that are ACTH independent include drug-induced
is not seen as quickly as that for T3, the T4 concentrations Cushing syndrome secondary to use of glucocorticoids,
progressively decline with increasing severity of illness medroxyprogesterone, and other drugs. Although adrenal
(Sakharova 2007). Changes to TSH may or may not occur. In excess rarely presents as an emergency, life-threatening
primary thyroid disorders, TSH can be considered a sensitive complications may occur, including infection, pulmonary
marker for diagnosis; however, in nonthyroidal illnesses TSH thromboembolism, cardiovascular issues (e.g., acute heart
may remain in the normal range, despite severe alterations in failure), respiratory failure, pancreatitis, bowel perforation,
T3 and possibly T4 (Nylen 2004). and acute psychosis (Nieman 2015).
The specific culprit for the thyroidal changes seen in crit- Surgical intervention is the recommended first-line
ical illness is less known. Cytokines are known to suppress management of Cushing syndrome. For life-threatening
several aspects of the HPT axis, whereas several medications hypercortisolism, treatment is recommended within 24–72
used in critical illness, such as phenytoin and furosemide, hours of onset, but immediate therapy may be warranted
inhibit T4 binding, and other medications increase clearance. before confirmative diagnosis. Etomidate is the only guide-
Corticosteroids can decrease secretion of TSH, in addition line-recommended treatment for critically ill patients who
to the effect of acute malnutrition and dopamine on limiting are not surgical candidates and cannot take oral medica-
TSH (Khardori 2012). Few human studies have investigated tions. By its inhibition of the enzyme 11β-hydroxylase, which
the effect of corticosteroid dose on thyroid hormones, but catalyzes the production of cortisol from 11-deoxycortisol,
one study of dexamethasone 2 mg every 6 hours showed etomidate can lower serum cortisol concentrations within
altered thyroid function within 8 hours after the first dose 12   hours. An intravenous loading dose of 2–5 mg should be
(Chopra 1975). Corticosteroids, amiodarone, iodinated con- followed by a continuous infusion of 0.03–0.05 mg/kg/hour or
trast, and other medications are also known to decrease type 2.5–3 mg/hour, although doses up to 30 mg/hour have been
1 deiodinase activity. Carbohydrate restriction prevents T4 to reported (Preda 2012). Typical doses required to achieve the
T3 conversion. goals are not sedative and are not likely to result in respiratory

CCSAP 2021 Book 1 • Pulmonary and Endocrinology 120 Critical Care Endocrinology
depression; however, close monitoring is required. Given that
Box 1. Causes of Absolute Adrenal
etomidate preparations contain propylene glycol, toxicity mon-
Insufficiency
itoring is required. Combination therapy with mitotane (3–5 g/
day), metyrapone (3–4.5 g/day), and ketoconazole (400–1200 Primary
mg/day) orally is an effective alternative to bilateral adrenal- • Autoimmune
ectomy in patients with ACTH-dependent Cushing syndrome
• Infectious: cytomegalovirus, human immunodeficiency
virus, Pneumocystis jiroveci, tuberculosis, among others
(Kamenický 2011). Mitotane has a direct cytotoxic action on • Adrenalectomy
the adrenal cortex but has a prolonged half-life with measur- • Bilateral adrenal hemorrhage and/or infarction
able plasma concentrations for months after a dose. Adverse • Genetic causes
effects include teratogenicity, GI effects, gynecomastia, low • Malignancy
white blood cell count, low T4 concentration, and increased
• Drugs: ketoconazole, etomidate, rifampin, tyrosine kinase
inhibitors, phenytoin, phenobarbital
liver enzymes and cortisol-binding globulin. Metyrapone
Secondary
inhibits 11-β hydroxylase, thereby preventing cortisol pro-
duction. Adverse effects include GI symptoms, hypertension,
• Treatment for Cushing syndrome
• Panhypopituitarism
hirsutism, and hypokalemia. Ketoconazole decreases adrenal • Pituitary tumors
and gonadal steroidogenesis by inhibition of adverse chain • Transsphenoidal pituitary surgery
cleavage and enzymatic activity. Adverse effects for this med- • Traumatic brain injury
ication are also GI symptoms, as well as possible hepatic • Drugs: progestins or glucocorticoids
dyscrasia and hypogonadism. Given the number of potential Tertiary
drug interactions present with mitotane and ketoconazole, • Hypothalamic dysfunction
pharmacists must remain aware of interacting medications
(Nieman 2015). Although these agents can be used inde-
pendently, combination is most likely required for the setting lower back pain. Patients with primary AI typically present with
of severe hypercortisolism requiring ICU admission. Additional pyrexia, hypotension, abdominal tenderness, and impaired
care should be patient-specific based on presenting complica- consciousness, as well as hyperpigmentation. Laboratory
tions. In patients with a urinary free cortisol 5-times normal abnormalities coincident with diagnosis of AI include hypo-
(30–145 nmol/L over 24 hours), prophylaxis for Pneumocystis natremia, hypoglycemia, hypercalcemia, mild normocytic
jiroveci pneumonia is reasonable. In patients receiving therapy anemia, and altered immune cell populations, such as lympho-
directed at cortisol reduction, therapy should be titrated to a cytosis, eosinophilia, or neutropenia. Hyperkalemia can also be
cortisol concentration of 10–20 mcg/dL, with serum cortisol seen in adrenal crisis secondary to primary AI.
monitoring every 4–6 hours (Nieman 2015). Typically the diagnosis of AI is made by administration of
cosyntropin to evaluate response. Suggested alternatives
Absolute Adrenal Insufficiency include the low-dose ACTH, insulin-induced hypoglycemia,
Definitions and Etiologies metyrapone, and CRH stimulation tests; however, as previ-
Absolute AI can be classified as primary, secondary, or tertiary ously noted, these tests remain unreliable within the critically
(Box 1). Primary AI, also termed Addison disease, accounts for ill population, further complicating diagnosis. Therefore,
70% of endogenous causes of AI. Tertiary AI is rare. adrenal crisis should be considered in the setting of absolute
Adrenal crisis, otherwise known as acute adrenal insuffi- hypotension (systolic blood pressure less than 100 mm Hg) or
ciency or an Addisonian crisis, is a life-threatening complication relative hypotension (systolic blood pressure reduction of 20
of AI. Adrenal crisis may be the first presentation of undi- mm Hg or more compared with the usual value). Resolution of
agnosed AI and can also be caused by nonadherence to hypotension within 1–2 hours after parenteral glucocorticoid
glucocorticoid therapy in patients with known AI. Adrenal crisis administration (as descried in following text) remains confir-
may be precipitated by stressors such as infections, surgery, matory in patients who do not experience a response to fluid
and chemotherapy. Despite its clinical relevance, no uniform and vasopressor therapy alone (Rushworth 2019).
criteria exist for diagnosis of adrenal crisis (Rushworth 2019).
This crisis occurs in patients with a primary or secondary AI, Pathophysiology
with a slightly higher incidence in those with primary AI pos- Primary AI evolves from an issue with the adrenal cortex,
sibly because of their complete lack of cortisol production/ resulting in dampened production of cortisol (Dang 2014).
secretion relative to those with alternative AI etiologies. Secondary AI is a pituitary issue, resulting in a decrease in
Adrenal insufficiency is a difficult diagnosis within the criti- ACTH. Tertiary is AI secondary to a hypothalamic issue,
cally ill population given its relatively nonspecific and insidious with a decrease in CRH as well as ACTH. Because primary
symptoms (Tompkins 2020). Symptoms include profound AI involves the adrenal cortex, a decrease in mineralocorti-
fatigue and weakness; dizziness and syncope; confusion; coid activity (aldosterone) may be concurrent. Once cortisol
abdominal pain; nausea and vomiting; anorexia; and limb or deprivation occurs, normal suppression of cytokines is lost,

CCSAP 2021 Book 1 • Pulmonary and Endocrinology 121 Critical Care Endocrinology
resulting in an abrupt cytokine surge and causing several of
Box 2. Signs of Critical Illness–Related
the symptoms just described. Furthermore, the loss of syn-
Corticosteroid Insufficiency
ergism with catecholamines results in an overall decrease in
Confusion and/or delirium
vascular reactivity, resulting in hypotension. Hyponatremia
Eosinophilia
can occur either secondary to aldosterone insufficiency in Hemorrhage and/or necrosis of the hypothalamus, pituitary,
primary AI or uninhibited vasopressin renal action in second- or adrenal gland on imaging
ary AI. Hypercalcemia is rare but can occur from decreased High cardiac index
excretion and increased bone resorption, whereas gluconeo- Hypotension refractory to fluids and catecholamines
Hypoxia
genesis and hypoglycemia occur secondary to intermediary
Intolerance to enteral nutrition with nausea/vomiting
metabolism alterations (Dang 2014).
Metabolic disturbances: hypoglycemia, hyponatremia, hyper-
kalemia, metabolic acidosis
Treatment
Treatment of adrenal crisis remains relatively straightforward
and unchallenged in recent years. A bolus of 100 mg hydrocor-
axis could be present at any axis level, including the hypo-
tisone should be administered intravenously followed by 200
thalamus. Regardless, whether CIRCI remains a pathologic
mg every 24 hours either by continuous infusion or divided as
response or a beneficial adaption to critical illness has not
50 mg every 6 hours. After resolution of shock, the dose can
been elucidated (Box 2). Furthermore, differential diagnosis
be reduced with a taper to oral hydrocortisone (15–25 mg/
from patients without CIRCI remains difficult given the lack of
day in split doses) dosing within 3 days (Husebye 2014). Once
specific diagnostic criteria (Annane 2017).
oral therapy is started and hydrocortisone doses decrease to
less than 50 mg, fludrocortisone should be added at a dose Pathophysiology
of 0.05–0.1 mg/day taken once in the morning, particularly in Critical illness–related corticosteroid insufficiency is char-
those patients with primary AI. Guidelines recommend 1000 acterized by an insufficient availability and use of systemic
mL of 0.9% sodium chloride within the first hour of treatment cortisol in the setting of critical illness. Recent evidence
in patients presenting in AI; however minimal data support showed that the true elicitation of the HPA cascade in the
this recommendation. Furthermore, caution should be used context of ICU stress may be less clear than once thought.
in patents with comorbid conditions as well as the poten- Considering the many types and duration of organ fail-
tial for concomitant DI (Husebye 2014). Fluid administration ure that can occur during critical illness, ACTH production
greater than 1 L should be on the basis of hemodynamic mon- is hypothesized to be increased in such patients. However,
itoring parameters and/or serum electrolytes. a recent study reported that systemic cortisol during criti-
During treatment, cortisol replacement can induce water cal illness likely is irrelevant to ACTH secretion (Song 2019).
diuresis; in patients with secondary AI, ADH can be sup- In some patients, ACTH is only transiently elevated despite
pressed. Serum electrolytes, specifically sodium, should be presence of continued elevated cortisol without an acceler-
monitored often, with the recommendation to ensure that the ation in cortisol production, showing both a discordance in
sodium increase is not greater than 9 mEq/L over the first 24 both positive and negative feedback. As mentioned previ-
hours (Puar 2016). Frequent hemodynamic and urine output ously in text, cortisol binding is altered as cortisol binding
monitoring is also recommended while the patient is in the ICU. globulin is decreased and binding is altered secondary to
cleavage induced by neutrophil elastases, whereas metabo-
Relative Adrenal Insufficiency and Critical lism by 5α- reductase, 5β- reductase, and 11β-hydroxysteroid
Illness–Related Corticosteroid Insufficiency dehydrogenase type 2 and plasma clearance of cortisol
Separate from absolute adrenal insufficiency in the ICU is are decreased overall. In addition, a downregulation of glu-
the concept of relative adrenal insufficiency, termed critical cocorticoid receptor-α and upregulation of glucocorticoid
illness–related corticosteroid insufficiency. Initially the con- receptor-β has been demonstrated in some studies, resulting
cept was introduced to present a phenomenon in which, in what is termed glucocorticoid resistance, a process that has
despite activation of the adrenal cortex, cortisol production actually been shown to be worsened by the administration of
was insufficient to effectively mitigate the acuity of the ill- corticosteroids during illness. If present, an alteration in cor-
ness. Differing from absolute AI of any etiology, relative AI tisol binding despite elevated free concentrations of cortisol
or CIRCI can occur even in context of normal or supranormal then classifies the insufficiency of the HPA axis all the way
serum cortisol concentrations (Téblick 2019). Although most to the tissue level as well. Cortisol-binding globulin can be
commonly associated with septic shock, CIRCI can present measured by serum assays; however, the guidelines do not
in all critically ill patients, including those with acute respi- recommend for or against its use. Although it is important to
ratory distress syndrome and trauma (Annane 2017). The understand the underlying mechanisms of CIRCI, the trans-
term relative adrenal insufficiency was replaced with CIRCI in lation to a diagnosis and subsequent treatment approach is
2008 after it was hypothesized that an impairment of the HPA nonspecific, given the current diagnostic limitations.

CCSAP 2021 Book 1 • Pulmonary and Endocrinology 122 Critical Care Endocrinology
Guideline Update (3) Suggest against salivary cortisol over serum cortisol
In 2017, a Society of Critical Care Medicine and the European (4) Suggest high-dose (250 mcg) rather than the low-dose
Society of Intensive Care Medicine task force developed (1 mcg) ACTH stimulation test
guidelines surrounding the concept of CIRCI (Annane 2017). (5) Suggest 250-µg ACTH stimulation test rather than the
The key factors resulting in CIRCI defined by this group were: hemodynamic response to hydrocortisone
dysregulation of HPA function, altered cortisol metabo- (6) Suggest against corticotropin levels
lism, and tissue resistance to glucocorticoids. Specifically,
the authors addressed the following recommendations and Treatment
suggestions regarding the diagnosis of CIRCI: Several randomized controlled trials have sought to answer
(1) No recommendation for the use of delta cortisol after the the question of whether a benefit exists for steroid treatment
cosyntropin test in the critically ill population, with most attention focused on
(2) Recommend against plasma free cortisol over plasma CIRCI in septic shock (Table 1). In 2018, two separate trials
total cortisol were published—ADRENAL and APROCCHSS (Annane 2018;

Table 1. Major Trials of Corticosteroids in Critically Ill Patients

Pertinent
Mortality Secondary
Trial Population Treatment Taper ACTH Test Outcome Outcomes ADE

Annane 299, septic HCT 50 mg None Benefit in 28 day: ↓ Shorter shock None
2002 shock and IV Q6 hr + nonresponders with HCT reversal with HCT
MV FCT 50 mcg (OR 0.54; (7 vs. 10 days)
PO Q24 hr 95% CI
for 7 days vs 0.31–0.97)
placebo

Sprung 499, mostly HCT 50 mg Over No difference 28 day: No Shorter shock Hyperglycemia (OR
2008 septic IV Q6 hr for 6 days difference reversal with HCT 1.18, 95% CI, 1.07–1.31),
shock 4 days vs (34% (3.3 vs. 5.8 days hypernatremia (OR 1.59;
placebo vs. 32%; p<0.001) 95% CI, 1.13–2.22),
p=0.51) superinfection (OR 1.37;
95% CI, 1.05–1.79) with
HCT

Annane 1241, HCT 50 mg None No difference 90 day: ↓ More vasopressor- Hyperglycemia with
2018 probable IV Q6 hr + with HCT free days at 28 HCT (RR 1.07; 95% CI,
or proven FCT 50 mcg (RR 0.88; days with HCT 1.03–1.12)
septic PO Q24 hr 95% CI (15±11 vs. 17±11;
shock for 7 days vs 0.78–0.99; p<0.001)
placebo p=0.03)

Venkate 3658, septic HCT 200 mg None Not used 90 day: No Shorter shock More ADE with HCT
2018 shock and IV continuous difference reversal (HR 1.32; (21 vs. 6 events)
MV infusion for (OR 0.95; 95% CI ,1.23–1.41)
7 days or ICU 95% CI shorter MV
discharge vs 0.82–1.10) weaning with HCT
placebo (HR 1.13; 95% CI,
1.05–1.22)

↓ = decreased; ACTH = adrenocorticotropic; ADE = adverse drug event; FCT = fludrocortisone; HCT = hydrocortisone; IV = intravenous;
MV = mechanical ventilation; PO = oral.
Information from: Annane D, Renault A, Brun-Buisson C, et al. Hydrocortisone plus fludrocortisone for adults with septic shock.
N Engl J Med. 2018;378:809-18; Annane D, Sébille V, Charpentier C, et al. Effect of treatment with low doses of hydrocortisone and
fludrocortisone on mortality in patients with septic shock. JAMA. 2002;288:862-71; Sprung CL, Annane D, Keh D, et al. CORTICUS
Study Group. Hydrocortisone therapy for patients with septic shock. N Engl J Med. 2008;358:111-24; Venkatesh B, Finfer S, Cohen J,
et al. Adjunctive glucocorticoid therapy in patients with septic shock. N Engl J Med. 2018;378:797-808.

CCSAP 2021 Book 1 • Pulmonary and Endocrinology 123 Critical Care Endocrinology
Venkatesh 2018). Corticosteroids in both trials improved the 1. Suggest against corticosteroids in sepsis without shock
time to shock resolution and weaning from mechanical ven- 2. Suggest using corticosteroids in septic shock refractory
tilation; however, APROCCHSS, which was initially a study of to fluids and moderate- to high-dose vasopressors
activated protein C and corticosteroids, found a 90-day mor- 3. Suggest using long-course and low-dose administration
tality reduction, whereas ADRENAL did not. The durations (hydrocortisone less than 400 mg/day for 3 or more days)
and patient populations of these studies differed: ADRENAL 4. Suggest using corticosteroids in patients with early
was performed over 4 years in five countries whereas moderate to severe acute respiratory distress syndrome,
APROCCHSS was a much longer trial performed over 7 years classified as a ratio of arterial oxygen partial pressure to
fractional inspired oxygen less than 200 within 14 days of
only in France. The ADRENAL study contained also 3 times
onset
the number of patients and included patients with lower
5. Suggest against the use of corticosteroids in major trauma
vasopressor duration and dosage relative to APROCCHSS.
6. Suggest the use of corticosteroids for 5–7 days at a dose
Other differences included a higher number of patients
less than 400 mg/day of intravenous hydrocortisone in
requiring renal replacement therapy and with pneumonia in hospitalized patients with community-acquired pneumonia
APROCCHSS (Yeh 2018).
7. Suggest against the use of corticosteroids in adults with
A recent Cochrane review evaluated 61 randomized con- influenza
trolled trials of corticosteroids versus placebo or usual care 8. Recommend the use of corticosteroids in bacterial
(antimicrobials, fluid replacement, and vasopressor therapy meningitis
as needed) in children and adults with sepsis (Annane 2019). 9. Suggest the use of corticosteroids in patients undergoing
Moderate certainty of the evidence was shown for the use cardiopulmonary bypass surgery
of glucocorticoids for the reduction of 28-day (RR 0.91; 95% 10. Suggest the use of corticosteroids in the setting of
CI, 0.84–0.99) and hospital mortality (RR 0.90; 95% CI, 0.82– cardiac arrest
0.99) in patients with sepsis, in addition to a large reduction in
ICU and hospital length of stay, with a mean difference of 1.07 The impact of fludrocortisone combination therapy
days (95% CI, —1.95 to —0.19) and 1.63 days (95% CI, —2.93 to remains unclear. Some authors have suggested that fludro-
—0.33), respectively. cortisone may be the cause of differences in the clinical trial
The 2017 guidelines further provided detailed recommen- evidence that is currently available; others disagree, given
dations regarding use of corticosteroids as follows (Annane that hydrocortisone doses greater than 40 mg provide max-
2017; Pastores 2018): imal activation of mineralocorticoid receptors (Gunst 2018).

Patient Care Scenario


A 67-year-old woman (weight 82 kg) is admitted to after admission, the patient has received empiric antibi-
the ICU after presenting with altered mental status. otics with cefepime, metronidazole, and vancomycin, and
At admission, she is hypotensive with a lactate of 4.2 the current norepinephrine dose is 0.3 mcg/kg/minute in
mg/dL. After receiving 30 mL/kg of balanced crystalloid addition to vasopressin at 0.03 units/minute. What role,
solution, the patient is started on norepinephrine infu- if any, should corticosteroids have for this patient? If
sion at 0.05 mcg/kg/minute. On evaluation now, 3 hours administered, what regimen is best?

ANSWER
Corticosteroids have most recently been used to improve Blood glucose and serum electrolytes should be moni-
shock resolution and weaning from mechanical ventila- tored in this patient, as well as overall muscular weakness
tion with a potential impact to reduce overall mortality. if possible. No strong evidence supports the use of a delta
This patient should be started on hydrocortisone at a dose cortisol in response to a cosyntropin test before admin-
of 200 mg/day for at least 3 days per guideline recommen- istration of stress-dose corticosteroids, but the use of
dations. Fludrocortisone should not be recommended a cosyntropin test is recommended over evaluation of
given the lack of evidence proving its benefit over the hemodynamic response. If used, the high-dose (250 mcg)
hydrocortisone dose administered. rather than the low-dose (1 mcg) ACTH stimulation test
should be performed.

1. Annane D, Pastores SM, Arlt W, et al. Critical illness-related corticosteroid insufficiency (CIRCI): a narrative review from a Multispecialty
Task Force of the Society of Critical Care Medicine (SCCM) and the European Society of Intensive Care Medicine (ESICM). Intensive Care
Med 2017;43:1781-92.
2. Annane D, Pastores SM, Rochwerg B, et al. Guidelines for the diagnosis and management of critical illness-related corticosteroid insuffi-
ciency (CIRCI) in critically ill patients (Part I): Society of Critical Care Medicine (SCCM) and European Society of Intensive Care Medicine
(ESICM) 2017. Intensive Care Med 2017;43:1751-63. Published correction appears in Intensive Care Med 2018;44:401-2.
3. Rhodes A, Evans LE, Alhazzani W, et al. Surviving Sepsis Campaign: International Guidelines for Management of Sepsis and Septic
Shock: 2016. Intensive Care Med 2017;43:304-77.

CCSAP 2021 Book 1 • Pulmonary and Endocrinology 124 Critical Care Endocrinology
Monitoring 48 hours, override the ability of the natural physiologic mech-
Given the current guidelines for use of corticosteroids, it anisms to counteract these shifts in intracellular volume
is important to be aware of known adverse events and rec- (Harrois 2019). Because SIADH is a diagnosis of exclusion,
ommendations for monitoring. In the Cochrane review just determination can be extremely challenging in the ICU, with
described, glucocorticoids were noted to increase muscle appropriate laboratory tests for diagnosis performed in less
weakness (RR 1.21; 95% CI, 1.01–1.44) and the incidence than 50% of patients (Verbalis 2016). Diagnostic criteria
of hypernatremia (RR 1.66; 95% CI, 1.34–2.06) and well as include serum osmolality, urine osmolality, and urine sodium
hyperglycemia (RR 1.20; 95% CI, 1.10–1.31) (Annane 2019). as well as a workup of adrenal, thyroid, pituitary, and renal
No difference was seen in neuropsychiatric events, rates function. Recent evidence demonstrates that the use of
of superinfection, GI bleeding, or other complications. The exogenous vasopressin as a vasoactive agent may be asso-
guidelines do not recommend any specific monitoring param- ciated with the development of hyponatremia (OR 3.58; 95%
eters for corticosteroid use. CI, 1.02–12.5) in subarachnoid hemorrhage after adjusting for
other factors (Marr 2017). Volume status must be assessed,
which can be limited secondary to inaccurate documenta-
DISORDERS OF ANTI-DIURETIC
tion, transfer between hospitals and/or units, and inaccuracy
HORMONE
of volume assessment methods within the critically ill popu-
Disorders of ADH, which is synonymous with arginine vaso-
lation (Kalantari 2013). Furthermore, the cumulative sodium
pressin, are often secondary diseases for the critically ill
balance within the ICU must be considered to assist in differ-
patient. Syndrome of inappropriate antidiuretic hormone
ential diagnosis—a significant portion of ICU volume is from
secretion is a state of hypotonic hyponatremia, whereas DI
sources other than fluid boluses or maintenance fluids (Gritti
is a hypernatremia secondary to free water loss and overall
2014; Magee 2018). In addition, renal disorders, occurring in
volume depletion. Although opposites in many ways, both
more than 50% of all ICU patients, can alter interpretation of
SIADH and DI are associated with worsened outcomes and
the urine biochemistry required for diagnosis; also, the use of
increased length of hospital stay. Diagnosis of either remains
mannitol or diuretics for volume overload or in the setting of
a challenge given the frequency of additional causes of
renal injury may impact interpretation (Jiang 2019).
sodium imbalance within the ICU (Harrois 2019). Synthesis
Minimal research has focused on the management of
of ADH occurs in the hypothalamic supraoptic and paraven-
SIADH over the past decade, despite reported failure rates
tricular nuclei, and the ADH is stored in the posterior pituitary.
of 55%–64% with current first-line therapies (Verbalis 2016).
Release of ADH is primarily triggered by fluctuations in vol-
Guidelines suggest fluid restriction for mild symptoms and
ume status or osmotic pressures. Working in the distal
the use of hypertonic saline for a sodium less than 135
convoluted tubule and collecting ducts of the kidneys, ADH
mmol/L accompanied by severe symptoms, including cardio-
increases water permeability, thereby increasing total blood
respiratory distress, vomiting, somnolence, seizures, and a
volume and decreasing plasma osmolality (Harrois 2019).
Glasgow Coma Scale score less than 9. Infusion of 150 mL
of 3% hypertonic saline over 20 minutes is recommended, fol-
Syndrome of Inappropriate Antidiuretic lowed by a second dose if serum sodium does not increase
Hormone Secretion by 5 mmol/L at repeat evaluation or if symptoms do not
Syndrome of inappropriate antidiuretic hormone secretion resolve. Repeat sodium evaluations should occur after 20
is a state of normal-expanded volume with decreased urine minutes after treatment. Sodium correction should be limited
output and serum sodium, with sodium concentrations to 10 mmol/L on the first day for patients with chronic hypo-
typically less than 130 mmol/L. The etiology of SIADH var- natremia; however, some have argued for more conservative
ies. In the literature to date, the most common causes of goals (4–6 mmol/L/day) in patients at high risk of osmotic
SIADH include central nervous system diseases, lung dis- demyelination, including those with alcoholism, malnutrition,
ease, cancer, and medications; however, these data were advanced liver disease, hypokalemia, or baseline sodium less
collected from patients who were not in the ICU. Common than 105 mmol/L (Spasovski 2014; Verbalis 2016).
medication-induced SIADH causes include antidepressants, Vaptans have minimal evidence to support their use in
anticonvulsants, antipsychotics, cytotoxic agents and pain the ICU, but recent studies have evaluated their potential for
medications. Specifically, citalopram, escitalopram, venla- in this population. Small reports suggest that single doses
faxine, carbamazepine, amitriptyline, phenytoin, valproic of oral 7.5-mg tolvaptan, less than the typical starting dose
acid, risperidone, paroxetine, vincristine, cyclophospha- of 15 mg, can be effective to treat hyponatremia (Llompart-
mide, and duloxetine have been often cited (Shepshelovich Pou 2017; Umbrello 2016). For patients receiving doses of 15
2017). The syndrome can be life threatening, particularly as mg or greater, the incidence of overcorrection, defined as an
a result of the increase in intracranial hypertension and her- increase in serum sodium concentration of at least 12 mmol/L
niation that can occur from a volume shift to the intracellular in a 24-hour period, is high, up to 19% (Der-Nigoghossian
space. Acute fluctuations, defined as those occurring within 2017). Single-bolus doses of 20 mg of conivaptan have also

CCSAP 2021 Book 1 • Pulmonary and Endocrinology 125 Critical Care Endocrinology
shown to be effective, with minimal evidence citing over- Recent evidence has shown the benefit of the use of
correction (Der-Nigoghossian 2017; Marik 2013). In these copeptin and urea for differentiation of DI from other causes
cohorts, repeat doses are often required after 48 hours, with of hypernatremia in the critically ill population. Copeptin is
some reports of effect up to 96 hours. a glycoprotein of the vasopressin prohormone and relatively
Desmopressin has been evaluated as an adjunctive therapy stable in plasma, whereas urea reabsorption from the kidneys
during sodium correction. Administration of desmopressin is dependent on vasopressin. In the only ICU study to date, a
is thought to hold urine electrolyte-free water excretion at a copeptin of 3.39 pmol/L (interquartile range [IQR] 1.99–3.90;
constant level, decreasing risk of overcorrection and abrupt normal, 1.70–11.25 pmol/L) and urea of 4 mmol/L (IQR 3.1–
sodium changes. In a retrospective observational study, 3.7; normal, 2.5 to 7.1 mmol/L) were significantly associated
combined use of 1–2  mcg intravenous desmopressin with with central DI (Nigro 2018).
hypertonic saline may have allowed gradual increase in serum In patients with suspected DI, desmopressin at a dose of
sodium concentrations without risking overcorrection (Sood 0.5–2 mcg should be administered intravenously for ADH
2013); however, a major study limitation was the observational replacement, with supplemental doses occurring after 2
design with no control group. Guidelines recommend consid- hours if a urine output of 1–2 mL/kg/hour is not achieved.
eration of desmopressin only after expert consultation and If urine output is not controlled by desmopressin, consider-
dosing no more often than every 8 hours (Spasovski 2014). ation should be made for nephrogenic DI because its relative
resistance to ADH repletion is characteristic. In these cases,
Diabetes Insipidus chlorothiazide, indomethacin, and amiloride have been used,
Diabetes insipidus causes hypernatremia, defined as sodium with amiloride recommended as a first-line choice for lithi-
concentration 145 mmol/L or less, by four key etiologies: cen- um-induced nephrogenic DI (Harrois 2019; Robertson 2016).
tral DI, nephrogenic DI, primary polydipsia, and gestational DI. The second hallmark of DI treatment relies on replenish-
Central DI, occurring secondary to a loss of ADH production, ment of water loss with fluid resuscitation. To prevent rapid
is the most clinically relevant to the treatment of critically ill osmolarity shifts, fluids should be selected based on sim-
patients, occurring in those with traumatic brain injury, after ilar osmolarity. A rate of sodium correction of 0.5 mmol/L/
pituitary surgery, and with brain death (Fenske 2012; Imaizumi hour is recommended with a maximum rate of 10 mmol/L in
2017). Nephrogenic DI, a loss of renal sensitivity to ADH, occurs 24 hours. However, patient factors should be taken into con-
most often in response to medications or hypercalcemia, with sideration, with patients who have intracranial hypertension
lithium remaining the most common medication-induced warranting slower correction or maintenance of hypernatre-
cause of DI (Moeller 2013). Case reports of transient DI after mia and patients who have acute hypernatremia possibly
discontinuation of vasopressin therapy in neurosurgical tolerating faster resolution (Harrois 2019).
ICU have also been reported (Carman 2019; Kristeller 2004).
Primary polydipsia occurs in patients with excessive water
THYROID DISORDERS
intake resulting in decreased ADH secretion, whereas gesta-
tional DI occurs secondary to increased ADH degradation from Thyroid Storm
an enzyme produced by the placenta. In ICU patients DI can Thyroid storm is the most severe form of severe thyrotoxico-
remain undetected secondary to the inability to detect one of sis and is a medical emergency. Thyrotoxicosis is defined as
its primary symptoms, polydipsia. Polydipsia is a compensa- a state of excess thyroid hormone, evident by elevated con-
tory response for the ongoing hypotonic polyuria in DI. Given centrations of T3 and T4 with decreased TSH. In a recent,
the high incidence of mortality with DI and hypernatremia in 18-year retrospective study of 31 French ICUs, only 92 cases
critically ill patients, it is likely not appropriate to reply on the of thyroid storm were reported. Amiodarone-induced thyro-
currently recommended criteria for diagnosis, including a high toxicosis was responsible for 33% of the cases, with Graves
urine output (more than 3000 mL/day), hypotonic urine with disease representing 26%. Graves disease is the most com-
low urine osmolality (less than 300 mOsm/kg), and polydip- mon cited etiology of thyroid storm and is characterized by
sia. Instead, patients with a urine output more than 300 mL/ TSH-receptor stimulating antibodies. In this study, ICU and
hour or 4 mL/kg/hour for 2 hours in the setting of an increase 6-month postadmission mortality rates were 17% and 22%,
in serum sodium of 2–3 mmol/L should receive further eval- respectively (Bourcier 2020).
uation for urine and plasma osmolality (Harrois 2019). Urine In thyroid storm, thyrotoxicosis occurs alongside sys-
osmolality of less than 300 mOsm/kg and a plasma osmolal- tematic decompensation (Box 3). All of these criteria are a
ity more than 300 mOsm/kg are indicative of DI. In patients part of the Burch-Wartofsky Point Scale, the guideline-rec-
receiving concomitant therapies that potentially contribute to ommended scoring tool to assist with diagnosis of thyroid
higher urine osmolality, such as high loads of sodium or man- storm (Ross 2016). Although excess thyroid can induce con-
nitol, or in patients with ongoing hyperglycemia, the use of sumption of oxygen, an increase in sympathetic drive, and
urine density may be considered, with a specific gravity less a hypermetabolic state, further details regarding the patho-
than 1.005 correlated with hypotonic urine. physiology of thyroid storm are unknown. It is hypothesized

CCSAP 2021 Book 1 • Pulmonary and Endocrinology 126 Critical Care Endocrinology
other complications, such as fever, central nervous system
Box 3. Systemic Symptoms of
manifestations, convulsions, and heart failure, standard of
Thyrotoxicosis
care therapies are suggested (Satoh 2016).
Agitation Hyperpyrexia
Careful monitoring for potential adverse effects such
Arrhythmias Nausea
Coma Psychosis as pruritus/rashes, agranulocytosis, and liver dysfunction
Congestive heart failure Stupor should occur with propylthiouracil therapy. Hemodynamic
Delirium Tachycardia monitoring should occur as well as monitoring of thyroid hor-
Diarrhea Vomiting mones. Improvement should occur within 3 days of therapy,
Hepatic failure
so assessment of T3 and T4 every 24 hours within this time
frame is acceptable. In one study, T3 concentrations dropped
by 45% in the first 24 hours of therapy. (Pokhrel 2020) After
that an increase in β1-adrenergic receptor exposure and cat-
hemodynamics, thermoregulation, and neurologic status
echolamine concentrations as well as displacement of free
have normalized, the patient can be converted to mainte-
thyroid hormones by circulating inhibitors of binding occur
nance therapy, which is then continued until euthyroidism is
during critical illness (Carroll 2010).
achieved (Carroll 2010).

Treatment Myxedema Coma


Treatment of thyroid storm consists of therapy to mitigate Myxedema coma is an endocrine emergency occurring sec-
the thyrotoxicosis as well as damage to organ systems. ondary to the progression of untreated hypothyroidism and
Propylthiouracil, which stops T3 conversion and T4 pro- typically thought to be induced secondary to another precip-
duction, is recommended in the setting of thyroid storm. A itating factor, such as a trauma or surgery (Jacobi 2019). The
loading dose of 600–1000 mg orally is followed by 250 mg phenomenon is rare, with one database showing its occur-
orally every 4 to 6 hours. Lower doses can be used as symp- rence in 149 patients of 19 million inpatients (Ono 2017).
toms resolve or in patients with a less severe condition. Patients present with a wide range of symptoms, a low serum
Methimazole is usually less preferred in the setting of thy- T4 and T3, and an elevated TSH concentration. Symptoms
roid storm given its slower onset of action. Of note, when may include lethargy, alveolar hypoventilation, hypother-
destructive thyroiditis is the etiology of the thyroid storm, mia, bradycardia, narrowed pulse pressure, hypertension,
propylthiouracil and methimazole are not recommended hypodynamic precordium, acute renal failure, edema, and
because of their lack of efficacy. An inorganic iodine should constipation. Additional abnormal laboratory findings include
also be administered to decrease hormone synthesis by hyponatremia and hypoglycemia.
iodide oxidation and organification. Typically the recom- Supportive care should be used in addition to hormone
mendation is the administration of potassium iodide 250 mg treatment. Although controversies do exist, the current stan-
orally every 6 hours starting 1 hour after the antithyroid drug dard approach to treatment is 300–500 mcg intravenous T4
therapy is initiated (Ross 2016; Satoh 2016). Corticosteroids therapy with the addition of T3 if no response is observed,
are the third treatment recommended for thyroid storm man- or starting dual therapy with 200–300 mcg T4 plus 10–25
agement. Corticosteroids are administered as prophylaxis mcg T3 orally (Kwaku 2007). After initial doses, T4 at a dose
for relative AI that is hypothesized to occur in thyroid storm. of 50–100 mcg/day with or without T3 10 mcg every 8 hours
Despite guidelines recommending hydrocortisone 300 mg is recommended (Kwaku 2007). Other supportive care mea-
intravenously as a loading dose, followed by 100 mg every sures should be considered as appropriate, particularly with
8 hours for prophylaxis as well as potential blockade of T4 consideration for the precipitating event. Given that infec-
to T3 conversion, it must be noted that most of these rec- tion is considered a possible precipitating etiology, some
ommendations are hypothetical. In fact, in one study using recommend liberal use of antibiotics in the initial phases.
multiple regression analyses, disease severity and mortal- Corticosteroids, specifically 100-mg intravenous hydrocor-
ity were significantly higher in patients who were treated tisone every 8 hours, have also been recommended in this
with corticosteroids than in those who were not (Isozaki setting given the possibility of AI; however, it is worth not-
2016). For supportive care, the use of β-adrenergic blocking ing that the occurrence of AI concomitantly with myxedema
agents such as propranolol is recommended, given its selec- is ill-defined and corticosteroids are suppressive of thyroid
tive ability to inhibit type 1 deiodinase (Ross 2016). Similarly, function (Wiersinga 2000). For these reasons, a cortisol
however, clinical trial data to support the use of propranolol concentration should be collected before administration of
are limited, and some case reports have documented wors- corticosteroids with discontinuation of glucocorticoids if AI
ening circulatory failure with its use (Abubakar 2017). For is not present. Further T3 and T4 hormone monitoring should
patients who experience failure of response to these ther- occur every 24–48 hours in addition to supportive monitoring
apies, plasmapheresis and emergency surgery have been measures, such as electrolytes, hemodynamics, and infec-
used to treat refractory thyroid storm. Regarding therapy for tious disease workup.

CCSAP 2021 Book 1 • Pulmonary and Endocrinology 127 Critical Care Endocrinology
DYSGLYCEMIA IN THE ICU Continuous infusion intravenous administration of insu-
Three key areas have been associated with worse outcomes lin remains the recommended approach for management of
in the critically ill population: stress hyperglycemia, hypogly- hyperglycemia in the ICU, with minimal evidence for the effi-
cemia, and high glycemic variability. cacy or safety of subcutaneous insulin (Umpierrez 2017).

Hyperglycemia Hypoglycemia
Hyperglycemia in the ICU can be secondary to underlying Alterations of blood glucose in either direction are an
diabetes or a product of stress hyperglycemia secondary important consideration for treatment, and the impact of
to critical illness. Stress hyperglycemia results from exces- either hyper- or hypoglycemia can be clinically significant.
sive hepatic production of glucose by glycogenolysis and Hypoglycemia is typically iatrogenic in nature in response to
gluconeogenesis, as well as a relative insulin resistance, insulin therapy, and targeting tight glucose control increases
often correlative with severity of illness (Lheureux 2019). the risk of hypoglycemia 5-fold. However, spontaneous epi-
This condition can be stimulated from a multitude of fac- sodes of hypoglycemia may occur in patients with hepatic
tors, including glucagon, catecholamines, growth hormone, failure or adrenal failure, and comorbid renal failure or mal-
cortisol, adipokines, and cytokines. The damage can be addi- nutrition also increase the risk (Lacherade 2009). Severe
tive given that hyperglycemia may exacerbate the cytokine hypoglycemia is defined as a blood glucose less than 40 mg/
and inflammatory process, resulting in even worsened hyper- dL (Lheureux 2019). Although the exact mechanism of harm
glycemia. The potential catabolic state resulting in lipolysis is unknown, it is suspected to be multifactorial, including a
can further add to this cascade by the alteration of end-or- neurologic contribution, considering the limited glucose
gan insulin signaling and glycogen synthase (Lheureux reserves and high-energy requirement of the brain, as well as
2019). Stress hyperglycemia is defined as a fasting glucose a cardiac contribution, with the potential predisposition of the
greater than 125 mg/dL or a single reading greater than 200 myocardium to arrhythmias in the setting of hypoglycemia
mg/dL in patients without diabetes. Although the impact of (Tickoo 2019). To manage hypoglycemia, patient responsive-
hyperglycemia has been debated at some cut-off values, it ness must first be assessed. In nonresponsive patients, 25 g
is clear that mortality gradually increases within the range of 50% dextrose should be administered intravenously, or
of 145–245 mg/dL in patients without diabetes. (Lheureux intramuscular glucagon 1 mg if intravenous access is not
2019). Several clinical trials have attempted to mitigate this available. In responsive patients who have an intake restric-
hyperglycemia with mixed outcomes. Even meta-analyses of tion of nothing by mouth, the same recommendations should
glucose control within the ICU have found markedly different be followed with a lower dose (15–20 g) of dextrose adminis-
results (Pittas 2006). tered and a repeat glucose test after 15 minutes. If recurrent
Hyperglycemic crises, including diabetic ketoacidosis hypoglycemia exists, priority should be placed on feeding
and hyperglycemic hyperosmolar syndrome secondary to if possible. If not, then continued administration of 15–20 g
underlying diabetic disorders, are relatively well-described dextrose should be given with 15-minute follow-up to assess
in the literature. Increasing evidence demonstrates that fur- the glucose concentrations reached (Tomky 2005). Although
ther research is needed to reassess these current standards. dextrose infusions may be used in practice, no evidence sup-
Specifically, the use of moderate-intensity insulin therapy ports their safety or clinical benefit on long-term outcomes in
rather than high-intensity insulin therapy for the treatment patients with continued hypoglycemia.
of diabetic ketoacidosis and hyperglycemic hyperosmolar
syndrome has been associated with decreased glycemic vari- Standard Approaches to Glycemic Control
ability and improved hospital and ICU length of stay (Firestone The first hallmark study of glucose control was published in
2019). Moderate-intensity insulin was defined as: 1) a more 2001, demonstrating a significant improvement in mortal-
liberal target blood glucose (200–300 mg/dL); 2) avoidance ity for patients treated with more aggressive insulin therapy
of intravenous bolus dose of insulin; 3) conservative initial (Van Den Berghe 2001). Since then, many studies have failed
infusion rate (0.05–0.1 units/kg/hour); 4) gradual insulin titra- to find the same benefit. With the NICE-SUGAR study in 2009,
tion, such as dose adjustments by 25%; and 5) target decline the largest glucose control study at the time, the standard of
in blood glucose of 26–50 mg/dL per hour. Another study care for management of blood glucose in critically ill patients
of 8553 patients demonstrated that partial early correction shifted from intensive insulin therapy targets (80–110 mg/dL)
of hyperglycemia to more than 180 mg/dL instead of inten- to relaxed targets of 140–180 mg/dL (Finfer 2009). Current
sive correction of hyperglycemia to 180 mg/dL or less was guidelines still recommend achievement of 140 mg/dL or less
independently associated with decreased hypoglycemia (OR to 180 mg/dL for most critically ill patients once insulin treat-
0.38; 95% CI, 0.30–0.48) and hospital mortality (OR, 0.44; 95% ment is started by continuous infusion (Moghissi 2009). A
CI, 0.22–0.86) (Mårtensson 2017). However, the 2014 guide- recent meta-analysis of 17,996 patients supported this rec-
lines remain the current standard of care; therefore, this ommendation, with a target glucose of 140–180 resulting in
chapter will not provide a detailed review (Gosmanov 2014). similar mortality as targets of 80–110 mg/dL, 110–140 mg/dL,

CCSAP 2021 Book 1 • Pulmonary and Endocrinology 128 Critical Care Endocrinology
or 180–220 mg/dL, without the increase in hypoglycemia coefficient of variation of greater than 20% suggested to cor-
seen with more aggressive targets (Yamada 2017). Current relate with worsened outcomes (Bagshaw 2009; Lheureux
recommendations suggest against the use of capillary blood 2019). To date, no studies have developed bedside protocols
testing and encourage the use of dynamic-scale algorithms targeting insulin therapy on glycemic variability, and more
with consideration for patient-specific factors; however, more information is needed to determine how to approach its use
data are needed on the various computer-guided protocols from a therapeutic standpoint. While awaiting further trials,
studied to date. pharmacists today should consider personalization of ther-
apy with avoidance of glycemic variability greater than 20%
Glycemic Variability and focusing on a target range of 140–180 mg/dL per current
Given the differences seen in previous clinical trials with vary- guidelines (Finfer 2009).
ing patient populations, it is likely that the best approach to
glucose management in the ICU is a personalized strategy.
One specific consideration, a phenomenon known as the CHRONIC CRITICAL ILLNESS
“diabetes paradox” has come to light, demonstrating that Recent attention has focused on the concept of prolonged
hyperglycemia does not appear to have the impact on mortal- critical illness and the impact on overall endocrine function.
ity for patients with diabetes that it does for patients without Chronic critical illness has been ill-defined, with some defi-
diabetes (Krinsley 2013). In an observational multicenter nitions ranging from 72 hours up to 21 days of mechanical
study of 90,644 patients with sepsis, patients with insulin-de- ventilation (Loss 2017). Most of the previous discussion in
pendent diabetes had lower adjusted hospital mortality with this chapter has focused on the acute phase of most critical
higher peak blood glucose concentrations, whereas patients illness, with increased pituitary hormone release; however,
who did not have insulin-dependent diabetes had increased the prolonged phase of critical illness is characterized by
mortality (p=0.012), a finding replicated when assessing gly- suppression of all hypothalamic–anterior pituitary axes (Van
cemic variability (p=0.048) (Magee 2018). Den Berghe 2016). The biphasic—both acute and chronic—
Furthermore, the patient population is an important con- response to critical illness has been well documented in the
sideration. Historically, it is known that hyperglycemia has literature and includes alteration in the growth hormone, thy-
been associated with 2-fold increases of the incidence of roid, and adrenal axis, particularly dependent on nutritional
wound infection in patients who undergo cardiothoracic status. Muscle and skeletal wasting has been associated
surgical procedures, but considerations for other ICU popu- with a loss of pulsatility in growth hormone secretion.
lations also likely exist (Latham 2001). For example, patients Regarding the HPT axis, both T4 and T3 are low in prolonged
with brain injury are suspected to have greater sensitivity to critical illness. The standard pulsatile TSH secretion is lost,
hypoglycemia; however, in observational studies hypergly- and evidence suggests a decrease in TRH as well. Lack of
cemia enhanced neurologic injury of traumatic brain injury hypothalamic TRH production prevents TSH activation of the
and intracerebral hemorrhage (Shi 2016). Similar to the ICU thyroid. Once recovery is finally reached, a TSH spike is seen
population overall, studies of glycemic control in patients again followed by hormone recovery. The impact of this phe-
with neurologic injury have failed to show consistent sig- nomenon, termed nonthyroidal illness, is currently unknown.
nificant differences in clinical outcomes with aggressive In one retrospective study of 213 patients, severity of nonthy-
glucose control and an increased incidence of hypoglyce- roidal illness demonstrated a significant association with
mia. Most recently, in the SHINE randomized controlled trial mortality (OR 3.101; 95% CI, 1.711–5.618) and ICU length of
of patients with acute ischemic stroke and hyperglycemia stay (Lee 2016). Worth noting, of the 213 patients with thy-
(glucose greater than 110 mg/dL with diabetes or 150 mg/dL roid function tested, 94.9% had some degree of nonthyroidal
or more without diabetes), the use of intravenous insulin to a illness.
glucose target of 80–130 mg/dL did not result in a significant In prolonged critical illness, the HPA axis also remains
difference in favorable functional outcome at 90 days com- altered. Hypercortisolism still remains with ACTH-cortisol
pared with subcutaneous insulin to a target of 80–178 mg/dL dissociation, for reasons not yet clear. A decrease in corti-
(Johnston 2019). sol production is noted, but clearance of cortisol is markedly
The concept of evaluating changes in glucose over time decreased as well, with an increase in the half-life of corti-
has gained increasing interest. In vitro studies demon- sol 5-times that of normal (Boonen 2013). Studies have
strate that fluctuations in blood glucose are associated demonstrated that despite reductions in ACTH and cortisol
with increased oxidative stress, more so than that of hyper- pulsatile secretion, the feedback loop and control remains
glycemia. Specifically, in a study of 7049 patients, glycemic intact (Peeters 2018). In a prospective study of ICU patients
variability that was defined as the standard deviation of blood admitted for at least 1 week, all 347 of whom were free of glu-
glucose values correlated better with increased mortality cocorticoid treatment up to ICU day 28, plasma ACTH always
compared with the average blood glucose concentration (Egi remained low to normal, whereas free cortisol remained high
2006). Many studies have confirmed this correlation, with a (p≤0.002), which was explained by reduced binding proteins

CCSAP 2021 Book 1 • Pulmonary and Endocrinology 129 Critical Care Endocrinology
low incidence of adverse events (less than 20%); however, the
Practice Points
data were not comparative (Kanji 2018).
The implications of critical illness on neuroendo- Studies on corticosteroids are conflicting as well, with
crine function and overall endocrine physiology are
none targeting patients specifically with signs of adrenal fail-
vast. The determination of when these responses are
physiologic and when they become pathophysiologic ure in late critical illness. Current doses of corticosteroids
remains unknown. In this context, clinical pharmacists studied in the critically ill population (200–300 mg/day) are 6
must remain aware of the endocrinologic alterations times that of normal cortisol production during critical illness
that occur within the critically ill population as well as and likely to continue to show potential harm in the setting
the implications and considerations for treatment as of delayed clearance, particularly late in the intensive care
follows:
course (Peeters 2018).
• The stress response to critical illness occurs in three Whether the late endocrine changes that occur in criti-
phases, ranging from acute adaption to eventually either
cal illness are of therapeutic consideration remain unknown.
exhaustion or normalization.
• Adrenal activation increases with illness severity, but can Current evidence surrounding therapies targeted at pro-
be inadequate, whereas thyroidal alterations result from longed critical illness changes is limited to animal studies
various causes, including cytokines and medications, often and retrospective reports. Further research targeting man-
inhibiting thyroidal hormone conversion. agement of these endocrine abnormalities is needed.
• Laboratory evaluation of the HPA and –thyroid hormones is
difficult given limitations in current testing methods.
• Pharmacotherapy for Cushing syndrome is second-line to
CONCLUSION
surgical intervention, whereas absolute AI is treated with Endocrine abnormalities are ubiquitous in the ICU, rang-
corticosteroid replacement. ing from rare emergencies of chronic conditions to acute
• Recent attention surrounds the phenomenon of critical
fluctuations relative to critical illness. Given the need for addi-
illness–related corticosteroid insufficiency; however, its
tional data in a multitude of disease states, it is imperative
implications remain unknown.
• Concomitant corticosteroids are recommended in patients that clinical pharmacists understand the basic principles of
with refractory septic shock, early moderate to severe endocrine pathophysiology and the recommended therapeu-
acute respiratory distress syndrome, bacterial meningitis, tic regimens. Furthermore, providers must remain aware of
cardiac arrest, cardiopulmonary bypass surgery, and com- the repercussions of not only chronic endocrine disease and
munity-acquired pneumonia.
implications for the treatment of these patients in the acute
• Disorders of ADH are predominately considered diagnoses
of exclusion, with recent evidence supporting fluid restric- setting but also the importance of identifying these abnor-
tion, hypertonic saline, and potential for low-dose vaptans malities within the critically ill patient.
for SIADH and ADH supplementation in DI.
• Thyroid storm should be treated with combination therapy,
REFERENCES
whereas either T3/T4 combination therapy or T4 therapy
can be administered for myxedema coma. Abubakar H, Singh V, Arora A, et al. Propranolol-induced
• Blood glucose targets in the ICU should be personalized, circulatory collapse in a patient with thyroid crisis and
with consideration for baseline diabetes, presenting illness, underlying thyrocardiac disease: a word of caution.
and glycemic variability. J Investig Med High Impact Case Rep 2017;5:
• Chronic critical illness is at times characterized by a failure 2324709617747903.
of the endocrine responses to normalize, and consideration
Annane D, Bellissant E, Bollaert PE, et al. Corticosteroids for
for treatment should be made on a patient-specific basis.
treating sepsis in children and adults. Cochrane Database
Syst Rev 2019;12:CD002243.

(p≤0.02) and suppressed cortisol breakdown (p≤0.001). Annane D, Pastores SM, Arlt W, et al. Critical illness-related
Cortisol concentrations did not normalize until after 28 days corticosteroid insufficiency (CIRCI): a narrative review
from a Multispecialty Task Force of the Society of Critical
(Peeters 2018). In a separate prospective study of 54 surgi-
Care Medicine (SCCM) and the European Society of
cal ICU patients, including patients 55 years and older and
Intensive Care Medicine (ESICM). Intensive Care Med
with ICU stays of 14 days or greater, 11% were adrenally insuf- 2017A;43:1781-92.
ficient after 2 weeks of ICU stay (Barquist 1997).
Annane D, Pastores SM, Rochwerg B, et al. Guidelines for the
diagnosis and management of critical illness-related corti-
Considerations for Treatment
costeroid insufficiency (CIRCI) in critically ill patients (part
Some authors have reported treating impaired thyroid func- I): Society of Critical Care Medicine (SCCM) and European
tion for critically ill patients who have prolonged hypotension, Society of Intensive Care Medicine (ESICM) 2017. Crit Care
respiratory failure, and altered mental status. A case series Med 2017B;45:2078-88.
suggested patient improvement with initiation of levothyrox-
Annane D, Renault A, Brun-Buisson C, et al. Hydrocortisone
ine (Kumar 2018). A retrospective study of enteral T3 showed plus fludrocortisone for adults with septic shock. N Engl J
improvement in 70 patients treated for low serum T3 and a Med 2018;378:809-18.

CCSAP 2021 Book 1 • Pulmonary and Endocrinology 130 Critical Care Endocrinology
Annane D, Sébille V, Troché G, et al. A 3-level prognostic Fenske W, Allolio B. Clinical review: current state and future
classification in septic shock based on cortisol levels perspectives in the diagnosis of diabetes insipidus: a clini-
and cortisol response to corticotropin. J Am Med Assoc cal review. J Clin Endocrinol Metab 2012;97:3426-37.
2000;283:1038-45.
Ferreira JA, Bissell BD. Misdirected sympathy: the role of
Bagshaw SM, Bellomo R, Jacka MJ, et al. The impact of early sympatholysis in sepsis and septic shock. J Intensive
hypoglycemia and blood glucose variability on outcome in Care Med 2018;33.
critical illness. Crit Care 2009;13:R91.
Finfer S, Bellomo R, Blair D, et al. Intensive versus conven-
Barquist E, Kirton O. Adrenal insufficiency in the surgical tional glucose control in critically ill patients. N Engl J Med
intensive care unit patient. J Trauma 1997;42:27-31. 2009;360:1283-97.

Boonen E, Van Den Berghe G. Endocrine responses to critical Firestone RL, Parker PL, Pandya KA, et al. Moderate-
illness: novel insights and therapeutic implications. intensity insulin therapy is associated with reduced length
J Clin Endocrinol Metab 2014;99:1569-82. of stay in critically ill patients with diabetic ketoacido-
sis and hyperosmolar hyperglycemic state. Crit Care Med
Boonen E, Vervenne H, Meersseman P, et al. Reduced cor- 2019;47:700-5.
tisol metabolism during critical illness. N Engl J Med
2013;368:1477-88. Gosmanov AR, Gosmanova EO, Dillard-Cannon E.
Management of adult diabetic ketoacidosis. Diabetes
Bourcier S, Coutrot M, Kimmoun A, et al. Thyroid storm in Metab Syndr Obes 2014;7:255-64.
the ICU: a retrospective multicenter study. Crit Care Med
2020;48:83-90. Gritti P, Lanterna LA, Rotasperti L, et al. Clinical evaluation
of hyponatremia and hypovolemia in critically ill adult
Carman N, Kay C, Petersen A, et al. Transient central diabe- neurologic patients: contribution of the use of cumulative
tes insipidus after discontinuation of vasopressin. Case balance of sodium. J Anesth 2014;28:687-95.
Rep Endocrinol 2019:4189525.
Gunst J, Van den Berghe G. Glucocorticoids with or with-
Carroll R, Matfin G. Endocrine and metabolic emergencies: out fludrocortisone in septic shock. N Engl J Med
thyroid storm. Ther Adv Endocrinol Metab 2010;1:139-45. 2018;379:893-6.
Cuesta JM, Singer M. The stress response and critical ill- Harrois A, Anstey JR. Diabetes insipidus and syndrome of
ness. Crit Care Med 2012 Dec;40:3283-9. inappropriate antidiuretic hormone in critically ill patients.
Crit Care Clin 2019;35:187-200.
Dalegrave D, Silva RL, Becker M, et al. Relative adrenal insuf-
ficiency as a predictor of disease severity and mortality in Husebye ES, Allolio B, Arlt W, et al. Consensus statement on
severe septic shock. Rev Bras Ter Intensiva 2012;24:362-8. the diagnosis, treatment and follow-up of patients with pri-
mary adrenal insufficiency. J Intern Med 2014;275:104-15.
Dang DK, Chen JT, Pucino F, et al. Adrenal gland disorders.
In: Chisholm-Burns MA, Schwinghammer TL, Wells BG, et Imaizumi T, Nakatochi M, Fujita Y, et al. The association
al., eds. Pharmacotherapy Principles & Practice, 4th ed. between intensive care unit-acquired hypernatraemia and
New York, McGraw-Hill Education, 2014:1327-44. mortality in critically ill patients with cerebrovascular dis-
eases: a single-centre cohort study in Japan. BMJ Open
Demiselle J, Fage N, Radermacher P, Asfar P. Vasopressin
2017;7:e016248.
and its analogues in shock states: a review. Ann Intensive
Care 2020;10:9. Isozaki O, Satoh T, Wakino S, et al. Treatment and manage-
ment of thyroid storm: analysis of the nationwide surveys:
Der-Nigoghossian C, Lesch C, Berger K. Effectiveness and
the taskforce committee of the Japan Thyroid Association
tolerability of conivaptan and tolvaptan for the treat-
and Japan Endocrine Society for the establishment of
ment of hyponatremia in neurocritically ill patients.
diagnostic criteria and nationwide surveys for thyroid
Pharmacotherapy 2017;37:528-34.
storm. Clin Endocrinol (Oxf) 2016;84:912-8.
Dichtel LE, Schorr M, Loures De Assis C, et al. Plasma free
Jacobi J. Management of endocrine emergencies in the ICU.
cortisol in states of normal and altered binding globulins:
J Pharm Pract 2019;32:314-26.
implications for adrenal insufficiency diagnosis. J Clin
Endocrinol Metab 2019;104:4827-36. Jiang L, Zhu Y, Luo X, et al. Epidemiology of acute kidney
injury in intensive care units in Beijing: the multi-center
Dorin RI, Qualls CR, Torpy DJ, et al. Reversible increase in
BAKIT study. BMC Nephrol 2019;20:468.
maximal cortisol secretion rate in septic shock. Crit Care
Med 2015;43:549-56. Johnston KC, Bruno A, Pauls Q, et al. Intensive vs standard
treatment of hyperglycemia and functional outcome in
Egi M, Bellomo R, Stachowski E, et al. Variability of blood patients with acute ischemic stroke: the SHINE random-
glucose concentration and short-term mortality in criti- ized clinical trial. JAMA 2019;322:326-35.
cally ill patients. Anesthesiology 2006;105:244-52.
Kalantari K, Chang JN, Ronco C, et al. Assessment of intra-
Eiden LE, Jiang SZ. What’s new in endocrinology: the chro- vascular volume status and volume responsiveness in
maffin cell. Front Endocrinol (Lausanne) 2018;9:711. critically ill patients. Kidney Int 2013;83:1017-28.

CCSAP 2021 Book 1 • Pulmonary and Endocrinology 131 Critical Care Endocrinology
Kamenický P, Droumaguet C, Salenave S, et al. Mitotane, Marik PE, Linde-Zwirble WT, Bittner EA, et al. Fluid admin-
metyrapone, and ketoconazole combination therapy as istration in severe sepsis and septic shock, patterns
an alternative to rescue adrenalectomy for severe ACTH- and outcomes: an analysis of a large national database.
dependent Cushing’s syndrome. J Clin Endocrinol Metab Intensive Care Med 2017;43:625-32.
2011;96:2796-804.
Marik PE, Rivera R. Therapeutic effect of conivaptan
Kanji S, Neilipovitz J, Neilipovitz B, et al. Triiodothyronine bolus dosing in hyponatremic neurosurgical patients.
replacement in critically ill adults with non-thyroidal ill- Pharmacotherapy 2013;33:51-5.
ness syndrome. Can J Anesth 2018;65:1147-53.
Marr N, Yu J, Kutsogiannis DJ, et al. Risk of hyponatremia
Khardori R, Castillo D. Endocrine and metabolic in patients with aneurysmal subarachnoid hemorrhage
changes during sepsis: an update. Med Clin North Am treated with exogenous vasopressin infusion. Neurocrit
2012;96:1095-105. Care 2017;26:182-90.
Krinsley JS, Egi M, Kiss A, et al. Diabetic status and the rela- Mårtensson J, Bailey M, Venkatesh B, et al. Intensity of early
tion of the three domains of glycemic control to mortality correction of hyperglycaemia and outcome of critically
in critically ill patients: an international multicenter cohort ill patients with diabetic ketoacidosis. Crit Care Resusc
study. Crit Care 2013;17:R37. 2017;19:266-73.
Kristeller JL, Sterns RH. Transient diabetes insipi-
Moeller HB, Rittig S, Fenton RA. Nephrogenic diabetes
dus after discontinuation of therapeutic vasopressin.
insipidus: essential insights into the molecular back-
Pharmacotherapy 2004;24:541-5.
ground and potential therapies for treatment. Endocr Rev
Kumar E, McCurdy MT, Koch CA, et al. Impairment of thyroid 2013;34:278-301.
function in critically ill patients in the intensive care units.
Moghissi ES, Korytkowski MT, DiNardo M, et al.; American
Am J Med Sci 2018;355:281-5.
Association of Clinical Endocrinologists; American
Kwaku MP, Burman KD. Myxedema coma. J Intensive Care Diabetes Association. American Association of Clinical
Med 2007;22:224-31. Endocrinologists and American Diabetes Association con-
sensus statement on inpatient glycemic control. Diabetes
Lacherade JC, Jacqueminet S, Preiser JC. An overview of
Care 2009;32:1119-31.
hypoglycemia in the critically ill. J Diabetes Sci Technol
2009;3:1242-9. Molenaar N, Groeneveld ABJ, de Jong MFC. Three calcu-
Latham R, Lancaster AD, Covington JF, et al. The associ- lations of free cortisol versus measured values in the
ation of diabetes and glucose control with surgical-site critically ill. Clin Biochem 2015;48:1053-8.
infections among cardiothoracic surgery patients. Infect Nieman LK, Biller BM, Findling JW, et al.; Endocrine Society.
Control Hosp Epidemiol 2001;22:607-12. Treatment of Cushing’s syndrome: an Endocrine Society
Lee WK, Hwang S, Kim D, et al. Distinct features of nonthyroi- clinical practice guideline. J Clin Endocrinol Metab
dal illness in critically ill patients with infectious diseases. 2015;100:2807-31.
Medicine (Baltimore) 2016;95:e3346.
Nigro N, Winzeler B, Suter-Widmer I, et al. Copeptin levels
Lheureux O, Prevedello D, Preiser JC. Update on glucose in and commonly used laboratory parameters in hospi-
critical care. Nutrition 2019;59:14-20. talised patients with severe hypernatraemia. Crit Care
2018;22:1-9.
Llompart-Pou JA, Pérez-Bárcena J, Novo M, et al. Effect of
single-dose of tolvaptan in neurocritical patients with Nylen ES, Muller B. Endocrine changes in critical illness. J
hyponatremia due to syndrome of inappropriate antidi- Intensive Care Med 2004;19:67-82.
uretic hormone secretion. Med Intensiva 2017;41:501-3.
Ono Y, Ono S, Yasunaga H, et al. Clinical characteristics and
Loss SH, Nunes DSL, Franzosi OS, et al. Doença crítica outcomes of myxedema coma: analysis of a national inpa-
crônica: estamos salvando ou criando VÍtimas? Revista tient database in Japan. J Epidemiol 2017;27:117-22.
Brasileira de Terapia Intensiva, vol 29. Sao Paulo, Brazil:
Associacao de Medicina Intensiva Brasileira (AMIB), Pastores SM, Annane D, Rochwerg B, et al. Guidelines for the
2017:87-95. diagnosis and management of critical illness-related corti-
costeroid insufficiency (CIRCI) in critically ill patients (part
Magee CA, Bastin MLT, Laine ME, et al. Insidious harm of II): Society of Critical Care Medicine (SCCM) and European
medication diluents as a contributor to cumulative volume Society of Intensive Care Medicine (ESICM) 2017. Crit Care
and hyperchloremia: a prospective, open-label, sequential Med 2018;46:146-8.
period pilot study. Crit Care Med 2018;46:1217-23.
Peeters B, Meersseman P, Vander Perre S, et al.
Magee F, Bailey M, Pilcher D V., et al. Early glycemia and Adrenocortical function during prolonged critical illness
mortality in critically ill septic patients: interaction with and beyond: a prospective observational study. Intensive
insulin-treated diabetes. J Crit Care 2018;45:170-7. Care Med 2018;44:1720-9.
Marik PE. Mechanisms and clinical consequences of criti-
Pittas AG, Siegel RD, Lau J. Insulin therapy and in-hospital
cal illness associated adrenal insufficiency. Curr Opin Crit
mortality in critically ill patients: systematic review and
Care 2007;13:363-9.

CCSAP 2021 Book 1 • Pulmonary and Endocrinology 132 Critical Care Endocrinology
meta-analysis of randomized controlled trials. JPEN J Téblick A, Peeters B, Langouche L, et al. Adrenal func-
Parenter Enteral Nutr 2006;30:164-72. tion and dysfunction in critically ill patients. Nat Rev
Endocrinol 2019;15:417-27.
Puar THK, Stikkelbroeck NMML, Smans LCCJ, et al. Adrenal
crisis: still a deadly event in the 21st century. Am J Med Tickoo M. The long and winding road to personalized glyce-
2016;129:339.e1-9. mic control in the intensive care unit. Semin Respir Crit
Care Med 2019;40:571-9.
Preda VA, Sen J, Karavitaki N, et al. Etomidate in the man-
agement of hypercortisolaemia in Cushing’s syndrome: a Tomky D. Detection, prevention, and treatment of hypoglyce-
review. Eur J Endocrinol 2012;167:137-43. mia in the hospital. Diabetes Spectrum 2005;18:39-44.

Rai R, J, Cohen J, Venkatesh B. Assessment of adrenocorti- Tompkins MG, Bissell BD, Sowders V, et al. Inclusion of adre-
cal function in the critically ill. Crit Care Resusc 2004;6. nal crisis in the differential diagnosis of distributive shock.
Am J Health Syst Pharm 2020;77:415-7.
Rhodes A, Evans LE, Alhazzani W, et al. Surviving sep-
sis campaign: international guidelines for management Ullian M. The role of corticosteroids in the regulation of vas-
of sepsis and septic shock: 2016. Intensive Care Med cular tone. Cardiovasc Res 1999;41:55-64.
2017;43:304-77.
Umbrello M, Mantovani ES, Formenti P, et al. Tolvaptan for
Robertson GL. Diabetes insipidus: differential diagnosis hyponatremia with preserved sodium pool in critically ill
and management. Best Pract Res Clin Endocrinol Metab patients. Ann Intensive Care 2016;6:1-9.
2016;30:205-18.
Umpierrez GE, Pasquel FJ. Management of inpatient hyper-
Ross DS, Burch HB, Cooper DS, et al 2016 American Thyroid glycemia and diabetes in older adults. Diabetes Care
Association Guidelines for diagnosis and management 2017;40:509-17.
of hyperthyroidism and other causes of thyrotoxicosis.
Thyroid 2016;26:1343-421. Van den Berghe G. On the neuroendocrinopathy of critical ill-
ness. perspectives for feeding and novel treatments. Am J
Rushworth RL, Torpy DJ, Falhammar H. Adrenal crisis. Respir Crit Care Med 2016;194:1337-48.
N Engl J Med 2019;381:852-61.
Van Den Berghe G, Wouters P, Weekers F, et al. Intensive
Sakharova OV, Inzucchi SE. Endocrine assessments during insulin therapy in critically ill patients. N Engl J Med
critical illness. Crit Care Clin 2007;23:467-90. 2001;345:1359-67.

Sapolsky RM, Romero LM, Munck AU. How do glucocorti- Venkatesh B, Finfer S, Cohen J, et al. Adjunctive glucocorti-
coids influence stress responses? integrating permissive, coid therapy in patients with septic shock. N Engl J Med
suppressive, stimulatory, and preparative actions. Endocr 2018;378:797-808.
Rev 2000;21:55-89.
Verbalis JG, Greenberg A, Burst V, et al. Diagnosing and
Satoh T, Isozaki O, Suzuki A, et al. 2016 guidelines for the treating the syndrome of inappropriate antidiuretic hor-
management of thyroid storm from the Japan Thyroid mone secretion. Am J Med 2016;129:537.e9-23.
Association and Japan Endocrine Society (first edition).
Endocr J 2016;63:1025-64. Wiersinga WM. Myxedema and coma (severe hypothyroid-
ism) [monograph on the Internet]. In: Feingold KR, Anawalt
Selye H. A syndrome produced by diverse nocuous agents. B, Boyce A, et al., eds. Endotext [Internet], 2018 April 25.
1936 [classical article]. J Neuropsychiatry Clin Neurosci South Dartmouth, MA: MDText.com, 2000-2020. Available
1998;10:230-1. at www.endotext.org.

Shepshelovich D, Schechter A, Calvarysky B, et al. Yamada T, Shojima N, Noma H, et al. Glycemic control,
Medication-induced SIADH: distribution and characteriza- mortality, and hypoglycemia in critically ill patients: a sys-
tion according to medication class. Br J Clin Pharmacol tematic review and network meta-analysis of randomized
2017;83:1801-7. controlled trials. Intensive Care Med 2017;43:1-15.

Song JH, Kim JH, Lee SM, et al. Prognostic implication of Yeh Y-C, Chiu C-T. ADRENAL trial versus APROCCHSS
adrenocortical response during the course of critical ill- trial: to steroid or not to steroid? J Emerg Crit Care Med
ness. Acute Crit Care 2019;34:38-45. 2018;2:44-44.

Sood L, Sterns RH, Hix JK, et al. Hypertonic saline and des-
mopressin: a simple strategy for safe correction of severe
hyponatremia. Am J Kidney Dis 2013;61:571-8.

Spasovski G, Vanholder R, Allolio B, et al.; Hyponatraemia


Guideline Development Group. Clinical practice guideline
on diagnosis and treatment of hyponatraemia.
Eur J Endocrinol 2014;170:G1-47.

CCSAP 2021 Book 1 • Pulmonary and Endocrinology 133 Critical Care Endocrinology
Self-Assessment Questions
1. A 60-year-old man is transferred to your ICU with sep- A. Add to decrease ICU mortality in this patient.
tic shock secondary to gram-negative rod bacteremia B. Add to decrease time on the mechanical ventilator in
and acute respiratory failure on norepinephrine 0.7 this patient.
mcg/kg/min and vasopressin 0.03 units/min. After deter- C. Add to decrease time to shock resolution in this
mining his medical history includes hypothyroidism, patient.
which one of the following laboratory values is best to D. Do not add given concern for increased incidence of
obtain to assess this patient’s current thyroidal status? superinfection.
A. Triiodothyronine 5. A 71-year-old man is admitted to the medical ICU with
B. Total thyroxine acute hypoxia and adrenal crisis. His medical history is
C. Thyroid-stimulating hormone significant for congestive heart failure and secondary
D. Thyroxin-blinding globulin adrenal insufficiency after previous transsphenoidal pitu-
itary surgery 10 years ago. The patient reportedly missed
2. A 26-year-old woman (weight 69 kg) is admitted with
a week of his corticosteroid therapy because he could
lethargy, nausea, and vomiting. In the ED, she was tachy-
not drive to his home pharmacy. He receives 1000 mL of
cardic (heart rate, 127 beats/minute), and hypotensive
crystalloid fluid en route by emergency medical services.
(blood pressure, 91/41 mm Hg). A random cortisol was
Which one of the following is best to recommend in the
collected and found to be 4 mcg/dL. The patient has
early management of this patient?
received 2.5 L of crystalloid fluid and is currently on
norepinephrine 0.5 mcg/kg/min when the ICU team is A. Hydrocortisone 100 mg IV
consulted for admission. Which one of the following B. Crystalloid fluid 30 mL/kg
hydrocortisone doses is best to recommend for this C. Norepinephrine 0.02 mcg/kg/min
patient? D. Ceftriaxone 2 g IV
A. 200 mg IV continuous infusion over 24 hours 6. A 54-year-old woman is admitted with new-onset sepsis
B. 100 mg IV bolus and acute respiratory distress syndrome secondary to
C. 50 mg IV every 6 hours suspected community-acquired pneumonia versus influ-
D. 50 mg IV every 6 hours + fludrocortisone enza A. Which one of the following is best to recommend
regarding the use of corticosteroids in this patient?
3. A 38-year-old man with a medical history significant
for tuberculosis (10 years ago) is now stable in your A. Methylprednisolone 2 mg/kg/day IV
ICU after a motor vehicle crash resulting in polytrauma, B. Hydrocortisone 200 mg/day IV
including traumatic brain injury. Overnight, the patient C. Prednisone 40 mg/day orally
experiences acute hypoxia resulting in pulseless electri- D. No corticosteroids
cal activity cardiac arrest. Which one of the following is
7. Given the recent price increase of vasopressin, you are
best to recommend regarding the use of corticosteroids
charged with developing guidelines for its use within
in this patient?
your Neuro ICU. Given the incidence of DI in your patient
A. Administer corticosteroids in the setting of population, which one of the following is best to include
hemodynamic compromise in trauma. in development of such guidelines?
B. Do not administer corticosteroids for cardiac arrest.
A. The development of DI during vasopressin infusion
C. When considering corticosteroids, this patient is
should be treated with infusion discontinuation.
most likely to have primary AI.
B. A precaution that the discontinuation of vasopressin
D. When considering corticosteroids, this patient is
may result in the development of DI.
most likely to have secondary AI.
C. Prohibit use of vasopressin in patients presenting
4. A 54-year-old woman is admitted to your ICU with septic with severe hypernatremia.
shock secondary to E. coli bacteremia and hydronephro- D. Prohibit use of vasopressin in patients presenting
sis post-ureter stent placement complicated by acute with central DI.
respiratory failure, resulting in need for mechanical
8. A 49-year-old man with a history of hyperthyroidism
ventilation. During patient care rounds on day 2 of ICU
and congestive heart failure presents with fever and an
stay, you notice she is current receiving 0.9 mcg/kg/min
altered mental status. The patient was recommended to
of norepinephrine and 0.03 units/min of vasopressin.
receive thyroidectomy; however, he cancelled his sur-
Which one of the following is best to recommend regard-
gery secondary to an ongoing pandemic. His free T4 is
ing the use of corticosteroids in this patient?

CCSAP 2021 Book 1 • Pulmonary and Endocrinology 134 Critical Care Endocrinology
9.2 ng/dL and TSH is 0.02 mIU/L. Which one of the fol- A. 50-year-old man presenting post-cardiothoracic
lowing is best to recommend for this patient? procedure
A. Beta blocker to decrease hormone synthesis B. A 22-year-old woman in septic shock with a history
B. Corticosteroids to inhibit deiodinases of insulin-dependent diabetes
C. Potassium iodide to stop T3 conversion and C. A 45-year-old man with a history of atrial fibrillation
production of T4 presenting with acute stroke
D. Propylthiouracil to stop T3 conversion D. A 65-year-old man with a history of BPH presenting
after a motor vehicle collision and open right
9. A 37-year-old woman is admitted to your ICU after a neigh-
tibia/fibula fracture
bor called for a wellness check and the patient was found
down, unconscious. Upon arrival, the patient is noted 13. A 73-year-old man has a medical history of insulin-depen-
to have a temperature of 31.4°C, with a heart rate of 42  dent diabetes on insulin glargine 20 units daily and aspart
beats/minute and mean arterial pressure of 53 mm Hg. sliding scale, hypertension, hyperlipidemia, and degenera-
The team is concerned for septic shock versus myx- tive disc disease. The patient presents with septic shock
edema coma in the setting of unknown medical history. secondary to gram-negative rod bacteremia. He is found to
Broad- spectrum anti-infectives are started, as well as have a glucose of 274 mg/dL on presentation. The patient
hydrocortisone, and levothyroxine IV. Which one of the is found to have an acute kidney injury with an SCr of 1.2
following is best to obtain before the initiation of this mg/dL (baseline 0.6 mg/dL). Which one of the following
patient’s steroid therapy? recommendations for management of this patient’s hyper-
A. Serum cortisol glycemia best aligns with current guidelines?
B. Blood cultures A. Insulin IV infusion to goal 140-180 mg/dL
C. Thyroid panel B. Insulin glargine 10 units daily SQ plus insulin sliding
D. Serum electrolytes
scale q6h to goal 140-180 mg/dL
10. A 45-year-man with a history of hyperthyroidism pres- C. Insulin IV infusion to goal 180-220 mg/dL
ents in thyroid storm, diagnosed via a free T4 is 8.4 ng/dL D. Insulin glargine 10 units daily SQ plus insulin sliding
and an undetectable TSH. Which one of the following is scale q6h to goal 180-220 mg/dL
best to obtain during the management of confirmed thy-
roid storm to monitor pharmacotherapy in this patient? Questions 14 and 15 pertain to the following case.

A. Basic metabolic panel F.G. is a 45-year-old woman originally admitted with acute
B. Complete blood count respiratory distress syndrome. She is now day 22 of ICU
C. Pulmonary arterial catheter results stay status-post tracheostomy and septic shock secondary
D. Urine electrolytes to line-related bloodstream infection. F.G. remains somno-
lent and remains on low-dose norepinephrine infusion 0.04
11. You receive an email from your medication supply
mcg/kg/min despite multiple attempts to wean.
administrator that the hospital has an impending short-
age of regular insulin for intravenous use, warranting a 14. Which one of the following is most likely contributing to
re-evaluation of your current insulin protocol. Your cur- F.G.’s current clinical status?
rent guidelines recommend the initiation of an insulin
A. Elevated TSH
infusion to target a blood glucose of 140 mg/dL for all
B. Decreased T4
ICU patients. Regarding clinical outcomes with glycemic
C. Increased reverse T3
changes in the ICU, which one of the following modifica-
D. Elevated TRH
tions is most pertinent?
15. F.G.’s care team collects a serum cortisol with morn-
A. Loosen goals for diabetics because mortality risk is
ing labs given concern for adrenal insufficiency which
lower with higher glucose targets.
returns at 20 mg/dL. Which one of the following best
B. Loosen goals for non-diabetics because mortality
risk is lower with higher glucose targets. assesses F.G.’s cortisol value?
C. Tighten goals for diabetics because mortality risk is A. Increased ACTH is likely contributing to increases in
lower with lower glucose targets. plasma cortisol.
D. Tighten goals for non-diabetics because mortality B. Return of cortisol blinding globulin levels to normal
risk is lower with lower glucose targets. has likely increased accuracy of total cortisol over
12. Which one of the following patients is most likely to war- free cortisol values.
rant glycemic goal personalization to achieve higher C. Decreased feedback to the HPT axis may contribute
goals within the typical range of 140-180 mg/dL in to higher plasma cortisol levels.
comparison to the general ICU population? D. Increased cortisol half-life is likely present
secondary to decreased clearance.

CCSAP 2021 Book 1 • Pulmonary and Endocrinology 135 Critical Care Endocrinology
Learner Chapter Evaluation: Endocrinology in the ICU

As you take the posttest for this chapter, also evaluate the 8. The teaching and learning methods used in the chapter
material’s quality and usefulness, as well as the achievement were effective.
of learning objectives. Rate each item using this 5-point scale: 9. The active learning methods used in the chapter were
effective.
• Strongly agree
10. The learning assessment activities used in the chapter
• Agree
were effective.
• Neutral
• Disagree 11. The chapter was effective overall.
• Strongly disagree 12. The activity met the stated learning objectives.
13. If any objectives were not met, please list them here.
1. The content of the chapter met my educational needs.
2. The content of the chapter satisfied my expectations.
OTHER COMMENTS
3. The author presented the chapter content effectively.
14. Please provide any specific comments related to any
4. The content of the chapter was relevant to my practice
perceptions of bias, promotion, or advertisement of
and presented at the appropriate depth and scope.
commercial products.
5. The content of the chapter was objective and balanced.
15. Please expand on any of your above responses, and/or
6. The content of the chapter is free of bias, promotion, and provide any additional comments regarding this chapter:
advertisement of commercial products.
7. The content of the chapter was useful to me.

CCSAP 2021 Book 1 • Pulmonary and Endocrinology 136 Critical Care Endocrinology
Immune-Mediated Reactions
By Abby M. Bailey, Pharm.D., BCCCP; and Jaclyn M. Stoffel, Pharm.D., BCPS

Reviewed by Kyle A. Weant, Pharm.D., FCCP, BCPS, BCCCP; and Heather Johnson, Pharm.D., MS, BCCP, BCCCP

LEARNING OBJECTIVES

1. Evaluate critically ill patients for the immunology and pathophysiology of immune-mediated reactions.
2. Develop evidence-based recommendations for the management of refractory hypotension in the setting of anaphylactic
shock.
3. Distinguish differences in the presentation of dermatologic emergencies, including toxic epidermal necrolysis and
Stevens-Johnson syndrome.
4. Develop a treatment and monitoring plan for therapy in the setting of a dermatologic emergency.
5. Distinguish differences in etiology and pathophysiology between types of angioedema.
6. Design a pharmacologic treatment plan for management of acute angioedema.

INTRODUCTION
ABBREVIATIONS IN THIS CHAPTER
This chapter focuses on the treatment of immune-mediated reac-
ACE Angiotensin-converting enzyme
tions in critically ill patients. Therapeutic advances are presented
EMM Erythema multiforme
for each content area with an emphasis on how pharmacists can
FFP Fresh frozen plasma
develop and recommend safe and effective treatment options for this
HAE Hereditary angioedema
patient population.
Ig Immunoglobulin
IL Interleukin
Immune-Mediated Reactions in a Critically Ill
IVIG Intravenous immune globulin Population
rhC1-INH Recombinant human C1 inhibitor The life-threatening disorders discussed in this chapter are anaphy-
SCORTEN Score of Toxic Epidermal Necrosis laxis, dermatologic emergencies, and angioedema. These disorders
SJS Stevens-Johnson syndrome can be triggered by allergic (IgE) or nonallergic (non-IgE-mediated)
TEN Toxic epidermal necrolysis mechanisms (LoVerde 2018). Disorders that involve the airway can
TNF Tumor necrosis factor lead to fatal airway obstruction, resulting in cardiorespiratory col-
TXA Tranexamic acid lapse, or shock, requiring management in the ICU.

Table of other common abbreviations.


IgE-Mediated Reactions
A variety of reactions can occur minutes to hours after exposure to
an antigen. Antigens may be a drug therapy, an environmental trigger,
or iatrogenic. These hypersensitivity reactions may be characterized
as immunogenic (IgE-mediated) or nonimmunologic (non-IgE-medi-
ated). The IgE-mediated reactions tend to have a rapid onset, usually
minutes after exposure (Stone 2014).

Immunology and Pathophysiology of Anaphylaxis


Data for the pathophysiologic mechanisms of anaphylaxis are mostly
limited to animal models, but a few human studies exist. Anaphylaxis
is an immediate hypersensitivity reaction, mediated predominantly
by IgE. The result is the release of mast cells and basophil mediators

CCSAP 2021 Book 1 • Pulmonary and Endocrinology 137 Immune-Mediated Reactions


(LoVerde 2018). Mouse models have also identified the poten- Researchers have also sought to identify whether patients
tial for IgG-dependent anaphylaxis. In addition to IgE, the who present with delayed anaphylaxis have a distinct patho-
complement cascade as well as the anaphylatoxins (C3a, physiology compared with those who present following an
C4a, C5a), tryptase, and neutrophils are known to play a role immediate reaction. A prospective study in Australia noted
in the instigation and propagation of anaphylactic reactions that delayed responses accounted for 55 of 315 (17%) of the
(Shaker 2020). Several studies have sought to differentiate anaphylaxis cases. These patients had elevated concentra-
the mediators in severe versus less severe presentations of tions of histamine, tryptase, IL-6, IL-10, and TNF-receptor 1
anaphylaxis. In a prospective study in emergency department based on the values obtained on emergency department
patients with anaphylaxis that caused hypotension, the medi- arrival, 1 hour later, and at discharge. These five mediators
ators were tryptase, as well as histamine, IL-6, IL-10, and TNF are the same are those correlated with severe anaphylaxis
receptor-1. Platelet-activating factor is a lipid-derived media- (Brown 2013).
tor that has significance in mouse models of anaphylaxis, but
it has not produced mast cell degranulation in human studies ANAPHYLAXIS
(Brown 2013, Stone 2009). Anaphylaxis is an acute, life-threatening systemic allergic
Mast cells are the single most important type of effec- reaction with a wide range of clinical manifestations, for
tor cells involved in anaphylaxis pathophysiology. However, which many patients require intensive care. The estimated
some models also implicate basophils and neutrophils, lifetime prevalence is 1.6%–5.1% (Lieberman 2006, Shaker
monocytes and macrophages, and platelets (Reber 2017). 2020, Wood 2014). The overall prevalence of fatal anaphylaxis
Mast cells and basophils have high-affinity receptors for IgE. is between 0.47–0.69 cases per million persons (Jerschow
Mediator release from mast cells is a result of IgE receptor 2014, Liew 2009, Turner 2015). Stinging insects are a lead-
cross-linking on the surface of mast cells by antigen and ing trigger in a combined population of adult and pediatric
anti-IgE. One of the several substances that can activate this patients, whereas medications are a leading trigger in adults
process is histamine, a well described mediator of anaphy- and foods are a leading trigger in children. More importantly,
laxis that produces hallmark symptoms of flushing, airway adverse drug reactions affect up to 20% of hospitalized
edema, and hypotension. However, histamine is not the only patients (Grabenhenrich 2016, Liew 2013).
identified mediator of anaphylaxis, which informs the appro-
priate approach to treatment modalities as described later in Diagnostic Considerations
text (Marone 1989). Anaphylaxis has a diverse range of clinical manifestations,
making specific diagnostic criteria elusive and resulting in
poor sensitivity for the available confirmatory tests. These
factors make the overall management for anaphylaxis
BASELINE KNOWLEDGE STATEMENTS inconsistent and lead to underdiagnosis. Clinicians are chal-
lenged to accurately and rapidly identify this life-threatening
Readers of this chapter are presumed to be familiar
immune-mediated reaction to operationalize effective treat-
with the following:
ment. The Food Allergy and Anaphylaxis Network (FAAN)
• General knowledge of the pathophysiology of diagnostic criteria to diagnose anaphylaxis were published in
anaphylaxis
2006 (Box 1).
• Drug knowledge of the various parenteral agents
used to treat allergic reactions, including the onset
Distinguishing Anaphylaxis From Other Shock
and duration of antihistamines and the dosing and
States
onset of corticosteroids
Anaphylaxis is distinguished from other forms of shock by
• General knowledge of topical agents used in the the mediators that instigate the cascade of inflammation and
management of burns
vasodilation. The most important cells implicated in ana-
Table of common laboratory reference values. phylaxis are mast cells, basophils, neutrophils, monocytes,
macrophages, and platelets. Histamine remains a primary
ADDITIONAL READINGS mediator that affects mast cells. Degranulation of mast
cells can produce the characteristic cutaneous manifesta-
The following free resources have additional back- tions commonly associated with anaphylaxis, such as rash,
ground information on this topic: urticaria, and pruritus. More importantly, anaphylaxis is also
• NIH. Genetic and Rare Diseases Information Center. distinguished from other forms of shock in that the onset of
Hereditary Angiodema. anaphylaxis is often far more rapid compared with cardio-
• American Academy of Allergy, Asthma, and genic, distributive, and hypovolemic shock. Both distributive
Immunology. Anaphlaxis.
and hypovolemic shock result in decreased venous return
because of intravascular volume depletion; to compensate,

CCSAP 2021 Book 1 • Pulmonary and Endocrinology 138 Immune-Mediated Reactions


and microischemia, ultimately causing systolic and diastolic
Box 1. Food Allergy and Anaphylaxis
dysfunction.
Network Diagnostic Criteria for
Anaphylaxis Patterns of Presentation for Anaphylaxis
Anaphylaxis is likely when 1 of 3 criteria are fulfilled: The three patterns of anaphylaxis are uniphasic, biphasic,
1. Acute onset of an illness (minutes to hours) with involve- and protracted. Uniphasic anaphylaxis peaks close to 60 min-
ment of the skin, mucosal tissue, or both with either utes after initiation and then resolves over the next hour, with
respiratory involvement or reduced blood pressure and/or
no recurrence. Most cases (70%–90%) of anaphylaxis are uni-
associated symptoms of end-organ dysfunction
2. ≥2 of the following that occur rapidly after exposure to a phasic. Biphasic anaphylaxis is an episode that recurs without
likely allergen: repeated exposure to the trigger. The timeframe for symptom
• Involvement of skin-mucosal tissue recurrence ranges from 1–72 hours after resolution. The inci-
• Respiratory involvement dence of biphasic anaphylaxis is highly variable, but it may
• Reduced blood pressure or associated symptoms occur in up to 20% of patients. Protracted anaphylaxis per-
• Gastrointestinal symptoms sists for days to weeks, but this pattern of anaphylaxis is rare
3. Reduced blood pressure as a result of exposure to a known
allergen trigger (Alqurashi 2015, Grunau 2015, LoVerde 2018, Zisa 2009).

Information from: Shaker M, Wallace D, Golden D. Anaphylaxis-a


2020 practice parameter update, systematic review, and grading Classification of Anaphylaxis
of recommendations, assessment, development and evaluation Anaphylactic reactions are classified as immune-mediated
(GRADE) analysis. J Allergy Clin Immunol 2020;145:1082-123;
LoVerde D, Eginli A, Krishnaswarmy G. Anaphylaxis. Chest
(both IgE-mediated and non-IgE-mediated), nonimmunologic,
2018;153:528-43. and idiopathic. IgE-mediated reactions are the most common
and include food-, venom-, and medication-triggered reac-
tions. Non-IgE-mediated reactions may include reactions to
the heart rate typically increases to preserve cardiac out- intravenous immunoglobulins and dialysis membranes; these
put. In contrast, myocardial depression and bradycardia are reactions are thought to be facilitated by the complement
more common features in anaphylactic shock. This manifes- cascade. Nonimmunologic reactions are a result of direct mast
tation is not completely understood, but it is thought to be cell degranulation and can include such triggers as exercise
a complex response to mast cell degranulation that results and temperature. Idiopathic reactions account for 30-60% of
in decreased intravascular volume, coronary vasospasm, cases and have no obvious trigger (LoVerde 2018). (Table 1).

Table 1. Classification of Anaphylaxis Based on Mechanism

Mechanism Triggers and Most Common Examples

Immune-mediated (IgE) Food allergens—milk, eggs, soy, wheat, tree nuts, peanuts, shellfish
Latex
Airborne allergens—pollen, aerosolized food, animal dander
Hymenoptera envenomation—honey bee, wasp, hornet
Medication allergens
Radiocontrast media

Immune-mediated (non-IgE) Intravenous immunoglobulin


Aspirin or nonsteroidal anti-inflammatory drugs
Dialysis membranes
Dextran
Heparin

Nonimmunologic Exercise
Physical triggers—heat, cold, sunlight

Idiopathic Differential diagnosis should include other causes of anaphylaxis as well as respiratory
conditions such as asthma

Information from: Shaker M, Wallace D, Golden D. Anaphylaxis—a 2020 practice parameter update, systematic review, and grading of
recommendations, assessment, development and evaluation (GRADE) analysis. J Allergy Clin Immunol 2020;145:1082-123; LoVerde
D, Eginli A, Krishnaswarmy G. Anaphylaxis. Chest 2018;153:528-43.

CCSAP 2021 Book 1 • Pulmonary and Endocrinology 139 Immune-Mediated Reactions


Treatment Antihistamines and corticosteroids are considered second-
Epinephrine is the cornerstone of treatment, and delayed line agents in the management of anaphylaxis because of
epinephrine has been associated with an increased risk of their relatively delayed onset and inability to stabilize or pre-
fatality. This drug treats all of the symptoms associated with vent mast cell degranulation. In addition, neither agent can
anaphylaxis as well as prevents progression or intensification target additional mediators of anaphylaxis, including those
of symptom severity. Epinephrine is a nonselective agonist that may cause cardiovascular or respiratory symptoms.
of the adrenergic receptors, and it reverses cardiovascular Antihistamines, however, can provide symptomatic relief
instability, bronchoconstriction, and airway mucosal edema. of cutaneous manifestations, including urticaria, and pruri-
Epinephrine also prevents further release of histamine and tis. Leukotriene receptor antagonists, such as montelukast,
other mediators by activation of β-2 receptors. The dose is do not have a role in the acute management of anaphylaxis,
0.01 mg/kg to a maximum of 0.5 mg in adults. To prepare despite working to stabilize mast cell degranulation. However,
a dose, a concentration of 1 mg/mL epinephrine is used to these agents have a role in preventing the occurrence of exer-
limit the volume of the intramuscular injection. Commercially cise-induced anaphylaxis when used in combination with
available autoinjectors deliver doses of 0.15 mg for pediatric daily antihistamines. In addition, desensitization protocols
patients or 0.3 mg for adults or children weighing more than use montelukast as part of the strategy to stabilize mast cells
30 kg (66 lb). from degranulation before introduction of the suspected
Study investigators continue to advocate for alternative trigger.
delivery routes for epinephrine. Currently accepted routes Steroids and antihistamines have been administered
for emergency treatment are subcutaneous, intramuscular, using several routes, types, doses, and durations. The
and intravenous. No specific comparator studies have eval- intravenous route of administration for management of ana-
uated which route of administration is most efficacious for phylaxis is advantageous, particularly if airway or tongue
treatment of anaphylaxis. In studies specifically evaluat- swelling is noted. If corticosteroids are administered, rea-
ing intramuscular administration of epinephrine in healthy sonable regimens include methylprednisolone (1–2 mg/
volunteers, this route produced rapid and high plasma concen- kg), hydrocortisone (100 mg), or dexamethasone (10 mg)
trations of epinephrine. Experts advocate for intramuscular (Grunau 2015, Liyanage 2017, Simons 2011, 2013, 2015).
administration as a first-line approach. The preference for Antihistamines targeting the histamine 1 (H1) receptor are
intramuscular administration over subcutaneous adminis- the most commonly used in the course of treating anaphy-
tration additionally stems from the concerns that distributive laxis. Assuming symptoms have resolved with the use of
shock alters peripheral absorption and pharmacokinetics epinephrine, evidence is insufficient that either antihista-
as seen in healthy volunteers. Intramuscular injection into mines or corticosteroids should be empirically used in the
the anterolateral thigh also avoids accidental injection into prevention of a biphasic anaphylactic reaction. The Joint
the osseous space in the deltoid region. Intravenous admin- Task Force of Practice Parameters (JTFPP) of the American
istration increases the risk of arrhythmias and adverse College of Allergy, Asthma and Immunology Epidemiology of
cardiovascular outcomes, and therefore is not recommended Anaphylaxis Working Group conducted an analysis of data
as a first-line route. Instead, intravenous administration published on the use of antihistamines and corticosteroids to
should be reserved for cases in which intramuscular admin- prevent either biphasic or immune-mediated drug reactions
istration has failed. Epinephrine can be administered as an in the setting of anaphylaxis (Lieberman 2006). The number
intravenous bolus (0.1 mg), followed by a continuous infusion needed to treat to prevent 1 episode of biphasic anaphylaxis
titrated to mean arterial pressure and oxygenation. The 0.1- was 72 with H1 antihistamines and 161 with glucocorticoids.
mg/mL concentration should always be used for intravenous However, the JTFPP analysis found benefit in using both
administration. drugs in premedication protocols for chemotherapy to pre-
Despite the evidence being clear that epinephrine is the vent sensitivity reactions.
preferred treatment for patients with anaphylaxis, it remains
underused. Epinephrine is available in several vial sizes and Monitoring
product formulations; however, variances in inpatient and In patients without severe features of anaphylaxis, an obser-
outpatient management make patient and provider education vation period of 1 hour is reasonable assuming they are
and operationalization difficult across a health system. In a asymptomatic after initial management strategies have
report of the pitfalls of epinephrine administration in anaphy- been used. Patients who have a higher risk of a biphasic
laxis, uncertainty of when or how to administer epinephrine, reaction or risk factors for anaphylaxis fatality should have
accidental injury, and dosing concerns were commonly cited an observation period of at least 6 hours. The ability to pre-
by either patients or providers (Cohen 2017). Pharmacists are dict biphasic anaphylaxis is inconsistent. However, the
uniquely poised to help increase the early and appropriate clinician can evaluate risk factors for biphasic anaphylaxis to
use of epinephrine as well as to mitigate medication errors in determine an appropriate observation period. Biphasic ana-
the execution of a pharmacotherapy plan. phylaxis is most commonly associated with a more severe

CCSAP 2021 Book 1 • Pulmonary and Endocrinology 140 Immune-Mediated Reactions


initial presentation (odds ratio ([OR], 2.11; 95% CI, 1.23–3.61)
Box 2. Causative Agents Associated
or repeated epinephrine doses (more than 1 dose) required
with Stevens-Johnson Syndrome or
on initial presentation (OR 4.82; 95% CI, 2.70–8.58). Patients
Toxic Epidermal Necrolysisa
at risk of anaphylaxis fatality include those with previously
diagnosed cardiovascular comorbidities, lack of access to High Risk (>50% of cases) Lower Risk (<50% of cases)
Sulfa antibiotics Aminopenicillins
an epinephrine autoinjector, or limited access to emergency
Carbamazepine Cephalosporins
medical services. Phenobarbital Corticosteroids
Phenytoin Acetaminophen
Oxicam NSAIDs Acetic-acid NSAIDs
SEVERE DERMATOLOGIC Allopurinol Macrolides
EMERGENCIES Lamotrigine Quinolones
Nevirapine Tetracyclines
Dermatologic emergencies encompass myriad conditions Sulfasalazine Tramadol
that can span from burns (thermal, electrical, or chemical)
to adverse cutaneous drug reactions. Drug hypersensitiv- Items listed in decreasing order of risk.
a

ity reactions are further subdivided into several types (see NSAIDs = nonsteroidal anti-inflammatory drugs.
Table 1). Most present as generalized maculopapular ery-
thema, which is mild and self-limiting after withdrawal of the
trigger. Few patients present with a severe drug reaction; EMM are male rather than female, and these patients more
however, these reactions are life-threatening and consist of often have few, if any, systemic symptoms (Alqurashi 2015).
distinct clinical entities with their own characteristic cuta-
neous presentations, causative drugs, clinical courses, and Epidemiology
possible treatment modalities (Cho 2017). Certain medica- The first publication describing a severe, cutaneous reaction
tions or classes of medications have been characterized as appeared in 1922, written by Drs. Albert Stevens and Frank
conferring a higher risk of adverse cutaneous drug reactions Johnson wherein they describe the case of two children
(Table 2, Box 2). who presented with whole-body cutaneous lesions involving
Erythema multiforme (sometimes referred to as erythema the eyes and mucous membranes. Not until 1993 were TEN
multiforme major or EMM), differs from SJS and TEN in sev- and SJS formally recognized as a clinical diagnosis EMM
eral ways. Namely, the lesions in EMM are typically localized (Bastuji-Garin 1993). The overall combined incidence of SJS,
versus diffuse and will not coalesce. Also, more patients with SJS/TEN overlap, and TEN is estimated to be 2 to 7 per million

Table 2. Adverse Cutaneous Drug Reactions

Drug Reaction Morphology and Physical Characteristics Symptom Onset

Erythema multiforme (EMM) Small papules on extremities evolve to a classic target appearance 1–2 wk
with concentric rings of erythema; typically <10% TBSA

Stevens-Johnson syndrome (SJS) Blistering on trunk and face to >10% TBSA with associated dermal 1–8 wk
desquamation

Toxic epidermal necrolysis (TEN) Desquamation >30% TBSA 1–8 wk

Drug reaction with eosinophilia Fever, internal organ involvement (liver, kidneys, hematologic system), 3–8 wk
and systemic symptoms (DRESS) and rash that varies in presentation from erythematous scaly rash to
a blistering/bullous rash (similar to SJS)

Acute generalized exanthematous Nonfollicular sterile pustules with diffuse, edematous erythema, 1–5 days
pustulosis (AGEP) irregularly dispersed or grouped, usually starting in skinfolds or on
the face

Generalized bullous fixed drug Well demarcated red (or even brown) areas that may be accompanied 1–24 hr
eruptions (GBFDE) by bullae

TBSA = total body surface area.


Information from: Sullivan JR, Shear NH. The drug hypersensitivity syndrome: what is the pathogenesis? Arch Dermatol 2001;137-57;
Wolkenstein P, Revuz J. Drug-induced severe skin reactions—incidence, management and prevention. Drug Saf 1995;13:56.

CCSAP 2021 Book 1 • Pulmonary and Endocrinology 141 Immune-Mediated Reactions


cases/year. The incidence of SJS is 1.2 to 6 per million cases/ leukocyte antigen-B12 antigen in patients with TEN (Table 4).
year, outnumbering TEN by about 3-fold. However, TEN occurs Anecdotal evidence suggests that SJS/TEN is associated
in all age groups and is more often found in women and older with an impaired capacity to detoxify drug metabolites. This
adult patients (Schwartz 2013a). impairment is thought to be initiated by an immune response
to an antigenic complex formed by the reaction of such
Pathophysiology metabolites with host tissues (French 2006, Schwartz 2013a).
The exact pathophysiologic mechanism involved in these Cytotoxic T cells are also known to mediate cutaneous
reactions is not yet elucidated. The immune system is a del- lesions associated with SJS/TEN. These T cells are likely to
icate interplay of innate and adaptive pathways, sometimes be drug-specific cytotoxic T cells given that medications are
resulting in unanticipated consequences. Coombs and Gell implicated in more than 80% of cutaneous reactions. Other
proposed classification of immune-mediated drug reactions important cytokines such as IL-6, TNF-α, interferon γ, IL-18,
into four (I–IV) categories (Pichler 2010). This system relies and Fas ligand are also present in the epidermis and blister
on the formation of IgE antibodies that bind to high-affinity fluid of patients with TEN. The constitutional symptoms of
IgE receptors on mast cells (I) and basophilic leukocytes (II), TEN may be explained by the actions of these inflammatory
on complement-fixing antibodies (III), and in T-cell reactions mediators. The idea that SJS/TEN is mediated by a cytotoxic
(IV). The T cells have heterogeneous functions that conclude T cell pathway is also supported by the interval between the
in different manifestations of inflammation. Therefore, the onset of drug exposure and symptom onset. The time between
fourth type of immune-mediated reaction is subdivided into exposure and the onset of cutaneous lesions may be 1–3
IVa, IVb, IVc, and IVd (Table 3). weeks, suggesting that a period of sensitization occurs first
Genetic variation is known to play a role in specific cases and then continued or repeat exposure. This interval period
of SJS/TEN, such as the increased incidence of the human (referred to as memory) will be shortened in patients who are

Table 3. Gell–Coombs Classification of Hypersensitivity Reactions

Type Type 1 Type II Type IIIa Type IVa Type IVb Type IVc Type IVd

Immune IgE IgG/IgM IgG/IgM TNF-α, TH1 TH2 cells CTL GM-CSF
reactant cells

Onset of Seconds– Minutes–hours 3–12 hours Days–weeks


reaction minutes

Effector Mast cell Phagocytes Complement Monocyte Eosinophil CD4+ T cells Neutrophils
cell NK cells Macro- CD8+ T cells
phage

Clinical Urticaria/ Petechiae Vasculitis, Contact Chronic allergic Contact AGEP,


symptoms angioedema of caused by urticaria, serum dermatitis rhinitis, dermatitis, Beçhet
skin/mucosa, thrombocyto- sickness maculopapular maculopapular disease,
edema of other penic purpura, exanthema and bullous SJS/TEN
organs, and drug-induced with exanthema
anaphylactic pemphigus eosinophilia
shock

Causative Antibiotics Penicillin, Immunoglobulins, Sulfamethoxazole


agent sulfonamides, antibiotics, phenytoin
examples quinidine, rituximab, carbamazepine
isoniazid infliximab, allopurinol
vaccines

a
Type III reaction is also known as the Arthus reaction.
AGEP = acute generalized exanthematous pustulosis; CTL = cytotoxic T-cell; GM-CSF = granulocyte-macrophage colony-stimulating
factor; Ig = immunoglobulin; NK = natural killer; SJS = Stevens-Johnson syndrome; TEN = toxic epidermal necrolysis; TH1 = T-helper 1
cell; TH2 = T-helper 2 cell; TNF = tumor necrosis factor.
Information from Rajan TV. The Gell-Coombs classification of hypersensitivity reactions: a re-interpretation. Trends Immunol 2003;
24:376-9; Coombs PR, Gell PG. Classification of allergic reactions responsible for clinical hypersensitivity and disease. In: Gell RR,
editor. Clinical Aspects of Immunology. Oxford: Oxford University Press, 1968:575-96.

CCSAP 2021 Book 1 • Pulmonary and Endocrinology 142 Immune-Mediated Reactions


Table 4. Genetic Polymorphisms Associated with Box 3. Score of Toxic Epidermal
Adverse Cutaneous Drug Reactions Necrosis (SCORTEN)
Criteria: scored as 1 point each
Drug Phenotype HLA Association • Age >40 years
• Presence of malignancy
Abacavir DRESS/SJS/TEN B*5701
• Heart rate >120 beats/minute
Allopurinol DRESS/SJS/TEN B*5801 • Epidermal detachment >10% total body surface area at
admission
Carbamazepine SJS/TEN B*1502 • Blood urea nitrogen >28 mg/dL
DRESS A*3101 • Serum glucose >250 mg/dL
• Bicarbonate <20 mEq/L
Lamotrigine SJS/TEN A*6801
Scoring: number of criteria = likelihood of mortality (%)
B*38
1 = 4% 5 = 85%
Nevirapine SJS/TEN B*3505 2 = 12% 6 = 95%
Phenytoin SJS/TEN B*1502 3 = 32% 7 = 99%
4 = 62%

DRESS = Drug rash with eosinophilia and systemic symp-


toms; HLA = human leukocyte antigen; SJS = Stevens-
in TEN usually results from sepsis or multiorgan failure. The
Johnson syndrome; TEN = toxic epidermal necrolysis.
SCORTEN is a validated scoring tool used to predict mortality
and should be calculated within 24 hours of diagnosis (Box 3).
The SCORTEN predicted mortality has been validated, but
re-exposed to that drug (French 2006). The relationship with
this tool may underestimate mortality in patients with respi-
all the potential cytokines culminates in keratinocyte apopto-
ratory involvement (Bastuji-Garin 2000, Guégan 2006, Hague
sis and epidermal necrolysis.
2007).

Clinical Course of Severe Mucocutaneous Potential Long-Term Sequela


Reactions
Recovering from SJS/TEN involves slow re-epithelialization
The most severe adverse cutaneous drug reactions are cer- of affected cutaneous lesions. Chronic changes can include
tainly SJS and TEN. In both conditions, a prodromal phase lasts alterations in pigmentation and nail deformities, as well as
between 1–4 days with the symptoms of fever, malaise, cough, ocular irregularities. Given that sloughing may extend into
rhinorrhea, and diffuse erythema. It is necessary to differenti- the esophagus and upper airway, serious complications and
ate SJS/TEN from less severe cutaneous reactions such as scarring are possible. Ocular involvement can be complicated
contact dermatitis, which is a localized reaction in an affected by pseudomembranes, synechiae formation, and permanent
area and lacks other cutaneous or respiratory symptoms. The corneal scarring.
localized reactions in contact dermatitis appear shortly after
exposure. In SJS/TEN, the prodromal phase typically begins
1–3 weeks after exposure to the trigger. The acute phase lasts ERYTHEMA MULTIFORME
2–3 weeks; during this phase the cutaneous lesions appear. Erythema multiforme is dichotomized into erythema mul-
Lesions first appear on the trunk and face, including the tiforme minor and erythema multiforme major or EMM, the
mucous membranes and eyes, and then spread to the extremi- difference being EMM involves the mucous membranes.
ties. The cutaneous lesions are coalescing, dusky red macules Overall, the surface area involvement and systemic inflam-
with purpuric centers that progress into bullae, leading to skin mation is milder compared with SJS/TEN and no sloughing
sloughing. The hallmark sign of SJS/TEN is a positive Nikolsky of the dermal layers occurs. Typically EMM occurs as an
sign, which is the separation of superficial epidermis when immune response to infection, and drugs are implicated in
light lateral pressure is applied. Mucosal involvement appears less than 10% of cases. Herpes simplex virus type 1 is com-
in most patients, most often presenting as stomatitis and con- monly implicated (Trayes 2019).
junctivitis (Eginli 2017, Gerull 2011).
Systemic symptoms follow the cutaneous manifestations Treatment and Monitoring
of disease. These symptoms can cause acute renal failure, Management of EMM is focused around treating the under-
pneumonitis, acute respiratory distress syndrome, bronchiol- lying infection or discontinuing the medication (Table 5).
itis obliterans, and hepatitis. The estimated mortality rate for Because of the mild, self-limiting, nature of the disease,
patients with SJS is 1%–3%. In contrast, mortality rates for management is largely symptomatic. Topical steroids or
TEN range from 30%–50%, depending on the degree of cuta- antihistamines can provide relief of cutaneous lesions. Oral
neous involvement as well as end-organ involvement. Death acyclovir can reduce the severity and duration of the EMM

CCSAP 2021 Book 1 • Pulmonary and Endocrinology 143 Immune-Mediated Reactions


Table 5. Treatment of Erythema Multiforme

Symptom Therapy Drugs and Doses

Associated herpes simplex Antiviral Acyclovir 400 mg 3 times daily for 5–10 days
virus infection

Painful mucosal lesions Oral analgesics Acetaminophen or nonsteroidal anti-inflammatory drugs.

Topical steroid Triamcinolone 0.1% oral paste: 1–2 times daily

Topical anesthetic solution Lidocaine 2% viscous solution 15 mL to swish and spit as needed
(maximum: 4.5 mg/kg)

Cutaneous lesions Topical corticosteroida Triamcinolone 0.5% (cream or ointment) 1–2 times daily

Topical antihistamine Diphenhydramine 1%–2% topical cream 3–4 times daily as needed

a
Appropriate selection based on location, lesion size, and desired steroid potency.

eruptions if associated with herpes simplex virus. When oral As with thermal injuries, fluid resuscitation is a cornerstone
mucosa is involved, treatment options include high-potency of supportive care. Predictive models for fluid supplementa-
topical corticosteroid paste and oral anesthetic solutions. tion in burn cases, such as the Parkland formula, overpredict
Severe cases of mucocutaneous involvement may cause the requirements for patients with SJS/TEN. Although epi-
decreased oral intake, which leads to hospitalization for dermal detachment occurs, replacement volumes are lower
administration of intravenous fluids and repletion of electro- because the severe hypoalbuminemia, capillary leak, and
lytes (Sokumbi Wetter 2012). sodium sequestration seen with thermal burn injuries are not
present in SJS/TEN. A study by Shiga and Cartotto of 21 TEN
patients with extensive epidermal loss recorded fluid require-
STEVENS-JOHNSON SYNDROME ments over the first 3 days of admission and determined 2 mL/
AND TOXIC EPIDERMAL NECROLYSIS kg body weight/% total body surface area epidermal detach-
General Management Considerations ment is the volume needed for the first 24 hours. Subsequent
Given the degree of cutaneous involvement, it is usually rec- fluids can be titrated to a urine output of 0.5–1 mL/kg/hour
ommended to transfer patients with SJS/TEN to a verified (Creamer 2016, Shiga 2010).
burn facility. Burn centers are equipped to handle patients Sloughing or denuded wounds should be covered with
with extensive epidermal damage, including ocular, gen- topical antibacterial agents. Wound management will help
ital, and pulmonary involvement. The mortality rate may reduce fluid and protein losses and improve pain control.
be reduced with early transfer to a burn unit. However, if a Both silver-containing agents as well as topical bacitracin
patient has nonprogressing disease, SCORTEN scores less are used in various burn centers. Most experts advise that
than 2, and an affected total body surface area of less than silver-containing compounds be used conservatively to avoid
30% (preferably less than 10%), a burn unit may not be neces- systemic absorption (Creamer 2016).
sary (Palmieri 2002, Revuz 1987, Schwartz 2013b).
Systemic Immunomodulating Therapies
Treatment Corticosteroids
Management of SJS/TEN is mostly analogous to treating Corticosteroids have been a mainstay in the treatment of SJS/
patients with severe thermal injuries. First, clinicians should TEN for decades. The proinflammatory disease pathology
obtain a very thorough and detailed history of drug exposure seems a natural target for the anti-inflammatory mechanism
in the previous 2 months and the trigger should be discontin- of steroids. Although no prospective studies have evaluated
ued if still an active medication. The survival of most patients steroids versus other therapies, many retrospective studies
hinges on good supportive care that includes wound manage- have investigated steroids as part of the standard of care
ment and fluids. The ambient room temperature should be (Table 6). A recent meta-analysis of systemic immune mod-
raised to reduce energy consumption and associated meta- ulating therapies in patients with SJS/TEN compared 96
bolic stress. Early nutrition is recommended given the energy studies and a total of 367 patients from 26 studies (30.4%)
expenditures above the baseline metabolic rate, which, for who were treated with corticosteroids. Of these studies, 11
example, is 120% of baseline in a patient with 80% skin loss of evaluated corticosteroids compared with standard of care.
the total body surface area. The combined point estimate showed a beneficial treatment

CCSAP 2021 Book 1 • Pulmonary and Endocrinology 144 Immune-Mediated Reactions


Table 6. Select Studies Evaluating Corticosteroids in Stevens-Johnson Syndrome and Toxic Epidermal Necrolysis
Compared With Standard of Care

Primary Author Year Design n Steroid Regimen Mortality

Dorafshar 2008 Retrospective 48 Not reported 5/48

Brand 2000 Retrospective 12 Not reported 4/12

Schneck (FR) 2008 Retrospective, case control 23 Variable, but 40–150 mg prednisone equivalents 28/159

Schneck (DE) 2008 Retrospective, case control 136 Variable, but 120–500 mg daily prednisone
equivalents

Chen 2010 Retrospective, case series 82 Hydrocortisone 100–700 mg once daily or 5/82
Methylprednisolone 40–80 mg once daily

Kim 2005 Retrospective, case series Methylprednisolone 250–1000 mg once daily 6/21

Ioannides 1994 Retrospective 18 Not reported 5/18

Kamaliah 1998 Retrospective 27 Not reported 2/27

Koh 2010 Retrospective 5 Prednisolone 0.5–1.5 mg/kg daily up to 1 month 0/5


or
Hydrocortisone 10–15 mg/kg divided daily for an
average of 4 days (range 2–6)

Léauté-Labrèze 2000 Retrospective 17 Prednisone 1 mg/kg daily decreasing over 1 week 0/17

Sethuraman 2012 Retrospective 20 Dexamethasone 3–8 mg daily 3/20


or
Prednisolone 30–40 mg daily

Yamane 2007 Retrospective 117 Prednisolone 10–600 mg daily 4/117

2005 Retrospective 28 Prednisolone 1 mg/kg once daily 2/28


Yip

Azfar 2010 Prospective, observational 40 Not reported 8/40

Information from: Azfar N, Zia M, Malik L, et al. Role of systemic steroids in the outcomes of Stevens-Johnson syndrome in toxic epi-
dermal necrolysis. J Pak Assoc Derm 2010,20:158-62; Brand R, Rohr JB. Toxic epidermal necrolysis in Western Australia. Australas J
Dermatol 2000;41:31-3; Chen J, Wang B, Zeng Y, et al. High-dose intravenous immunoglobulins in the treatment of Stevens-Johnson
syndrome and toxic epidermal necrolysis in Chinese patients: a retrospective study of 82 cases. Eur J Dermatol 2010;20:743-7;
Dorafshar AH, Dickie SR, Cohn AB, et al. Antishear therapy for toxic epidermal necrolysis: an alternative treatment approach. Plast
Reconstr Surg 2008;122:154-60; Ioannides D, Vakali G, Chrysomallis F, et al. Toxic epidermal necrolysis: a study of 22 cases. J Europ
Acad Derm Vener 1994;3:266-75; Kamaliah MD, Zainal D, Mokhtar N, et al. Erythema multiforme, Stevens-Johnson syndrome and
toxic epidermal necrolysis in northeastern Malaysia. Int J Dermatol 1998;37: 520-3; Kim KJ, Lee DP, Suh HS, et al. Toxic epidermal
necrolysis: analysis of clinical course and SCORTEN-based comparison of mortality rate and treatment modalities in Korean patients.
Acta Derm Venereol 2005;85:497-502; Koh MJ, Tay YK: Stevens-Johnson syndrome and toxic epidermal necrolysis in Asian children.
J Am Acad Dermatol 2010;62:54-60; Léauté-Labrèze C, Lamireau T, Chawki D, et al. Diagnosis, classification, and management of ery-
thema multiforme and Stevens-Johnson syndrome. Arch Dis Child 2000;83:347-52; Schneck J, Fagot JP, Sekula P, et al. Effects of
treatments on the mortality of Stevens-Johnson syndrome and toxic epidermal necrolysis: A retrospective study on patients included
in the prospective EuroSCAR Study. J Am Acad Dermatol 2008;581:33-40; Yamane Y, Aihara M, Ikezawa Z: Analysis of Stevens-
Johnson syndrome and toxic epidermal necrolysis in Japan from 2000 to 2006. Allergol Int 2007;56:419-25; Sethuraman G, Sharma
VK, Pahwa P, et al. Causative drugs and clinical outcome in Stevens Johnson syndrome (SJS), toxic epidermal necrolysis (TEN), and
SJS-TEN overlap in children. Indian J Dermatol 2012;57:199-200; Yip LW, Thong BY, Tan AW, et al. High-dose intravenous immunoglob-
ulin in the treatment of toxic epidermal necrolysis: a study of ocular benefits. Eye (Lond) 2005;19:846-53.

effect, although it was not statistically significant (OR, 0.54; admission. The primary end point was mortality in a cohort
95% CI, 0.29–1.01) (Zimmermann 2017). of 281 patients with SJS or TEN enrolled in a registry span-
The EuroSCAR registry is one of the largest evaluations in ning several countries in Europe. Data specific to outcomes
patients with SJS/TEN. This study was a retrospective evalu- with the use of IVIG and corticosteroids were gathered from
ation on the effects of treatment administered after hospital the two centers with more than 80% of the registry patients

CCSAP 2021 Book 1 • Pulmonary and Endocrinology 145 Immune-Mediated Reactions


(France and Germany). In addition to overall mortality, the centers treating patients with TEN. These studies demon-
authors evaluated the risk of death in patients treated with strated that patients who received corticosteroids were at an
IVIG or corticosteroids compared with supportive care only. increased risk of mortality. However, contemporary scrutiny
The registry provided the ORs from logistic regression mod- of these data show that poor outcomes may have resulted
els, adjusting for all factors shown to have an impact on the from late referral to the treatment facility and prolonged cor-
outcome. Dosing strategies for both corticosteroids and IVIG ticosteroid therapy that promoted infectious complications
were variable. No treatment significantly affected the risk of (Halebian 1986, Kelemen 1995).
mortality compared with supportive care only. These results Corticosteroids are considered part of standard of care for
did not support the prior opinion of many experts that corti- many centers, despite little evidence pointing to a definitive
costeroids were possibly harmful in the treatment of patients clinical benefit. There is no preferred agent or dose, but spe-
with SJS or TEN because no increase in mortality was cialists suggest early implementation (within 72 hours) and
observed with this approach (Schneck 2008). shortest duration possible to avoid complications including
Corticosteroids are not benign therapies, particularly infection.
at doses used in the treatment of immune-mediated reac-
tions such as SJS/TEN. No single steroid is preferred, but Intravenous Immune Globulin
the agent should be administered at a high dose (at least 1 Intravenous immune globulin is a product derived from
mg/kg prednisone equivalents daily) and initiated early in pooled plasma. It can nonspecifically inhibit cytokine
the course of disease. Well known complications of steroids release, can antagonize certain receptors, and is regularly
are hyperglycemia, impaired wound healing, and infectious used to treat other inflammatory or autoimmune disorders.
complications, all of which negatively impact outcomes in Histologic samples from patients with SJS/TEN show kerati-
critically ill patients. Specifically, in SJS/TEN, some experts nocytes with up-regulated lytically active Fas ligand, and IVIG
argue that steroids pose a significant risk of harm because of is known to inhibit Fas ligand-mediated keratinocyte apopto-
risk of impaired wound healing and increased risk of sepsis sis (Table 7).
and thus should be contraindicated. This concern originated Several centers prioritize IVIG therapy in the treatment
from studies performed in the mid-1980s and 1990s at burn of patients with SJS/TEN. However, the perceived benefit

Table 7. Select Studies Evaluating Intravenous Immune Globulin in Stevens-Johnson Syndrome and Toxic Epidermal
Necrolysis Compared With Standard of Care

Primary
Author Year Design n Regimen Mortality

Cartotto 2008 Retrospective 27 0.7 g/kg daily for 4 days 7/27

Dorafshar 2008 Retrospective 48 Not reported 4/48

Firoz 2012 Prospective 23 4 g/kg None

Gravante 2007 Retrospective, case series 17 0.4 g/kg daily for 5 days 7/17

Schneck 2008 Retrospective, case control 35 1.9 g/kg over 1–7 days 12/35

Shortt 2004 Retrospective, case control 16 0.7 g/kg 4/16

Yip 2005 Retrospective, historical control 18 2 g/kg over 2 days 2/18

Paquet 2006 Prospective 11 1 g/kg daily for 3 days 1/6

Information from: Cartotto M, Mayich D, Nickerson M, et al. SCORTEN accurately predicts mortality among toxic epidermal necrolysis
patients treated in a burn center. J Burn Care Res 2008;29:141-6; Dorafshar AH, Dickie SR, Cohn AB, et al. Antishear therapy for toxic
epidermal necrolysis: an alternative treatment approach. Plast Reconstr Surg 2008;122:154-60; Firoz B, Henning J, Zarzabal L, et al.
Toxic epidermal necrolysis: five years of treatment experience from a burn unit. J Am Acad Dermatol 2012;67:630-5; Gravante, G,
Delogu, D, Marianetti, M, et al. Toxic epidermal necrolysis and Steven-Johnson syndrome in oncologic patients. Eur Rev Med
Pharmacol Sci 2007;11:269-74; Paquet P, Kaveri S, Jacob, E, et al. Skin immunoglobulin deposition following intravenous immuno-
globulin therapy in toxic epidermal necrolysis. Exp Dermatol 2006;15:381-6; Schneck J, Fagot JP, Sekula P, et al. Effects of treatments
on the mortality of Stevens-Johnson syndrome and toxic epidermal necrolysis: A retrospective study on patients included in the pro-
spective EuroSCAR Study. J Am Acad Dermatol 2008;581:33-40; Shortt, R, Gomez, M, Mittman, N, et al. Intravenous immunoglobulin
does not improve outcome in toxic epidermal necrolysis. J Burn Care Rehabil 2004;25:246-55; Yip LW, Thong BY, Tan AW, et al. High-
dose intravenous immunoglobulin in the treatment of toxic epidermal necrolysis: a study of ocular benefits. Eye (Lond)
2005;19:846-53.

CCSAP 2021 Book 1 • Pulmonary and Endocrinology 146 Immune-Mediated Reactions


continues to be debated. A recent meta-analysis pooled data Etanercept is considered an investigational therapy in the
from 10 evaluations of IVIG compared with standard of care. treatment of SJS/TEN. Etanercept is a TNF-α inhibitor, which
Although evidence was lacking to routinely support admin- is labeled for the treatment of rheumatoid arthritis and pso-
istration of IVIG in patients with SJS/TEN, these studies riatic conditions. In a series of 10 SJS/TEN patients treated
favored the use IVIG with higher doses (greater than 0.7 g/ with a single 50-mg subcutaneous dose of etanercept, all
kg once daily) for the duration of treatment. Other reviews patients responded with a mean re-epithelialization time of
drew similar conclusions; however, given the rarity of the 8.5 days. None of the patients died, despite a mean SCORTEN-
condition and the heterogeneity of the patients, mixed predicted mortality rate of 50% (Paradisi 2014). In contrast,
conclusions are not surprising (Barron 2015, Huang 2012, a retrospective study evaluated a combination of IVIG and
Zimmermann 2017). etanercept in which 13 SJS/TEN patients were treated with
Different dosing strategies have been compared to eluci- etanercept and compared to patients who did not receive
date whether the cumulative dose of IVIG has an impact on etanercept. No significant differences were observed in mor-
mortality outcomes. A systematic review in 2012 assessed tality (15.4% vs. 10%; p=0.58), ICU days (6.9 vs. 15.1; p=0.08),
literature published before 2011 (Huang 2012). One of the length of stay (9.8 vs. 16.4; p=0.11), or infections (38.5% vs.
aims was to compare the clinical differences between high 57.5%; p=0.58) (Pham 2019).
(2 g/kg or more) and low cumulative doses (less than 2 g/kg) Cyclosporine is another therapy considered investi-
of IVIG and to compare IVIG treatment in adult and pediatric gational in the treatment of SJS/TEN and is based on
patients. Among the 17 studies reviewed, the overall mortality disease mediation by cytotoxic T cells, natural killer cells,
rate was 19.9%. The pooled OR for mortality from six obser- and cytokine-induced apoptosis. Cyclosporine inhibits
vational control studies comparing IVIG and supportive care the amplification of this pathway, making it an ideal can-
was 1.00 (95% CI, 0.58–1.75; p=0.99). Adults treated with high- didate for treatment. An open pilot study assessed the
versus low-dose IVIG initially experienced a lower mortality effect of cyclosporine 3 mg/kg daily for 10 days and then
rate (18.9% vs. 50%; p=0.022). However, a multivariate logis- a tapered dose. The study evaluated mortality at 1 month
tic regression model adjustment indicated that IVIG dose in 29 patients compared with the SCORTEN prediction and
did not, in fact, correlate with mortality (high vs. low dose: found no deaths, a shorter length of stay compared with con-
OR 0.49; 95% CI, 0.106–2.300; p=0.369). Pediatric patients trols, and a faster rate of epithelialization. However, despite
treated with IVIG had significantly lower mortality than adults some positive results, treatment was stopped in 3 cases for
(0% vs. 21.6%; p=0.001) (Huang 2012). cyclosporine-related adverse events including encephalopa-
A retrospective review of 64 patients treated with IVIG for thy, neutropenia, and severe infection. Sepsis was reported
SJS/TEN specifically compared those who received a high in 24% of patients (Roujeau 2017). Other studies have eval-
cumulative dose (3 g/kg or more) to those who received a low uated cyclosporine in the treatment of SJS/TEN and have
cumulative dose (less than 3 g/kg). The mean dose of IVIG used similar doses but were limited by small population
over the study population was 2.4 g/kg. The mean delay from sizes and lack of comparator or control arms. The possibility
the onset of epidermal detachment to administration was of drug-related adverse events is a potential barrier for more
3.2 days, which is within the timeframe seen in other publica- widespread use of cyclosporine.
tions. The mortality rate was 31%, and no mortality difference
was observed between patients who receive high (3 g/kg or Monitoring
more) versus low aggregative doses (less than 3 g/kg) of IVIG Monitoring patients with SJS/TEN is similar to caring for burn
(Lee 2013). patients. Important aspects of monitoring are hemodynamic,
Currently IVIG remains one of the more common fluid, and pulmonary status, as well as regulating tempera-
immune-modulating therapies in the treatment of SJS/TEN. ture and assessing for compartment syndrome and wound
The consensus is that, if IVIG is to be used, the ideal approach re-epithelialization.
is to initiate early (within the first 72 hours) in the course the Fluid resuscitation is a cornerstone for supportive
disease and doses should be close to 0.75 g/kg daily for 3–4 care in SJS/TEN patients. Clinicians should be vigilant
days (less than 3 g/kg as an aggregative dose). Also, IVIG regarding the patient’s fluid and electrolyte balance. The
can be added to corticosteroids if already ordered. The rec- use of a Foley catheter may aid in the prevention of ure-
ommended approach is that IVIG should be dosed based on thral strictures. Given that oral mucosa may be involved,
adjusted body weight or ideal body weight, rather than actual it is imperative to assess respiratory function and provide
body weight, because it does not distribute into body fat and supplemental oxygen, intubation, and mechanical venti-
it is only present in the intravascular space and extracellu- lation as necessary. High-volume resuscitation can place
lar fluids. The use of adjusted body weight, rather than ideal patients at risk of compartment syndrome. Urine output
body weight is based on the presumption that extracellular is not a sensitive marker, but bladder pressures can be
fluid is increased in patients with increased adipose tissue measured as part of overall fluid status and management
(Ameratunga 2017, Rocchio 2013). strategies (Latenser 2009).

CCSAP 2021 Book 1 • Pulmonary and Endocrinology 147 Immune-Mediated Reactions


ANGIOEDEMA causing vasodilation. Unfortunately, excess bradykinin in the
Angioedema is a manifestation of other pathophysiologic endothelial cells leads to increased vascular permeability
processes that results in localized, nonpitting, asymmet- and subsequent angioedema. Nonallergic angioedema can
ric edema of the subcutaneous and submucosal tissues. It be further subcategorized into the following primary types:
is associated with various causes, such as food, infection, hereditary angioedema, acquired angioedema, and drug-induced
drugs, and genetics. About 15% of the general population angioedema. Each of these disease states interacts with the
experiences angioedema. Facial swelling that involves the bradykinin pathway differently and results in either excess
tongue, lips, and periorbital area is common; however, laryn- production or reduced breakdown of bradykinin.
geal swelling may result in upper airway compromise and the
need for intensive care. In cases involving airway obstruction, Hereditary Angioedema
mortality is as high as 25%–40% (Temiño 2008). Hereditary angioedema is a genetic form of angioedema
that is most commonly caused by a deficiency in C1-INH
Epidemiology and Pathophysiology concentrations. This rare condition estimated to affect 1 in
Angioedema is generally categorized as either allergic or 10,000 to 1 in 50,000 persons (Cicardi 1996, Zuraw 2008).
nonallergic. Allergic angioedema, also known as histamine-me- The inheritance pattern is most often heterozygous and
diated angioedema, is a type I hypersensitivity reaction. Most autosomal dominant; however, spontaneous mutations in
forms of nonallergic angioedema are thought to be a brady- persons with no family history can occur (Nzeako 2001). No
kinin-mediated reaction. Bradykinin is an end-product of the known racial or sex predominance is associated with HAE
kallikrein–kinin system, which provides a counterbalance overall. As a regulator of the kallikrein–kinin system, C1-INH
for the vasoconstrictive properties of the renin-angiotensin- functions by preventing complement autoactivation and is
aldosterone system (Figure 1). In the kallikrein–kinin sys- responsible for inactivating factor XIIa and subsequently
tem, activated factor XII converts prekallikrein to kallikrein kallikrein. In the case of C1-INH quantitative or functional
by a positive feedback loop. In the final step of the cascade, deficiency, these factors are left unchecked, which leads to
kallikrein cleaves high-molecular-weight kininogen to form increased formation and release of several kinins, including
bradykinin. Bradykinin then binds the bradykinin B2-receptors, bradykinin.

Figure 1. Pathophysiology of bradykinin-mediated angioedema. Dotted lines signify inhibition.

ACE = angiotensin-converting enzyme; APP = aminopeptidase P; C1-INH = C1 esterase inhibitor; DPP-IV = dipeptidyl peptidase IV;
NEP = neutral endopeptidase (neprilysin).
Information from: Bhardwaj N, Craig TJ. Treatment of hereditary angioedema: a review (CME). Transfusion 2014;54:2989-996;
Hirschy R, Shah T, Davis, T, et al. Treatment of life–threatening ACE–inhibitor-induced angioedema. Adv Emerg Nurs J 2018;
40:267-77.

CCSAP 2021 Book 1 • Pulmonary and Endocrinology 148 Immune-Mediated Reactions


Table 8. Distinguishing Complement Findings for Types of Angioedema

Antigenic C1-INH Functional C1-INH


Type C4 Concentration C1q Concentration Concentration Concentration

HAE Type I Decreased Normal Decreased Decreased

HAE Type II Decreased Normal Normal Decreased

HAE Type III Normal Normal Normal Normal

AAE Decreased Decreased Decreased Decreased

ACE inhibitor induced Normal Normal Normal Normal

Allergic Normal Normal Normal Normal

ACE = angiotensin-converting enzyme; AAE = acquired angioedema; C1-INH = C1 esterase inhibitor; HAE = hereditary angioedema
Information from: Nzeako UC. Hereditary angioedema. Arch Intern Med 2001;161:2417. Zuraw BL, Bernstein JA, Lang DM, et al. A
focused parameter update: hereditary angioedema, acquired C1 inhibitor deficiency, and angiotensin-converting enzyme inhibitor-
associated angioedema. J Allergy Clin Immunol 2013;131:1491-3.

At least three forms of HAE are recognized. Type I HAE Drug-Induced Angioedema
occurs most often, comprising 80%–85% of all HAE cases. This Drug-induced angioedema can also be differentiated based
genetic mutation leads to a decrease in C1-INH production, on mechanism: allergic (IgE-mediated, histamine-mediated)
resulting in a quantitative deficiency in C1-INH concentrations or nonallergic (non-IgE-mediated). β-Lactam antibiotics
(Table 8) (Zuraw 2013). Type II HAE, which comprises about are the most common cause of IgE-mediated drug-induced
15% of HAE cases, is a result of a genetic defect that allows for angioedema. Alternatively, ACE inhibitor-induced angioedema
transcription of C1-INH with reduced function, which results is the most widely studied form of nonallergic, drug-induced
in normal or even elevated concentrations of dysfunctional angioedema. Recent evidence also suggests that dipepti-
C1-INH (Warin 1986). Finally, type III HAE is a condition that dyl peptidase-IV (DPP-IV) inhibitors and sacubitril/valsartan
was originally described in the 1980s but has not garnered may increase the risk of nonallergic angioedema when used
much research until the 2000s (Binkley 2000, Bork 2000). in combination with ACE inhibitors. Other known culprits of
Although the pathophysiology and epidemiology of type III nonallergic, drug-induced angioedema include angiotensin
HAE is not fully understood, it does appear to present with II inhibitors, nonsteroidal anti-inflammatory drugs (NSAIDs),
normal quantitative and functional C1-INH concentrations. and fibrinolytic agents.
It is seen predominantly in women, and symptoms appear to Wilkin and colleagues reported the first cases of ACE inhib-
worsen during periods of increased estrogen concentrations, itor-induced angioedema in 1980 (Wilkin 1980). Although
such as pregnancy. Estrogen has previously been reported the incidence of angioedema in patients taking ACE inhibi-
to increase factor XII concentrations. Thus, the hypothesis is tors is 0.1%–0.7%, the widespread use of this drug therapy
that, in the setting of type III HAE, increased estrogen concen- adds to this significance of this condition (Bezalel 2015). In
trations would result in a downstream increase in bradykinin fact, it accounts for up to one-third of all angioedema cases
production (Levy 2010, Zuraw 2008). seen in the emergency department (Baş 2015, Bezalel 2015).
Although most cases occur within the weeks after initiation
Acquired Angioedema of therapy, angioedema caused by ACE inhibitors can occur
Acquired angioedema is a term used to describe angioedema at any point during therapy, and some cases have occurred
caused by a C1-INH deficiency that is not a result of a genetic after several years of therapy (Byrd 2006, Temiño 2008).
defect or other mechanisms, as is the case with HAE or Known risk factors include African American ethnicity, age
drug-induced angioedema, respectively. The incidence older than 65 years, female sex, chronic heart failure, tobacco
of acquired angioedema is very rare with only a few hun- use, and history of allergies or drug-induced rash (Byrd 2006,
dred cases ever reported, and the pathophysiology has not Hirschy 2018, Miller 2008). African Americans in particular
been fully elucidated (Cicardi 2010). Two forms of acquired are 3 to 5 times more likely than whites to experience ACE
angioedema have been described in literature. Type I results inhibitor-induced angioedema (Brown 1996).
from an increase in the breakdown of C1-INH, whereas type The pathophysiology of ACE inhibitor-induced angioedema
II appears to result from an autoantibody to C1-INH (Cicardi is thought to be related to bradykinin accumulation as well as
2010). Because of the limited research, acquired angioedema a failure in bradykinin breakdown. In addition to its role in con-
is managed similarly to HAE. verting angiotensin I to angiotensin II, ACE is responsible for

CCSAP 2021 Book 1 • Pulmonary and Endocrinology 149 Immune-Mediated Reactions


bradykinin degradation. Thus, when ACE is inhibited, bradyki- Presentation
nin can accumulate. Bradykinin breakdown is also dependent Angioedema can involve the skin (face, extremities, gen-
on other enzymes including neutral endopeptidase/neprilysin, italia), upper airway (laryngeal, pharyngeal) and GI tract.
aminopeptidase-P, carboxypeptidase-M, and carboxypepti- Facial edema commonly effects the tongue, lips, and perior-
dase-N. Although ACE inhibitors may affect these enzymes bital area. Gastrointestinal edema presents with abdominal
to some degree, it is hypothesized that angioedema in some pain, cramping, and vomiting. These symptoms may be mis-
patients who use ACE inhibitors may also be caused by a taken for other abdominal conditions, particularly if facial or
defect in other pathways for bradykinin breakdown (Blais other extremity swelling is not present. Genital edema usu-
1999). Other medications also interact with these enzymes, ally only occurs after a trauma (Cicardi 1996, Nzeako 2001).
including sacubitril/valsartan and DPP-IV inhibitors. Although symptoms of HAE and drug-induced angioedema
Sacubitril/valsartan is an angiotensin receptor-neprilysin overlap, cutaneous and GI symptoms are more commonly
inhibitor. Because neprilysin is one of the enzymes involved in HAE. Angioedema caused by ACE inhibitors primarily
in bradykinin breakdown, angioedema is a possible adverse effects the face and upper airway. Although a rare complica-
effect. Safety concerns with neprilysin inhibitors first arose tion, laryngeal swelling may result in airway obstruction and
with early trials of omapatrilat, a combination ACE inhibitor compromise, resulting in intubation and death (Banerji 2008,
and neprilysin inhibitor, which caused a significantly higher Bluestein 2009, Javaud 2015, Nzeako 2001).
rate of angioedema than ACE inhibitors alone (Owens 2017). Acute HAE manifestations typically last 1–5 days.
Based on the decreased incidence of angioedema in therapy Symptoms may resolve spontaneously or require exogenous
with angiotensin II receptor blockers versus ACE inhibitors, C1-esterase therapy. Gastrointestinal symptoms typically
research into angiotensin receptor-neprilysin inhibitor ther- subside within 12 to 24 hours, whereas cutaneous symp-
apy began. A phase III study comparing sacubitril/valsartan to toms may persist for several days. Patients with HAE may
enalapril found a 0.45% and 0.24% incidence of angioedema, experience as many as 20–100 symptomatic attacks per
respectively (p=0.13) (Shi 2018). year. Alternatively, if angioedema is a result of ACE inhibitor
In addition to its role in incretin metabolism, DPP-IV serves therapy, resolution of symptoms is anticipated within 24–72
as a salvage mechanism for bradykinin breakdown when the hours after discontinuation of the medication (Chiu 2001,
ACE mechanism is diminished. Substance P, a substrate of Heymann 1997, Warrier 2004).
DPP-IV, exhibits vasodilatory activity similar to bradykinin.
Therefore, DPP-IV inhibitors (e.g., sitagliptin, saxagliptin,
linagliptin) may result in the accumulation of bradykinin PHARMACOLOGIC THERAPY FOR
and substance P particularly when ACE activity is inhibited. ACUTE ANGIOEDEMA
Researchers have suggested that concomitant use of DPP-IV Management of angioedema occurs in two main phases:
inhibitors may increase the risk of angioedema when used acute episode management and prophylaxis. For the pur-
concomitantly with ACE inhibitors (Brown 2009, Byrd 2008). poses of this review, the discussion focuses on management
Angioedema is also a rare adverse effect associated with of acute episodes. Airway management, with a low threshold
tissue plasminogen activator (tPA), with an incidence of for establishment of a definitive airway, should be prioritized
1%–5% (Fugate 2012, Pahs 2016, Werner 2014). Development in the treatment of angioedema regardless of cause. If the
of angioedema after tPA administration is thought to be patient shows signs of stridor, drooling, hoarseness, dys-
caused by activation of both the complement cascade and the pnea, or dysphagia, advanced airway management may be
kallikrein-kinin system, resulting in release of histamine and required (Levy 2010).
bradykinin, respectively. Patients who are taking ACE-inhibitors In the case of allergic or drug-induced angioedema, removal
when they receive tPA may be at a higher risk of angioedema. of the trigger should occur immediately. Further acute treat-
Finally, although not related to the bradykinin pathway, ment depends on determining the cause of angioedema.
NSAIDs are another commonly used medication that can Historically, pharmacologic treatment for acute angioedema,
cause angioedema. Although the exact incidence of NSAID- regardless of cause, has targeted histaminergic and anti-in-
induced angioedema is unknown, NSAID hypersensitivity flammatory reactions. These therapies include epinephrine,
reactions are thought to account for 20%–30% of all drug-re- antihistamines and steroids. For allergic angioedema, these
lated reactions (Kowalski 2013, Laidlaw 2017). Interference agents would be considered the mainstay of treatment, as
with arachidonic acid metabolism is believed to be the mech- previously discussed in depth in this review. Most patients
anism behind NSAID-induced angioedema. Nonselective with allergic angioedema respond to this therapy. However,
NSAIDs inhibit cyclooxygenase-1 and -2 pathways, which leads for nonallergic, bradykinin-mediated angioedema, these
to a shunting of arachidonic acid metabolism to the 5-lipoxy- medications have limited use. Therefore, a therapeutic trial
genase pathway. This shift results in an increase in synthesis of histamine-targeted therapies can serve as both a thera-
and release of cysteinyl leukotrienes, which increases vascu- peutic and diagnostic tool. For nonallergic angioedema, only
lar permeability and can result in angioedema. 2%–10% of patients improve after steroids and antihistamines

CCSAP 2021 Book 1 • Pulmonary and Endocrinology 150 Immune-Mediated Reactions


Table 9. Overview of Available Medications for Acute Angioedema

FDA-
Generic Name Approved
(Trade Name) Drug Description Mechanism Indication Dosage and Route AWP Per Dose

C1-INH (Berinert) Lyophilized, Replacement of Treatment for 20 units/kg IV over $2605.11–$5210.22b


human pdC1-INH deficient protein acute HAE 10 min
(C1-INH)
sC1-INH (Cinryze) Nanofiltered, Prophylaxis 1000 units IV; repeat if $5738.28b
human pdC1-INH for HAE no response in 1 hra

Conestat alfa rhC1-INH Treatment for 50 units/kg IV; max 4200 $6234–$12,468b
(Ruconest) acute HAE units if >84 kg

Ecallantide Recombinant Selective plasma Treatment for 30 mg SC (three 10-mg $14,767.62


(Kalbitor) human protein kallikrein inhibitor acute HAE in injections); may repeat
age ≥12 yr once

Icatibant (Firazyr) Synthetic Bradykinin Treatment for 30 mg SC; may repeat $3434.19
decapeptide B2-receptor inhibitor acute HAE every 6 hr; max 3 doses

a
Cinryze is only FDA-approved for prophylaxis. Dose listed is representative of current off-label recommendations for acute hereditary
angioedema.
b
Listed price is representative of 2 vials for 1 dose.
AWP = average wholesale price; C1-INH = C1 esterase inhibitor; HAE = hereditary angioedema; IV = intravenous; max = maximum;
pdC1-INH = plasma-derived C1 esterase inhibitor; rhC1-INH = recombinant human C1-esterase inhibitor; SC = subcutaneous.
Information from: manufacturers’ package inserts; Red Book Online [database online]. Greenwood Village, CO: Truven Health
Analytics. Available at http://redbook.solutions.aap.org/redbook.aspx. Accessed June 4, 2020.
Zuraw B, Cicardi M, Levy RJ, et al. Recombinant human C1-inhibitor for the treatment of acute angioedema attacks in patients with
hereditary angioedema. J Allergy Clin Immunol 2010;126:821-7.

(Bramante 2011, Lenschow 2018, Richman 2012). Table 9 sum- with C1-INH (4.9 hours) than placebo (7.8 hours; p=0.0237).
marizes the currently available medications for treatment of As an extension study of IMPACT1, IMPACT2 was designed
HAE and ACE inhibitor-induced angioedema. to provide additional data on long-term use of CI-INH concen-
trate for treatment of subsequent HAE. A total of 57 patients
C1-Inhibitor Concentrates were enrolled in IMPACT2 for a median duration of 24 months
Replacement therapy is a mainstay treatment for several (0–51 months), during which time they received C1-INH for
inherited protein deficiencies, including HAE. The two formu- a median of 7 acute HAE episodes (1–184 episodes). The
lations of human C1-INH concentrate available in the United median time of onset was 0.46 hours, similar to the results of
States are Berinert and Cinryze. These medications act as a IMPACT1. Time to complete resolution of symptoms was 15.5
replacement for C1 esterase inhibitor, which is either quan- hours. These results suggest that C1-INH replacement ther-
titatively or functionally deficient in patients with HAE, as apy is effective at providing rapid relief for symptoms in acute
previously discussed. HAE (Craig 2009, 2011).
Berinert was first approved for the treatment of acute Based on the results of IMPACT1 and IMPACT2, Berinert
abdominal and facial HAE in 2009. Since then, it has was approved for acute HAE attacks starting at a dose of 20
also been approved for laryngeal HAE. The International units/kg by a slow intravenous push. Typical doses range
Multicenter Prospective Angioedema C1-inhibitor Trial 1 from 500 units (1 vial) to 1000 units (2 vials). The pharmacoki-
(IMPACT1) established the safety and efficacy of Berinert netic properties for Berinert are provided in Table 10. Median
at 20 units/kg intravenous push for acute HAE. In this ran- time to onset is 30–60 minutes when a 20 unit/kg dose is
domized, double-blind, placebo-control trial, patients were administered. At a dose of 10 units/kg, the time to onset of
enrolled if they presented within 5 hours of onset of moder- symptom relief appears to be similar to placebo (1.2 hours vs.
ate to severe abdominal or facial symptoms of HAE. Median 1.5 hours; p=0.2731). Berinert is well tolerated; the most com-
time to onset of symptom relief was significantly shorter with mon adverse effects are abdominal pain, nausea, vomiting,
C1 esterase inhibitor (at a dose of 20 units/kg) compared diarrhea, headache, and muscle spasms (Craig 2009, 2011).
with placebo (0.5 hours vs. 1.5 hours; p=0.025). The median Cinryze is also a human C1-INH concentrate with similar
time to complete resolution of symptoms was also shorter activity to Berinert. It differs from Berinert in that it undergoes

CCSAP 2021 Book 1 • Pulmonary and Endocrinology 151 Immune-Mediated Reactions


Table 10. Pharmacokinetic Properties of Available Medications Used to Acute Angioedema Attacks

Generic Name Elimination


(Trade Name) Route Onset Metabolism Clearance Half-Life Vd

C1-INH (Berinert) Intravenous 30–60 min Unknown 1.4 mL/kg/hr 32–62 35 L

C1-INH (Cinryze) 45 min–2 hr Unknown 0.85 mL/mina 40–56 hr —

Conestat alfa 60–90 min Hepatic (receptor-mediated 1207 mL/hrb 3 hr 3L


(Ruconest) endocytosis, then hydrolysis)

Ecallantide Subcutaneous 30 min–4 hr Unknown 156 mL/min 2 hr 26 L


(Kalbitor)

Icatibant (Firazyr) 30 min–2 hr Hepatic (10% icatibant; 90% 245 mL/min 1.5 hr 29 L
metabolites)

a
Based on single dose of 1000 units.
b
Based on 50 unit/kg dosing regimen.
C1-INH = C1 esterase inhibitor; Vd = Steady-state volume of distribution
Information from: Bork K. Pasteurized and nanofiltered, plasma-derived C1 esterase inhibitor concentrate for the treatment of
hereditary angioedema. Immunotherapy 2014;6:533-51; Craig TJ, Levy RJ, Wasserman RL, et al. Efficacy of human C1 esterase
inhibitor concentrate compared with placebo in acute hereditary angioedema attacks. J Allergy Clin Immunol 2009;124:801-8;
Craig TJ, Bewtra AK, Bahna SL, et al. C1 esterase inhibitor concentrate in 1085 hereditary angioedema attacks—final results of
the I.M.P.A.C.T.2 study. Allergy Eur J Allergy Clin Immunol 2011;66:1604-11; Zuraw B, Cicardi M, Levy RJ, et al. Recombinant
human C1-inhibitor for the treatment of acute angioedema attacks in patients with hereditary angioedema. J Allergy Clin
Immunol. 2010;126:821-7; manufacturers’ package inserts.

an additional nanofiltration step during preparation, which is for safety and efficacy analysis. The median times to start
theorized to reduce the risk of viral transmissions. Cinryze of symptom relief were 66 and 122 minutes for 100 units/
was approved by the FDA for use in prophylactic treatment of kg and 50 units/kg rhC1-INH, respectively, in comparison to
HAE in 2008. The approved dosing is 1000 units over 10 min- 295 minutes with saline (p<0.001 and p=0.013). Both doses
utes every 3 to 4 days. Although Cinryze has not received FDA of rhC1-INH also resulted in a significantly shorter time to
approval for use in acute attacks, it has been studied in this minimal symptoms compared with saline (p<0.001 and
population. Patients in these studies received C1-INH 1000 p=0.001). With both doses found to be similarly efficacious,
units over 10 minutes, which could be repeated if no symp- rhC1-INH 50 units/kg dosing was approved for treatment of
tom relief was evident at 60 minutes. Patients with laryngeal acute HAE (Craig 2011).
symptoms were included in only one study, but none required Subsequent studies, with larger patient sample sizes,
intubation after treatment with C1-INH. Median time to onset have supported the efficacy and safety of rhC1-INH for acute
of symptom relief ranged from 0.75–2 hours (Riedl 2012, HAE. Headache is the most common adverse event; however,
Zuraw 2010). For the pharmacokinetic properties of Cinryze, patients may also experience vertigo, nasopharyngitis, and
see Table 10. injection-site discomfort. Conestat alfa is contraindicated in
patients with an allergy to rabbits or rabbit-derived products;
Recombinant C1-Inhibitor therefore, a thorough allergy history is imperative before use
Conestat alfa (Ruconest) is an rhC1-INH produced from (Riedl 2013, 2014, Zuraw 2010).
transgenic rabbit milk. Similar to endogenous C1-INH and The main difference between conestat alfa (rhC1-INH)
human C1-INH concentrate, conestat alfa prevents com- and other plasma-derived C1-INH concentrates is its shorter
plement autoactivation which results in factor XIIa and half-life (see Table 10). Because untreated angioedema may
kallikrein inactivation. Conestat alfa received FDA approval last several days, repeat dosing of rhC1-INH may be required;
for the treatment of acute HAE in 2014. Approval was based however, it appears to be uncommon, with only one study
primarily on two randomized, double-blind saline-control, reporting a single patient who had a recurrence within 24
trials conducted in North American and Europe. Eligible hours. Of interest, if repeat doses are required, the risk of
patients were randomized to receive 100 units/kg rhC1- adverse events does not appear to be increased. Therefore,
INH, 50 units/kg rhC1-INH, or saline in the North American conestat alfa is approved such that if symptoms persist, an
study, or to receive 100 units/kg rhC1-INH or saline in the additional dose may be administered (with no more than 2
European study. Results from the two studies were pooled doses in a 24-hour period) (Riedl 2013, 2014, Zuraw 2010).

CCSAP 2021 Book 1 • Pulmonary and Endocrinology 152 Immune-Mediated Reactions


Kallikrein Inhibitor in responders based on the location of the angioedema in a
Ecallantide (Kalbitor) is a 60-amino-acid recombinant pro- comparison of laryngeal angioedema to all other locations
tein produced in Pichia pastoris yeast. This agent functions (OR 1.05; p=0.97).
as a plasma kallikrein inhibitor, which results in decreased Ecallantide appears to be well-tolerated, with the most
production of kinins, including bradykinin. In 2009, the drug common symptoms being headache, nausea, diarrhea, and
received FDA approval for the treatment of acute HAE attacks pyrexia. Symptoms of hypersensitivity were not observed
in adults older than 16 years of age. The recommended dose is in EDEMA3 or EDEMA4 (Cicardi 2010b; Levy RJ 2010). The
30 mg subcutaneously, administered in three separate 10-mg pharmacokinetic profile of ecallantide is shown in Table 10.
injections. Unlike lyophilized human C1-INH concentrate Because it is administered as 3 separate 10-mg/mL injec-
products, ecallantide is not approved for self-administration tions, care should be taken to ensure that each injection is at
by patients because of the risk of anaphylaxis and anaphylac- least 5 cm apart from the previous injection site and in a dif-
toid reactions. ferent anatomic location than the site of the acute episode.

Hereditary Angioedema ACE Inhibitor-Induced Angioedema


Two phase III, placebo-control trials—EDEMA-3 and Ecallantide has also been suggested as a treatment alterna-
EDEMA-4—evaluated the use of ecallantide for the treatment tive for ACE inhibitor-induced angioedema (Table 11). Similar
of moderate to severe acute HAE. For both studies, patients to C1-esterases, ecallantide is believed to decrease the exag-
were eligible if they presented within 8 hours of a moderate gerated effects of bradykinin through blockade of further
to severe HAE episode. Patients were randomized to receive bradykinin production. However, in the setting of ACE inhib-
30-mg subcutaneous ecallantide or placebo (Cicardi 2010b, itor-induced angioedema, bradykinin accumulation occurs
Levy RJ 2010). The primary end point in EDEMA3 was treat- as a result of a failure in metabolism rather than overpro-
ment outcome score (TOS) at 4 hours after administration duction. The effectiveness of this “upstream” blockade is
of the study drug. The treatment outcome score is a com- unproven, and was the subject of 2 phase II, randomized, pla-
posite, patient-reported outcome measure that ranges from cebo-control trials evaluating the role of ecallantide for the
+100 (significant improvement) to –100 (significant worsen- treatment of ACE inhibitor-induced angioedema (Bernstein
ing). At 4 hours, the median TOS was 50 in the ecallantide 2015, Lewis 2015).
group (n=36) and 0 in the placebo group (n=36) (p=0.004),
indicating a better response with ecallantide. Alternatively,
the primary end point in EDEMA4 was the change from base-
line in the mean symptom complex severity (MSCS) score at
Table 11. Therapies for the Treatment of Angiotensin-
4 hours after administration of study drug. The MSCS score
Converting-Enzyme Inhibitor-Induced Angioedema
is a patient-reported measure of symptom burden at a single
point, ranging from 0 (normal) to 3 (severe). The mean change Drug or Drug Class Suggested Place in Therapy
in MSCS score from baseline to 4 hours was significantly
H1RA, H2RA Low evidence
greater with ecallantide (n=48) compared with placebo (n=48)
(–0.8 vs. –0.4; p=0.01) (Levy RJ 2010). An integrated analysis Corticosteroids Low evidence
of these two trials found that ecallantide resulted in a signifi- Epinephrine Low evidence
cantly better response than placebo with regard to both the
Fresh-Frozen Plasma Low evidence
TOS and MSCS score at 4 hours (Sheffer 2011).
In EDEMA3, the authors also reported a shorter median Tranexamic Acid Low evidence
time to significant improvement in overall response with C1-INH (Berinert) Not recommended
ecallantide (165 minutes) compared with placebo (greater
C1-INH (Cinryze) Not recommended
than 240 minutes); however, this finding did not reach sta-
tistical significance (p=0.14) (Cicardi 2010b). A post-hoc Ecallantide (Kalbitor) Not recommended
analysis was conducted to evaluate response based on time Icatibant (Firazyr) May consider in severe or life-
elapsed between symptom onset and treatment. Change in threatening cases (off-label use)
the TOS and MSCS scores was significantly better with ecal-
lantide compared with placebo when patients were treated
C1-INH = C1 esterase inhibitor; H1RA = H1-receptor antago-
within 6 hours of symptom onset. These findings suggest nist; H2RA = H2-receptor antagonist.
that early administration of ecallantide results in better out- Information from: Hirschy R, Shah T, Davis T, et al. Treatment
comes (Banta 2011). It should be noted that in these studies, of life-threatening ACE-inhibitor-induced angioedema. Adv
a large percentage of the patients presented with GI or cuta- Emerg Nurs J 2018;40:267-77; Wilkerson RG. Angioedema in
the emergency department: an evidenced-based review.
neous attacks. In EDEMA3, only 13 patients (18%) presented
Emerg Med Pract 2012;14:1-21.
with laryngeal symptoms. Also, EDEMA4 found no difference

CCSAP 2021 Book 1 • Pulmonary and Endocrinology 153 Immune-Mediated Reactions


Lewis and colleagues randomized 79 patients who pre- and may be repeated every 6 hours if HAE symptoms persist.
sented within 12 hours of symptom onset to either placebo Injection-site reactions are common, although most reported
(n=18) or to 10 mg (n=20), 30 mg (n=19), or 60 mg (n=19) of are minor.
ecallantide. Patients were excluded if advanced airway man-
Hereditary Angioedema
agement was determined to be imminent. Corticosteroids,
antihistamines, or epinephrine were allowed based on physi- The FAST (For Angioedema Subcutaneous Treatment) clinical
cian discretion. No difference was observed in the primary trial series evaluated icatibant in patients with cutaneous or
outcome of eligibility for discharge at 6 hours (88% ecallan- abdominal HAE episodes. Although FAST-1 and FAST-2 were
tide vs. 72% placebo; 95% CI, –11%–41%). The ICU admission published together, the differences in their study designs are
rates were low overall, and no differences between groups worth noting. Although both were randomized, double-blind,
were observed (9% ecallantide vs. 17% placebo; 95% CI, 19%– prospective studies, FAST-1 (n=56) was placebo-control trial,
34%). The study was ultimately terminated early because of whereas FAST-2 (n=74) was a double-dummy trial comparing
futility (Lewis 2015). icatibant to TXA. In both studies, patients in the treatment arm
In the second study, ecallantide 30 mg (n=26) was com- received 30-mg subcutaneous icatibant. In the FAST-2 trial
pared with placebo (n=24) in patients presenting within the TXA dose was 3 g daily for 2 days. In FAST-1, no difference
12 hours of symptom onset. Patients were enrolled if they was noted in the primary outcome (2.5 hours vs. 4.6 hours
remained symptomatic despite receiving standard therapy, with placebo; p=0.14). In the FAST-2 trial, icatibant produced a
which could include corticosteroids, antihistamines, and/ shorter time to clinically significant symptom relief compared
or epinephrine. During the observation period after study with TXA (2.0 hours vs. 12 hours; p<0.001). Both trials had
drug administration, an open-label dose of ecallantide could a significantly shorter time to first symptom improvement
be administered if patients were determined to be at risk of in patients who received icatibant based on assessment by
severe upper airway compromise or if symptoms worsened both the investigator and the patient. A total of 8 patients in
after 2 hours. Open-label ecallantide could be repeated once. FAST-1 and 3 patients in FAST-2 received open-label icatibant
Although a 10% improvement in eligibility for emergency for laryngeal symptoms (Cicardi 2010a).
department discharge at 4 hours was observed with ecallan- A subsequent trial, FAST-3 (n=88), was a phase III, ran-
tide, this finding did not reach statistical significance (31% domized, double-blind, placebo control, multicenter study
ecallantide vs. 21% placebo; 95% CI, 14%–34%). Open-label that further investigated the role of icatibant in acute HAE
treatment was required in 11 patients (42%) and 13 patients episodes. One specific difference in FAST-3 compared with
(54%) in the ecallantide and placebo groups, respectively. All the FAST-1 and FAST-2 study designs is that patients with
patients who required an open-label dose were considered laryngeal symptoms (mild to moderate severity) were also ran-
to have experienced treatment failure and were not eligible domized to either icatibant or placebo. Patients with severe
for discharge at 4 hours. Ecallantide was well-tolerated, with laryngeal symptoms received open-label icatibant. In a popu-
no significant difference in adverse events compared with lation of patients with cutaneous or abdominal attacks, time
placebo. to resolution of symptoms was shorter with icatibant versus
These randomized control trials failed to find a significant placebo (8 hours vs. 36 hours; p=0.012). Time to initial symp-
improvement in discharge from the emergency department tom relief as assessed by both the investigator and patient
within 4 and 6 hours for patients who received ecallantide was also shorter with icatibant. In total, 5 patients with mild
versus placebo, respectively. It must be noted, however, that to moderate laryngeal symptoms were randomized to receive
both studies had very small sample sizes, which increases either icatibant (n=3) or placebo (n=2). The small sample size
the risk of a type II error. The studies included an even smaller limits statistical comparison; however, time to complete reso-
percentage of patients with severe angioedema, a population lution was reported as 6 hours (95% CI, 3.5–44.8) and 4 hours
that theoretically would benefit most from ecallantide or other (95% CI, 1.5–6.4) in the icatibant and placebo groups, respec-
bradykinin-mediated therapy. Data are currently insufficient tively (Lumry 2011).
to support the use of ecallantide for ACE inhibitor-induced
angioedema. Ecallantide should not be used unless HAE is ACE Inhibitor-Induced Angioedema
suspected. Although not FDA-approved for this indication, numerous case
reports have addressed the use of icatibant for ACE inhibi-
Bradykinin B2-Receptor Antagonist tor-induced angioedema (Baş 2010; Bova 2015, Fok 2015).
Icatibant (Firazyr) is a synthetic decapeptide that is struc- In 2015, Baş and colleagues published the first randomized
turally similar to bradykinin. Therefore, it functions as a control trial comparing icatibant (n=13) to standard therapy
selective antagonist at the bradykinin B2 receptor, thus (n=14; intravenous prednisolone 500 mg and clemastine 2
inhibiting bradykinin-induced vasodilation (Cicardi 2010a). mg) for ACE inhibitor-induced upper-airway angioedema. The
Icatibant is currently approved for treatment of acute HAE epi- primary end point of median time to resolution of symptoms
sodes. It is administered as a 30-mg subcutaneous injection was 8 hours in the icatibant group compared with 27.1 hours

CCSAP 2021 Book 1 • Pulmonary and Endocrinology 154 Immune-Mediated Reactions


in the standard therapy group (p=0.002). Furthermore, the Fresh Frozen Plasma
median time to onset of symptom relief was also shorter with Although FFP is most often used for management of hemo-
icatibant (2 hours vs. 11.7 hours; p=0.03). stasis, FFP is also thought to target the bradykinin pathway.
These results, however, must be interpreted with caution. The enzymes C1-INH and kininase II that are contained in FFP
First, the overall baseline severity score was low, indicating supplement the breakdown of bradykinin and may lead to
mild angioedema symptoms among the patients enrolled. resolution of angioedema. Although no randomized control
Only 1 patient required advanced airway management. In trials exist to support this theory, several case reports seem
addition, the standard therapy used in this study was not con- to support its use in acute HAE episodes and ACE inhibitor-in-
sistent with current practice in the United States. The steroid duced angioedema. The most common dosing associated
dose, prednisolone 500 mg, far exceeds the common dosing with improvement in symptoms is 1 to 4 units of FFP. Onset of
for angioedema reactions and the antihistamine, clemastine, symptom improvement is expected to occur 30–90 minutes
is not widely available (Baş 2015). after FFP administration. Complete resolution of symptoms
Other prospective, randomized trials have not been able may occur within 2–4 hours of FFP; however, resolution may
to replicate the positive results found by Baş et al. in 2015 be delayed as long as 12 hours (Bolton 2012, Hassen 2013,
(Table 12) (Sinert 2017, Straka 2017). Sinert et al. compared Pekdemir 2007, Pickering 1969, Warrier 2004).
icatibant to placebo in patients with moderate to severe ACE Despite potentially treating acute angioedema, some
inhibitor-induced angioedema. Patients in either group could concerns exist that FFP may also worsen symptoms. This par-
receive “conventional treatment” with antihistamines, epi- adoxical effect is theorized to be a consequence of excess C4
nephrine, and steroids. Not only was no difference observed and other substrates (e.g., high-molecular-weight kininogen)
in time to onset of symptom resolution with icatibant, but also present in FFP. To date, however, there are no reports
the study failed to find an improvement in time to meeting describing cases of worsening acute HAE angioedema after
discharge criteria (Sinert 2017). These conflicting results administration of FFP (Hassen 2013, Hill 2004, Prematta
suggest against widespread use of icatibant for ACE inhib- 2007). Other potential adverse effects associated with FFP
itor-induced angioedema. Further research is warranted to include transmission of bloodborne pathogens and transfu-
determine whether icatibant prevents emergency intuba- sion reactions. Furthermore, the risk of fluid overload and
tions, decreases hospital or ICU admissions, or decreases subsequent acute lung injury precludes widespread use of
hospital lengths of stay. this therapy. Compared with C1-INH concentrate, 50 times the

Table 12. Selected Studies Evaluating Icatibant in ACE Inhibitor-Induced Angioedema

Primary Symptom
Author Design N Onset Comparator Arms Primary Findings

Baş Randomized, double- 27 ≤10 hr Icatibant 30 mg SC Median time to symptom resolution:


blind, double-dummy, injection vs. SOC (IV icatibant 8 hr (IQR 3–16) vs. SOC 27.1 hr
multicenter prednisolone 500 mg (IQR 20.3–40.8) (p=0.002); 1 tracheotomy
+ clemastine 2 mg) in SOC group

Sinert Randomized, double- 121 ≤12 hr Icatibant 30 mg SC Median time to meeting discharge criteria:
blind, placebo injection vs. placebo icatibant 4 hr (IQR 2–6) vs. placebo 4 hr
control (IQR 1–6) (p=0.63); median time to onset of
symptom relief: icatibant 2 hr (IQR 0.6–3.1)
vs. placebo 1.6 hr (IQR 0.5–3.9 hr) (p=0.57);
1 intubation in icatibant group

Straka Randomized, double- 30 ≤6 hr Icatibant 30 mg SC Time to symptom resolution: no difference


blind, placebo injection vs. placebo (p=0.20); intubations: 1 in icatibant vs. 2 in
control, multicenter placebo groups

ACE = angiotensin-converting-enzyme; IQR = interquartile range; SOC = standard of care.


Information from: Baş M, Greve J, Stelter K, et al. Therapeutic efficacy of icatibant in angioedema induced by angiotensin-converting
enzyme inhibitors: a case series. Ann Emerg Med 2010;56:278-82; Sinert R, Levy P, Bernstein JA, et al. Randomized trial of icatibant
for angiotensin-converting enzyme inhibitor–induced upper airway angioedema. J Allergy Clin Immunol Pract 2017;5:1402-9;
Straka BT, Ramirez CE, Byrd JB, et al. Effect of bradykinin receptor antagonism on ACE inhibitor-associated angioedema. J Allergy
Clin Immunol 2017;140:242-8.

CCSAP 2021 Book 1 • Pulmonary and Endocrinology 155 Immune-Mediated Reactions


volume is required of FFP to achieve the same plasma con- universally available, and relatively safe. It is also easy
centrations of the enzyme (Longhurst 2005). Practitioners to administer and does not require time to thaw, as is the
should be familiar with the volume of FFP administered per case with FFP. Several case reports and series have been
unit (200–300 mL/unit) because some patients may require published on using TXA for acute HAE and ACEI-induced
fluid restriction. angioedema (Horiuchi 2018, Mansi 2015, van den Elzen 2018).
Despite the evidence that FFP is generally effective for Tranexamic acid was used as a comparator in the FAST-2
treatment of acute angioedema, this practice remains con- study, which evaluated the role of icatibant for HAE. In FAST-
troversial. The decision to use FFP should consider the 2, TXA was dosed 3 g daily for 2 days (Cicardi 2010a). In 2018,
availability of other novel medications. If FFP is used for an a case series (n=33) was published in France that reported
acute episode, the medical team should be prepared to man- avoidance of intubation in all patients who were not intubated
age a paradoxical exacerbation should it occur. before receiving TXA. Dosing ranged from 0.5–4 g, with either
intravenous or oral administration, but the most common
Antifibrinolytics dosing regimen was 1 g intravenous infusion (Beauchêne
Antifibrinolytic agents, such as TXA, bind to plasminogen 2018). Evidence is limited and conflicting with regard to the
and inhibit the formation of plasmin. Plasmin directly initiates role of TXA in acute angioedema. However, with its low cost
complement activation and can consume C1-INH. By inhib- and low risk profile, some practitioners may consider TXA
iting plasmin, antifibrinolytics indirectly prevent kallikrein early in their treatment algorithm before progressing to novel
activation. Antifibrinolytics have been shown to reduce the fre- therapies if TXA fails to relieve symptoms.
quency and intensity of angioedema episodes; however, use is
most commonly reserved for HAE prophylaxis in patients who CONCLUSION
have experienced a failure of steroid therapy (Frank 1972). The management of patients experiencing immune-mediated
The use of TXA for acute angioedema episodes has gar- reactions can be complex and involves an understanding
nered attention in recent years because TXA is inexpensive, of how to manage shock as well as the delicate interplay of

Patient Care Scenario


A 40-year-old woman (weight 64.5 kg [142 lb]) presents to the acute angioedema episode and to specifically address
the ED with acute onset of swelling of the face and tongue. the use of C1 esterase inhibitors or therapies that mitigate
She has a known history of HAE and her episodes are typ- bradykinin. Which one of the following is best to recom-
ically triggered by stress. She states that she providing mend as first-line therapy for this patient?
care for her sick parent tonight when she developed acute A. Cinryze (sC1-INH) 1000 units intravenous
onset swelling of the lips and tongue, which was preceded B. Epinephrine 0.3 mg intramuscular
by perioral tingling. The emergency medicine physician
C. FFP 2 units
asks you to recommend a medication strategy to manage
D. Ruconest (conestat alfa) 50 units/kg intravenous

ANSWER
Management of angioedema occurs in two main phases: administration of steroids and antihistamines. If no his-
acute episode management and prophylaxis. If the patient tory of allergic triggers or history of anaphylaxis were
exhibits signs of stridor, drooling, hoarseness, dyspnea present, the choice of Ruconest (conestat alfa) would be
or dysphagia, advanced airway management may be the next best answer (Answer D is correct). Answer A is
required. Most patients with allergic angioedema respond incorrect because Cinryze has only received FDA approval
to epinephrine. A trial of therapies targeted at mitigating for a dose as prophylaxis for an episode of HAE. Given
histamine can serve as both a therapeutic and diagnos- that an option is provided for a concentrated C1-INH prod-
tic tool if this type of angioedema is suspected. Given uct, FFP would not be the best answer. Although FFP is
that this patient has a known history of HAE and reports effective, potential adverse effects associated with FFP
no possible allergic trigger, a histamine trial of 0.3 mg include transmission of bloodborne pathogens and trans-
given intramuscularly is not a reasonable first choice. fusion reactions. Furthermore, the risk of fluid overload
For nonallergic, bradykinin-mediated angioedema, these and subsequent acute lung injury precludes widespread
agents have limited use. For patients with nonaller- use of this therapy.
gic angioedema, only 2%–10% of patients improve after

1. Bramante RM. Angioedema. N Engl J Med 2011;365:e4.


2. Longhurst HJ. Emergency treatment of acute attacks in hereditary angioedema due to C1 inhibitor deficiency: what is the evidence? Int
J Clin Pract 2005;59:594-9.
3. Lenschow M, Bas M, Johnson F, et al. A score for the differential diagnosis of bradykinin- and histamine-induced head and neck swell-
ings. Eur Arch Otorhinolaryngol 2018;275:1767-73.
4. Richman MJ, Talan DA, Lumry WR. Treatment of laryngeal hereditary angioedema. J Emerg Med 2012;42:44-7.

CCSAP 2021 Book 1 • Pulmonary and Endocrinology 156 Immune-Mediated Reactions


of the therapies, pharmacists in these practice areas provide
Practice Points
drug therapy recommendations regarding selection, dosing,
Clinical pharmacists face many challenges as and monitoring of medications to prevent and treat complica-
they optimize pharmacotherapy for patients with
tions in these patients.
immune-mediated reactions. New data continue to
emerge regarding how to treat these rare conditions. As
a result, guidelines and recommendations, new indica- REFERENCES
tions for existing medications, new therapeutic entities, Alqurashi W, Stiell I, Chan K, et al. Epidemiology and clinical
and safety issues continue to evolve. An awareness of predictors of biphasic reactions in children with anaphy-
the following points is critical for the clinical pharma- laxis. Ann Allergy Asthma Immunol 2015;115:217-23.
cist working with this patient population:
Ameratunga R. Initial intravenous immunoglobulin doses
• Pharmacists should be aware of the types of hypersensi- should be based on adjusted body weight in obese
tivity reactions and have a strategy for identifying the likely patients with primary immunodeficiency disorders
trigger. Allergy Asthma Clin Immunol 2017;13:47.
• Early administration of epinephrine is recommended in the
treatment of anaphylaxis and anaphylactic shock. Banerji A, Clark S, Blanda M, et al. Multicenter study
• The clinical pharmacist should be aware of the delayed of patients with angiotensin-converting enzyme
onset of steroids and antihistamines in the treatment of inhibitor-induced angioedema who present to the
anaphylaxis. emergency department. Ann Allergy Asthma Immunol
• Angioedema is further subdivided into allergic and non- 2008;100:327-32.
allergic angioedema. Allergic angioedema may respond
to therapies such as antihistamines, epinephrine, and
Banta E, Horn P, Craig TJ. Response to ecallantide treatment
steroids. of acute attacks of hereditary angioedema based on time
• Nonallergic angioedema encompasses ACE inhibitor- to intervention: results from the EDEMA clinical trials.
induced angioedema as well as HAE. Allergy Asthma Proc 2011;32:319-24.
• Depending on the type of angioedema, the patient may Barron S, Del Vecchio MT, Aronoff SC. Intravenous immuno-
respond to C1 esterase inhibitor supplementation, globulin in the treatment of Stevens-Johnson syndrome
kallikrein inhibitors, or FFP. and toxic epidermal necrolysis: a meta-analysis with
• Dermatologic emergencies, although rare, are a widely meta-regression of observational studies. Int J Dermatol
complex group of conditions with the SJS and TEN being of 2015;54:108-15.
most significant concern.
• Supportive care of SJS/TEN is important, and experts Baş M, Greve J, Stelter K, et al. A randomized trial of icati-
suggest transferring these patients to a burn center. bant in ACE-inhibitor-induced angioedema. N Engl J Med
Resuscitation mimics burn resuscitation, except fluid 2015;372:418-25.
estimates are calculated using 2 mL/kg/hour for the first
Baş M, Greve J, Stelter K, et al. Therapeutic efficacy of
24 hours.
icatibant in angioedema induced by angiotensin-con-
• Although no study or guideline can confirm the clinical
verting enzyme inhibitors: a case series. Ann Emerg Med
benefit of corticosteroids or IVIG, many centers will incor-
porate these as part of standard of care.
2010;56:278-82.
• No single steroid is preferred, but the chosen agent should Bastuji-Garin S, Fouchard N, Bertocchi M, et al. SCORTEN: a
be administered at a high dose (at least 1 mg/kg pred- severity-of-illness score for toxic epidermal necrolysis.
nisone equivalents per day), used early in the course of J Invest Dermatol 2000;115:149-53.
disease, and used for the shortest duration possible. Also,
IVIG should be used at a high dose (at least 0.75 g/kg daily) Bastuji-Garin S, Rzany B, Stern RS, et al. Clinical classification
for 3-4 days. of cases of toxic epidermal necrolysis, Stevens-Johnson
syndrome, and erythema multiforme. Arch Dermatol
1993;129:92-6.
immune-modulating therapies. The life-threatening disorders Beauchêne C, Martins-Héricher J , Denis D, et al. Tranexamic
discussed in this chapter include anaphylaxis, dermatologic acid as first-line emergency treatment for episodes of bra-
emergencies, and angioedema. These disorders can be trig- dykinin-mediated angioedema induced by ACE inhibitors.
Rev Med Interne 2018;39:772-6.
gered by allergic (IgE) or nonallergic (non-IgE-mediated)
mechanisms. Although these disorders are distinct in many Bernstein JA, Moellman JJ, Collins SP, et al. Effectiveness
ways, all can involve the airway and lead to fatal airway of ecallantide in treating angiotensin-converting enzyme
obstruction that requires management in the ICU. inhibitor-induced angioedema in the emergency depart-
ment. Ann Allergy Asthma Immunol 2015;114:245-9.
After initial stabilization of these patients, the focus shifts
to identifying the trigger or cause of the reaction and the pre- Bezalel S, Mahlab-Guri K, Asher I, et al. Angiotensin-converting
vention of complications. Pharmacists play a substantial enzyme inhibitor-induced angioedema. Am J Med 2015;
128:120-5.
role in assisting in both the acute phases of these conditions
as well as in the prophylaxis, or prevention, of subsequent Binkley KE, Davis A, 3rd. Clinical, biochemical, and genetic
attacks. Given the cost as well as pharmacologic complexity characterization of a novel estrogen-dependent

CCSAP 2021 Book 1 • Pulmonary and Endocrinology 157 Immune-Mediated Reactions


inherited form of angioedema. J Allergy Clin Immunol Cohen MB, Saunders SS, Wise SK, et al. Pitfalls in the use
2000;106:546-50. of epinephrine for anaphylaxis: patient and provider
opportunities for improvement. Int Forum Allergy Rhinol
Blais C, Rouleau JL, Brown NJ, et al. Serum metabolism 2017;7:276-86.
of bradykinin and des-Arg9-bradykinin in patients with
angiotensin-converting enzyme inhibitor-associated Craig T, Levy RJ, Wasserman RL, et al. Efficacy of human C1
angioedema. Immunopharmacology 1999;43:293-302. esterase inhibitor concentrate compared with placebo
in acute hereditary angioedema attacks. J Allergy Clin
Bluestein HM, Hoover TA, Banerji AS, et al. Angiotensin-
Immunol 2009;124:801-8.
converting enzyme inhibitor-induced angioedema in a
community hospital emergency department. Ann Allergy Craig TJ, Bewtra AK, Bahna SL, et al. C1 esterase inhibitor
Asthma Immunol 2009;103:502-7. concentrate in 1085 hereditary angioedema attacks—final
results of the I.M.P.A.C.T.2 study. Allergy 2011;66:1604-11.
Bolton MR, Dooley-Hash SL. Angiotensin-converting enzyme
inhibitor angioedema. J Emerg Med 2012;43:e261-2. Creamer D, Walsh SA, Dziewulski P, et al. UK guidelines for
the management of Stevens-Johnson syndrome/toxic epi-
Bork K, Barnstedt S-E, Koch P, et al. Hereditary angioedema
dermal necrolysis in adults 2016. J Plast Reconstr Aesthet
with normal C1-inhibitor activity in women. Lancet
Surg 2016;69:e119-53.
2000;356:213-7.

Bova M, Guilarte M, Sala-Cunill A, et al. Treatment of ACEI- Eginli A, Shah K, Watkins C, et al. Stevens-Johnson syndrome
related angioedema with icatibant: a case series. In and toxic epidermal necrolysis. Ann Allergy Asthma
Emerg Med 2015;10:345-50. Immunol 2017;118:143-7.

Bramante RM, Rand M. Angioedema. N Engl J Med 2011; Fok JS, Katelaris CH, Brown AF, et al. Icatibant in angio-
365:e4. tensin-converting enzyme (ACE) inhibitor-associated
angioedema. Intern Med J 2015;45:821-7.
Brown NJ, Byiers S, Carr D, et al. Dipeptidyl peptidase-IV
inhibitor use associated with increased risk of ACE inhibi- Frank MM, Sergent JS, Kane MA, et al. Epsilon aminocaproic
tor-associated angioedema. Hypertension 2009;54:516-23. acid therapy of hereditary angioneurotic edema: a double-
blind study. N Engl J Med 1972;286:808-12.
Brown NJ, Ray WA, Snowden M, et al. Black Americans have
an increased rate of angiotensin converting enzyme French LE. Toxic epidermal necrolysis and Stevens Johnson
inhibitor-associated angioedema. Clin Pharmacol Ther syndrome: our current understanding. Allergol Int 2006;
1996;60:8-13. 55:9-16.

Brown SG, Stone SF, Fatovich DM, et al. Anaphylaxis: clinical Fugate JE, Kalimullah EA, Wijdicks EFM. Angioedema after
patterns, mediator release, and severity. J Allergy Clin tPA: what neurointensivists should know. Neurocrit Care
Immunol 2013;132:1141-9. 2012;16:440-3.

Byrd JB, Touzin K, Sile S, et al. Angiotensin-converting enzyme Gerull R, Nelle M, Schaible T. Toxic epidermal necrolysis
inhibitor-associated angioedema. Immunol Allergy Clin and Stevens-Johnson syndrome: a review. Crit Care Med
North Am 2006;26:725-37. 2011;39:1521-32.

Byrd JB, Touzin K, Sile S, et al. Dipeptidyl peptidase IV in Grabenhenrich LB, Dolle S, Moneret-Vautrin A, et al.
angiotensin-converting enzyme inhibitor-associated Anaphylaxis in children and adolescents: the European
angioedema. Hypertension 2008;51:141-7. Anaphylaxis Registry. J Allergy Clin Immunol 2016;
137:1128-37.e1.
Chiu AG, Newkirk KA, Davidson BJ, et al. Angiotensin-
converting enzyme inhibitor–induced angioedema: a Grunau BE, Wiens MO, Rowe BH, et al. Emergency depart-
multicenter review and an algorithm for airway management. ment corticosteroid use for allergy or anaphylaxis is not
Ann Otol Rhinol Laryngol 2001;110:834-40. associated with decreased relapses. Ann Emerg Med
2015;66:381-9.
Cho YT, Chu CY. Treatments for severe cutaneous adverse
reactions. J Immunol Res 2017;1503709. Guégan S, Bastuji-Garin S, Poszepczynska-Guigné E, et al.
Performance of the SCORTEN during the first five days
Cicardi M, Zanichelli A. Acquired angioedema. Allergy Asthma
of hospitalization to predict the prognosis of epidermal
Clin Immunol 2010;6:14.
necrolysis. J Invest Dermatol 2006;126:272-6.
Cicardi M, Agostoni A. Hereditary angioedema. N Engl J Med
Hague JS, Goulding JMR, Long TMW, et al. Respiratory
1996;334:1666-7.
involvement in toxic epidermal necrolysis portends a poor
Cicardi M, Banerji A, Bracho F, et al. Icatibant, a new bradyki- prognosis that may not be reflected in SCORTEN. Br J
nin-receptor antagonist, in hereditary angioedema. N Engl Dermatol 2007;157:1294-6.
J Med 2010a;363:532-41.
Halebian PH, Corder VJ, Madden MR, et al. Improved
Cicardi M, Levy RJ, McNeil DL, et al. Ecallantide for the treat- burn center survival of patients with toxic epidermal
ment of acute attacks in hereditary angioedema. N Engl J necrolysis managed without corticosteroids. Ann Surg
Med 2010b;363:523-31. 1986;204:503-12.

CCSAP 2021 Book 1 • Pulmonary and Endocrinology 158 Immune-Mediated Reactions


Hassen GW, Kalantari H, Parraga M, et al. Fresh frozen for acute attacks of hereditary angioedema. Ann Allergy
plasma for progressive and refractory angiotensin-con- Asthma Immunol 2010;104:523-9.
verting enzyme inhibitor-induced angioedema. J Emerg
Med 2013;44:764-72. Lewis LM, Graffeo C, Crosley P, et al. Ecallantide for the acute
treatment of angiotensin-converting enzyme inhibitor-in-
Heymann WR. Acquired angioedema. J Am Acad Dermatol duced angioedema: a multicenter, randomized, controlled
1997;36:611-5. trial. Ann Emerg Med 2015;65:204-13.
Hill BJ, Thomas SH, McCabe C. Fresh frozen plasma for Lieberman P, Camargo CA, Jr., Bohlke K, et al. Epidemiology
acute exacerbations of hereditary angioedema. Am J of anaphylaxis: findings of the American College of
Emerg Med 2004;22:633. Allergy, Asthma and Immunology Epidemiology of
Anaphylaxis Working Group. Ann Allergy Asthma Immunol
Hirschy RA, Shah T, Davis T, et al. Treatment of life-threaten-
2006;97:596-602.
ing ACE-inhibitor-induced angioedema. Adv Emerg Nurs J
2018;40:267-77. Liew WK, Chiang WC, Goh AE, et al. Paediatric anaphylaxis in
a Singaporean children cohort: changing food allergy trig-
Horiuchi T, Hide M, Yamashita K, et al. The use of tranexamic
gers over time. Asia Pac Allergy 2013;3:29-34.
acid for on-demand and prophylactic treatment of hered-
itary angioedema—a systematic review. J Allergy Clin Liew WK, Williamson E, Tang MLK. Anaphylaxis fatalities
Immunol 2018;1:126-38. and admissions in Australia. J Allergy Clin Immunol
Huang YC, Li YC, Chen TJ. The efficacy of intravenous immu- 2009;123:434-42.
noglobulin for the treatment of toxic epidermal necrolysis: Liyanage CK, Galappatthy P, Seneviratne SL. Corticosteroids
a systematic review and meta-analysis. Br J Dermatol in management of anaphylaxis:a systematic review of
2012;167:424-32. evidence. Eur Ann Allergy Clin Immunol 2017;49:196-207.
Javaud N, Charpentier S, Lapostolle F, et al. Angiotensin- Longhurst HJ. Emergency treatment of acute attacks in
converting enzyme inhibitor-induced angioedema and hereditary angioedema due to C1 inhibitor deficiency:
hereditary angioedema: a comparison study of attack what is the evidence? Int J Clin Pract 2005;59:594-9.
severity. Intern Med 2015;54:2583-8.
LoVerde D, Iweala OI, Eginli A, et al. Anaphylaxis. Chest 2018;
Jerschow E, Lin RY, Scaperotti MM, et al. Fatal anaphy- 153:528-43.
laxis in the United States, 1999-2010: temporal patterns
and demographic associations. J Allergy Clin Immunol Lumry WR, Li HH, Levy RJ, et al. Randomized placebo-
2014;134:1318-28. controlled trial of the bradykinin B2 receptor antagonist
icatibant for the treatment of acute attacks of heredi-
Kelemen JJ, 3rd, Cioffi WG, McManus WF, et al. Burn center
tary angioedema: the FAST-3 trial. Ann Allergy Asthma
care for patients with toxic epidermal necrolysis. J Am
Immunol 2011;107:529-37.
Coll Surg 1995;180:273-8.
Mansi M, Zanichelli A, Coerezza A, et al. Presentation, diag-
Kowalski ML, Asero R, Bavbek S, et al. Classification and
nosis and treatment of angioedema without wheals: a
practical approach to the diagnosis and management of
retrospective analysis of a cohort of 1058 patients.
hypersensitivity to nonsteroidal anti-inflammatory drugs.
J Intern Med 2015;277:585-93.
Allergy 2013;68:1219-32.
Marone G, Casolaro V, Cirillo R, et al. Pathophysiology of
Laidlaw TM, Cahill KN. Current knowledge and management
human basophils and mast cells in allergic disorders.
of hypersensitivity to aspirin and NSAIDs. Allergy Clin
Clin Immunol Immunopathol 1989;50:S24-40.
Immunol Pract 2017;5:537-45.

Latenser BA. Critical care of the burn patient: the first Miller DR, Oliveria SA, Berlowitz DR, et al. Angioedema inci-
48 hours. Crit Care Med 2009;37:2819-26. dence in US veterans initiating angiotensin-converting
enzyme inhibitors. Hypertension 2008;51:1624-30.
Lee HY, Lim YL, Thirumoorthy T, et al. The role of intravenous
immunoglobulin in toxic epidermal necrolysis: a retro- Nzeako UC, Frigas E, Tremaine WJ. Hereditary angioedema:
spective analysis of 64 patients managed in a specialized a broad review for clinicians. Arch Intern Med
centre. Br J Dermatol 2013;169:1304-9. 2001;161:2417-29.

Lenschow M, Bas M, Johnson F, et al. A score for the dif- Owens RE, Oliphant C. Angioedema spotlight: a closer exam-
ferential diagnosis of bradykinin- and histamine-induced ination of sacubitril/valsartan safety results. J Am Board
head and neck swellings. Eur Arch Otorhinolaryngol Fam Med 2017;30:556-7.
2018;275:1767-73. Pahs L Droege C, Kneale H, et al. A novel approach to the
Levy JH, Freiberger DJ, Roback J. Hereditary angioedema: treatment of orolingual angioedema after tissue plas-
current and emerging treatment options. Anesth Analg minogen activator administration. Ann Emerg Med
2010;110:1271-80. 2016;68:345-8.

Levy RJ, Lumry WR, McNeil DL, et al. EDEMA4: a phase 3, Palmieri TL, Greenhalgh DG, Saffle JR, et al. A multicenter
double-blind study of subcutaneous ecallantide treatment review of toxic epidermal necrolysis treated in U.S. burn

CCSAP 2021 Book 1 • Pulmonary and Endocrinology 159 Immune-Mediated Reactions


centers at the end of the twentieth century. J Burn Care clinical features, systemic manifestations, etiology, and
Rehabil 2002;23:87-96. immunopathogenesis. J Am Acad Dermatol 2013a;69:173.
e171-13.
Paradisi A, Abeni D, Bergamo F, et al. Etanercept therapy
for toxic epidermal necrolysis. J Am Acad Dermatol Schwartz RA, McDonough P, Lee BW. Toxic epidermal
2014;71:278-83. necrolysis: part II. Prognosis, sequelae, diag-nosis, differ-
ential diagnosis, prevention, and treatment. J Am Acad
Pekdemir M, Ersel M, Aksay E, et al. Effective treatment of Dermatol 2013b;69:187.e181-16.
hereditary angioedema with fresh frozen plasma in an
emergency department. J Emerg Med 2007;33:137-9. Shaker MS, Wallace DV, Golden DBK, et al. Anaphylaxis—a
2020 practice parameter update, systematic review, and
Pham CH, Gillenwater TJ, Nagengast E, et al. Combination grading of recommendations, assessment, development
therapy: etanercept and intravenous immunoglobulin for and evaluation (GRADE) analysis. J Allergy Clin Immunol
the acute treatment of Stevens-Johnson syndrome/toxic 2020;145:1082-123.
epidermal necrolysis. Burns 2019;45:1634-8.
Sheffer AL, Campion M, Levy RJ, et al. Ecallantide (DX-88) for
Pichler WJ, Adam J, Daubner B, et al. Drug hypersensitivity acute hereditary angioedema attacks: integrated analysis
reactions: pathomechanism and clinical symptoms. Med of 2 double-blind, phase 3 studies. J Allergy Clin Immunol
Clin North Am 2010;94:645-64. 2011;128:153-9.
Pickering RJ, Good RA, Kelly JR, et al. Replacement therapy Shi V, Senni M, Streefkerk H, et al. Angioedema in heart
in hereditary angioedema. Lancet 1969;293:326-30. failure patients treated with sacubitril/valsartan (LCZ696)
or enalapril in the PARADIGM-HF study. Int J Cardiol
Prematta M, Gibbs JG, Pratt EL, et al. Fresh frozen plasma
2018;264:118-23.
for the treatment of hereditary angioedema. Ann Allergy
Asthma Immunol 2007;98:383-8. Shiga S, Cartotto R. What are the fluid requirements in toxic
epidermal necrolysis? J Burn Care Res 2010;31:100-4.
Reber LL, Hernandez JD, Galli SJ. The pathophysiology of
anaphylaxis. J Allergy Clin Immunol 2017;140:335-48. Simons FE, Ardusso LR, Bilo MB, et al. World allergy organi-
zation guidelines for the assessment and management of
Revuz J, Penso D, Roujeau JC, et al. Toxic epidermal necroly-
anaphylaxis. World Allergy Organ J 2011;4:13-37.
sis. Clinical findings and prognosis factors in 87 patients.
Arch Dermatol 1987;123:1160-5. Simons FE, Ardusso LR, Dimov V, et al. World Allergy
Organization anaphylaxis guidelines: 2013 update of the
Richman MJ, Talan DA, Lumry WR. Treatment of laryngeal
evidence base. Int Arch Allergy Immunol 2013;162:193-204.
hereditary angioedema. J Emerg Med 2012;42:44-7.
Simons FE, Ebisawa M, Sanchez-Borges M, et al. 2015
Riedl MA, Bernstein JA, Li H, et al. Recombinant human
update of the evidence base: World Allergy Organization
C1-esterase inhibitor relieves symptoms of hereditary
anaphylaxis guidelines. World Allergy Organ J 2015;8:32.
angioedema attacks: phase 3, randomized, placebo-con-
trolled trial. Ann Allergy Asthma Immunol 2014;112:163-9. Sinert R, Levy P, Bernstein JA, et al. Randomized trial of icati-
bant for angiotensin-converting enzyme inhibitor–induced
Riedl MA, Levy RJ, Suez D, et al. Efficacy and safety of
upper airway angioedema. J Allergy Clin Immunol Pract
recombinant C1 inhibitor for the treatment of hereditary
2017;5:1402-9.
angioedema attacks: a North American open-label study.
Ann Allergy Asthma Immunol 2013;110:295-9. Sokumbi O, Wetter DA. Clinical features, diagnosis, and treat-
ment of erythema multiforme: a review for the practicing
Riedl MA, Hurewitz DS, Levy R, et al. Nanofiltered C1 ester-
dermatologist. Int J Dermatol 2012;51:889-902.
ase inhibitor (human) for the treatment of acute attacks
of hereditary angioedema: an open-label trial. Ann Allergy Stone SF, Cotterell C, Isbister GK, et al. Elevated serum
Asthma Immunol 2012;108:49-53. cytokines during human anaphylaxis: identification of
potential mediators of acute allergic reactions. J Allergy
Rocchio MA, Hussey AP, Southard RA, et al. Impact of ideal
Clin Immunol 2009;124:786-92.
body weight dosing for all inpatient i.v. immune globulin
indications. Am J Health Syst Pharm 2013;70:751-2. Stone SF, Phillips EJ, Wiese MD, et al. Immediate-type
hypersensitivity drug reactions. Br J Clin Pharmacol
Roujeau JC, Mockenhaupt M, Guillaume JC, et al. New 2014;78:1-13.
evidence supporting cyclosporine efficacy in epidermal
necrolysis. J Invest Dermatol 2017;137:2047-9. Straka BT, Ramirez CE, Byrd JB, et al. Effect of bradykinin recep-
tor antagonism on ACE inhibitor-associated angioedema.
Schneck J, Fagot JP, Sekula P, et al. Effects of treatments J Allergy Clin Immunol 2017;140:242-8.
on the mortality of Stevens-Johnson syndrome and toxic
epidermal necrolysis: a retrospective study on patients Temiño VM, Stokes Peebles R Jr. The spectrum and treat-
included in the prospective EuroSCAR Study. J Am Acad ment of angioedema. Am J Med 2008;121:282-6.
Dermatol 2008;58:33-40.
Trayes KP, Love G, Studdiford JS. Erythema multiforme: rec-
Schwartz RA, McDonough P, Lee BW. Toxic epidermal ognition and management. Am Fam Physician 2019;
necrolysis: part I. Introduction, history, classi-fication, 100:82-8.

CCSAP 2021 Book 1 • Pulmonary and Endocrinology 160 Immune-Mediated Reactions


Turner PJ, Gowland MH, Sharma V, et al. Increase in anaphy- Zimmermann S, Sekula P, Venhoff M, et al. Systemic immu-
laxis-related hospitalizations but no increase in fatalities: nomodulating therapies for Stevens-Johnson syndrome
an analysis of United Kingdom national anaphylaxis data, and toxic epidermal necrolysis: a systematic review and
1992-2012. J Allergy Clin Immunol 2015;135:956-63. meta-analysis. JAMA Dermatol 2017;153:514-22.

van den Elzen M, Go MFCL, Knulst AC, et al. Efficacy of Zisa G, Riccobono F, Calamari AM, et al. A case of protracted
treatment of non-hereditary angioedema. Clin Rev Allergy hypotension as unique symptom of a biphasic anaphylaxis
Immunol 2018;54:412-31. to amoxicillin. Eur Ann Allergy Clin Immunol 2009;41:60-1.

Warin RP, Cunliffe WJ, Greaves TB, et al. Recurrent Zuraw B, Cicardi M, Levy RJ, et al. Recombinant human
angioedema: familial and oestrogen-induced. Br J C1-inhibitor for the treatment of acute angioedema
Dermatol 1986;115:731-4. attacks in patients with hereditary angioedema. J Allergy
Clin Immunol 2010;126:821-7.
Warrier MR, Copilevitz CA, Dykewicz MS, et al. Fresh frozen
plasma in the treatment of resistant angiotensin-convert- Zuraw BL. Clinical practice. Hereditary angioedema. N Engl J
ing enzyme inhibitor angioedema. Ann Allergy Asthma Med 2008;359:1027-36.
Immunol 2004;92:573-5.
Zuraw BL, Bernstein JA, Lang DM, et al. A focused parame-
Werner R, Keller M, Woehrle JC. Facial angioedema and ter update: hereditary angioedema, acquired C1 inhibitor
stroke. Cerebrovasc Dis 2014;38:101-6. deficiency, and angiotensin-converting enzyme inhib-
itor-associated angioedema. J Allergy Clin Immunol
Wilkin JK, Hammond JJ, Kirkendal WM. The captopril-in- 2013;131:1491-3.
duced eruption: a possible mechanism: cutaneous kinin
potentiation. Arch Dermatol 1980;116:902-5.

Wood RA, Camargo CA, Jr., Lieberman P, et al. Anaphylaxis


in America: the prevalence and characteristics of ana-
phylaxis in the United States. J Allergy Clin Immunol
2014;133:461-7.

CCSAP 2021 Book 1 • Pulmonary and Endocrinology 161 Immune-Mediated Reactions


Self-Assessment Questions
Questions 16 and 17 pertain to the following case. epinephrine 5 minutes after the initial dose. The provider
K.T., a 20-year-old woman, presents to the ED in respiratory on shift would like to discharge the patient. Which one of
distress. She complains of an erythematous rash involving the following is best to recommend as the duration and
her face, neck and chest after eating at a restaurant 1 hour type of monitoring for this patient?
ago. K.T. is currently hemodynamically stable with a MAP of
A. At least 1-hour observation in the ED
70 mm Hg. She endorses respiratory involvement with the
B. Admission to the ICU
sensation of her throat tightening as well as tachypnea.
C. At least 6-hour observation in the ED
16. Which one of the following is best to recommend admin- D. Admission with a 12–24 hour monitoring period
istering to K.T.?
20. A patient is admitted to the hospital with complaints of a
A. Diphenhydramine 50 mg intravenously rash with target-appearing lesions. The patient endorses
B. Epinephrine at 0.3 mg as an intramuscular injection that this rash started recently 2 weeks after developing
C. Epinephrine as a continuous infusion an outbreak of herpes lesions. The lesions are limited to
D. Oral prednisone at 1 mg/kg as a single dose the extremities. There is a negative Nikolsky sign. He has
17. K.T.’s physician requests to use intravenous immune otherwise felt fine and is in his normal state of health.
globulin (IVIG) with the rationale that the patient has a Which one of the following best evaluates this patient’s
strong family history of hypersensitivity reactions and presentation?
will likely progress to SJS. Which one of the following is
A. Herpetic lesions associated with herpes simplex
best to recommend for K.T.?
virus type 1
A. Administer IVIG at a dose of 2 g/kg as a single dose B. Erythema multiforme (EMM)
in addition to supportive care. C. SJS
B. Administer IVIG at a dose of 0.4 g/kg daily for a D. Toxic epidermal necrolysis (TEN)
duration of 3–5 days.
21. A 65-year-old African American man is transferred to
C. Initiate the process of enrolling her in a clinical trial.
your burn unit from an outside facility with a 4-day his-
D. Continue management with fluids and vasoactive
tory of fever, malaise, and cough. This was followed by
medications.
a widespread, diffuse cutaneous reaction with a pos-
18. You are a pharmacist practicing in the ED. Epineph- itive Nikolsky sign. The patient’s family states that he
rine is stocked in the automated dispensing cabinets in was started on phenytoin about 3 weeks ago for man-
concentrations of 1 mg/mL and 0.1 mg/mL. During the agement of seizures. The patient was intubated prior to
resuscitation of an adult patient experiencing anaphy- arrival because the lesions involved the entirety of the
laxis, a provider asks for “one of epinephrine”, which a trunk, head, face, upper extremities, and mouth/oral
nurse mistakenly interprets as 1 mg of epinephrine. This cavity. Based on review from the outside facility, his
is administered intravenously. The patient experiences a renal and hepatic function are within normal limits for
ventricular dysrhythmia as a result, requiring admission age. Which one of the following best characterizes this
to the ICU. Which one of the following best classifies the patient’s cutaneous reaction?
degree of severity of this patient’s adverse drug event
A. EMM
(ADE)?
B. SJS
A. Mild C. TEN
B. Moderate D. Drug reaction with eosinophilia and systemic
C. Severe symptoms (DRESS)
D. No classification
19. A 19-year-old man presents to the hospital after devel- Questions 22 and 23 pertain to the following case.
oping symptoms of anaphylaxis 15 minutes after eating You are a pharmacist in a certified burn treatment center. L.P.
something at a football game. The patient endorses is 42-year-old woman (weight 85 kg) who arrives through the
a history of anaphylaxis with multiple admissions in ED. She has been diagnosed with TEN, and about 55% of her
the past, the most recent being 1 year ago. His current total body surface area (TBSA) has full epidermal detach-
symptoms include diffuse, erythematous rash as well ment. L.P. has been intubated for airway protection and is
as orolaryngeal edema. Pre-hospital staff adminis- being mechanically ventilated and appropriately sedated. Her
tered a dose of epinephrine 0.3 mg intramuscularly. The family reports L.P.’s cutaneous lesions developed over the
patient responds after a second dose of intramuscular last 24 hours and have progressively worsened over that time.

CCSAP 2021 Book 1 • Pulmonary and Endocrinology 162 Immune-Mediated Reactions


The ED has started 2 L of lactated Ringers (LR). L.P.’s pre- 26. A 28-year-old white woman is admitted for severe, dif-
senting heart rate is 110 beats/minute, serum glucose is 200 fuse abdominal pain and vomiting. She has had 15 ED
mg/dL, BUN is 29 mg/dL, lactate is 6 mg/dL and her serum visits for similar symptoms over the course of the last
bicarbonate is 18 mEq/L. year. The patient denies infection, recent travel, and any
changes in diet or medications. She denies any contrib-
22. Which one of the following equations would best esti-
utory medical history or family history and is only on an
mate L.P.’s fluid needs for the first 24 hours?
oral contraceptive. An abdominal CT reveals small bowel
A. 2 mL/kg/hour/% TBSA of lactated Ringers edema. The admitting team is concerned for heredi-
B. 4 mL/kg/hour/% TBSA of 0.9% sodium chloride tary angioedema (HAE). A complement panel is ordered
C. 0.5 mL/kg/% TBSA of 5% albumin and reveals the following: C4 level low, C1-INH level low,
D. 1.5 mL/kg/% burn plus colloids 0.5 mL/kg/% burn C1-INH function low, C1q level normal. Which one of
plus 2000 mL glucose in water the following best classifies the cause of this patient’s
23. Using the Score of Toxic Epidermal Necrosis (SCORTEN) angioedema?
scoring tool, which one of the following best estimates A. HAE type I
L.P.’s calculated risk of mortality? B. HAE type II
A. 1 point = 4% C. AAE type II
B. 2 points = 12% D. ACE inhibitor-induced angioedema
C. 3 points = 32% 27. A 56-year-old man presents to the hospital for unilateral
D. 4 points = 62% tongue and facial swelling. The swelling began last night,
24. Your institution’s formulary committee has asked you to but he decided to come to the hospital after he noticed
review literature pertaining to active therapies for SJS difficulty swallowing. Physical examination is posi-
and TEN. Which one of the following responses best tive for stridor and dyspnea and negative for urticaria
assesses current evidence? or rash. The patient has no contributory family history
or history of similar attacks. He has no known medica-
A. IVIG showed a shorter time to complete resolution of
tion allergies. A home drug reconciliation is completed
symptoms compared with corticosteroids.
and lists the following: metformin 500 mg twice daily,
B. Etanercept showed a shorter ICU length of stay
lisinopril 10 mg daily, metoprolol succinate 50 mg daily,
compared with placebo.
sacubitril/valsartan 24/26 mg twice daily, aspirin 81 mg
C. Cyclosporine showed a reduction in mortality with
daily, and furosemide 20 mg twice daily. He reports that
no increase in adverse events.
the sacubitril/valsartan is a new prescription started 2
D. No treatment has been shown to reduce mortality
days ago. He also acknowledges recently completing a
when compared with supportive care.
course of cefdinir for a UTI with his last dose about 3
25. A patient is being managed in your burn unit for treat- days ago. A decision is made to intubate and the patient
ment of SJS/TEN. The patient is now 48 hours out from is admitted to your ICU service. Which one of the follow-
the time of initial presentation and fluid resuscitation. ing is the most likely cause of this patient’s drug-induced
The patient is intubated and appropriately sedated while angioedema?
being mechanically ventilated. There are no signs of pul-
A. Cefdinir
monary edema. Bowel sounds are present and optimal
B. Aspirin
nutrition has been initiated. A Foley catheter is in place
C. Furosemide
with a bladder pressure of 3 mm Hg and a urine output of
D. Sacubitril/valsartan
0.5 mL/kg/hour. The team is concern for fluid creep and
overall volume status. Which one of the following is best 28. A 45-year-old African American woman with a medical
to recommend for this patient? history of hypertension and diabetes presents to the ED
for lip swelling and tingling. Symptom onset occurred
A. Consult a nephrologist for evaluation for dialysis to
about 4 hours ago. The patient felt flushed and noticed
manage fluid status.
the swelling after breakfast. Although the swelling has
B. Monitor urine output as that is the most sensitive
not gotten worse, she decided to come in because it
marker for overall fluid status.
hasn’t gone away. The patient denies any recent trauma,
C. Consider diuretics to increase urine output.
infection, or changes to diet or medications. Her home
D. Continue use of bladder pressures for monitoring
drugs include aspirin 81 mg daily, metformin 500 mg
for volume overload and compartment syndrome
twice daily, amlodipine 10 mg daily, and lisinopril 5 mg
following the initial resuscitation period.
daily. She does not endorse any difficulty swallowing,

CCSAP 2021 Book 1 • Pulmonary and Endocrinology 163 Immune-Mediated Reactions


hoarseness, or voice changes. Which one of the follow- 30. A 33-year-old woman with a medical history of type I HAE
ing is best to recommend for initial treatment of this presents to the hospital for persistent abdominal pain.
patient’s angioedema? The patient states that she administered icatibant 30 mg
subcutaneously at home when she felt the symptoms
A. Administer icatibant 30 mg subcutaneously.
beginning. However, her abdominal pain has persisted
B. Administer C1 esterase inhibitor (Berinert) 20
which led her to come to the hospital. She brought a
units/kg intravenously.
repeat dose of her icatibant to the hospital with her, and
C. Administer diphenhydramine, famotidine, and
your hospital does have a patient home medication pol-
methylprednisolone.
icy that allows for use. Bedside ultrasound is concerning
D. Prepare medications for rapid sequence intubation
for free fluid in her abdomen. In addition to treating her
due to airway compromise.
pain, which one of the following is best to recommend as
29. Your institution’s formulary committee has asked you to initial treatment for this patient’s angioedema?
review literature pertaining to target therapies for acute
A. C1 esterase inhibitor (Cinryze) 1000 units
HAE attacks. Which one of the following statements best
intravenously
reflects current evidence?
B. Repeat dose of icatibant 30 mg subcutaneously
A. IMPACT2 showed a shorter time to complete C. Diphenhydramine, famotidine, methylprednisolone
resolution of symptoms when using C1 esterase and epinephrine
inhibitor (Berinert) compared with FFP. D. 1 unit FFP
B. EDEMA-3 reported a statistically shorter median
time to significant improvement with ecallantide
compared with placebo.
C. FAST-2 showed similar time to clinically significant
symptom relief between icatibant and tranexamic
acid.
D. IMPACT1 established time to onset of symptom
relief was improved with C1 esterase inhibitor
(Berinert) 20 units/kg dosing regimen compared
with placebo.

CCSAP 2021 Book 1 • Pulmonary and Endocrinology 164 Immune-Mediated Reactions


Learner Chapter Evaluation: Immune-Mediated Reactions

As you take the posttest for this chapter, also evaluate the 25. The learning assessment activities used in the chapter
material’s quality and usefulness, as well as the achievement were effective.
of learning objectives. Rate each item using this 5-point scale: 26. The chapter was effective overall.

• Strongly agree 27. The activity met the stated learning objectives.
• Agree 28. If any objectives were not met, please list them here.
• Neutral
• Disagree OTHER COMMENTS
• Strongly disagree
29. Please provide any specific comments related to any
16. The content of the chapter met my educational needs. perceptions of bias, promotion, or advertisement of
17. The content of the chapter satisfied my expectations. commercial products.
18. The author presented the chapter content effectively. 30. Please expand on any of your above responses, and/or
provide any additional comments regarding this chapter:
19. The content of the chapter was relevant to my practice
and presented at the appropriate depth and scope.
20. The content of the chapter was objective and balanced. Questions 31–33 apply to the entire learning module.

21. The content of the chapter is free of bias, promotion, and 31. How long did it take you to read the instructional materi-
advertisement of commercial products. als in this module?

22. The content of the chapter was useful to me. 32. How long did it take you to read and answer the assess-
ment questions in this module?
23. The teaching and learning methods used in the chapter
were effective. 33. Please provide any additional comments you may have
regarding this module:
24. The active learning methods used in the chapter were
effective.

CCSAP 2021 Book 1 • Pulmonary and Endocrinology 165 Immune-Mediated Reactions


Pulmonary and Endocrinology IV
Pulmonary and Endocrinology IV Panel

Series Editors: Reviewers


Bradley A. Boucher, Pharm.D., FCCP, FNAP, MCCM, BCPS Patrick M. Wieruszewski, Pharm.D., BCCCP
Professor of Clinical Pharmacy and Translational Science Pharmacist, Cardiothoracic Surgery/Anesthesia ICU
Associate Dean for Strategic Initiatives and Operations Assistant Professor of Anesthesiology and Pharmacy
College of Pharmacy Department of Pharmacy
University of Tennessee Health Science Center Mayo Clinic
Memphis, Tennessee Rochester, Minnesota
Curtis E. Haas, Pharm.D., FCCP Yanjun Maggie Zhao, Pharm.D., BCCCP, BCPS
Chief Pharmacy Officer Clinical Pharmacist, Medication Knowledge Management
University of Rochester Medical Center Department of Pharmacy
Rochester, New York Mayo Clinic
Rochester, Minnesota
Faculty Panel Chair:
Alexander H. Flannery, Pharm.D., FCCM, BCCCP, BCPS
Interactive Case: Neuromuscular
Assistant Professor
University of Kentucky College of Pharmacy Blocking Agents
Lexington, Kentucky Author
Sara J. Hyland, Pharm.D., BCCCP
Interactive Case: Acute Respiratory Clinical Pharmacist
Distress Syndrome Department of Pharmacy Services
Grant Medical Center (OhioHealth)
Author
Columbus, Ohio
Stephanie Bass, Pharm.D., BCPS, BCCCP
Medical ICU Clinical Pharmacist Reviewers
Department of Pharmacy Eric G. Johnson, Pharm.D., BCCCP
Cleveland Clinic Perioperative Critical Care Pharmacist
Cleveland, Ohio Department of Acute Care Pharmacy Services
University of Kentucky HealthCare
Reviewers
Lexington, Kentucky
Mitchell S. Buckley, Pharm.D., FCCP, FASHP, FCCM, BCCCP
Jaimini S. Patel, Pharm.D., BCCCP
Clinical Pharmacy Specialist
Pharmacy Clinical Specialist—Critical Care
Department of Pharmacy
Department of Pharmacy
Banner University Medical Center Phoenix
Cleveland Clinic Akron General Medical Center
Phoenix, Arizona
Akron, OH
Mahmoud A. Ammar, Pharm.D., BCPS, BCCCP
The American College of Clinical Pharmacy and the authors
Critical Care Clinical Pharmacy Specialist, Trauma/Surgical
Department of Pharmacy Services thank the following individuals for their careful review of the
Yale-New Haven Health Pulmonary and Endocrinology IV features:
New Haven, Connecticut Jennifer Le, Pharm.D., MAS, FCCP, FCSHP, FIDSA, BCPS-ID
Professor of Clinical Pharmacy
Interactive Case: Extracorporeal Skaggs School of Pharmacy and Pharmaceutical Sciences
University of California, San Diego
Membrane Oxygenation La Jolla, California
Author Michael C. Thomas, Pharm.D., FCCP, BCPS
Jennifer Falvey, Pharm.D., BCPS Professor, Pharmacy Practice 
Cardiovascular Pharmacy Specialist McWhorter School of Pharmacy
University of Rochester Medical Center Samford University
Rochester, New York Birmingham, Alabama
DISCLOSURE OF POTENTIAL CONFLICTS OF INTEREST
Consultancies: Stephanie Bass (Entasis); Mitchell S. Buckley (Wolters Kluwer); Chris Droege (Deloitte Consulting LLP); Eric Johnson
(Slayback Pharmaceuticals); Patrick M. Wieruszewski (La Jolla Pharmaceutical Company); G. Christopher Wood (ASHP)

Stock Ownership:

Royalties:

Grants: Brittany Bissell (ASHP, University of Kentucky); Chris Droege (ACCP); Alexander H. Flannery (ACCP, American Society
of Nephrology, La Jolla Pharmaceutical Company); Anthony Hawkins (University of Georgia College of Pharmacy); Heather P.
May (ACCP, Mayo Clinic); Andrea Sikora Newsome (NIH); Joseph M. Swanson (Cubist/Merck)

Honoraria: Abby Bailey (ACCP EM PRN); Joseph M. Swanson (United Arab Emirates – Ministry of Health)

Other:

Nothing to disclose: Mahmoud A. Ammar, Paige Garber Bradshaw, Jennifer Falvey, Maria Guido, Leslie A. Hamilton, Sara Jordan
Hyland, Heather A. Johnson, Debbie Liang, Brian P. Murray, Andrea M. Nei, Jaimini S. Patel, Michael A. Peters, Michael A. Rudoni,
Poorvi Shah, Zachary R. Smith, Jaclyn Stoffel, Kyle A. Weant, Maggie Zhao

ROLE OF BPS: The Board of Pharmacy Specialties (BPS) is an autonomous division of the American Pharmacists Association
(APhA). To maintain its strict, independent standards for certification, BPS does NOT endorse or provide review information,
preparatory courses, or study guides for Board Certification Examinations. The Board, through its specialty councils, is respon-
sible for specialty examination content, administration, scoring, and all other aspects of its certification programs. BPS is totally
separate and distinct from ACCP. CCSAP has been approved by BPS for use in BCCCP recertification. Information about the BPS
recertification process is available online.

Questions regarding BCCCP recertification should be directed to:

Board of Pharmacy Specialties


2215 Constitution Avenue NW
Washington, DC 20037
(202) 429-7591
CONTINUING PHARMACY EDUCATION
AND RECERTIFICATION INSTRUCTIONS
Continuing Pharmacy Education Credit: The American College of Clinical Pharmacy is accredited by the Accreditation
Council for Pharmacy Education (ACPE) as a provider of continuing pharmacy education (CPE).

CCSAP Target Audience: The target audience for Pulmonary and Endocrinology is not only ICU and ED pharmacists across the
spectrum of care but also any pharmacist caring for acutely ill patients whose management may be complicated by these
challenging scenarios.

Available CPE credits: Purchasers who successfully complete all posttests for CCSAP 2021 Book 1 (Pulmonary and Endocrinology)
can earn 22.5 contact hours of continuing pharmacy education credit. The universal activity numbers are as follows: Pulmonary
and Endocrinology I, 0217-0000-21-030-H01-P, 5.5 contact hours; Pulmonary and Endocrinology II, 0217-0000-21-031-H01-P,
5.0 contact hours; Pulmonary and Endocrinology III, 0217-0000-21-032-H01-P, 5.5 contact hours; and Pulmonary and Endocrinology IV,
0217-0000-21-033-H01-P, 6.5 contact hours. You may complete one or all available modules for credit. Tests may not be submitted
more than one time.

TO EARN CPE CREDITS FROM THIS CCSAP BOOK

Posttest access: Go to www.accp.com and sign in with your e-mail address and password. Technical support is available from 8 a.m.
to 5 p.m. (Central) weekdays by calling (913) 492-3311. CCSAP products are listed under My Products on your My Account page.

BCCCP test deadline: 11:59 p.m. (Central) on September 15, 2021.


ACPE test deadline: 11:59 p.m. (Central) on March 15, 2024.

BCCCP Recertification Credit: To receive BCCCP recertification CPE credit, a CCSAP posttest must be submitted
within the 6-month period after the book’s release (see above). Only completed tests are eligible for credit; no
partial or incomplete tests will be processed. You may complete one or all available modules for credit. Tests may
not be submitted more than one time.

The passing point to earn BCCCP recertification credit is based on an expert analysis of the assessment items in each posttest
module. Any posttest submitted before the BCCCP test deadline that meets this passing point will earn BCCCP recertification
credits. These credits will be assigned as of the date of test submission and reported within 48 hours to BPS. For statements of
recertification credit, visit www.bpsweb.org.

Questions regarding the number of hours required for BCCCP recertification should be directed to BPS at (202) 429-7591 or
www.bpsweb.org. The ACCP Recertification Dashboard is a free online tool that can track recertification credits as they are
earned through ACCP and schedule new opportunities for credits from upcoming ACCP professional development programs.

ACPE CPE Credit: To receive ACPE CPE credit for a CCSAP module, a posttest must be submitted within 3 years after the book’s
release (see above). Only completed tests are eligible for credit; no partial or incomplete tests will be processed. You may complete
one or all available modules for credit. Tests may not be submitted more than one time.

Any posttest submitted before the ACPE deadline that scores 50% or greater will be awarded the appropriate CPE. These
credits will be assigned as of the date of test submission and reported within 48 hours. For statements of CPE credit, visit
www.mycpemonitor.net.

Posttest answers: The explained answers—with rationale and supporting references—will be posted 2 weeks after the BCCCP
test deadline and will be available to anyone who has either (1) submitted a posttest or (2) waived the right to receive credit from
a posttest (see below). Go to www.accp.com and sign in with your e-mail address and password. Click the CCSAP book on your
My Account page and you will see a link to the explained answers.

Test Waivers: To access the explained answers without submitting a posttest, sign in to your My Account page, select the
CCSAP book, and click on the waiver link for that module. By completing the waiver form for a module, you waive the opportunity
to receive CPE credit for that module. After you submit a waiver, you will see a link to the PDF file that contains the answers for
the module you waived. Answers will be available starting 2 weeks after the BCCCP test deadline.
Interactive Case: Acute Respiratory
Distress Syndrome
By Stephanie Bass, Pharm.D., BCPS, BCCCP

Reviewed by Mitchell S. Buckley, Pharm.D., FCCP, FASHP, FCCM, BCCCP; and Mahmoud A. Ammar, Pharm.D., BCPS, BCCCP

LEARNING OBJECTIVES

1. Apply guideline recommendations for the use of prone positioning, extracorporeal membrane oxygenation, extracorporeal
carbon dioxide removal, and inhaled vasodilators in acute respiratory distress syndrome (ARDS).
2. Distinguish the use of neuromuscular blocking agents and glucocorticoids in the treatment of ARDS on the basis of recent
evidence.
3. Evaluate current evidence regarding the potential role of vitamins in ARDS treatment.
4. Design an appropriate fluid management strategy for a patient with ARDS.
5. Assess patients for the various phenotypes of ARDS and recommend specific drug therapy on the basis of phenotype.

ABBREVIATIONS IN THIS FEATURE


ABG Arterial blood gas BASELINE KNOWLEDGE STATEMENTS

AKI Acute kidney injury Readers of this feature are presumed to be familiar
ARDS Acute respiratory distress syndrome with the following:
COVID-19 Coronavirus disease 2019 • General definitions and pathophysiology of ARDS
CVP Central venous pressure • Sedation and analgesia strategies and monitoring
ECCO2R Extracorporeal carbon dioxide (e.g., RASS)
removal
• Common nonpharmacologic and pharmacologic
ECMO Extracorporeal membrane management of ARDS
oxygenation
LCA Latent class analysis
• Basic mechanical ventilator parameters including
tidal volume, PEEP, and plateau pressure and goals
NMBA Neuromuscular blocking agent for these settings with lung-protective ventilation
PCWP Pulmonary capillary wedge
pressure Table of common laboratory reference values.
Po2/Fio2 Ratio of arterial partial oxygen
pressure to fraction of inspired ADDITIONAL READINGS
oxygen
RASS Richmond Agitation-Sedation The following resources have additional background
Scale (score) information on this topic:
SAPS II Simplified Acute Physiology II • Bos LD, Martin-Loeches I, Schultz MJ. ARDS:
(score) challenges in patient care and frontiers in research.
SOFA Sequential organ failure Eur Respir Rev 2018;27:170107.
assessment (score)
• Cawley MJ. Mechanical ventilation: a tutorial for
sRAGE Soluble form of receptor for pharmacists. Pharmacotherapy 2007;27:250-66.
advanced glycation end products
• Fan E, Brodie D, Slutsky AS. Acute respiratory
TOF Train-of-four distress syndrome: advances in diagnosis and
treatment. JAMA 2018;319:698-710.
Table of other common abbreviations.
• Ranieri VM, Rubenfeld GD, Thompson BT, et al.
Acute respiratory distress syndrome: the Berlin
definition. JAMA 2012;307:2526-33.

CCSAP 2021 Book 1 • Pulmonary and Endocrinology 173 Interactive Case: Acute Respiratory Distress Syndrome
Bein T, Weber-Carstens S, Goldmann A, et al. Lower tidal
Practice Points volume strategy (approximately 3 ml/kg) combined
• Acute respiratory distress syndrome (ARDS) is a broad with extracorporeal CO2 removal versus “conventional”
definition for acute hypoxemia with bilateral opacities not protective ventilation (6 mL/kg) in severe ARDS: the pro-
explained by heart failure or volume overload; therefore, spective randomized Xtravent-study. Intensive Care Med
treatment of ARDS is usually based on both timing (early or 2013;39:847-56.
late) and severity (based on degree of hypoxemia).
• Rescue therapies for ARDS, including prone positioning, Bhimraj A, Morgan RL, Shumaker AH, et al. Infectious
inhaled vasodilators, and extracorporeal membrane oxy- Diseases Society of America guidelines on the treatment
genation (ECMO), are limited to use in patients with severe and management of patients with COVID-19. Clin Infect
hypoxemia despite optimal lung-protective ventilation Dis 2020.
strategies.
Bos LD, Martin-Loeches I, Schultz MJ. ARDS: challenges
• Use of continuous infusion neuromuscular blocking agents
in patient care and frontiers in research. Eur Respir Rev
(NMBAs) should be reserved for patients with moderate to
2018;27.
severe ARDS who develop patient-ventilator dyssynchrony
or refractory hypoxemia while using optimal lung-protec- Brant EB, Angus DC. Is high-dose vitamin C beneficial for
tive ventilation strategies. patients with sepsis? JAMA 2019;322:1257-8.
• Corticosteroids in ARDS appears to be most beneficial
when used early in the course of moderate to severe ARDS. Calfee CS, Delucchi K, Parsons PE, et al. Subphenotypes in
• The role of high-dose vitamin C and D therapy in ARDS acute respiratory distress syndrome: latent class analy-
management requires evaluation in well-designed studies sis of data from two randomised controlled trials. Lancet
before it is broadly applied. Respir Med 2014;2:611-20.
• Conservative fluid management in patients with ARDS who
are hemodynamically stable is associated with improved Combes A, Hajage D, Capellier G, et al. Extracorporeal mem-
ventilator-free days and reduced ICU length of stay. brane oxygenation for severe acute respiratory distress
• Many ARDS management strategies, including fluid man- syndrome. N Engl J Med 2018;378:1965-75.
agement, will likely be evaluated on the basis of various
Dancer RC, Parekh D, Lax S, et al. Vitamin D deficiency
ARDS phenotypes to best identify which patient subpopu-
contributes directly to the acute respiratory distress
lations will benefit from therapy.
syndrome (ARDS). Thorax 2015;70:617-24.

Devlin JW, Skrobik Y, Gelinas C, et al. Clinical practice


INTERACTIVE CASE: ARDS guidelines for the prevention and management of pain,
agitation/sedation, delirium, immobility, and sleep
• Click here to start this CCSAP activity. disruption in adult patients in the ICU. Crit Care Med
2018;46:e825-e73.
REFERENCES
Famous KR, Delucchi K, Ware LB, et al. Acute respiratory
Adhikari NK, Dellinger RP, Lundin S, et al. Inhaled nitric oxide
distress syndrome subphenotypes respond differently to
does not reduce mortality in patients with acute respira-
randomized fluid management strategy. Am J Respir Crit
tory distress syndrome regardless of severity: systematic
Care Med 2017;195:331-8.
review and meta-analysis. Crit Care Med 2014;42:404-12.
Afshari A, Brok J, Moller AM, et al. Aerosolized prostacyclin Forel JM, Roch A, Marin V, et al. Neuromuscular blocking
for acute lung injury (ALI) and acute respiratory distress agents decrease inflammatory response in patients pre-
syndrome (ARDS). Cochrane Database Syst Rev senting with acute respiratory distress syndrome. Crit
2010;8:CD007733. Care Med 2006;34:2749-57.

Akoumianaki E, Lyazidi A, Rey N, et al. Mechanical venti- Fowler AA, Syed AA, Knowlson S, et al. Phase I safety trial of
lation-induced reverse-triggered breaths: a frequently intravenous ascorbic acid in patients with severe sepsis.
unrecognized form of neuromechanical coupling. Chest J Transl Med 2014;12:32.
2013;143:927-38.
Fowler AA, Truwit JD, Hite RD, et al. Effect of vitamin C infu-
Alhazzani W, Moller MH, Arabi YM, et al. Surviving Sepsis sion on organ failure and biomarkers of inflammation and
Campaign: guidelines on the management of critically ill vascular injury in patients with sepsis and severe acute
adults with coronavirus disease 2019 (COVID-19). Intensive respiratory failure: the CITRIS-ALI randomized clinical
Care Med 2020;46:854-87. trial. JAMA 2019;322:1261-70.
Amrein K, Schnedl C, Holl A, et al. Effect of high-dose vitamin
Gainnier M, Roch A, Forel JM, et al. Effect of neuromuscular
D3 on hospital length of stay in critically ill patients with
blocking agents on gas exchange in patients presenting
vitamin D deficiency: the VITdAL-ICU randomized clinical
with acute respiratory distress syndrome. Crit Care Med
trial. JAMA 2014;312:1520-30.
2004;32:113-9.
Arabi YM, Mandourah Y, Al-Hameed F, et al. Corticosteroid
therapy for critically ill patients with Middle East Gattinoni L, Caironi P, Cressoni M, et al. Lung recruitment in
Respiratory Syndrome. Am J Respir Crit Care Med patients with the acute respiratory distress syndrome.
2018;197:757-67. N Engl J Med 2006;354:1775-86.

CCSAP 2021 Book 1 • Pulmonary and Endocrinology 174 Interactive Case: Acute Respiratory Distress Syndrome
Gattinoni L, Chiumello D, Caironi P, et al. COVID-19 pneu- survivors of acute lung injury. Am J Respir Crit Care Med
monia: different respiratory treatments for different 2012;185:1307-15.
phenotypes? Intensive Care Med 2020;46:1099-102.
Moss M, Huang DT, Brower RG, et al. Early neuromuscular
Gattinoni L, Pelosi P, Suter PM, et al. Acute respiratory blockade in the acute respiratory distress syndrome.
distress syndrome caused by pulmonary and extrapulmo- N Engl J Med 2019;380:1997-2008.
nary disease. Different syndromes? Am J Respir Crit Care
Munshi L, Del Sorbo L, Adhikari NKJ, et al. Prone position for
Med 1998;158:3-11.
acute respiratory distress syndrome. A systematic review
Gebistorf F, Karam O, Wetterslev J, et al. Inhaled nitric and meta-analysis. Ann Am Thorac Soc 2017;14:S280-S8.
oxide for acute respiratory distress syndrome (ARDS)
Papazian L, Aubron C, Brochard L, et al. Formal guidelines:
in children and adults. Cochrane Database Syst Rev
management of acute respiratory distress syndrome.
2016;6:CD002787.
Ann Intensive Care 2019;9:69.
Ginde AA, Brower RG, Caterino JM, et al. Early high-dose
Papazian L, Forel JM, Gacouin A, et al. Neuromuscular
vitamin D3 for critically ill, vitamin D-deficient patients.
blockers in early acute respiratory distress syndrome.
N Engl J Med 2019;381:2529-40.
N Engl J Med 2010;363:1107-16.
Goligher EC, Tomlinson G, Hajage D, et al. Extracorporeal Park S, Schmidt M. Early neuromuscular blockade in mod-
membrane oxygenation for severe acute respiratory dis- erate to severe acute respiratory distress syndrome: do
tress syndrome and posterior probability of mortality not throw the baby out with the bathwater! J Thorac Dis
benefit in a post hoc Bayesian analysis of a randomized 2019;11:E231-E4.
clinical trial. JAMA 2018;320:2251-9.
Park SY, Kim HJ, Yoo KH, et al. The efficacy and safety of
Greenberg SB, Vender J. The use of neuromuscular block- prone positioning in adults patients with acute respira-
ing agents in the ICU: where are we now? Crit Care Med tory distress syndrome: a meta-analysis of randomized
2013;41:1332-44. controlled trials. J Thorac Dis 2015;7:356-67.
Griffiths MJD, McAuley DF, Perkins GD, et al. Guidelines on Peek GJ, Mugford M, Tiruvoipati R, et al. Efficacy and
the management of acute respiratory distress syndrome. economic assessment of conventional ventilatory sup-
BMJ Open Respir Res 2019;6:e000420. port versus extracorporeal membrane oxygenation for
severe adult respiratory failure (CESAR): a multicentre
Grissom CK, Hirshberg EL, Dickerson JB, et al. Fluid man-
randomised controlled trial. Lancet 2009;374:1351-63.
agement with a simplified conservative protocol for
the acute respiratory distress syndrome. Crit Care Med Putzu A, Belletti A, Cassina T, et al. Vitamin D and outcomes
2015;43:288-95. in adult critically ill patients: a systematic review and
meta-analysis of randomized trials. J Crit Care 2017;
Guerin C, Reignier J, Richard JC, et al. Prone positioning in
38:109-14.
severe acute respiratory distress syndrome. N Engl J Med
2013;368:2159-68. Ranieri VM, Rubenfeld GD, Thompson BT, et al. Acute respi-
ratory distress syndrome: the Berlin definition. JAMA
Horby P, Lim WS, Emberson JR, et al. Dexamethasone in
2012;307:2526-33.
hospitalized patients with COVID-19 – preliminary report.
N Engl J Med 2020. DOI: 10.1056/NEJMoa2021436. Ruan SY, Huang TM, Wu HY, et al. Inhaled nitric oxide ther-
apy and risk of renal dysfunction: a systematic review and
Jabaudon M, Blondonnet R, Pereira B, et al. Plasma sRAGE meta-analysis of randomized trials. Crit Care 2015;19:137.
is independently associated with increased mortality in
ARDS: a meta-analysis of individual patient data. Intensive Shi H, Han X, Jiang N, et al. Radiological findings from 81
Care Med 2018;44:1388-99. patients with COVID-19 pneumonia in Wuhan, China: a
descriptive study. Lancet Infect Dis 2020;20:425-34.
Long CL, Maull KI, Krishnan RS, et al. Ascorbic acid
dynamics in the seriously ill and injured. J Surg Res Silversides JA, Major E, Ferguson AJ, et al. Conservative
2003;109:144-8. fluid management or deresuscitation for patients with sep-
sis or acute respiratory distress syndrome following the
Meduri GU, Bridges L, Shih MC, et al. Prolonged gluco- resuscitation phase of critical illness: a systematic review
corticoid treatment is associated with improved ARDS and meta-analysis. Intensive Care Med 2017;43:155-70.
outcomes: analysis of individual patients’ data from four
randomized trials and trial-level meta-analysis of the Sinha P, Calfee CS. Phenotypes in acute respiratory distress
updated literature. Intensive Care Med 2016;42:829-40. syndrome: moving towards precision medicine. Curr Opin
Crit Care 2019;25:12-20.
Meduri GU, Golden E, Freire AX, et al. Methylprednisolone
infusion in early severe ARDS: results of a randomized Slutsky AS, Villar J. Early paralytic agents for ARDS? Yes, no,
controlled trial. Chest 2007;131:954-63. and sometimes. N Engl J Med 2019;380:2061-3.

Mikkelsen ME, Christie JD, Lanken PN, et al. The adult Steinberg KP, Hudson LD, Goodman RB, et al. Efficacy and
respiratory distress syndrome cognitive outcomes safety of corticosteroids for persistent acute respiratory
study: long-term neuropsychological function in distress syndrome. N Engl J Med 2006;354:1671-84.

CCSAP 2021 Book 1 • Pulmonary and Endocrinology 175 Interactive Case: Acute Respiratory Distress Syndrome
Sterne JAC, Murthy S, Diaz JV, et al. Association between Villar J, Perez-Mendez L, Kacmarek RM. The Berlin definition
administration of systemic corticosteroids and mortality met our needs: no. Intensive Care Med 2016;42:648-50.
among critically ill patients with COVID-19: a meta-
analysis. JAMA 2020. DOI: 10.1001/jama.2020.17023. Wiedemann HP, Wheeler AP, Bernard GR, et al. Comparison
of two fluid-management strategies in acute lung injury.
Teng J, Pourmand A, Mazer-Amirshahi M. Vitamin C: the next N Engl J Med 2006;354:2564-75.
step in sepsis management? J Crit Care 2018;43:230-4.
Wilson KC, Chotirmall SH, Bai C, et al. COVID-19: Interim
Torbic H, Krishnan S, Duggal A. Neuromuscular blocking Guidance on Management Pending Empirical Evidence.
agents for acute respiratory distress syndrome: how did Last updated April 3, 2020.
we get conflicting results? Crit Care 2019;23:305.

Villar J, Ferrando C, Martinez D, et al. Dexamethasone


treatment for the acute respiratory distress syndrome: a
multicentre, randomised controlled trial. Lancet Respir
Med 2020;8:267-76.

CCSAP 2021 Book 1 • Pulmonary and Endocrinology 176 Interactive Case: Acute Respiratory Distress Syndrome
Self-Assessment Questions
1. A 29-year-old man (height 72 inches, weight 130 kg) pres- placed in prone positioning for 12 hours, and the team
ents to the medical ICU with acute respiratory distress decides to initiate inhaled vasodilators. After 18 hours of
syndrome (ARDS) secondary to coronavirus disease this therapy, results of a repeat ABG are as follows: pH
2019 (COVID-19). On admission, he is placed on vol- 7.23, Pco2 63 mm Hg, HCO3− 25 mEq/L, Po2 65 mm Hg,
ume assist-control mechanical ventilation with the and Sao2 90% on the same ventilator settings. Which one
following settings: tidal volume 400 mL, respiratory rate of the following is best to recommend regarding the use
12 breaths/minute, fraction of inspired oxygen (Fio2) of inhaled vasodilators for this patient?
100%, and positive end-expiratory pressure (PEEP) 16 cm
A. Discontinue inhaled vasodilators; no improvement in
H2O. Results of his arterial blood gas (ABG) after 6 hours
oxygenation has been appreciated.
on these settings are as follows: pH 7.15, Pco2 62 mm Hg,
B. Check SCr and only continue inhaled vasodilators if
HCO3− 22 mEq/L, Po2 60 mm Hg, and Sao2 88%. According
there is no sign of renal dysfunction.
to the PROSEVA trial, which one of the following is best to
C. Check ECHO and only continue inhaled vasodilators if
recommend regarding prone positioning for this patient?
there are signs of severe right ventricular dysfunction.
A. Not a candidate for prone positioning; ratio of arterial D. Continue inhaled vasodilators with severe hypoxemia.
partial oxygen pressure to fraction of inspired oxygen
4. For which one of the following patients would it be most
(Po2/Fio2) exceeds the criteria for enrollment.
appropriate to recommend the use of dexamethasone for
B. Is a candidate for prone positioning; initiate daily
ARDS?
with 12 hours of prone positioning and 12 hours of
supine positioning. A. 39-year-old man with Po2/Fio2 of 210 mm Hg on
C. Is a candidate for prone positioning; initiate daily day 1 of ARDS
with 16 hours of prone positioning and 8 hours of B. 69-year-old man with Po2/Fio2 of 120 mm Hg on
supine positioning. day 1 of ARDS
D. Not a candidate for prone positioning; should C. 81-year-old woman with Po2/Fio2 of 140 mm Hg on
first optimize mechanical ventilation using lung- day 5 of ARDS
protective ventilation strategies. D. 65-year-old man with Po2/Fio2 of 225 mm Hg on
day 5 of ARDS
2. According to the EOLIA trial, which one of the following
patients with ARDS would be the best candidate for venove- 5. A 53-year-old woman presents to the medical ICU with
nous extracorporeal membrane oxygenation (ECMO)? moderate ARDS (baseline Po2/Fio2 145 mm Hg on Fio2
60% and PEEP 12 cm H2O) secondary to aspiration pneu-
A. 55-year-old woman on 100% Fio2 with ABG results of
monitis. During morning rounds, the team asks about the
pH 7.28/Pco2 53 mm Hg/Po2 72 mm Hg/HCO3−
newly published DEXA-ARDS and the use of dexametha-
22 mEq/L while using prone positioning and continuous
sone for ARDS in this patient. To most closely resemble
infusion neuromuscular blocking agents (NMBAs)
DEXA-ARDS, which one of the following would be most
B. 78-year-old man on 80% Fio2 with ABG results of
appropriate for initiating and monitoring dexamethasone?
pH 7.35/Pco2 43 mm Hg/Po2 97 mm Hg/HCO3−
23 mEq/L while using prone positioning and A. Initiate within 24 hours of onset of moderate to
continuous infusion NMBAs severe ARDS; continue regardless of Po2/Fio2.
C. 37-year-old man on 100% Fio2 with ABG results of pH B. Initiate within 72 hours of onset; continue regardless
7.19/Pco2 68 mm Hg/Po2 80 mm Hg/HCO3HCO3− of Po2/Fio2.
26 mEq/L while using intermittent bolus NMBAs C. Initiate within 24 hours of onset; discontinue if Po2/
D. 49-year-old woman on 85% Fio2 with ABG results of Fio2 improves to above 200 mm Hg in first 24 hours.
pH 7.33/Pco2 49 mm Hg/Po2 112 mm Hg/HCO3− D. Initiate within 72 hours of onset; discontinue if Po2/
24 mEq/L while using prone positioning Fio2 improves to above 200 mm Hg in first 72 hours.

3. A 73-year-old man (height 70 inches, weight 105 kg) pres- 6. For a patient with moderate to severe ARDS, which one
ents to the medical ICU with severe ARDS secondary to of the following is the most significant consequence of
necrotizing pancreatitis. He is placed on volume control receiving deep sedation compared with light sedation?
mechanical ventilation with the following settings: tidal A. Prolonged mechanical ventilation
volume 400 mL, respiratory rate 14 breaths/minute, Fio2 B. Increased sedation requirements and total sedation
75%, and PEEP 12 cm H2O. Results of his initial ABG on costs
these settings are as follows: pH 7.20, Pco2 66 mm Hg, C. Unrecognized patient-ventilator dyssynchrony
HCO3− 25 mEq/L, Po2 60 mm Hg, and Sao2 85%. He is D. Increased sedation-associated adverse effects

CCSAP 2021 Book 1 • Pulmonary and Endocrinology 177 Interactive Case: Acute Respiratory Distress Syndrome
7. One critique of the CITRIS-ALI study is that it did not 10. A 34-year-old man (height 72 inches, weight 85 kg) is
account for multiple comparisons in the many second- admitted to the surgical ICU with moderate ARDS (base-
ary outcomes it evaluated. One conservative way to line Po2/Fio2 110 mm Hg) secondary to bilateral pulmonary
adjust for multiple comparisons is to use the Bonferroni contusions from a motor vehicle crash. He is placed on
correction. Using the Bonferroni correction, which one volume assist-control mechanical ventilation with the
of the following is the most appropriate adjusted p value following settings: tidal volume 400 mL, respiratory rate
accounting for the 46 secondary outcome tests that 12 breaths/minute, Fio2 60%, and PEEP 12 cm H2O. He is
were performed (α = 0.05)? currently receiving propofol as a continuous infusion tar-
geted to light sedation (Richmond Agitation-Sedation
A. 0.05
Scale [RASS] score 0 to -2); his most recent RASS score
B. 0.01
is –1. The patient is also receiving fentanyl as a continu-
C. 0.005
ous infusion targeted to pain; his current pain score is 0.
D. 0.001
His most recent ABG results are as follows: pH 7.32, Pco2
8. A 67-year-old woman (height 56 inches, weight 80 kg) 52 mm Hg, HCO3− 26 mEq/L, Po2 95 mm Hg, and Sao2
presents to the burn ICU with moderate ARDS (Po2/Fio2 91%. He is synchronous with the mechanical ventilator,
120 mm Hg) secondary to inhalation injury. During her though he is breathing spontaneously over the ventilator
hospital course, the patient has received NMBAs for 48 with a respiratory rate of 18 breaths/minute. Which one of
hours and prone positioning for her refractory hypoxemia. the following is best to recommend for this patient?
However, she continues to have persistent severe ARDS.
A. Maintain current sedation.
On hospital day 16, the team approaches you regarding
B. Decrease sedation goal to target RASS -4 to -5 (i.e.,
the use of corticosteroids for the patient’s severe per-
deep sedation).
sistent ARDS. Her most recent ABG results are as follows:
C. Decrease sedation goal to target RASS -4 to -5 and
pH 7.40, Pco2 65 mm Hg, HCO3− 34 mEq/L, Po2 95 mm Hg,
administer intermittent bolus of NMBA.
and Sao2 93% on Fio2 65%, and PEEP 14 cm H2O. Which
D. Decrease sedation goal to target RASS -4 to -5 and
one of the following is best to recommend regarding cor-
initiate continuous NMBA for 48 hours.
ticosteroid use for this patient’s ARDS treatment?
A. Initiate dexamethasone intravenously 20 mg daily 11. A 72-year-old man (height 68 inches, weight 110 kg) pres-
for 5 days; then taper. ents to the medical ICU with moderate ARDS (Po2/Fio2
B. Initiate methylprednisolone intravenously 120 mg 170 mm Hg) secondary to massive transfusions after
once, then 30 mg every 6 hours for 2 weeks; then taper. hemorrhagic shock. Because of the volume of fluids the
C. Initiate methylprednisolone intravenously 60 mg patient has received, the team wants to use a conserva-
once, then 60 mg daily for 2 weeks; then taper. tive fluid management strategy. The patient’s current vital
D. Do not initiate corticosteroids at this time. signs and laboratory values are as follows: heart rate 110
beats/minute, blood pressure 85/52 mm Hg on norepi-
9. A 57-year-old man (weight 85 kg) presents to the med-
nephrine 15 mcg/minute, CVP 13 mm Hg, SCr 1.2 mg/dL,
ical ICU with moderate ARDS (Po2/Fio2 165 mm Hg)
and urinary output 25 mL/hour. Which one of the follow-
secondary to COVID-19. The team uses a conservative
ing is best to recommend regarding this patient’s fluid
fluid management strategy. The patient’s current vital
management?
signs and laboratory values are as follows: heart rate
95 beats/minute, blood pressure 90/55 mm Hg (patient A. Give 1 L of lactated Ringer solution; reassess in 1 hour.
had previously required norepinephrine but has been off B. Give 40 mg of intravenous furosemide; reassess in
vasopressors for about 15 hours), CVP 9 mm Hg, SCr 0.7 1 hour.
mg/dL, and urinary output 40 mL/hour. His most recent C. Give 40 mg of intravenous furosemide; reassess in
ABG results are as follows: pH 7.22, Pco2 48 mm Hg, 4 hour.
HCO3− 19 mEq/L, Po2 108 mm Hg, Sao2 92% on Fio2 65%, D. Perform a passive leg raise to determine whether
and PEEP 12 cm H2O. Which one of the following is best patient is fluid responsive.
to recommend for this patient’s fluid management? 12. Certain medications for ARDS (e.g., corticosteroids)
A. Administer 1 L of intravenous normal saline; have a variable role depending on when they are initiated
reassess in 1 hour. during the course of ARDS. Which one of the following is
B. Administer 20 mg of intravenous furosemide; the most appropriate classification of timing of ARDS as
reassess in 1 hour. a phenotype of ARDS?
C. Administer 20 mg of intravenous furosemide; A. Physiologically derived phenotype
reassess in 4 hours. B. Biologically derived phenotype
D. Do not use conservative fluid management; the C. Clinically derived phenotype
patient has hyperinflammatory phenotype. D. Data-derived phenotype

CCSAP 2021 Book 1 • Pulmonary and Endocrinology 178 Interactive Case: Acute Respiratory Distress Syndrome
Learner Feature Evaluation: Interactive Case: Acute Respiratory Distress
Syndrome

As you take the posttest for this feature, also evaluate the 8. The teaching and learning methods used in the feature
material’s quality and usefulness, as well as the achievement were effective.
of learning objectives. Rate each item using this 5-point scale: 9. The active learning methods used in the feature were
effective.
• Strongly agree
10. The learning assessment activities used in the feature
• Agree
were effective.
• Neutral
• Disagree 11. The feature was effective overall.
• Strongly disagree 12. The activity met the stated learning objectives.
13. If any objectives were not met, please list them here.
1. The content of the feature met my educational needs.
2. The content of the feature satisfied my expectations.
3. The author presented the feature content effectively.
OTHER COMMENTS
4. The content of the feature was relevant to my practice 14. Please provide any specific comments related to any
and presented at the appropriate depth and scope. perceptions of bias, promotion, or advertisement of
commercial products.
5. The content of the feature was objective and balanced.
15. Please expand on any of your above responses, and/or
6. The content of the feature is free of bias, promotion, and
provide any additional comments regarding this feature:
advertisement of commercial products.
7. The content of the feature was useful to me.

CCSAP 2021 Book 1 • Pulmonary and Endocrinology 179 Interactive Case: Acute Respiratory Distress Syndrome
Interactive Case: Extracorporeal
Membrane Oxygenation
By Jennifer Falvey, Pharm.D., BCPS

Reviewed by Patrick M. Wieruszewski, Pharm.D., BCCCP; and Yanjun Maggie Zhao, Pharm.D., BCCCP, BCPS

LEARNING OBJECTIVES

1. Evaluate the impact of extracorporeal membrane oxygenation (ECMO) circuits on pharmacokinetic alterations of medica-
tions and the interplay of circuit, medication, and patient factors.
2. Design an appropriate anticoagulation regimen for a patient requiring ECMO.
3. Design an appropriate pain and sedation regimen for a patient receiving ECMO.
4. Develop an optimal antimicrobial regimen and perform therapeutic drug monitoring as indicated for a patient receiving
ECMO to maximize therapeutic efficacy.

ABBREVIATIONS IN THIS FEATURE


ACT Activated clotting time BASELINE KNOWLEDGE STATEMENTS

AT Antithrombin Readers of this feature are presumed to be familiar


ECMO Extracorporeal membrane with the following:
oxygenation
ELSO Extracorporeal Life Support
• Basic pharmacokinetic principles including
volume of distribution, clearance, protein binding,
Organization and partition coefficient
HIT Heparin-induced thrombocytopenia
• General understanding of ECMO indications,
TDM Therapeutic drug monitoring circuitry, clinical management, and complications
TEG Thromboelastography
• General medication knowledge of anticoagulation,
VA-ECMO Venoarterial ECMO sedation, analgesia, and antimicrobials
Vd Volume of distribution
VV-ECMO Venovenous ECMO Table of common laboratory reference values.

Table of other common abbreviations. ADDITIONAL READINGS

The following free resources have additional back-


ground information on this topic:
• Sorokin V, MacLaren G, Vidanapathirana P, et al.
Choosing the appropriate configuration and
cannulation strategies for extracorporeal mem-
brane oxygenation: the potential dynamic process
of organ support and importance of hybrid modes.
Eur J Heart Fail 2017;19:75-83.
• Extracorporeal Life Support Organization (ELSO).
ELSO General Guidelines. 2017.
• Cheng V, Abdul-Aziz M, Roberts J, et al. Optimising
drug dosing in patients receiving extracorporeal
membrane oxygenation. J Thorac Dis 2018;10
(suppl 5):S629-41.

CCSAP 2021 Book 1 • Pulmonary and Endocrinology 181 Interactive Case: Extracorporeal Membrane Oxygenation
Beiderlinden M, Treschan T, Gorlinger K, et al. Argatroban
• Ha M, Sieg A. Evaluation of altered drug pharmacoki- in extracorporeal membrane oxygenation. Artif Organs
netics in critically ill adults receiving extracorporeal 2007;31:461-5.
membrane oxygenation. Pharmacotherapy 2017;
Berei T, Lillyblad M, Wilson K, et al. Evaluation of systemic
37:221-35.
heparin versus bivalirudin in adult patients supported by
• Chlebowski M, Baltagi S, Carlson M, et al. Clinical extracorporeal membrane oxygenation. ASAIO J 2018;
controversies in anticoagulation monitoring and 64:623-9.
antithrombin supplementation for ECMO. Crit Care
2020;24:19. Branick K, Taylor M, Trump M, et al. Apparent interference
with extracorporeal membrane oxygenation by liposomal
amphotericin B in a patient with disseminated blastomy-
cosis receiving continuous renal replacement therapy.
Am J Health Syst Pharm 2019;76:810-4.
Practice Points Cardinale M, Ha M, Liu M, et al. Direct thrombin inhibitor
• Extracorporeal membrane oxygenation (ECMO) is a life- resistance and possible mechanisms. Hosp Pharm
saving modality used to bridge patients to recovery or an 2016;51:922-7.
advanced therapy.
• ECMO can alter drug pharmacokinetics significantly, which
Cheng R, Hachamovitch R, Kittleson M, et al. Complications
may require therapeutic intervention. of extracorporeal membrane oxygenation for treatment of
• Heparin has long been the standard of anticoagulation in cardiogenic shock and cardiac arrest: a meta-analysis of
ECMO. Emerging data suggest that direct thrombin inhib- 1,866 adult patients. Ann Thorac Surg 2014;97:610-6.
itors such as argatroban and bivalirudin are acceptable Cheng V, Abdul-Aziz M, Roberts J, et al. Optimising drug
alternatives. Each ECMO program should create antico- dosing in patients receiving extracorporeal membrane
agulation protocols and guidelines individualized to that oxygenation. J Thorac Dis 2018;10(suppl 5):S629-41.
center.
• Common analgesic and sedative medications may have Choi J, Luc J, Weber M, et al. Heparin-induced thrombocy-
significant pharmacokinetic alterations on ECMO, neces- topenia during extracorporeal life support: incidence,
sitating careful dose titration guided by desired clinical management and outcomes. Ann Cardiothorac Surg
outcomes. 2019;8:19-31.
• Pharmacokinetic alterations of anti-infective agents should
be accounted for when treating a patient receiving ECMO in
Chung Y, Cho D, Sohn D, et al. Is stopping heparin safe in
order to optimize effective dosing, achieve adequate serum patients on extracorporeal membrane oxygenation treat-
concentrations, and maximize therapeutic efficacy. ment? ASAIO J 2017;63:32-6.
Cies J, Moore W, Giliam N, et al. Oxygenator impact on cef-
taroline in extracorporeal membrane oxygenation circuits.
INTERACTIVE CASE: Pediatr Crit Care Med 2018;19:1077-82.
EXTRACORPOREAL MEMBRANE Cies J, Moore W, Giliam N, et al. Oxygenator impact on
OXYGENATION voriconazole in extracorporeal membrane oxygenation
circuits. Perfusion 2020;35:529-33.
• Click here to start this CCSAP activity.
Colman E, Yin E, Laine G, et al. Evaluation of a heparin moni-
REFERENCES toring protocol for extracorporeal membrane oxygenation
and review of the literature. J Thorac Dis 2019;11:3325-35.
Aguilar G, Ferriols R, Carbonell J, et al. Pharmacokinetics of
anidulafungin during venovenous extracorporeal mem- Cuker A, Arepally G, Chong B, et al. American Society of
brane oxygenation. Crit Care 2016;20:325. Hematology 2018 guidelines for management of venous
thromboembolism: heparin-induced thrombocytopenia.
Arachchillage D, Laffan M, Khanna S, et al. Frequency of
Blood Adv 2018;2:3360-92.
thrombocytopenia and heparin-induced thrombocyto-
penia in patients receiving extracorporeal membrane Dallefeld S, Sherwin J, Zimmerman K, et al.
oxygenation compared with cardiopulmonary bypass Dexmedetomidine extraction by the extracorporeal mem-
and the limited sensitivity of pretest probability score. brane oxygenation circuit: results from an in vitro study.
Crit Care Med 2020;48:e371-9. Perfusion 2020;35:209-16.
Arena F, Marchetti L, Henrici De Angelis L, et al. Ceftolozane- DeGrado J, Hohlfelder B, Ritchie B, et al. Evaluation of seda-
tazobactam pharmacokinetics during extracorporeal tives, analgesics, and neuromuscular blocking agents in
membrane oxygenation in a lung transplant recipient. adults receiving extracorporeal membrane oxygenation.
Antimicrob Agents Chemother 2019;63:e02131-18. J Crit Care 2017;37:1-6.

Aubron C, McQuilten Z, Bailey M, et al. Low-dose thera- Delmas C, Jacquemin A, Vardon-Bounes F, et al.
peutic anticoagulation in patients on extracorporeal Anticoagulation monitoring under ECMO support: a com-
membrane oxygenation: a pilot randomized trial. Crit Care parative study between the activated coagulation time and
2019;47:e563-71. the anti-Xa assay. J Intensive Care Med 2020;35:679-86.

CCSAP 2021 Book 1 • Pulmonary and Endocrinology 182 Interactive Case: Extracorporeal Membrane Oxygenation
Dhanani J, Lipman J, Pincus J, et al. Pharmacokinetics Hellwig T, Peitz G, Gulseth M. High-dose argatroban for
of total and unbound cefazolin during veno-arterial treatment of heparin-induced thrombocytopenia with
extracorporeal membrane oxygenation: a case report. thrombosis: a case report and review of laboratory consid-
Chemotherapy 2019;64:115-8. erations. AJHP 2012;69:490-5.
Donadello K, Antonucci E, Cristallini S, et al. B-lactam Hohlfelder B, Kelly D, Hoang M, et al. Activated clotting times
pharmacokinetics during extracorporeal membrane oxy- demonstrate weak correlation with heparin dosing in adult
genation therapy: a case-control study. Int J Antimicrob extracorporeal membrane oxygenation. Am J Ther 2019.
Agents 2015;45:278-82. DOI: 10.1097/MJT.0000000000001113.
Donadello K, Roberts J, Cristallini S, et al. Vancomycin popu- Hohlfelder B, Szumita P, Lagambina S, et al. Safety of propo-
lation pharmacokinetics during extracorporeal membrane fol for oxygenator exchange in extracorporeal membrane
oxygenation therapy: a matched cohort study. Crit Care oxygenation. ASAIO J 2017;63:179-84.
2014;18:632.
Hynynen M, Hammaren E, Rosenberg P. Propofol seques-
Dzierba A, Brodie D, Bacchetta M, et al. Ketamine use in tration within the extracorporeal circuit. Can J Anaesth
sedation management in patients receiving extracor- 1994;41:583-8.
poreal membrane oxygenation. Intensive Care Med
Iapichino G, Protti A, Andreis D, et al. Antithrombin during
2016;42:1822-3.
extracorporeal membrane oxygenation in adults: national
Ellouze O, Abbad X, Constandache T, et al. Risk factors of survey and retrospective analysis. ASAIO J 2019;
bleeding in patients undergoing venoarterial extracor- 65:257-63.
poreal membrane oxygenation. Ann Thorac Surg 2020.
Keebler M, Haddad E, Choi C, et al. Venoarterial extracorpo-
Online ahead of print.
real membrane oxygenation in cardiogenic shock. JACC
Esper S, Levy J, Waters J, et al. Extracorporeal membrane Heart Fail 2018;6:503-16.
oxygenation in the adult: a review of anticoagulation moni-
Kennedy D, Alaniz C. Apparent argatroban resistance in a
toring and transfusion. Anesth Analg 2014;118:731-43.
patient with elevated factor VIII levels. Ann Pharmacother
Everett B, Yeh R, Foo S, et al. Prevalence of heparin/platelet 2013;47:e29.
factor 4 antibodies before and after cardiac surgery.
Kim Y, Lee H, Yang J, et al. Use of argatroban for extracor-
Ann Thorac Surg 2007;83:592-7.
poreal life support in patients with nonheparin-induced
Extracorporeal Life Support Organization (ELSO). ELSO thrombocytopenia. Medicine 2018;97:e13235.
General Guidelines. Ann Arbor, MI: ELSO, 2017.
Lam E, Rochani A, Kaushal G, et al. Pharmacokinetics of
Eyler R, Heung M, Pleva M, et al. Pharmacokinetics of osel- ketamine at dissociative doses in an adult patient with
tamivir and oseltamivir carboxylate in critically ill patients refractory status asthmaticus receiving extracorporeal
receiving continuous venovenous hemodialysis and/or membrane oxygenation therapy. Clin Ther 2019;41:994-9.
extracorporeal membrane oxygenation. Pharmacotherapy
Lamm W, Nagler B, Hermann A, et al. Propofol-based seda-
2012;32:1061-9.
tion does not negatively influence oxygenator running
Foulquier J, Berneau P, Frerou A, et al. Liposomal ampho- time compared to midazolam in patients with extra-
tericin B pharmacokinetics in a patient treated with corporeal membrane oxygenation. Int J Artif Organs
extracorporeal membrane oxygenation. Med Mal Infect 2019;42:233-20.
2019;49:69-71.
Landolf K, Rivosecchi R, Gomez H, et al. Comparison of
Gélisse E, Neuville M, de Montmollin E, et al. Extracorporeal hydromorphone versus fentanyl-based sedation in extra-
membrane oxygenation (ECMO) does not impact on ami- corporeal membrane oxygenation: a propensity-matched
kacin pharmacokinetics: a case-control study. Intensive analysis. Pharmacotherapy 2020;40:389-97.
Care Med 2016;42:946-8.
Lemaitre F, Hasni N, Leprince P, et al. Propofol, midazolam,
Glick D, Dzierba A, Abrams D, et al. Clinically suspected hep- vancomycin and cyclosporine therapeutic drug monitoring
arin-induced thrombocytopenia during extracorporeal in extracorporeal membrane oxygenation circuits primed
membrane oxygenation. J Crit Care 2015;30:1190-4. with whole human blood. Crit Care 2015;19:40.
Guglin M, Zucker M, Bazan V, et al. Venoarterial ECMO for Lemaitre F, Luyt C, Roullet-Renoleau F, et al.
adults. JACC scientific statement. JACC 2019;73:698-716. Pharmacokinetics of oseltamivir carboxylate in critically ill
patients with pandemic (H1N1) influenza. Ther Drug Monit
Heilmann C, Geisen U, Beyersdorf F, et al. Acquired von
2012;34:171-5.
Willebrand syndrome in patients with extracorporeal life
support (ECLS). Intensive Care Med 2012;38:62-8. Lequier L, Annich G, Al-Ibrahim O, et al. ELSO
Anticoagulation Guidelines. Ann Arbor, MI: ELSO, 2014.
Heith C, Hansen C, Bakken R, et al. Effects of ex vivo pedi-
atric extracorporeal membrane oxygenation circuit on Leven C, Fillatre P, Petitcollin A, et al. Ex vivo model to deci-
the sequestration of mycophenolate mofetil, tacrolimus, pher the impact of extracorporeal membrane oxygenation
hydromorphone, and fentanyl. J Pediatr Pharmacol Ther on beta-lactam degradation kinetics. Ther Drug Monit
2019;24:290-5. 2017;39:180-4.

CCSAP 2021 Book 1 • Pulmonary and Endocrinology 183 Interactive Case: Extracorporeal Membrane Oxygenation
Mazzeffi M, Greenwood J, Tanaka K, et al. Bleeding, transfu- adult respiratory failure (CESAR): a multicentre ran-
sion, and mortality on extracorporeal life support: ECLS domised controlled trial. Lancet 2009;374:1351-63.
Working Group on Thrombosis and Hemostasis. Ann
Thorac Surg 2016;101:682-9. Pieri M, Agracheva N, Bonaveglio E, et al. Bivalirudin versus
heparin as an anticoagulant during extracorporeal mem-
Mehta N, Halwick D, Dodson, et al. Potential drug seques- brane oxygenation: a case-control study. J Cardiothorac
tration during extracorporeal membrane oxygenation: Vasc Anesth 2013;27:30-4.
results from an ex vivo experiment. Intensive Care Med
2007;33:1018-24. Pollak U. Heparin-induced thrombocytopenia complicating
extracorporeal membrane oxygenation support: review
Menk M, Briem P, Weiss B, et al. Efficacy and safety of of the literature and alternative anticoagulants. J Thromb
argatroban in patients with acute respiratory distress Haemost 2019;17:1608-22.
syndrome and extracorporeal lung support. Ann Intensive
Care 2017;7:82. Preston T, Hodge A, Riley J, et al. In vitro adsorption and
plasma free hemoglobin levels associated with hollow
Millar J, Fanning J, McDonald C, et al. The inflammatory fiber oxygenators in the extracorporeal life support (ECLS)
response to extracorporeal membrane oxygenation circuit. JECT 2007;39:234-7.
(ECMO): a review of the pathophysiology. Crit Care 2016;
20:387. Ranucci M. Bivalirudin and post-cardiotomy ECMO: a word of
caution. Crit Care 2012;16:427.
Morrisette M, Zomp-Wiebe A, Bidwell K, et al. Antithrombin
supplementation in adult patients receiving extracorpo- Ranucci M, Ballotta A, Kandil H, et al. Bivalirudin-based
real membrane oxygenation. Perfusion 2020;35:66-72. versus conventional heparin anticoagulation for postcar-
diotomy extracorporeal membrane oxygenation. Crit Care
Mulla H, Peek G, Harvey C, et al. Oseltamivir pharmaco- 2011;15:R275.
kinetics in critically ill adults receiving extracorporeal
membrane oxygenation support. Anaesth Intensive Care Ruiz S, Papy E, Da Silva D, et al. Potential voriconazole and
2013;41:66-73. caspofungin sequestration during extracorporeal mem-
brane oxygenation. Intensive Care Med 2009;35:183-4.
Mulla H, Pooboni S. Population pharmacokinetic of van-
comycin in patients receiving extracorporeal membrane Ruiz-Ramos J, Gimeno R, Perez F, et al. Pharmacokinetics of
oxygenation. Br J Clin Pharmacol 2005;60:265-75. amikacin in critical care patients on extracorporeal device.
ASAIO J 2018;64:686-8.
Myers G, Voorhees C, Eke B, et al. The effect of Diprivan
(propofol) on phosphorylcholine surfaces during cardio- Sanfilippo F, Asmussen S, Maybauer D, et al. Bivalirudin for
pulmonary bypass – an in vitro investigation. Perfusion alternative anticoagulation in extracorporeal membrane
2009;24:349-55. oxygenation: a systematic review. J Intensive Care Med
2017;32:312-9.
Nigoghossian C, Dzierba A, Etheridge J, et al. Effect of
extracorporeal membrane oxygenation use on sedative Shekar K, Frase J, Taccone F, et al. The combined effects
requirements in patients with severe acute respiratory of extracorporeal membrane oxygenation and renal
distress syndrome. Pharmacotherapy 2016;36:607-16. replacement therapy on meropenem pharmacokinetics: a
matched cohort study. Crit Care 2014;18:565.
Panigada M, Iapichino G, Brioni M, et al.
Thromboelastography-based anticoagulation manage- Shekar K, Roberts J, Mcdonald C, et al. Protein-bound drugs
ment during extracorporeal membrane oxygenation: are prone to sequestration in the extracorporeal mem-
a safety and feasibility pilot study. Ann Intensive Care brane oxygenation circuit: results from an ex vivo study.
2018;8:7. Crit Care 2015;19:164.

Park J, Shin D, Lee S, et al. Investigation of key circuit Shekar K, Roberts J, Mcdonald C, et al. Sequestration
constituents affecting drug sequestration during extra- of drugs in the circuit may lead to therapeutic failure
corporeal membrane oxygenation treatment. ASAIO J during extracorporeal membrane oxygenation. Crit Care
2017;63:293-98. 2012a;16:R194.

Park S, Yang, J, Park H, et al. Trough concentrations of van- Shekar K, Roberts J, Mullany D, et al. Increased sedation
comycin in patients undergoing extracorporeal membrane requirements in patients receiving extracorporeal mem-
oxygenation. PLoS One 2015;10:e0141016. brane oxygenation for respiratory and cardiorespiratory
failure. Anaesth Intensive Care 2012b;40:648-55.
Patel M, Altshuler D, Lewis T, et al. Sedation requirements
in patients on venovenous or venoarterial extracor- Sherwin J, Heath T, Watt K. Pharmacokinetics and dosing of
poreal membrane oxygenation. Ann Pharmacother anti-infective drugs in patients on extracorporeal mem-
2020;54:122-30. brane oxygenation: a review of the current literature. Clin
Ther 2016;38:1976-94.
Peek G, Mugford M, Tiruvoipati R, et al. Efficacy and eco-
nomic assessment of conventional ventilatory support Spriet I, Annaert P, Meersseman P, et al. Pharmacokinetics
versus extracorporeal membrane oxygenation for severe of caspofungin and voriconazole in critically ill patients

CCSAP 2021 Book 1 • Pulmonary and Endocrinology 184 Interactive Case: Extracorporeal Membrane Oxygenation
during extracorporeal membrane oxygenation. Waele J, Cauwenberghe S, Hoste E. The use of the activated
J Antimicrob Chemother 2009;63:767-70. clotting time for monitoring heparin therapy in critically ill
patients. Intensive Care Med 2003;29:325-8.
Tellor B, Shin N, Graetz T, et al. Ketamine infusion for patients
receiving extracorporeal membrane oxygenation support: Wagner D, Pasko D, Phillips K, et al. In vitro clearance of dex-
a case series. F1000Res 2015;4:16. medetomidine in extracorporeal membrane oxygenation.
Perfusion 2012;28:40-6.
Touchard C, Aubry A, Eloy P, et al. Predictors of insufficient
peak amikacin concentrations in critically ill patients Walker E, Roberts A, Louie E, et al. Bivalirudin dosing require-
on extracorporeal membrane oxygenation. Crit Care ments in adult patients on extracorporeal life support with
2018;22:199. or without continuous renal replacement therapy. ASAIO J
2019;65:134-8.
Tron C, Leven C, Fillatre P, et al. Should we fear tubing
adsorption of antibacterial drugs in extracorporeal mem- Winiszewski H, Rougny A, Lagoutte-Renosi J, et al. The phar-
brane oxygenation? An answer for cephalosporins and macokinetic challenge of treating invasive aspergillosis
carbapenems. Clin Exp Pharmacol Physiol 2016;43:281-3. complicating severe influenzae assisted by extracorporeal
membrane oxygenation. Crit Care 2018;22:355.
Tsu L, Dager W. Bivalirudin dosing adjustments for
reduced renal function with or without hemodialysis in Wood K, Ayers B, Gosev I, et al. Venoarterial-extracorporeal
the management of heparin-induced thrombocytopenia. membrane oxygenation without routine systemic anti-
Ann Pharmacother 2011;45:1185-92. coagulation decreases adverse events. Ann Thorac Surg
2020;109:1458-66.
Turner R. Azithromycin pharmacokinetics in adults with
acute respiratory distress syndrome undergoing treat- Wu C, Shen L, Hsu L, et al. Pharmacokinetics of vancomycin
ment with extracorporeal-membrane oxygenation. in adults receiving extracorporeal membrane oxygenation.
Ann Pharmacother 2016;50:72-3. J Formos Med Assoc 2016;115:560-70.

Verkerk B, Dzierba A, Muir J, et al. Opioid and benzodiazepine


requirements in obese adult patients receiving extra-
corporeal membrane oxygenation. Ann Pharmacother
2020;54:144-50.

CCSAP 2021 Book 1 • Pulmonary and Endocrinology 185 Interactive Case: Extracorporeal Membrane Oxygenation
Self-Assessment Questions
Questions 13 and 14 pertain to the following case. Questions 15 and 16 pertain to the following case.

B.B. is a 33-year-old previously healthy man (weight 86 kg) L.B., a 45-year-old man with a history of nonischemic cardio-
in the medical ICU requiring mechanical ventilation for myopathy secondary to continued alcohol abuse, presents
acute respiratory distress syndrome. One day earlier, he with worsening dyspnea, fatigue, and a 20-kg weight gain over
had presented to the ED after 3 days of fever, fatigue, and the past 5 days because of medication and dietary nonadher-
productive cough. A sputum sample and respiratory viral ence. He has been initiated on venoarterial ECMO (VA-ECMO)
panel are sent. B.B. is initiated on antibiotics for presumed (left femoral artery, right femoral vein) with intra-aortic bal-
community-acquired pneumonia (ceftriaxone 2 g intrave- loon pump (IABP) for acute cardiogenic shock. Current ECMO
nously daily and azithromycin 500 mg intravenously daily). flows are 4.2 L/minute (3900 rpm), and IABP trigger is 1:2.
However, his condition rapidly deteriorates, and refrac- A bedside ECHO reveals adequate left ventricular emptying
tory hypoxemia persists despite measures to optimize gas with no evidence of smoke in any chamber. L.B.’s relevant
exchange and end-organ perfusion, including low-tidal- laboratory values include SCr 1.9 mg/dL, AST 300 U/L, ALT
volume ventilation, paralysis, and prone positioning. 450 U/L, total bilirubin 4.5 mg/dL, activated PTT (aPTT)
Venovenous extracorporeal membrane oxygenation 42 seconds, Hgb 10 g/dL, Hct 29%, Plt 250,000/mm3, and INR 1.7.
(VV-ECMO) is initiated to improve oxygenation. B.B.’s relevant
15. Which one of the following is the best initial anticoagula-
laboratory values are BUN 36 mg/d L, SCr 1.2 mg/dL, AST
tion regimen to recommend for L.B.?
70 U/L, and ALT 57 U/L.
A. Bivalirudin infusion initiated at 0.3 mg/kg/hour
13. B.B. continues to experience fever with rising WBC and
B. Unfractionated heparin infusion titrated to a target
hypotension requiring vasopressor initiation. New blood
aPTT of 50–70 seconds
and sputum cultures are sent. His relevant laboratory val-
C. Argatroban infusion initiated at 2 mcg/kg/minute
ues include BUN 42 mg/dL, SCr 2.2 mg/dL, AST 85 U/L,
D. Unfractionated heparin infusion titrated to a
and ALT 63 U/L. Which one of the following broad-spec-
target an activated clotting time (ACT) of 220–250
trum antibiotic regimens is best to recommend for B.B.?
seconds
A. Cefepime 2 g intravenously every 8 hours and
16. Two days later, while receiving systemic anticoagula-
vancomycin 1 g intravenously once with further
tion, L.B. develops GI bleeding requiring transfusion of
dosing on the basis of therapeutic drug monitoring
3 units of packed RBCs (PRBCs) to maintain an Hgb of
(TDM)
7 g/dL. L.B. remains hemodynamically stable on the
B. Piperacillin/tazobactam 4.5 g intravenously every
current ECMO settings without the need for vasopres-
6 hours and vancomycin 1.25 g intravenously every
sor titration. No significant fibrin stranding is identified
8 hours
in the cannulas or on the oxygenator. Which one of the
C. Cefepime 2 g intravenously every 8 hours and
following is best to recommend for L.B.?
vancomycin 2-g intravenous loading dose followed
by 1.25 g intravenously every 12 hours A. Use a reduced dose of argatroban 1 mcg/kg/minute.
D. Meropenem 1 g intravenously every 12 hours and B. Continue the current anticoagulation regimen.
vancomycin 1.75 g intravenously with further dosing C. Discontinue all anticoagulation and monitor closely
on the basis of TDM for the appearance of fibrin or thrombus within the
circuit.
14. B.B. has required sedation with a midazolam infusion
D. Use bivalirudin targeting an anti–factor Xa (anti-Xa)
since intubation. After ECMO initiation, which one of
goal of 0.1–0.3 IU/mL.
the following best evaluates B.B.’s midazolam dose
requirement? 17. A patient receiving VV-ECMO has significant thrombus
noted on the oxygenator; the team decides to exchange
A. Minimal change because studies have shown that
it and asks about the effect of an oxygenator change on
only 20% is lost to the circuit.
medication dose requirements. Which one of the follow-
B. Increased because ex vivo studies show that only
ing is best to recommend for this patient?
20% is recovered from the circuit at 24 hours.
C. Decreased after 1 day because drug adsorbed to the A. No adsorption will occur to the new oxygenator, so
circuit will be released and absorbed by the patient. the doses should remain unchanged.
D. Increased because sedation goals are higher for B. Significant loss will be expected because most drug
patients receiving ECMO (Richmond Agitation- loss will be absorbed to the oxygenator rather than
Sedation Scale [RASS] score -4 to -3). the cannulas.

CCSAP 2021 Book 1 • Pulmonary and Endocrinology 186 Interactive Case: Extracorporeal Membrane Oxygenation
C. Degree of adsorption varies depending on the team decides to continue with ECMO decannulation. An
medication, and the patient should be monitored for advanced practice provider on the team asks which of
decreased medication efficacy. C.K.’s drugs will require adjustment after the ECMO cir-
D. An additional dose of all medications is cuit is removed. Which one of the following is the most
recommended after the oxygenator exchange to appropriate response?
account for the adsorption to the new component.
A. Hydromorphone infusion requirements may
18. Which one of the following patients receiving ECMO is at decrease. The infusion should be titrated to achieve
highest risk of treatment failure? the patient’s pain and sedation goals.
B. Cefepime should be decreased from 1 g every
A. 21-year-old woman on VV-ECMO after a lung
8 hours to 1 g every 12 hours.
transplant receiving voriconazole 4 mg/kg
C. Midazolam requirements should not change
intravenously every 12 hours for an invasive fungal
significantly after decannulation.
infection
D. Dexmedetomidine infusion rate should be increased
B. 69-year-old man on VA-ECMO after an ST-segment
by 25% to account for the change in volume of
elevation myocardial infarction (STEMI) receiving
distribution (Vd).
cefepime 2 g intravenously every 8 hours for
pseudomonal bacteremia
Questions 22–24 pertain to the following case.
C. 55-year-old man on VV-ECMO receiving fluconazole
400 mg intravenously daily for bacteremia with B.R., a 64-year-old woman (weight 85 kg), has a medical his-
Candida albicans tory that includes heart failure with reduced ejection fraction,
D. 49-year-old woman on VV-ECMO receiving coronary artery disease, and diabetes. She presents to the ED
meropenem 2 g intravenously every 8 hours for with worsening dyspnea, lower-extremity edema, and weight
Enterobacter cloacae bacteremia gain. B.R. is now being treated on the advanced heart failure
service for cardiogenic shock. She continues to decompen-
Questions 19–21 pertain to the following case. sate despite medical optimization, including inotrope
initiation and aggressive diuresis. The decision is to cannu-
C.K., a 57-year old-man (weight 95 kg), is admitted to the car-
late B.R. for VA-ECMO as a bridge to heart transplantation.
diac ICU after a STEMI requiring VA-ECMO for cardiogenic
shock. His medical history includes diabetes, hypertension, 22. Which one of the following is the best intravenous load-
atrial fibrillation, and coronary artery disease. C.K.’s relevant ing dose of heparin to recommend for B.R. at the time of
laboratory results include BUN 37 mg/dL, SCr 1.4 mg/dL, AST ECMO cannulation?
220 U/L, ALT 300 U/L, total bilirubin 2.1 mg/dL, Hgb 8.4 g/dL,
A. 1250 units
Hct 26%, Plt 144,000/mm3, and baseline aPTT 34 seconds.
B. 1500 units
19. Which one of the following is the best initial heparin mon- C. 2000 units
itoring goal to recommend for C.K.? D. 5000 units
A. ACT 200–220 seconds 23. After 48 hours of continued heparin dosage escalation,
B. Thromboelastography (TEG) R-time 16–24 minutes B.R.’s antithrombin (AT) activity level is 57%. Her current
C. aPTT 80–100 seconds aPTT is 42 seconds (target 60–80 seconds). The team
D. Anti-Xa concentration 0.3–0.7 IU/mL asks about the role of AT supplementation. Which one of
the following is best to recommend regarding AT supple-
20. On day 4 of C.K.’s ECMO therapy, his platelet count drops
mentation for B.R.?
to 68,000/mm3. Heparin-induced thrombocytopenia (HIT)
is suspected with a high pretest probability of HIT. Which A. Initiate supplementation because her AT activity
one of the following is best to recommend for C.K.? level is less than 70%, and AT has consistently
reduced thrombus formation in the ECMO circuit.
A. Discontinue heparin and initiate bivalirudin 0.02
B. Consider AT supplementation; however, the benefits
mg/kg/hour.
of supplementation are unknown and use may lead
B. Discontinue heparin and initiate argatroban 1.5
to supratherapeutic levels of anticoagulation.
mcg/kg/minute.
C. Do not consider; AT activity levels have not been
C. Anticoagulation is no longer needed because his Plt
shown to increase after supplementation while
is less than 100,000/mm3.
receiving ECMO.
D. Continue heparin and wait for the results of the PF4
D. Do not consider; AT supplementation should only
enzyme-linked immunosorbent assay.
be considered when treating pediatric patients
21. C.K. undergoes transesophageal ECHO and a ECMO receiving ECMO.
wean that show improved left ventricular function. The

CCSAP 2021 Book 1 • Pulmonary and Endocrinology 187 Interactive Case: Extracorporeal Membrane Oxygenation
24. B.R.’s anticoagulation was changed from heparin to C. Continue the current bivalirudin infusion dose.
bivalirudin to allay concerns of heparin resistance and D. Change bivalirudin back to heparin. The incidence of
reduce the risk of HIT because she is awaiting a heart HIT in patients receiving ECMO is similar to that in
transplant. A bedside echocardiogram revealed “smoke” patients not receiving ECMO, so there is no benefit
in the right and left ventricles and inadequate left ven- to avoiding heparin exposure.
tricular emptying. Her current laboratory values are as
follows: BUN 42 mg/dL, SCr 1.5 mg/dL, AST 54 U/L, ALT
76 U/L, and aPTT 64 seconds (goal 50–70 seconds).
Given these findings, which one of the following is best
to recommend for B.R.?
A. Increase the bivalirudin infusion to target a higher
aPTT of 90–110 seconds to prevent formation of any
intracardiac thrombus.
B. Change bivalirudin to an argatroban infusion
starting at 0.2 mcg/kg/minute.

CCSAP 2021 Book 1 • Pulmonary and Endocrinology 188 Interactive Case: Extracorporeal Membrane Oxygenation
Learner Feature Evaluation: Interactive Case: Extracorporeal Membrane
Oxygenation

As you take the posttest for this feature, also evaluate the 24. The active learning methods used in the feature were
material’s quality and usefulness, as well as the achieve- effective.
ment of learning objectives. Rate each item using this 5-point 25. The learning assessment activities used in the feature
scale: were effective.
26. The feature was effective overall.
• Strongly agree
• Agree 27. The activity met the stated learning objectives.
• Neutral 28. If any objectives were not met, please list them here.
• Disagree
• Strongly disagree OTHER COMMENTS
16. The content of the feature met my educational needs. 29. Please provide any specific comments related to any
17. The content of the feature satisfied my expectations. perceptions of bias, promotion, or advertisement of
18. The author presented the feature content effectively. commercial products.

19. The content of the feature was relevant to my practice 30. Please expand on any of your above responses, and/or
and presented at the appropriate depth and scope. provide any additional comments regarding this feature:

20. The content of the feature was objective and balanced.


21. The content of the feature is free of bias, promotion, and
advertisement of commercial products.
22. The content of the feature was useful to me.
23. The teaching and learning methods used in the feature
were effective.

CCSAP 2021 Book 1 • Pulmonary and Endocrinology 189 Interactive Case: Extracorporeal Membrane Oxygenation
Interactive Case: Neuromuscular
Blocking Agents
By Sara J. Hyland, Pharm.D., BCCCP

Reviewed by Eric G. Johnson, Pharm.D., BCCCP; and Jaimini S. Patel, Pharm.D., BCCCP

LEARNING OBJECTIVES

1. Distinguish the pharmacologic and pharmacodynamic properties of various neuromuscular blocking agents (NMBAs) and
their reversal strategies.
2. Evaluate the depth of neuromuscular blockade and patient-specific factors to inform NMBA use and reversal in surgical
patients.
3. Design appropriate NMBA therapy for a patient requiring rapid sequence intubation.
4. Justify the appropriateness of NMBA use and reversal for nonrespiratory indications in critically ill patients.

ABBREVIATIONS IN THIS FEATURE


ACh Acetylcholine BASELINE KNOWLEDGE STATEMENTS

AChE Acetylcholinesterase Readers of this feature are presumed to be familiar


AChEI Acetylcholinesterase inhibitor with the following:
CICV “Can’t intubate, can’t ventilate” • Physiology of acetylcholine receptors
ICP Intracranial pressure • Indications for rapid sequence intubation and its
mAChR Muscarinic acetylcholine receptor components
nAChR Nicotinic acetylcholine receptor • Pathophysiology and general management of
NMB Neuromuscular blocking/blockade critical care conditions for which neuromuscular
NMBA Neuromuscular blocking agent blocking agents are used, including traumatic brain
injury, intra-abdominal hypertension, and targeted
NMJ Neuromuscular junction
temperature management
PNS Peripheral nerve stimulation
PPC Postoperative pulmonary Table of common laboratory reference values.
complication
PTC Posttetanic count ADDITIONAL READINGS
RSI Rapid sequence intubation
TBI Traumatic brain injury The following resources have additional background
TOF Train-of-four information on this topic:
TOFC Train-of-four count • Plaud B, Baillard C, Bourgain JL, et al. Guidelines on
muscle relaxants and reversal in anaesthesia.
TOFR Train-of-four ratio
Anaesth Crit Care Pain Med 2020;39:125-42.
Table of other common abbreviations. • Naguib M, Brull SJ, Kopman AF, et al. Consensus
statement on perioperative use of neuromuscular
monitoring. Anesth Analg 2018;127:71-80.
• Higgs A, McGrath BA, Goddard C, et al. Guidelines
for the management of tracheal intubation in
critically ill adults. Br J Anaesth 2018;120:323-52.
• Renew JR, Ratzlaff R, Hernandez-Torres V, et al.
Neuromuscular blockade management in the
critically ill patient. J Intensive Care Med 2020;8:37.
• Online Appendix: Review of NMJ Physiology and
NMBA Basic Pharmacology

CCSAP 2021 Book 1 • Pulmonary and Endocrinology 191 Interactive Case: Neuromuscular Blocking Agents
American College of Surgeons (ACS). ACS TQIP Best Practices
Practice Points in the Management of Traumatic Brain Injury. 2015.
• Neuromuscular blocking agents (NMBAs) are high-risk
medications with complex risk-benefit profiles in perioper- Andersson ML, Møller AM, Wildgaard K. Butyrylcholinesterase
ative and critical care populations. deficiency and its clinical importance in anaesthesia: a sys-
• NMBAs differ in mechanism and structure, and novel tematic review. Anaesthesia 2019;74:518-28.
reversal agents are revolutionizing their use.
Aniskevich S, Brull SJ, Naguib M. Chapter 9: neuromuscular
• Sugammadex is a novel encapsulating agent specific to
blocking agents. In: Johnson KB, ed. Clinical Pharmacology
aminosteroid NMBAs that can quickly reverse neuromus-
for Anesthesiology. New York: McGraw-Hill, 2015.
cular blockade (NMB) of any depth. Neostigmine is an
indirect reversal agent that can be used for any nondepo- April MD, Arana A, Pallin DJ, et al. Emergency department
larizing NMBA; however, it can only successfully antago- intubation success with succinylcholine versus rocuro-
nize shallow degrees of NMB. nium: a national emergency airway registry study.
• Clinical use of NMBAs in perioperative settings can be for Ann Emerg Med 2018;72:645-53.
anesthetic and/or surgical indications. Agent selection and
dosing should be patient-specific. Badaoui R, Cabaret A, Alami Y, et al. Reversal of neuromus-
• Perioperative NMB should be monitored and reversed using cular blockade by sugammadex in laparoscopic bariatric
train-of-four (TOF) peripheral nerve stimulation (PNS). Use surgery: in support of dose reduction. Anaesth Crit Care
of quantitative TOF monitoring to objectively determine Pain Med 2016;35:25-9.
train-of-four ratio (TOFR) is recommended but is not as
widely available as qualitative TOF monitoring to subjec- Bakhsh A. Rocuronium versus succinylcholine for rapid
tively assess train-of-four count (TOFC). sequence intubation. Acad Emerg Med 2020;27:66-8.
• Postoperative residual NMB represents a significant and
Baron Shahaf D, Hare GMT, Baker AJ, et al. A new index of
under-recognized source of morbidity and mortality. Ap-
coordinated posterior and anterior evoked EEG to detect
propriate NMBA monitoring and reversal to achieve a TOFR
recall under sedation – a pilot study. Sci Rep 2019;
of at least 0.9 before extubation are essential to mitigating
9:17859.
postoperative pulmonary complications (PPCs) and other
risks of residual NMB. Bartels K, Hunter JM. Neostigmine versus sugammadex:
• Optimal NMBA dosing and selection are paramount to the tide may be turning, but we still need to navigate the
successful intubation and to mitigating adverse effects in winds. Br J Anaesth 2020;124:504-7.
the setting of rapid sequence intubation (RSI).
• Many current nonrespiratory uses of NMBAs in critically ill Barthel F, Stojeba N, Lyons G, et al. Sugammadex in
patients are not supported by adequate evidence, and the rocuronium anaphylaxis: dose matters. Br J Anaesth
current practice guidelines are limited by this paucity of 2012;109:646-7.
data.
• Pharmacists caring for critically ill patients should have Batistaki C, Tentes P, Deligiannidi P, et al. Residual neuro-
the expertise necessary to inform interprofessional muscular blockade in a real life clinical setting: correlation
risk-benefit assessments regarding the use of NMBAs with sugammadex or neostigmine administration. Minerva
and reversal agents across the continuum of care. Anestesiol 2016;82:550-8.

Bellod A Jr, March X, Hernandez C, et al. Delayed recurari-


sation after sugammadex reversal. Eur J Anaesthesiol
INTERACTIVE CASE: 2014;31:710-2.
NEUROMUSCULAR BLOCKING Bhat R, Mazer-Amirshahi M, Sun C, et al. Accuracy of rapid
AGENTS sequence intubation medication dosing in obese patients
intubated in the ED. Am J Emerg Med 2016;34:2423-5.
• Click here to start this CCSAP activity.
Bhavani SS. Severe bradycardia and asystole after sugam-
madex. Br J Anaesth 2018;121:95-6.
REFERENCES
Abad-Gurumeta A, Ripollés-Melchor J, Casans-Francés R, Bisschops MMA, Holleman C, Huitink JM. Can sugammadex
et al. A systematic review of sugammadex vs neostig- save a patient in a simulated “cannot intubate, cannot ven-
mine for reversal of neuromuscular blockade. Anaesthesia tilate” situation? Anaesthesia 2010;65:936-41.
2015;70:1441-52.
Blichfeldt-Lauridsen L, Hansen BD. Anesthesia and myasthe-
Ahmad I, El-Boghdadly K, Bhagrath R, et al. Difficult Airway nia gravis. Acta Anaesthesiol Scand 2012;56:17-22.
Society guidelines for awake tracheal intubation (ATI) in
adults. Anaesthesia 2020;75:509-28. Bosch R, van Lierop MJ, de Kam PJ, et al. A PK-PD mod-
el-based assessment of sugammadex effects on
Alday E, Muñoz M, Planas A, et al. Effects of neuromuscular coagulation parameters. Eur J Pharm Sci 2016;84:9-17.
block reversal with sugammadex versus neostigmine on
postoperative respiratory outcomes after major abdomi- Bouju P, Tadié JM, Barbarot N, et al. Clinical assessment and
nal surgery: a randomized-controlled trial. Can J Anaesth train-of-four measurements in critically ill patients treated
2019;66:1328-37. with recommended doses of cisatracurium or atracurium

CCSAP 2021 Book 1 • Pulmonary and Endocrinology 192 Interactive Case: Neuromuscular Blocking Agents
for neuromuscular blockade: a prospective descriptive Carron M, De Cassai A, Linassi F. Sugammadex in the man-
study. Ann Intensive Care 2017;7:10. agement of myasthenic patients undergoing surgery:
beyond expectations. Ann Transl Med 2019;7(suppl 8):
Brett K, Farrah K. Sugammadex for the Reversal of S307.
Neuromuscular Blockade in Surgical Patients: A Review
of Clinical Effectiveness and Cost-Effectiveness. Carron M, Zarantonello F, Lazzarotto N, et al. Role of
Ottawa, Ontario, Canada: Canadian Agency for Drugs and sugammadex in accelerating postoperative discharge: a
Technologies in Health, August 6, 2019. meta-analysis. J Clin Anesth 2017;39:38-44.

Bronsert MR, Henderson WG, Monk TG, et al. Intermediate- Carron M, Zarantonello F, Tellaroli P, et al. Efficacy and safety
acting nondepolarizing neuromuscular blocking agents of sugammadex compared to neostigmine for reversal of
and risk of postoperative 30-day morbidity and mortality, neuromuscular blockade: a meta-analysis of randomized
and long-term survival. Anesth Analg 2017;124:1476-83. controlled trials. J Clin Anesth 2016b;35:1-12.

Brull SJ, Kopman AF. Current status of neuromuscular Chalermkitpanit P, Rodanant O, Thaveepunsan W, et al.
reversal and monitoring: challenges and opportunities. Determination of dose and efficacy of atracurium for rapid
Anesthesiology 2017;126:173-90. sequence induction of anesthesia: a randomised prospec-
tive study. J Anaesthesiol Clin Pharmacol 2020;36:37-42.
Brull SJ, Murphy GS. Residual neuromuscular block: lessons
unlearned. Part II: methods to reduce the risk of residual Che D, Rui L, Cao J, et al. Cisatracurium induces mast cell
weakness. Anesth Analg 2010;111:129-40. activation and pseudo-allergic reactions via MRGPRX2.
Int Immunopharmacol 2018;62:244-50.
Burbridge MA. Incidence of anaphylaxis to sugammadex in
a single-center cohort of 19,821 patients. Anesth Analg Checketts MR, Alladi R, Ferguson K, et al. Recommendations
2021;132:93-7. for standards of monitoring during anaesthesia and recov-
ery 2015: Association of Anaesthetists of Great Britain and
Callaway CW, Donnino MW, Fink EL, et al. Part 8: post-
Ireland. Anaesthesia 2016;71:85-93.
cardiac arrest care: 2015 American Heart Association
guidelines update for cardiopulmonary resuscitation Chen B. Sugammadex: a limited but important role in emer-
and emergency cardiovascular care. Circulation gency medicine. Pediatr Emerg Care 2020;36:296-301.
2015;132:S465-82.
Chesnut R, Aguilera S, Buki A, et al. A management algo-
Cammu G. Sugammadex: appropriate use in the context rithm for adult patients with both brain oxygen and
of budgetary constraints. Curr Anesthesiol Rep 2018; intracranial pressure monitoring: the Seattle International
8:178-85. Severe Traumatic Brain Injury Consensus Conference
(SIBICC). Intensive Care Med 2020a;46:919-29.
Cammu G. Residual neuromuscular blockade and postop-
erative pulmonary complications: what does the recent Chesnut RM, Temkin N, Videtta W, et al. Consensus-based
evidence demonstrate? Curr Anesthesiol Rep 2020 Mar management protocol (CREVICE Protocol) for the treat-
27;1-6. [Online ahead of print] ment of severe traumatic brain injury based on imaging
and clinical examination for use when intracranial
Cammu G, Van Vlem B, van den Heuvel M, et al. Dialysability
pressure monitoring is not employed. J Neurotrauma
of sugammadex and its complex with rocuronium in inten-
2020b;37:1291-9.
sive care patients with severe renal impairment. Br J
Anaesth 2012;109:382-90. Clancy M, Halford S, Walls R, et al. In patients with head
injuries who undergo rapid sequence intubation using
Carlson AB, Kraus GP. Physiology, cholinergic receptors.
succinylcholine, does pretreatment with a competitive
In: StatPearls. Treasure Island, FL: StatPearls, October 27,
neuromuscular blocking agent improve outcome?
2018.
A literature review. Emerg Med J 2001;18:373-5.
Carney N, Totten AM, O’Reilly C, et al. Guidelines for the
Corda D, Gravenstein N. Sugammadex: the anaphylactic risk.
Management of Severe Traumatic Brain Injury, Fourth
APSF Newsl 2018;30:3.
Edition. Neurosurgery 2017;80:6-15.
Cordero EI. Drug interactions with the use of neuromuscular
Carollo DS, White WM. Postoperative recurarization in a
blockers. Rev Cubana Anestesiol Reanim 2014;13:276-86.
pediatric patient after sugammadex reversal of rocu-
ronium-induced neuromuscular blockade. A A Pract Couto M, Nunes C, Vide S, et al. Rocuronium continuous
2019;13:204-5. infusion for profound neuromuscular blockade: a sys-
tematic review and meta-analysis. Clin Neuropharmacol
Carron M. Effects of sugammadex on coagulation: it does
2019;42:203-10.
not represent a bleeding risk in surgical patients. Int J Clin
Pharmacol Ther 2014;52:824. Curley JM, Ciceri DP, Culp WC Jr. Sugammadex admin-
istration to facilitate timely neurologic examination
Carron M, Baratto F, Pettenuzzo T, et al. Sugammadex as
in the traumatic brain injury patient. Neurocrit Care
rescue therapy for residual neuromuscular blockade in the
2020;32:880-2.
intensive care unit. Can J Anaesth 2016a;63:1384-5.

CCSAP 2021 Book 1 • Pulmonary and Endocrinology 193 Interactive Case: Neuromuscular Blocking Agents
Deana C, Barbariol F, D’Incà S, et al. SUGAMMADEX versus sugammadex vs. neostigmine. J Crit Care Med (Targu
neostigmine after ROCURONIUM continuous infusion Mures) 2015;1:61-7.
in patients undergoing liver transplantation. BMC
Anesthesiol 2020;20:70. Feltracco P, Tonetti T, Barbieri S, et al. Cisatracurium- and
rocuronium-associated residual neuromuscular dysfunc-
deBacker J, Hart N, Fan E. Neuromuscular blockade in the tion under intraoperative neuromuscular monitoring and
21st century management of the critically ill patient. postoperative neostigmine reversal: a single-blind ran-
Chest 2017;151:697-706. domized trial. J Clin Anesth 2016;35:198-204.

de Boer HD, Carlos RV. New drug developments for Fernández-Candil J. Sugammadex overdose? Randomized
neuromuscular blockade and reversal: gantacurium, trial with 1 or 2 mg/kg of sugammadex to reverse mod-
CW002, CW011, and calabadion. Curr Anesthesiol Rep erate rocuronium-induced neuromuscular blockade.
2018a;8:119-24. Presented at: European Society of Anesthesia Annual
Meeting; May 2016; London, UK.
de Boer HD, Carlos RV, Brull SJ. Is lower-dose sugammadex
a cost-saving strategy for reversal of deep neuromuscular Fodale V, Santamaria LB. Laudanosine, an atracurium
block? Facts and fiction. BMC Anesthesiol 2018b;18:159. and cisatracurium metabolite. Eur J Anaesthesiol
2002;19:466-73.
De Kam PJ, Grobara P, Prohn M, et al. Effects of sugam-
madex on activated partial thromboplastin time and Fortier LP, McKeen D, Turner K, et al. The RECITE study: a
prothrombin time in healthy subjects. Int J Clin Pharmacol Canadian prospective, multicenter study of the incidence
Ther 2014;52:227-36. and severity of residual neuromuscular blockade. Anesth
Analg 2015;121:366-72.
de Kam PJ, Nolte H, Good S, et al. Sugammadex hypersen-
sitivity and underlying mechanisms: a randomised study Frerk C, Mitchell VS, McNarry AF, et al. Difficult Airway
of healthy non-anaesthetised volunteers. Br J Anaesth Society 2015 guidelines for management of unanticipated
2018;121:758-67. difficult intubation in adults. Br J Anaesth 2015;115:827-48.
De Laet IE, Malbrain MLNG, De Waele JJ. A clinician’s guide Fuchs-Buder T, De Robertis E, Brunaud L. Neuromuscular
to management of intra-abdominal hypertension and block in laparoscopic surgery. Minerva Anestesiol
abdominal compartment syndrome in critically ill patients. 2018;84:509-14.
Crit Care 2020;24:97.
Fujita A, Ishibe N, Yoshihara T, et al. Rapid reversal of neu-
de Lemos JM, Carr RR, Shalansky KF, et al. Paralysis in the romuscular blockade by sugammadex after continuous
critically ill: intermittent bolus pancuronium compared infusion of rocuronium in patients with liver dysfunction
with continuous infusion. Crit Care Med 1999;27:2648-55. undergoing hepatic surgery. Acta Anaesthesiol Taiwan
2014;52:54-8.
de Souza CM, Tardelli MA, Tedesco H, et al. Efficacy and
safety of sugammadex in the reversal of deep neuromus- Gajewski M, Esochaghi S. Transient asystole after sugam-
cular blockade induced by rocuronium in patients with madex administration for immediate reversal of deep
end-stage renal disease: a comparative prospective clini- blockade while on dexmedetomidine infusion in a super
cal trial. Eur J Anaesthesiol 2015;32:681-6. obese patient. Case Rep Anesthesiol 2019;2019:2709568.
Dewachter P, Mouton-Faivre C. Anaphylaxis incidence with Gaszynski TM, Szewczyk T. Rocuronium for rapid sequence
rocuronium, succinylcholine, and atracurium: how risk induction in morbidly obese patients: a prospective study
communication can influence behavior. Anesthesiology for evaluation of intubation conditions after administra-
2015;123:735-6. tion 1.2 mg kg−1 ideal body weight of rocuronium.
Deyhim N, Beck A, Balk J, et al. Impact of sugammadex Eur J Anaesthesiol 2011;28:609-10.
versus neostigmine/glycopyrrolate on perioperative effi- Gavel G, Walker RWM. Laryngospasm in anaesthesia. Contin
ciency. Clinicoecon Outcomes Res 2020;12:69-79. Educ Anaesth Crit Care Pain 2014;14:47-51.
Engelman DT, Ben Ali W, Williams JB, et al. Guidelines for Gerlach RM, Chaney MA. Appropriate use of neuromus-
perioperative care in cardiac surgery: Enhanced Recovery cular blocking agents and reversal drugs to enhance
After Surgery Society recommendations. JAMA Surg recovery following cardiac surgery. Curr Anesthesiol Rep
2019;154:755-66. 2020;10:267-72.
Evron S, Abelansky Y, Ezri T, et al. Respiratory events with
Gerlach RM, Shahul S, Wroblewski KE, et al. Intraoperative
sugammadex vs. neostigmine following laparoscopic
use of nondepolarizing neuromuscular blocking agents
sleeve gastrectomy: a prospective pilot study assess-
during cardiac surgery and postoperative pulmonary com-
ing neuromuscular reversal strategies. Rom J Anaesth
plications: a prospective randomized trial. J Cardiothorac
Intensive Care 2017;24:111-4.
Vasc Anesth 2019;33:1673-81.
Ezri T, Evron S, Petrov I, et al. Residual curarization and
Goto T, Goto Y, Hagiwara Y, et al. Advancing emergency air-
postoperative respiratory complications following laparo-
way management practice and research. Acute Med Surg
scopic sleeve gastrectomy. The effect of reversal agents:
2019;6:336-51.

CCSAP 2021 Book 1 • Pulmonary and Endocrinology 194 Interactive Case: Neuromuscular Blocking Agents
Green NH. To BIS or not to BIS. Br J Anaesth 2016;116:726-7. Hunter JM, Naguib M. Sugammadex-induced bradycar-
dia and asystole: how great is the risk? Br J Anaesth
Guihard B, Chollet-Xémard C, Lakhnati P, et al. Effect of rocu- 2018;121:8-12.
ronium vs succinylcholine on endotracheal intubation
success rate among patients undergoing out-of-hospi- Hussain Z, Curtain C, Mirkazemi C, et al. Peri-operative med-
tal rapid sequence intubation: a randomized clinical trial. ication dosing in adult obese elective surgical patients: a
JAMA 2019;322:2303-12. systematic review of clinical studies. Clin Drug Investig
2018;38:673-93.
Gunduz Gul G, Ozer AB, Demirel I, et al. The effect of sugam-
madex on steroid hormones: a randomized clinical study. Iga S, Shinji IGA, Shimizu K, et al. Recurarization following
J Clin Anesth 2016;34:62-7. adequate dose of sugammadex reversal in a patient mon-
itored with acceleromyography. J Japan Soc Clin Anesth
Gustafsson UO, Scott MJ, Hubner M, et al. Guidelines 2016;36:1-6.
for perioperative care in elective colorectal surgery:
Enhanced Recovery After Surgery (ERAS®) society rec- Ingrande J, Lemmens HJ. Anesthetic pharmacology and the
ommendations: 2018. World J Surg 2019;43:659-95. morbidly obese patient. Curr Anesthesiol Rep 2013;3:10-7.

Haerter F, Eikermann M. Reversing neuromuscular blockade: Iwasaki H, Renew JR, Kunisawa T, et al. Preparing for the
inhibitors of the acetylcholinesterase versus the encapsu- unexpected: special considerations and complications
lating agents sugammadex and calabadion. Expert Opin after sugammadex administration. BMC Anesthesiol
Pharmacother 2016;17:819-33. 2017;17:140.
Hafeez KR, Tuteja A, Singh M, et al. Postoperative complica- Jahr JS, Miller JE, Hiruma J, et al. Sugammadex: a sci-
tions with neuromuscular blocking drugs and/or reversal entific review including safety and efficacy, update on
agents in obstructive sleep apnea patients: a systematic regulatory issues, and clinical use in Europe. Am J Ther
review. BMC Anesthesiol 2018;18:91. 2015;22:288-97.
Hammermeister K, Bronsert M, Richman JS, et al. Residual Juels A. Perioperative anaphylaxis: symptoms, diagnosis,
neuromuscular blockade (NMB), reversal, and periopera- and management. In: Anesthesiology News [Internet]. 21
tive outcomes. APSF Newsl 2016;30:74-5. Jun 2019 [cited 28 Jun 2020].
Hampton JP. Rapid-sequence intubation and the role of the Kang WS, Lim H, Kim BS, et al. Assessment of the effects of
emergency department pharmacist. Am J Health Syst sugammadex on coagulation profiles using thromboelas-
Pharm 2011;68:1320-30. tographic parameters. Sci Rep 2020;10:11179.
Hawryluk GWJ, Aguilera S, Buki A, et al. A management Katz JA, Murphy GS. Anesthetic consideration for neuromus-
algorithm for patients with intracranial pressure moni- cular diseases. Curr Opin Anaesthesiol 2017;30:435-40.
toring: the Seattle International Severe Traumatic Brain
Injury Consensus Conference (SIBICC). Intensive Care Keating GM. Sugammadex: a review of neuromuscular block-
Med 2019;45:1783-94. ade reversal. Drugs 2016;76:1041-52.

Hemmerling TM, Zaouter C, Geldner G, et al. Sugammadex Kheterpal S, Vaughn MT, Dubovoy TZ, et al. Sugammadex
– a short review and clinical recommendations for versus neostigmine for reversal of neuromuscular
the cardiac anesthesiologist. Ann Card Anaesth blockade and postoperative pulmonary complications
2010;13:206-16. (STRONGER): a multicenter matched cohort analysis.
Anesthesiology 2020;132:1371-81.
Higgs A, McGrath BA, Goddard C, et al. Guidelines for the
management of tracheal intubation in critically ill adults. Kirmeier E, Eriksson LI, Lewald H, et al. Post-anaesthesia
Br J Anaesth 2018;120:323-52. pulmonary complications after use of muscle relaxants
(POPULAR): a multicentre, prospective observational
Honing G, Martini CH, Bom A, et al. Safety of sugammadex
study. Lancet Respir Med 2019;7:129-40.
for reversal of neuromuscular block. Expert Opin Drug Saf
2019;18:883-91. Koo CH, Hwang JY, Min SW, et al. A meta-analysis on the
effect of dexamethasone on the sugammadex reversal
Howard J, Wigley J, Rosen G, et al. Glycopyrrolate: it’s time to
of rocuronium-induced neuromuscular block. J Clin Med
review. J Clin Anesth 2017;36:51-3.
2020;9:1240.
Hraiech S, Forel JM, Guervilly C, et al. How to reduce cisa-
Korinek JD, Thomas RM, Goddard LA, et al. Comparison of
tracurium consumption in ARDS patients: the TOF-ARDS
rocuronium and succinylcholine on postintubation seda-
study. Ann Intensive Care 2017;7:79.
tive and analgesic dosing in the emergency department.
Hristovska AM, Duch P, Allingstrup M, et al. The comparative Eur J Emerg Med 2014;21:206.
efficacy and safety of sugammadex and neostigmine in
Kotake Y, Ochiai R, Suzuki T, et al. Reversal with sugamma-
reversing neuromuscular blockade in adults. A Cochrane
dex in the absence of monitoring did not preclude residual
systematic review with meta-analysis and trial sequential
neuromuscular block. Anesth Analg 2013;117:345-51.
analysis. Anaesthesia 2018;73:631-41.

CCSAP 2021 Book 1 • Pulmonary and Endocrinology 195 Interactive Case: Neuromuscular Blocking Agents
Kounis NG, Koniari I, Soufras GD, et al. Sugammadex- Lentz S, Grossman A, Koyfman A, et al. High-risk airway
induced atropine-resistant bradycardia: clinical, management in the emergency department: diseases and
pathophysiologic, and electrocardiographic consider- approaches. Part II. J Emerg Med 2020b;59:573-85.
ations. JA Clin Rep 2020;6:31.
Levitan R. Safety of succinylcholine in myasthenia gravis.
Kramer N, Lebowitz D, Walsh M, et al. Rapid sequence intu- Ann Emerg Med 2005;45:225-6.
bation in traumatic brain-injured adults. Cureus 2018;
10:e2530. Lewis SR, Pritchard MW, Fawcett LJ, et al. Bispectral index
for improving intraoperative awareness and early postop-
Krause M, McWilliams SK, Bullard KJ, et al. Neostigmine ver- erative recovery in adults. Cochrane Database Syst Rev
sus sugammadex for reversal of neuromuscular blockade 2019;9:CD003843.
and effects on reintubation for respiratory failure or newly
initiated noninvasive ventilation: an interrupted time series Li YL, Liu YL, Xu CM, et al. The effects of neuromuscu-
design. Anesth Analg 2020;131:141-51. lar blockade on operating conditions during general
anesthesia for spinal surgery. J Neurosurg Anesthesiol
Kulkarni LM, Sanikop C, Shilpa H, et al. Anaesthetic manage- 2014;26:45-9.
ment in a patient with multiple sclerosis. Indian J Anaesth
2011;55:64-7. Lorinc AN, Lawson KC, Niconchuk JA, et al. Residual
weakness and recurarization after sugammadex admin-
Kyle BC, Gaylard D, Riley RH. A persistent “can’t intubate, istration in pediatric patients: a case series. A A Pract
can’t oxygenate” crisis despite rocuronium reversal with 2020;14:e01225.
sugammadex. Anaesth Intensive Care 2012;40:344-6.
Loupec T, Frasca D, Rousseau N, et al. Appropriate dos-
Laguna JJ, Archilla J, Doña I, et al. Practical guidelines ing of sugammadex to reverse deep rocuronium-induced
for perioperative hypersensitivity reactions. J Investig neuromuscular blockade in morbidly obese patients.
Allergol Clin Immunol 2018;28:216-32. Anaesthesia 2016;71:265-72.
Lamberg JJ, Answine JF. A surgeon’s assessment of inade-
Luckianow GM, Ellis M, Governale D, et al. Abdominal com-
quate neuromuscular antagonism in a case of prolonged
partment syndrome: risk factors, diagnosis, and current
neuromuscular blockade. J Anaesthesiol Clin Pharmacol
therapy. Crit Care Res Pract 2012;2012:908169.
2013;29:244-7.
Lundstrøm LH, Duez CH, Nørskov AK, et al. Avoidance versus
Lang LH, Parekh K, Tsui BYK, et al. Perioperative manage-
use of neuromuscular blocking agents for improving con-
ment of the obese surgical patient. Br Med Bull 2017;
ditions during tracheal intubation or direct laryngoscopy
124:135-55.
in adults and adolescents. Cochrane Database Syst Rev
Langeron O, Bourgain JL, Francon D, et al. Difficult intubation 2017;5:CD009237.
and extubation in adult anaesthesia. Anaesth Crit Care
Lundstrøm LH, Duez CHV, Nørskov AK, et al. Effects of
Pain Med 2018;37:639-51.
avoidance or use of neuromuscular blocking agents on
Le Corre F, Nejmeddine S, Fatahine C, et al. Recurarization outcomes in tracheal intubation: a Cochrane systematic
after sugammadex reversal in an obese patient. Can J review. Br J Anaesth 2018;120:1381-93.
Anaesth 2011;58:944-7.
Luo J, Chen S, Min S, et al. Reevaluation and update
Ledowski T, Hillyard S, O’Dea B, et al. Introduction of on efficacy and safety of neostigmine for reversal
sugammadex as standard reversal agent: impact on the of neuromuscular blockade. Ther Clin Risk Manag
incidence of residual neuromuscular blockade and postop- 2018;14:2397-406.
erative patient outcome. Indian J Anaesth 2013;57:46-51.
Madden LK, Hill M, May TL, et al. The implementation of
Lee BK, Cho IS, Oh JS, et al. Continuous neuromuscu- targeted temperature management: an evidence-based
lar blockade infusion for out-of-hospital cardiac arrest guideline from the Neurocritical Care Society. Neurocrit
patients treated with targeted temperature manage- Care 2017;27:468-87.
ment: a multicenter randomized controlled trial. PLoS One
2018;13:e0209327. Madsen MV, Staehr-Rye AK, Gätke MR, et al. Neuromuscular
blockade for optimising surgical conditions during abdom-
Lee DH, Lee BK, Jeung KW, et al. Neuromuscular blockade inal and gynaecological surgery: a systematic review. Acta
requirement is associated with good neurologic outcome Anaesthesiol Scand 2015;59:1-16.
in cardiac arrest survivors treated with targeted tempera-
ture management. J Crit Care 2017;40:218-24. Malbrain MLNG, Chiumello D, Cesana BM, et al. A system-
atic review and individual patient data meta-analysis on
Lee LA, Athanassoglou V, Pandit JJ. Neuromuscular block- intra-abdominal hypertension in critically ill patients:
ade in the elderly patient. J Pain Res 2016;9:437-44. the wake-up project. World Initiative on Abdominal
Lemmens HJM, Brodsky JB. The dose of succinylcholine in Hypertension Epidemiology, a Unifying Project
morbid obesity. Anesth Analg 2006;102:438-42. (WAKE-Up!). Minerva Anestesiol 2014;80:293-306.

Lentz S, Grossman A, Koyfman A, et al. High-risk airway Marehbian J, Muehlschlegel S, Edlow BL, et al. Medical
management in the emergency department. Part I: dis- management of the severe traumatic brain injury patient.
eases and approaches. J Emerg Med 2020a;59:85-95. Neurocrit Care 2017;27:430-46.

CCSAP 2021 Book 1 • Pulmonary and Endocrinology 196 Interactive Case: Neuromuscular Blocking Agents
Marsch SC, Steiner L, Bucher E, et al. Succinylcholine ver- of thoracic surgical patients evaluating hypoxic epi-
sus rocuronium for rapid sequence intubation in intensive sodes in the early postoperative period. J Clin Anesth
care: a prospective, randomized controlled trial. Crit Care 2020;64:109804.
2011;15:R199.
Moore L, Kramer CJ, Delcoix-Lopes S, et al. Comparison of
Martinez-Ubieto J, Ortega-Lucea S, Pascual-Bellosta A, et cisatracurium versus atracurium in early ARDS. Respir
al. Prospective study of residual neuromuscular block Care 2017;62:947-52.
and postoperative respiratory complications in patients
Mosier JM, Sakles JC, Law JA, et al. Tracheal intubation in
reversed with neostigmine versus sugammadex. Minerva
the critically ill. Where we came from and where we should
Anestesiol 2016;82:735-42.
go. Am J Respir Crit Care Med 2020;201:775-88.
Martyn JAJ, Richtsfeld M. Succinylcholine-induced hyperka-
Mosier JM, Sakles JC, Stolz U, et al. Neuromuscular block-
lemia in acquired pathologic states: etiologic factors and
ade improves first-attempt success for intubation in the
molecular mechanisms. Anesthesiology 2006;104:158-69.
intensive care unit. A propensity matched analysis.
Mashour GA, Avidan MS. Intraoperative awareness: Ann Am Thorac Soc 2015;12:734-41.
controversies and non-controversies. Br J Anaesth
Murata T, Kubodera T, Ohbayashi M, et al. Recurarization
2015;115(suppl 1):i20-6.
after sugammadex following a prolonged rocuronium
May TL, Riker RR, Fraser GL, et al. Variation in sedation and infusion for induced hypothermia. Can J Anaesth
neuromuscular blockade regimens on outcome after car- 2013;60:508-9.
diac arrest. Crit Care Med 2018;46:e975-80. Murphy G. The development and regulatory history
McDonnell NJ, Pavy TJG, Green LK, et al. Sugammadex in the of sugammadex in the United States. APSF Newsl
management of rocuronium-induced anaphylaxis. 2016;30:53-4.
Br J Anaesth 2011;106:199-201. Murphy GS, Brull SJ. Residual neuromuscular block: lessons
McKenzie AJ. Neostigmine anaphylaxis: a rare and missed unlearned. Part I: definitions, incidence, and adverse phys-
diagnosis. Anaesth Intensive Care 2020;48:59-60. iologic effects of residual neuromuscular block. Anesth
Analg 2010;111:120-8.
Mefford B, Donaldson JC, Bissell BD. To block or not:
updates in neuromuscular blockade in acute respiratory Murphy GS, Szokol JW, Avram MJ, et al. Residual neuro-
distress syndrome. Ann Pharmacother 2020;54:899-906. muscular block in the elderly: incidence and clinical
implications. Anesthesiology 2015;123:1322-36.
Mertes PM, Ebo DG, Garcez T, et al. Comparative epidemiol-
Murray MJ, DeBlock H, Erstad B, et al. Clinical practice guide-
ogy of suspected perioperative hypersensitivity reactions.
lines for sustained neuromuscular blockade in the adult
Br J Anaesth 2019;123:e16-28.
critically ill patient. Crit Care Med 2016;44:2079-103.
Mertes PM, Volcheck GW. Anaphylaxis to neuromuscu-
Mushambi MC, Kinsella SM, Popat M, et al. Obstetric
lar-blocking drugs: all neuromuscular-blocking drugs are
Anaesthetists’ Association and Difficult Airway
not the same. Anesthesiology 2015;122:5-7.
Society guidelines for the management of difficult and
Meyhoff CS, Lund J, Jenstrup MT, et al. Should dosing of failed tracheal intubation in obstetrics. Anaesthesia
rocuronium in obese patients be based on ideal or cor- 2015;70:1286-306.
rected body weight? Anesth Analg 2009;109:787-92.
Naguib M, Brewer L, LaPierre C, et al. The myth of rescue
Min KC, Woo T, Assaid C, et al. Incidence of hypersensitivity reversal in “can’t intubate, can’t ventilate” scenarios.
and anaphylaxis with sugammadex. J Clin 2018;47:67-73. Anesth Analg 2016;123:82-92.

Mirza K, Landoski K, Thakar D, et al. Sugammadex- Naguib M, Brull SJ, Kopman AF, et al. Consensus statement
associated hypotension, bradycardia, asystole, and death. on perioperative use of neuromuscular monitoring. Anesth
Case Rep Anesthesiol 2020;2020:8767195. Analg 2018;127:71-80.

Miskovic A, Lumb AB. Postoperative pulmonary complica- Naguib M, Flood P, McArdle JJ, et al. Advances in neurobi-
tions. Br J Anaesth 2017;118:317-34. ology of the neuromuscular junction: implications for the
anesthesiologist. Anesthesiology 2002;96:202-31.
Miyazaki Y, Sunaga H, Kida K, et al. Incidence of ana-
phylaxis associated with sugammadex. Anesth Analg Naguib M, Johnson KB. Innovative disruption in the world of
2018;126:1505-8. neuromuscular blockade: what is the “state of the art”?
Anesthesiology 2017;126:12-5.
Monk TG, Rietbergen H, Woo T, et al. Use of sugammadex
Naguib M, Lien CA. Pharmacology of muscle relaxants and
in patients with obesity: a pooled analysis. Am J Ther
their antagonists. In: Miller’s Anesthesia, 7th ed. New
2017;24:e507-16.
York: Churchill Livingston, 2010:859-911.
Moon TS, Reznik S, Pak T, et al. Sugammadex versus
Natt BS, Malo J, Hypes CD, et al. Strategies to improve first
neostigmine for reversal of rocuronium-induced neuro-
attempt success at intubation in critically ill patients.
muscular blockade: a randomized, double-blinded study
Br J Anaesth 2016;117(suppl 1):i60-8.

CCSAP 2021 Book 1 • Pulmonary and Endocrinology 197 Interactive Case: Neuromuscular Blocking Agents
Nolan JP, Soar J, Cariou A, et al. European Resuscitation Patanwala AE, Stahle SA, Sakles JC, et al. Comparison of
Council and European Society of Intensive Care Medicine succinylcholine and rocuronium for first-attempt intuba-
guidelines for post-resuscitation care 2015: section 5 tion success in the emergency department. Acad Emerg
of the European Resuscitation Council Guidelines for Med 2011;18:10-4.
Resuscitation 2015. Resuscitation 2015;95:202-22.
Petitpain N, Argoullon L, Masmoudi K, et al. Neuromuscular
Oh SK, Kwon WK, Park S, et al. Comparison of operating blocking agents induced anaphylaxis: results and trends
conditions, postoperative pain and recovery, and overall of a French pharmacovigilance survey from 2000 to 2012.
satisfaction of surgeons with deep vs. no neuromuscu- Allergy 2018;73:2224-33.
lar blockade for spinal surgery under general anesthesia:
a prospective randomized controlled trial. J Clin Med Res Pimentel MP, Billings F, Sivashanker K, et al. Reducing med-
2019a;8:498. ication waste while improving access to sugammadex: a
quality improvement project in medication stewardship.
Oh TK, Oh AY, Ryu JH, et al. Retrospective analysis of A A Pract 2020;14:e01223.
30-day unplanned readmission after major abdominal
surgery with reversal by sugammadex or neostigmine. Platt PR, Clarke RC, Johnson GH, et al. Efficacy of sugammadex
Br J Anaesth 2019b;122:370-8. in rocuronium-induced or antibiotic-induced anaphylaxis: a
case-control study. Anaesthesia 2015;70:1264-7.
Oh TK, Ryu JH, Nam S, et al. Association of neuromuscular
reversal by sugammadex and neostigmine with 90-day Plaud B, Baillard C, Bourgain JL, et al. Guidelines on muscle
mortality after non-cardiac surgery. BMC Anesthesiol relaxants and reversal in anaesthesia. Anaesth Crit Care
2020;20:41. Pain Med 2020;39:125-42.

Okubo M, Gibo K, et al.; Japanese Emergency Medicine Putzu A, Tramèr MR, Giffa M, et al. The optimal dose of
Network Investigators. The effectiveness of rapid succinylcholine for rapid sequence induction: a system-
sequence intubation (RSI) versus non-RSI in emergency atic review and meta-analysis of randomized trials. BMC
department: an analysis of multicenter prospective Anesthesiol 2020;20:54.
observational study. Int J Emerg Med 2017;10:1.
Rahe-Meyer N, Fennema H, Schulman S, et al. Effect of rever-
Orihara M, Takazawa T, Horiuchi T, et al. Comparison of sal of neuromuscular blockade with sugammadex versus
incidence of anaphylaxis between sugammadex and usual care on bleeding risk in a randomized study of surgi-
neostigmine: a retrospective multicentre observational cal patients. Anesthesiology 2014;121:969-77.
study. Br J Anaesth 2020;124:154-63.
Rajasurya V, Surani S. Abdominal compartment syndrome:
Ortiz VE, Wiener-Kronish J, eds. Perioperative Anesthetic often overlooked conditions in medical intensive care
Care of the Obese Patient. Boca Raton, FL: CRC Press, units. World J Gastroenterol 2020;26:266-78.
2016.
Raval AD, Deshpande S, Rabar S, et al. Does deep neuromus-
Padar M, Reintam Blaser A, Talving P, et al. Abdominal cular blockade during laparoscopy procedures change
compartment syndrome: improving outcomes with a patient, surgical, and healthcare resource outcomes? A
multidisciplinary approach – a narrative review. systematic review and meta-analysis of randomized con-
J Multidiscip Healthc 2019;12:1061-74. trolled trials. PLoS One 2020a;15:e0231452.

Park C. Risk factors associated with inpatient cardiac arrest Raval AD, Uyei J, Karabis A, et al. Incidence of residual neu-
during emergency endotracheal intubation at general romuscular blockade and use of neuromuscular blocking
wards. Acute Crit Care 2019;34:212-8. agents with or without antagonists: a systematic review
and meta-analysis of randomized controlled trials. J Clin
Partownavid P, Romito BT, Ching W, et al. Sugammadex: a Anesth 2020b;64:109818.
comprehensive review of the published human science,
including renal studies. Am J Ther 2015;22:298-317. Reddy JI, Cooke PJ, van Schalkwyk JM, et al. Anaphylaxis is
more common with rocuronium and succinylcholine than
Patanwala AE. Paralytic agents for intubation in the with atracurium. Anesthesiology 2015;122:39-45.
out-of-hospital setting. JAMA 2020;323:1506-7.
Renew JR. Avoiding postoperative residual weakness – a
Patanwala AE, Erstad BL, Roe DJ, et al. Succinylcholine cornerstone of any ERAS protocol. APSF Newsl 2019;
is associated with increased mortality when used for 34:48.
rapid sequence intubation of severely brain injured
patients in the emergency department. Pharmacotherapy Renew JR, Naguib M. Management of neuromuscular block-
2016;36:57-63. ade in the elderly and morbidly obese patient: what does
the data show? Curr Anesthesiol Rep 2020a;10:107-16.
Patanwala AE, Sakles JC. Effect of patient weight on first
pass success and neuromuscular blocking agent dosing Renew JR, Ratzlaff R, Hernandez-Torres V, et al.
for rapid sequence intubation in the emergency depart- Neuromuscular blockade management in the critically Ill
ment. Emerg Med J 2017;34:739-43. patient. J Intensive Care Med 2020b;8:37.

CCSAP 2021 Book 1 • Pulmonary and Endocrinology 198 Interactive Case: Neuromuscular Blocking Agents
Roantree RAG, Furtado CS, Goldstein S. EMS, facilitated intu- attenuating the haemodynamic response. Br J Anaesth
bation without paralytics. In: StatPearls. Treasure Island, 1985;57:550-3.
FL: StatPearls, 2020.
Shaw I, Trueger NS, Pirotte MJ. What is the time to muscle
Rodney G, Raju PKBC, Ball DR. Not just monitoring; a strat- relaxation after intramuscular administration of neuro-
egy for managing neuromuscular blockade. Anaesthesia muscular blockers? Ann Emerg Med 2015;66:390-3.
2015;70:1105-9.
Smith SE, Hamblin SE, Dennis BM. Effect of neuromuscu-
Roy M, Morissette N, Girard M, et al. Postoperative awake lar blocking agents on sedation requirements in trauma
paralysis in the intensive care unit after cardiac surgery patients with an open abdomen. Pharmacotherapy 2019;
due to residual neuromuscular blockade: a case report 39:271-9.
and prospective observational study. Can J Anaesth
2016;63:725-30. Smith SE, Hamblin SE, Guillamondegui OD, et al. Effectiveness
and safety of continuous neuromuscular blockade in
Saager L, Maiese EM, Bash LD, et al. Incidence, risk factors, trauma patients with an open abdomen: a follow-up study.
and consequences of residual neuromuscular block in the Am J Surg 2018;216:414-9.
United States: the prospective, observational, multicenter
RECITE-US study. J Clin Anesth 2019;55:33-41. Society for Obstetric Anesthesia and Perinatology (SOAP).
Statement on Sugammadex During Pregnancy and
Sadleir PHM, Clarke RC, Bunning DL, et al. Anaphylaxis to Lactation, 2019.
neuromuscular blocking drugs: incidence and cross-reac-
tivity in Western Australia from 2002 to 2011. Br J Anaesth Soleimanpour H, Safari S, Sanaie S, et al. Anesthetic consid-
2013;110:981-7. erations in patients undergoing bariatric surgery: a review
article. Anesth Pain Med 2017;7:e57568.
Sakızcı-Uyar B, Çelik S, Postacı A, et al. Comparison of the
effect of rocuronium dosing based on corrected or lean Sørensen MK, Bretlau C, Gätke MR. Rapid sequence induc-
body weight on rapid sequence induction and neuromus- tion and intubation with rocuronium–sugammadex
cular blockade duration in obese female patients. Saudi compared with succinylcholine: a randomized trial.
Med J 2016;37:60-5. Br J Anesthes 2012;108:682-9.
Sanfilippo F, Santonocito C, Veenith T, et al. The role of neuro- Soto RG, Dunipace D. Mivacurium: return of a drug seeking
muscular blockade in patients with traumatic brain injury: an indication? ASA Monitor 2017;81:30-1.
a systematic review. Neurocrit Care 2015;22:325-34.
Soto Mesa D, Fayad Fayad M, Pérez Arviza L, et al. Efficacy
Sanoja IA, Toth KS. Profound bradycardia and cardiac arrest of different doses of sugammadex after continuous infu-
after sugammadex administration in a previously healthy sion of rocuronium. World J Clin Cases 2015;3:360-7.
patient: a case report. A A Pract 2019;12:22-4.
Sottile PD, Kiser TH, Burnham EL, et al. An observational
Sasakawa T, Miyasaka K, Sawa T, et al. Postoperative recu- study of the efficacy of cisatracurium compared with
rarization after sugammadex administration due to vecuronium in patients with or at risk for acute respi-
the lack of appropriate neuromuscular monitoring: the ratory distress syndrome. Am J Respir Crit Care Med
Japanese experience. APSF Newsl 2020;35:42-3. 2018;197:897-904.
Sato N, Hagiwara Y, et al.; Japanese Emergency Medicine
Spoerl D, Nigolian H, Czarnetzki C, et al. Reclassifying ana-
Network Investigators. A comparison of emergency airway
phylaxis to neuromuscular blocking agents based on the
management between neuromuscular blockades alone
presumed patho-mechanism: IgE-mediated, pharmacolog-
and rapid sequence intubation: an analysis of multicenter
ical adverse reaction or “innate hypersensitivity”?
prospective study. BMC Res Notes 2017;10:6.
Int J Mol Sci 2017;18:1223.
Schaller SJ, Fink H. Sugammadex as a reversal agent for
Srivastava A, Hunter JM. Reversal of neuromuscular block.
neuromuscular block: an evidence-based review. Core
Br J Anaesth 2009;103:115-29.
Evid 2013;8:57-67.
Stöckl M, Testori C, Sterz F, et al. Continuous versus intermit-
Schepens T, Cammu G. Neuromuscular blockade: what was,
tent neuromuscular blockade in patients during targeted
is and will be. Acta Anaesthesiol Belg 2014;65:151-9.
temperature management after resuscitation from cardiac
Schreiber JU, Lysakowski C, Fuchs-Buder T, et al. Prevention arrest – a randomized, double blinded, double dummy,
of succinylcholine-induced fasciculation and myalgia: clinical trial. Resuscitation 2017;120:14-9.
a meta-analysis of randomized trials. Anesthesiology
2005;103:877-84. Stoelting R. APSF survey results: drug-induced muscle
weakness in the postoperative period safety initiative.
Schuller PJ, Newell S, Strickland PA, et al. Response of APSF Newsl 2014;28:49-70.
bispectral index to neuromuscular block in awake volun-
teers. Br J Anaesth 2015;115(suppl 1):i95-103. Stollings JL, Diedrich DA, Oyen LJ, et al. Rapid-sequence
intubation: a review of the process and consider-
Scott RP, Savarese JJ, Basta SJ, et al. Atracurium: clin- ations when choosing medications. Ann Pharmacother
ical strategies for preventing histamine release and 2014;48:62-76.

CCSAP 2021 Book 1 • Pulmonary and Endocrinology 199 Interactive Case: Neuromuscular Blocking Agents
Sturgess DJ, Greenland KB, Senthuran S, et al. Tracheal Trujillo R, West WP. Pseudocholinesterase deficiency. In:
extubation of the adult intensive care patient with a pre- StatPearls. Treasure Island, FL: StatPearls, 2019.
dicted difficult airway – a narrative review. Anaesthesia
2017;72:248-61. Tsai-Nguyen G, Modrykamien AM. Use of neuromuscular
blocking agents in acute respiratory distress syndrome.
Swaminathan AK, Mallemat H. Rocuronium should be the Proc (Bayl Univ Med Cent) 2018;31:177-9.
default paralytic in rapid sequence intubation. Ann Emerg
Med 2018;71:397-8. Tsur A, Kalansky A. Hypersensitivity associated with sugam-
madex administration: a systematic review. Anaesthesia
Tajaate N, Schreiber JU, Fuchs-Buder T, et al. 2014;69:1251-7.
Neostigmine-based reversal of intermediate acting neu-
romuscular blocking agents to prevent postoperative Van Lancker P, Dillemans B, Bogaert T, et al. Ideal versus
residual paralysis: a systematic review. Eur J Anaesthesiol corrected body weight for dosage of sugammadex in
2018;35:184-92. morbidly obese patients. Anaesthesia 2011;66:721-5.

Takazawa T, Horiuchi T, Yoshida N, et al. Flow cytometric Wardi G, Villar J, Nguyen T, et al. Factors and outcomes asso-
investigation of sugammadex-induced anaphylaxis. ciated with inpatient cardiac arrest following emergent
Br J Anaesth 2015;114:858-9. endotracheal intubation. Resuscitation 2017;121:76-80.

Takazawa T, Mitsuhata H, Mertes PM. Sugammadex and Warr J, Thiboutot Z, Rose L, et al. Current therapeutic uses,
rocuronium-induced anaphylaxis. J Anesth 2016;30:290-7. pharmacology, and clinical considerations of neuro-
muscular blocking agents for critically ill adults. Ann
Takazawa T, Miyasaka K, Sawa T, et al. Current status Pharmacother 2011;45:1116-26.
of sugammadex usage and the occurrence of sugam-
madex-induced anaphylaxis in Japan. APSF Newsl Welch JL, Seupaul RA. Update: does rocuronium create bet-
2018;33:1-13. ter intubating conditions than succinylcholine for rapid
sequence intubation? Ann Emerg Med 2017;69:e55-6.
Thapa S, Brull SJ. Succinylcholine-induced hyperkalemia
in patients with renal failure: an old question revisited. Workum JD, Janssen SHV, Touw HRW. Considerations
Anesth Analg 2000;91:237-41. in neuromuscular blockade in the ICU: a case report
and review of the literature. Case Rep Crit Care
Thilen SR, Weigel WA. Sugammadex: a costly simple solu- 2020;2020:8780979.
tion that is not really solving the problem. Anesth Analg
2020;131:e73-e74. Yanai M, Ariyoshi K. Two cardiac arrests that occurred after
the administration of sugammadex: a case of Kounis syn-
Todd MM, Hindman BJ, King BJ. The implementation of drome. Case Rep Emerg Med 2020;2020:6590101.
quantitative electromyographic neuromuscular monitor-
ing in an academic anesthesia department. Anesth Analg Yoshida T, Sumi C, Uba T, et al. A rare case of atropine-resis-
2014;119:323-31. tant bradycardia following sugammadex administration.
JA Clin Rep 2020;6:18.
Togioka BM, Yanez D, Aziz MF, et al. Randomised con-
trolled trial of sugammadex or neostigmine for reversal of Zwiers A, van den Heuvel M, Smeets J, et al. Assessment of
neuromuscular block on the incidence of pulmonary com- the potential for displacement interactions with sugam-
plications in older adults undergoing prolonged surgery. madex: a pharmacokinetic-pharmacodynamic modelling
Br J Anaesth 2020;124:553-61. approach. Clin Drug Investig 2011;31:101-11.

Tran DTT, Newton EK, Mount VAH, et al. Rocuronium versus


succinylcholine for rapid sequence induction intubation.
Cochrane Database Syst Rev 2015;2015:CD002788.

CCSAP 2021 Book 1 • Pulmonary and Endocrinology 200 Interactive Case: Neuromuscular Blocking Agents
Self-Assessment Questions
Questions 25–28 pertain to the following case. postoperatively because of her critical condition and
K.T. is a 19-year-old previously healthy woman (height open abdomen; hence, the anesthetist opts not to pur-
63 inches, weight around 104 kg [230 lb]) who presents sue reversal of her NMBA. Qualitative train-of-four (TOF)
emergently after a motor vehicle collision. Apparent injuries assessment yields a train-of-four count (TOFC) of 3 at
include blunt abdominal trauma and left open femur fracture. the time of operating room exit. Which one of the fol-
K.T. denies any contributory medical history or home drugs. lowing is best to communicate to the surgical ICU team
She appears to have morbid obesity. Her vital signs on pre- regarding K.T.’s postoperative care?
sentation to the trauma bay are blood pressure 86/44 mm Hg,
A. She should not have any residual NMB because
heart rate 126 beats/minute, temperature 93.2°F (34°C), and
it has been more than 1 hour since vecuronium
peripheral oxygen saturation 94%. Results of an arterial blood
administration.
gas with point-of-care assessment include pH 7.25, Hgb 8.2 g/
B. She has minimal residual NMB that should not
dL, K 3.8 mEq/L, and lactate 2.0 mmol/L. Her FAST (Focused
interfere with postoperative care.
Assessment with Sonography for Trauma) examination is pos-
C. She has moderate NMB that should be kept in mind
itive for fluid in the right upper quadrant. Although K.T.’s initial
when making neurologic assessments.
Glasgow Coma Scale score was 14 on arrival, it decreased to
D. She has deep NMB and aggressive analgosedation
8 during secondary trauma assessment. The ED physician
should be initiated promptly.
calls for rapid sequence intubation (RSI). K.T. is administered
etomidate 30 mg then succinylcholine 150 mg for RSI per the 27. Which one of the following best explains K.T.’s current
recommendation of the emergency department pharmacist. level of NMB?
The trauma surgeon attending orders the operating room to A. Prolongation of vecuronium duration from
prepare for emergency exploratory laparotomy. hypothermia and acidosis
25. K.T. is successfully intubated and taken to the operating B. Reduction of vecuronium duration from increased
room, where the anesthetist pursues general anesthe- pseudocholinesterase activity
sia with desflurane and initial doses of midazolam and C. Up-regulation of nicotinic acetylcholine receptors
fentanyl. The trauma surgeon prepares to incise the (nAChRs) from prolonged immobility
abdomen for damage control laparotomy and requests D. Down-regulation of NAChRs in obesity
surgical paralysis. Which one of the following strategies 28. K.T. is successfully transferred to the surgical ICU and
is best to recommend to K.T.’s surgical care team regard- stabilized. The next morning on rounds, the trauma sur-
ing surgical neuromuscular blockade (NMB)? gical attending proposes a continuous cisatracurium
A. Do not use neuromuscular blocking agents (NMBAs) infusion to expedite the time to abdominal fascial clo-
given lack of evidence supporting benefit in sure. Which one of the following best assesses clinical
laparotomy. considerations for NMB for K.T.?
B. Routinely use intraoperative NMBAs for emergency A. Its use has been studied in many high-quality trials
surgical cases. for this indication.
C. Avoid NMBA use in open abdominal cases because B. It may increase intra-abdominal pressure and is not
it may increase risk of intra-abdominal hypertension. advised.
D. Use NMB in laparotomy for optimizing operating C. Its use is associated with decreased length of stay
conditions. and mortality in abdominal indications.
26. K.T.’s anesthetist administers vecuronium 10 mg at the D. It has not consistently decreased time to abdominal
start of surgical incision. Crystalloid and blood prod- closure and may increase risks of acquired
ucts are also administered throughout the case. K.T. weakness and delirium.
is found to have a grade 4 liver laceration and multi-
ple bowel injuries, for which the trauma team pursues Questions 29–32 pertain to the following case.
extensive surgical interventions, completing the case M.K., an 82-year-old man, is brought to your center after
1 hour later. K.T.’s abdominal incision is left open and falling from the roof of his barn. He was minimally respon-
covered with a temporary abdominal closure dressing sive at the scene when medics arrived, so he was intubated
system. Her temperature is 96.8°F (36°C) despite use in the field, about 10 minutes ago. M.K. arrives motionless
of an intraoperative warming device. Recent arterial and receiving bag-mask ventilations by the medics, who
blood gas results include pH 7.30, Pao2 98 mm Hg, Paco2 relay they administered 20 mg of etomidate and 100 mg of
34 mm Hg, and HCO3 18 mEq/L. K.T. will remain intubated succinylcholine for induction of RSI. A brief review of his

CCSAP 2021 Book 1 • Pulmonary and Endocrinology 201 Interactive Case: Neuromuscular Blocking Agents
electronic medical record indicates M.K. has a medical his- albumin 4.3 g/dL, total bilirubin 0.4 mg/dL, AST 43 U/L,
tory of benign prostatic hypertrophy. His height and weight at and ALT 32 U/L. Repeat imaging suggests no worsening
his last encounter were 74 inches and 82 kg (180 lb). His vital of intracranial pathology, and brain oxygenation monitor-
signs on presentation today are blood pressure 120/84 mm ing is unavailable. Despite appropriate use of hypertonic
Hg, heart rate 116 beats/minute, temperature 95.9°F (35.5°C), saline for the past 24 hours, M.K.’s ICP is often 26–28
and peripheral oxygen saturation 99%. Because of high con- mm Hg. The neurosurgery attending physician on rounds
cern for traumatic brain injury (TBI) and/or spinal cord injury, wants to pursue NMB. Which one of the following best
the neurosurgery resident was paged in advance so that he evaluates the appropriateness of initial NMBA therapy
could assess M.K. promptly upon arrival. for M.K.?
29. In debriefing about his RSI, which one of the following A. Current quality program guidance supports a test
best evaluates the use of NMBA therapy for M.K.? dose of NMBA to assess efficacy in reducing ICP,
according to high-quality evidence suggesting a
A. Succinylcholine is always preferred in out-of-
mortality benefit.
hospital RSI because of its shorter duration of
B. Current quality program guidance supports a
action.
test dose of NMBA to assess efficacy in reducing
B. Rocuronium may have been preferred, had severe
ICP, according to limited evidence suggesting a
TBI been suspected.
reduction in ICP.
C. Succinylcholine was the ideal agent, but dosing
C. Current quality program guidance supports a
should have been optimized to 140 mg.
continuous infusion of NMBA to assess efficacy in
D. Rocuronium would have been preferred to limit the
reducing ICP, according to high-quality evidence
need for sedative administration during transport.
suggesting a mortality benefit.
30. Shortly after the neurosurgeon’s initial assessment, M.K. D. Current quality program guidance supports a
begins to have spontaneous movement. Imaging reveals continuous infusion of NMBA to assess efficacy
a large traumatic subdural hematoma with a 5-mm mid- in reducing ICP, according to limited evidence
line shift. An updated neurologic examination informs suggesting a reduction in ICP.
the surgical team’s decision to pursue emergency
32. M.K.’s neurosurgical team ultimately decides to initiate
decompressive craniectomy. In addition, the emergency
a continuous NMBA infusion but desires a direct NMB
medicine physician reports the temporary advanced
reversal strategy in case an urgent neurologic assess-
airway placed in the field must be replaced with an endo-
ment is deemed necessary. Which one of the following is
tracheal tube and requests a repeat RSI to accomplish
best to recommend for M.K.’s NMBA therapy?
this while the operating room is being prepared. It is now
40 minutes since medics performed M.K.’s initial intuba- A. Atracurium infusion titrated to TOFC of 2 or greater
tion. Which one of the following is best to recommend B. Atracurium infusion titrated to TOFC less than 2
regarding an NMBA regimen for M.K.’s RSI? C. Rocuronium infusion titrated to TOFC of 2 or greater
D. Rocuronium infusion titrated to TOFC less than 2
A. Further NMBAs should be avoided.
B. Administer succinylcholine 120 mg.
C. Administer rocuronium 100 mg. Questions 33–36 pertain to the following case.
D. Administer rocuronium 100 mg followed by
Three days ago, L.S., a 29-year-old woman, was admitted
sugammadex 8 mg/kg.
to your medical ICU with septic shock. She has no contrib-
31. M.K. is now on his second postoperative day in the utory medical history or chronic medications, though she
neurosurgery ICU, having undergone successful crani- appears cachectic (current weight is 54 kg). L.S.’s hospital
otomy with subdural hematoma evacuation earlier this course has been complicated by multiorgan failure, includ-
admission. Intracranial pressure (ICP) monitoring and ing acute kidney injury, shock liver, acute heart failure, and
intermittent ventricular drainage by external ventricular mixed respiratory failure necessitating the use of continuous
drain are ongoing. He has received no NMBAs since his positive airway pressure. She was found to have bacteremia
procedure and is achieving optimal sedation with fen- with endocarditis from methicillin-resistant Staphylococcus
tanyl and propofol infusions according to institutional aureus related to intravenous polysubstance use and has
protocol. His head of bed is elevated 30 degrees, and been receiving appropriate antimicrobial and resuscitative
he has sustained goals of normothermia, normoglyce- therapies. L.S. has been almost immobile and with subdued
mia, Pao2 100 mm Hg or more, Paco2 30–35 mm Hg, pH mentation since admission, though she can follow basic
7.35–7.45, and electrolytes within normal limits. Other commands and has not required mechanical ventilation.
values from this morning’s laboratory tests include SCr Although most of her critical conditions are stabilizing, she
1.5 mg/dL, BUN 23 mg/dL, total serum protein 6.8 g/dL, still requires inotropic support and supplemental oxygen.

CCSAP 2021 Book 1 • Pulmonary and Endocrinology 202 Interactive Case: Neuromuscular Blocking Agents
This morning’s laboratory values include Na 139 mEq/L, K 3.9 is desired to maintain optimal operating conditions for
mEq/L, Cl 102 mEq/L, CO2 39 mEq/L, BUN 28 mg/dL, SCr 1.9 the rest of surgery, which is only expected to be about 30
mg/dL, and glucose 102 mg/dL. After careful consideration of more minutes. The anesthesiologist elects to administer
her many critical conditions and associated risks, L.S.’s care rocuronium 50 mg because the team wishes to extubate
team elects to pursue surgical intervention for her severe L.S. shortly after her procedure. Once surgery is com-
infective endocarditis. plete, qualitative TOF assessment yields a TOFC of 4 with
fade. Which one of the following is best to recommend
33. L.S. is given cisatracurium 0.1-mg/kg intravenous push
regarding a reversal strategy for L.S.’s intraoperative
with induction of general anesthesia and successfully
NMBA therapy?
intubated. The cardiothoracic surgeon and anesthe-
siologist agree that deep NMB is indicated during the A. She has minimal residual NMB, and no
procedure to optimize operating conditions and reduce pharmacologic reversal is indicated.
energy expenditure during the therapeutic cooling period B. She has shallow residual NMB and should receive
during cardiopulmonary bypass. Which one of the follow- neostigmine 5 mg with glycopyrrolate 1 mg.
ing is best to recommend for L.S.’s intraoperative NMBA C. She has minimal residual NMB and should receive
therapy? sugammadex 4 mg/kg.
D. She has shallow residual NMB and should receive
A. Continuous NMBA infusion, titrated to maintain a
sugammadex 2 mg/kg.
TOFC of 0
B. Intermittent doses of NMBA, as needed to maintain 36. L.S.’s anesthesiologist administers neostigmine 4 mg
a TOFC of 0 with glycopyrrolate 0.8 mg at the end of the case, and
C. Continuous NMBA infusion, titrated to maintain a she is transferred to the cardiothoracic surgery ICU,
TOFC of 1–3 remaining intubated. Several hours later, she is hemody-
D. Intermittent doses of NMBA, as needed to maintain namically stable off vasoactive agents and is following
a TOFC of 1–3 commands; however, she cannot be weaned from the
ventilator. You suspect residual NMB is playing a role in
34. Which one of the following is best to recommend regard-
L.S.’s clinical presentation and request quantitative TOF
ing L.S.’s intraoperative NMBA selection and reversal
assessment, which yields a train-of-four ratio (TOFR)
strategy?
of 0.7. Perplexed, the intensivist asks for your thoughts
A. Cisatracurium may be preferred to rocuronium regarding reversal of residual NMB at this point. Which
because of its significantly shorter duration of one of the following is best to recommend for L.S.’s
action. NMBA reversal?
B. Cisatracurium may be preferred to rocuronium
A. Residual NMB is present, and sugammadex 1–2
because it can be reversed with neostigmine, even
mg/kg could be considered if likely benefits
at deep levels of NMB.
outweigh risks, including rare bradycardia and
C. Rocuronium may be preferred to cisatracurium
anaphylaxis.
because it can be reversed with sugammadex, even
B. Residual NMB is present, and sugammadex 16 mg/
at deep levels of NMB.
kg could be considered if likely benefits outweigh
D. Rocuronium may be preferred to cisatracurium
risks, including rare bradycardia and anaphylaxis.
because its duration of action is not altered by
C. Residual NMB is present, and extubation should
hypothermia or renal dysfunction.
be delayed until the TOFR is at least 0.8 to avoid
35. L.S.’s intraoperative peripheral nerve stimulation (PNS) postoperative pulmonary complications (PPCs).
monitoring shows spontaneous recovery of NMB about D. This degree of residual NMB has not been
120 minutes after her intubating dose of cisatracurium. associated with PPCs, so the likely benefits of
Hypothermic cardiopulmonary bypass is now complete, reversal agents are not outweighed by their risks.
and L.S. has regained normothermia. A redose of NMBA

CCSAP 2021 Book 1 • Pulmonary and Endocrinology 203 Interactive Case: Neuromuscular Blocking Agents
Learner Feature Evaluation: Interactive Case: Neuromuscular Blocking Agents

As you take the posttest for this feature, also evaluate the 4 0. The learning assessment activities used in the feature
material’s quality and usefulness, as well as the achievement were effective.
of learning objectives. Rate each item using this 5-point scale: 41. The feature was effective overall.
• Strongly agree 42. The activity met the stated learning objectives.
• Agree 4 3. If any objectives were not met, please list them here.
• Neutral
• Disagree
OTHER COMMENTS
• Strongly disagree
4 4. Please provide any specific comments related to any
31. The content of the feature met my educational needs.
perceptions of bias, promotion, or advertisement of
32. The content of the feature satisfied my expectations. commercial products.
33. The author presented the feature content effectively. 45. Please expand on any of your above responses, and/or
34. The content of the feature was relevant to my practice provide any additional comments regarding this feature:
and presented at the appropriate depth and scope.
35. The content of the feature was objective and balanced. Questions 46–48 apply to the entire learning module.
36. The content of the feature is free of bias, promotion, and 46. How long did it take you to read the instructional materi-
advertisement of commercial products. als in this module?
37. The content of the feature was useful to me. 47. How long did it take you to read and answer the assess-
38. The teaching and learning methods used in the feature ment questions in this module?
were effective. 4 8. Please provide any additional comments you may have
39. The active learning methods used in the feature were regarding this module:
effective.

CCSAP 2021 Book 1 • Pulmonary and Endocrinology 204 Interactive Case: Neuromuscular Blocking Agents

You might also like